You are on page 1of 190

0.1.

BIOL 111 2007/2008 1

0.1 BIOL 111 2007/2008 7. After leaf fall,the main method transpi-
ration in deciduos plants is by
1. Which of the following statement is true (a) i (b) ii (c) iii (d) all of the above
about transpiration in plants ? Solution
(a) Transpiration aids the descent of wa-
ter in trees .
(b) High leaf surface area to volume ratio 8. The brisk movement of desmids are known
reduces the rate of transpiration to be
(c) Transpiration helps to get rid of ex- (a) chemotactic (b) prototactic
cess water (c) thermotactic (d) none of these
(d) All of the above Solution
Solution C B
Use the following information to an- 9. Which of the following is not a condition
swer question 2 and 3 necessary for movement in plants ?
(i) Active transport (a) oxygen (b) Enzymes
(ii) Osmosis and active transport (c) oxidation stress (d) Water .
(iii) Root pressure Solution
(iv) Transpiration pull and root pres- B
sure .
10. Movement of curvature occurs in which
2. The driving force(s) for the transport of
of the following plants ?
water is/are
(a) Oscillatoria sp (b) Euglena sp
(a) i (b) ii (c) iii (d) iv
(c) Zea sp (d) All of the above
Solution D
Solution C
3. The movement of organic substance in
plant is driven by 11. Which of the following is not an external
(a) iv (b) iii (c) ii (d) i agent causing taxism in plants
Solution D (a) Temperature (b) Heat
(c) light (d) Sound
4. Through which of the following structures Solution
does the movement of water through the D
root hairs to the xylem occur ?
(a) Collenchyma cells (b) Protoplasm 12. The response of M umosa pudica leaves
(c) Golgi bodies (d) Cell vacuoles . to touch is an example of
Solution D (a) thigmotropism (b) haptotropism
Use the following information to an- (c) nyctinasty (d) seismonasty
swer question 5-7 Solution
(i) Stomata transpiration D
(ii) Cuticular transpiration
13. Which of the following factors is least
(iii) lenticular transpiration
likely to affect transpiration rate in plants
5. About 90% of water lost in higher plants (a) Air humidity (b) Wind speed
occurs via (c) Ambient temperature (d) Root pres-
(a) i (b) ii (c) i and ii (d) iii sure
Solution A Solution
D
6. 10% of transpiration in plant usually oc-
curs through 14. Which of the following represent active
(a) ii and iii (b) iii physiological processes in plant ?
(c) none of the above (d) all of the above (a) osmosis (b) Diffusion and cytoplas-
Solution mic streaming
2

(c) Osmosis and diffusion (d) Cytoplas- (c) optimum growth temperatures for trop-
mic streaming ical miaze is about 350 C
Solution (d) optimum temperature for minter wheat
C growth is about 350 C
Solution
15. Reproductive growth in plants occurs af- C
ter
(a) the period of young plant to produc- 21. Deficiency of calcuim results in
tion of flowers and fruits (a) stunded growth of stem and roots
(b) germination to period of young shoot (b) stunted growth of leaves and buds
production (c) Yellowing of leaves and buds
(c) germination to produce of fruits (d) poor fruiting
(d) Germination to vegetative growth . Solution
Solution A
D
22. The main function of phosphorous in the
16. The element which is essential for chloro- growth of plants is
phyll synthesis is (a) formation of chlorophyll
(a) P (b) Cu (c) K (d) Fe (b) formation of cell and regulation of cell
Solution activities
D (c) formation of cell wall and enhance-
ment of healthy growth .
17. Which of the following is the correct or-
Solution
der of occurrence of phases in a sigmoid
C
growth curve ?
(a) log-lag-stationary-declaration
23. In which of the following groups of ele-
(b) Lag-log-stationary-declaration
ment will all not be obtained by plants
(c) Deceleration-log-staionary-lag
from the mineral salts present in soil par-
(d) Staionary -lag-log-deceleration
ticles and humans
Solution
(a) Boron,molybdenum and zinc
(b) Carbon,sulphur and iron
18. Phase II of seed germination involves (c) Hydrogen,oxygen and carbon
(a) Active metabolism (b) passive metabolism (d) Boron,zinc and oxygen
(c) active imbibition (d) active metabolism Solution
and active water uptake . A
Solution
A 24. The highly toxic fertilizer that can only
be used when there is abundant wate is
19. The tissues involved in manufactured food (a) Single Super phosphate ......SSP
translocation in plants are the (b) Nitrogen phosphate and potassium
(a) phleom (b) xylem .....NPK
(c) cortex (d) phloem and xylem (c) Urea ....N
Solution (d) Nitrogen,potassium and super phos-
A phate
Solution C
20. Which of the following statement is cor-
rect ? 25. A condition whereby leaves grow down-
(a) Temperature range that favour plant ward as a result of exposure to very low
growth does not vary with plant species concentration of ethylene is known as
(b) Constant temperature favours plant (a) seismonasty (b) chemonasty
growth (c) epinasty (d) thigmonasty
0.1. BIOL 111 2007/2008 3

Solution 33. Which of the following is not an economic


B importance of sphagnum spp .
(a)it is used in aquaria decoration
26. The algea exist in all of the following (b) it is a raw material in toilet paper
forms except making
(a) unicellular (b) filamentous (c) it is a source of insecticide
(c) sclerenchmatous (d) parenchymatous (d) it is used in making cheap cloth
Solution Solution
C

27. Variation in colour of algai species gives 34. in pteridophytes,the spores are contained
priliminary identification,but conclusion in the structure called
has to be made using (a) sporophyte (b) sphenophyta
(a) biochemical analysis (b) anatomical (c) lycophyta (d) psilophyta
analysis Solution
(c) morphological analysis (d) none of C
the above
Solution 35. Which of the following taxonomic divi-
A sion is represented by a single family .
(a) pterophyta (b) sphenophyta
28. What colour is shown by the members of
(c) sporangium (d) strobilus
pyrophyta ?
Solution
(a) green (b) Brown red /blue
(c) Brown-red (d) Diamond-like
Solution 36. Which of the following aquatic fern is
grown in rice padies for nitrogen fixation
29. A typical example of Bacillariophyta is ?
(a) Fucus spp (b) Dinobryon spp (a) azolla (b) salvinia
(c) vaucheria spp (d) arachnoidiscis spp (c) msrsilea (d) Dryopteris
Solution Solution
D

30. The sac-fungi are otherwise called 37. A typical example of a gymnosperm with
(a) basidiomycetes (b) Oomucetes broad leaves is
(c) Ascomycetes (d) Deuteromycetes (a) pinus (b) cycas
Solution (c) yews (d) redwood
C Solution

31. Which of the following causes athlete’s


foot? 38. The Anthophyta are also called
(a) puccinia spp (b) penicilium notatum (a) gymnosperm (b) spermatophytes
(c) rhizopus stolonifer (d) erysiphae spp (c) flowing plants (d) pteridophytes .
Solution Ans C

39. A major characteristics of dicotyledonous


32. The sporophyte of hornworts is attached plants is
to the gametophyte by the (a) identical leaf surfaces
(a) elaters (b) foot (b) parallel venation
(d) rhiziods (d) seta (c) fibrous roots
Solution (d) persistent taproot
B Ans D
4

40. Which of the following is not one of the (c) 40-50 0 C (d) 50-600 C
challenges of terrestrial environment Ans A
(a) overcoming water shortage
(b) Transporting water and dissolved sub- 48. In the absence of which of the following
stance will photosynthesis not occur
(c) protection of embro aganist desicca- oxygen (b) Sulphur
tion (c) Ammonia (d) Hydrosulphide
(d) having chlorophyll for photosynthesis Ans A
Solution 49. The site of photosynthesis in plants is the
D (a) chloroplast (b) spongy mesophyll
41. slime moulds are characterized by wall- (c) palisade mesophyll (d) collenchyma
less assimilative states and Ans C
(a) reproduction by binary fision 50. The absence of iron and magnesium in a
(b) reproduction by budding plant’s nutrient leads to
(c) reproduction by spores (a) chlorosis (b) defoliation
(d) reproduction by fragmentation (c) stunted growth (d) witing
Ans C Ans A
42. The following are asexual spores except 51. The most abundant pigment found in all
(a) sporangiospore (b) conidospore photosynthesis plant is
(c) ascospore (d) chlamydospore . (a) chlorophyll (b) carotene
Ans (c) erythtoaphins (d) xanthaphyll
Ans A
43. In filamentous forms of chlorophyte,motile
cells are produced only as 52. Respectively ,the light dependent reac-
(a) somatic cells (b) reproductive cells tions and light independent reaction of
(d) vegetative cells (d) all of the above photosynthesis take place in the
Ans B (a) grana and stroma (b) stroma and grana
(c) outer membrane and inner membrane
44. The development of a root-like structure (d) chloroplast DNA and chloroplast ri-
started in the bosones
(a) chlophyta (b) bryophyta Ans A
(c) phaeophyta (c) pyrophyta
Ans B 53. The molecule produced by the process
of photophosphorylation during the light
45. The red algae is divided into two sub- dependent stage of photosynthesis is
classes namely (a) ADP (b) ATP (c) NADP (d) NADPH2
(a) Bangiophyceae and chlorophyceae Ans B
(b) bangiophyceae and rhodophyceac
(c) floridiophyceae and rhodophyceae 54. The two types of photosystems inside which
(d) banglophyceae and floridiophyceae are located chlorophyll and accessory pig-
Ans D ment molecules are
(a) III and IV (b) I and II
46. The concentration of carbon dioxide for (c) IV and V (d) IV and III
optimum photosynthesis is Ans B
(a) 0.03 % (b) 0.3% (d) 0.5% (d) 0.1%
Ans A 55. In which type of the following membrane
are photosystems visible as particles ?
47. The optimum temperature range for pho- (a) ribosomal (b) stroma
tosynthesis is (c) outer (d) Thylakiod
(a) 20-30 0 C (b) 30-400 C Ans C
0.1. BIOL 111 2007/2008 5

56. The series of reaction of the dark stage 63. Two example of monera are
of photosynthesis are cyclical and known (a) bacterial and blue-green algea
as the (b) bacterial and green algae
(a) calvin cycle (b) carbon cycle (c) bacterial and brown algea
(c) nitrogen cycle (d) Hatch-slack path- (d) bacterial and all the eukaryotic algae
way Ans A
Ans A
64. The statement ”Bacterial are struc-
57. The CO2 acceptor in the dark stage of turally simple but biochemically com-
photosynthesis is the plex ” is
(a) ribulose bisphosphate (a) completely false
(b) nicotinamide ademine dinucleotide phos- (b) completely true
phate (c) sometimes true
(c) glycerate phosphate (d) sometimes false
(d) glyceraldehydes 3-phosphate Ans B
Ans A
65. The gametophytes of Merchantia and Dry-
58. With respect to energy input and out- opteris are
put,the process of the dark stage of pho- (a) all haploid (b) haploid and diploid re-
tosynthesis can be described as spectively
(a) exoergic (b) endergonic (c) all diploid (d) diploid and haploid re-
(c) exothermic (d) endotropic spectively
Ans B Ans A
59. The first product of photosynthesis is 66. The gametophyte of selaginella is the
(a) glucose (b) glycerate phosphate (a) sporanginlla sp
(c) glyceraldehyde 3-phosphate (d) triose (b) Dryopteris sp
phosphate (c) lycopodium sp
Ans D (d) Pinus sp
Ans B
60. The major reproductive structure devel-
oped by the bryophytes over the fungi is 67. Which of the following is homosporous ?
(a) multicellular gametengia (a) selaginella sp (b) Dryopteris sp
(b) unicellular gametangia (c) lycopodium sp (d) pinus sp
(c) nutritive cells around gametangia Ans B
(d) numerous gametangia
Ans 68. The reproductive process that occurs only
in Angiosperms is
61. On the basis of form and function ,plants (a) pollination (b) fertilization
are divided into (c) double fertilization (d) double polli-
(a) vascular and non-vascular nation
(b) vascular and eukaryotes Ans C
(c) thallophytes and bryophytes
(d) photosynthetic and non-photosynthetic 69. Which of the following is true of the en-
Ans A dosperm ?
(a) it is diploid (b) it is haploid
62. one of these is not a feature of the non- (c) it is triploid (d) none of the above
vascular, non photosynthetic eukairyotes Ans C
(a) possession of definite nucleus
(b) possession of outer membrane-bound 70. The delay in formation of seed in the life
organelles cycle of the Gymnosperm is from
(c) Chlorophyllous (d) Achlorophyllous (a) microspore formation
Ans C (b) magaspore formation
6

(c) pollination (d) fertilization (c) class (d) order


Ans Ans D

3. The plant Baphia latiloi was so named


Use the diagram below to answer due to
question 71-74 (a) its country of origin (b) name of its
digram discover
(c) its colour (d) its habitat
71. The plant whose life cycle is illustrated Ans B
is
4. The bacterial pili are used for
(a) selaginella sp (b) pinus sp
(a) sexual reproduction
(c) Dryopteris sp (d) lycopodium sp
(b) vegetative reproduction
Ans B
(c) cell division
72. The stage labelled II in the digram rep- (d) infection
resent Ans A
(a) Antheridium (b) pollen grain
5. The naked mass of protoplasm called plas-
(c) Microsporocyte (d) Microsporophyll
modium is commonly found in
Ans B
(a) rhizopus spp (b) puccinia spp
73. The stages labelled III and IV are the (c) erysiphe spp (d) polytrichum spp
(a) ovulate and staminate cone Ans
(b) staminate and ovulate cone
(c) megasporophyll and microsporophyll 6. Which of the following is to make wool
(d) stobilus and cone for textile
Ans B (a) Marchanita spp (b) riccia spp
(c) sphagnum spp (d) polytrichum spp
74. The process that takes place in I is Ans
(a) meiosis (b) mitosis
(c) death of some spores 7. Division psilophyta is characterized by
(d) growth of some spores the following except
Ans A (a) absence of roots (b) small and scale-
like leaves
75. Alternative of generation in bryophytes (c) homomorphic alteration of generation
occurs between two (d) heteromorphic alteration of genera-
(a) distinct plants (b) distinct and inde- tion
pendent plants Ans C
(c) distinct and dependent plants
(d) independent and dependent plants 8. Scouring rushes are obtainable from
Ans B (a) lycopodium spp (b) epuisetum spp
(c) psilotun spp (d) isoetes spp
Ans B
0.2 BIOL 111 2008/2009
9. A typical aquatic fern that shows N-fixation
1. Which of the following types of classifi- is
cation reflects genetic and evolutionary (a) Salvinia spp (b) marilea spp
relationship ? (c) azolla (d) dryoptris spp
(a) Phenetic (b) Artificial Ans B
(c) Natural (d) Molecular
Ans C 10. In fruit bearing plants gametophytic stage
is
2. The suffix ”ales” usually signifies (a) dominant and reduced (b) indepen-
(a) kingdom (b) phylum dent and reduced
0.2. BIOL 111 2008/2009 7

(c) dependent and reduced (d) totally ab- reduces it .


sent (b) low leaf surface area to volume ratio
Ans C reduces it .
(c) High leaf surface area to volume ratio
11. Energy is released when potential energy increases it .
of (d) All of the above
(a) substrate is greater than product Ans C
(b) product is greater then substrate
(c) substrate is less than product 19. The route followed by the movement of
(d) product is less than product water through the casparian strip is the
Ans A ...... pathway
(a) apoplast (b) symplast
12. The form of sugar translocated by plants (c) vacuolar (d) all of the above
is Ans
(a) glucose (b) fructose
(c) sucrose (d) starch 20. Which of the following algae are incor-
Ans C rectly paired with their description
(a) Dioflagllates–marine plankton,whiling,spilling
13. The component of the endodermis which movement,characteristic wall
prevent passage of water is (b) Chrysophytes – golden algae ,xantho-
(a) cutin (b) lignin phylls predominant,flagellated,fresh wa-
(c) suberin (d) cellulose ter plankton
Ans C-Subein in the cell walls of the en- (c) Bacillariophyta–diatom,two-piece shells
dodermis and of bark it render the tissue of silica .
resistant to decay and entry of water (d) phaeophytes-multicellular brown al-
gae,seaweeds.
14. The main role played by translocation in
Ans
plants is distribution of
(a) water (b) heat 21. All Bryophytes (Mosses,liverworts and Horn-
(c) mineral salts (d) gases worts) have the following characteristics
Ans C (a) Reproductive cells in protective cham-
bers and a waxy cuticle
15. In which of the following movement is the
(b) A waxy cuticle ,true leaves and re-
degree of movement dependent on the in-
productive cells in protective chembers
tensity of stimulus ?
(c) Vascular tissues,true leaves and a waxy
(a) Nastic (b) Tropic
cuticle
(c) Mechanical (d) vital
(d) Reproductive cells in protective cham-
Ans B
bers and vascular tissues
16. Which ATP bond is broken and trans- Ans A
ferred together with the phosphate ion ? 22. Important terrestrial adaptation that evolved
(a) 1 (b) 2 (c) 3 (d) 4 exclusively in seed plants include all the
Ans following except
17. Which one of these process occurs in both (a) pollination by wind or animal instead
the pholem and xylem ? of fertilization by swimming sperm .
(a) Diffusion (b) Root pressure (b) transport of water through vascular
(c) Transpiration pull (d) osmosis tissue
Ans D (c) retention of gametophyte plant within
sporophyte
18. Which of the following is true of transpi- (d) production of numerous reproductive
ration rate ? cells
(a) High leaf surface area to volume ratio Ans D
8

23. In the sexual life cycle of fungi ..... usu- (d) pseudoparenchymatous thallus
ally occurs shortly after Ans C
(a) syngamy,karyogamy (b) meiosis,fusion
of hapliod nuclei 29. Fungi lack chlorophyll ,hence they are
(c) spore production,formation of a dikary- (a) autotrophic (b) heterotropic
otic mycelium (c) coenocytic (d) all of the above
(d) syngamy,meiosis Ans B
Ans B
30. The complex fungi includes all except
24. Slime moulds are structurally (a) mushrooms (b) toadstools
(a) thalloid with septation (b) wall-less (c) scouring rushes (d) pufballs
assimilative Ans C
(c) a mass of intertwined hyphae
(d) an erect,semi-prostrate thallus 31. Lateral roots are initial form the ..... while
Ans B root hairs are from
(a) endodermis,epidermis (b) epidermis,endodermis
25. The evolutionary lineage of the Blue-green (c) epidermis,pericycle (d) pericycle,epidermis
algae in the sequence Ans D
(a) Chroococcus-synechocytis-champaesiphon
(b) synechocytis-chamaesi (c) Chamae- 32. Two growth hormons involved in promot-
siphon Chroocccus-synechocytis ing fruit development are
(d) None of the above (a) ethylene & auxins (b) cytokinnins &
Ans auxins
(c) gibberllins & auxins (d)gibberellins &
26. Vegatative non-vascular photosynthetic al- cytokinins
gae form one of the following Ans C
(a) Unicellular,multicellular motile and
non-motile . 33. ........ is the type of NPK fertilizer that
(b) parenchymatoes non-motile,coloniel and can be applied at seed planting
non-spihonous (a) NPK-20-10-10
(c) siphonous,filamentous,colonial and non- (B) NPK-10-10-20
filamentous (c) NPK-15-15-15
(d) All of the above (d) NPK-27-13-13
Ans D Ans C

27. Structural differentiation depicting evo- 34. The iron content of ferredoxin and cy-
lutionary forms among the Phaeophyta tochrome oxidase promotes their involve-
include ment in
(a) Ectocarpales-pilayella and Desmares- (a) synthetic reaction (b) hydrelytic and
tia oxidation reaction
(b) Ectocarpales-filamentous forms with (c) oxidation and reduction reaction
apical growth and Trichothallic form. (d) hydrolytic and synthetic reaction
Ans C
(c) Ectocarpales-pilayella and pseudoparenchyceae
(d) All of the above
Ans A 35. The rate of growth of plant species varies
with
28. A unique advanced characteristic of the (a) plant species,plant age,light intensity
florideophyceae is (b) plant type ,light intensity ,environ-
(a) simple uniserrate branching mental factors
(b) Branches fuse to varying degree (c) plant species ,plant age ,environmen-
(c) primary pit connection between adja- tal factors
cent cells. (d) plant species,light intensity,plant growth
0.2. BIOL 111 2008/2009 9

stage 44. The light dependent reaction take place


Ans C in the
(a) Thylakiod (b) Stroma
36. The tunica layer gives rise to (c) Lamella (d) polysome
(a) Stem primordial (b) flower primordal Ans B
(c) fruit primordal (d) leaf primordia
Ans D 45. The compound that first react with CO2
in the dark reaction of photosynthesis is
37. Within the temperature range,the high- known as
est growth rate is obtained at ..... tem- (a) nicotinamide adenine dinuceotide phos-
perature phate
(a) maximum (b) normal (b) ribulose biphosphate
(c) minimum (d) optimum (c) adenosine triphosphate
Ans D (d) ribulose biphosphate carboxylase
38. Gibberellins can stimulate flowering in Ans B
(a) onion (b) maize
46. The gametophyte of selaginella is the
(c) soybean (d) none of the above
(a) strobilus (b) sporophyll
Ans D
(c) prothallus (d) perigynium
39. Cysteine cannot be formed without ........ Ans C
(a) nitrogen (b) zinc
47. Which of the following is strictly monoc-
(c) iron (d) sulphur
cious ?
Ans D
(a) Marchantia (b) selaginellc
40. The deficiency of these elements does not (c) pethia (d) pinus
result in chlorosis Ans D
(a) Potassium,calcium,sulphur
48. pteridophyte and bryophyte gametangia
(b) potassium,zinc,Nitrogen
always have
(c) calcium,zinc,phosphorous
(a) protective covering of fertile cells
(d) potassium,sulphur ,Nitrogen
(b) protective jacket of fertile cells
Ans C
(c) protective endosperm of dormant cells
41. The chemical structure of the most abun- (d) none of the above
dant chlorophyll is Ans D
(a) C55 H70 O4 N4 M g
(b) C40 H56 O2 49. In fern,the sorus has a protective cover-
(c) C55 H72 O5 N4 M g ing knows as
(d) C55 H71 O6 N4 M g (a) protective sterile jacket
Ans C (b) indusium
(c) integument (d) none of the above
42. The site of photosynthesis in plants is Ans B
(a) chloroplast (b) spongy mesophyll
(c) leaves (d) pallisade mesophyll 50. The life cycle of Mosses and Dryopteris
Ans A shows that the
(a)gametophyte is dependent on sporo-
43. The most important raw material ,which phyte
also affect the rate of photosynthesis is (b) gametophyte is dependent of sporo-
(a) light and water (b) temperature and phyte
carbon dioxide (c)sporophyte is dependent on gameto-
(c) mineral salts and chlorophyll content phyte
(d) carbon dioxide and water (d) all of the above
Ans D Ans D
10

51. The fertile fronds of fern bear sori on the (c) double fertilization (d) triple fertiliza-
(a) upper surface of the fronds tion
(b) lower surface of the fronds Ans C
(c)lower surface of the prothallus
(d) upper surface of the capsule 59. The production of antherozoids in Marchan-
Ans B tia involves
(a) meiosis in the mother cell
52. Reduction division that starts with 16 (b) mitosis and meiosis in the mother
spores and end up with 64 spores occurs cells
in (c) mitosis in the thallus
(a) selaginella and marchantia (d) mitosis in the mother cell
(b) marchantia and fern Ans A
(c) dryopteris and selaginella
(d) All of the above 60. Selaginella plant is
Ans C (a) homosporous (b) heterosporous
(d) magasporous (d) microsporous
53. The primitive fern are referred to as Ans B
(a) lepstosporangiate
(b) eusporangiate
(c) monosporangiate 0.3 BIOL 111 2010/2011
(d) disporangiate
Ans B 1. Which of the following taxonomic rank
ends with the suffix ”phyta”
54. Example of a group of plant that have
(a) Division (b) family
seeds but no fruit is
(c) species (d) class
(a) Grass (b) Thevetia
Ans A
(c) Conifer (d) ferns
Ans C 2. Only two division of the class phycotinae
are known to have chlorophyll .... and ....
55. The ovuliferous scales of pinus cone bear
(a) a,b (b) a,c
(a) three ovules (b) one ovule
(c) a, xanthophylls (d) a,carotene
(c) two ovules (d) four ovules
Ans B-the algae (class phycotinae ) are
Ans C
mostly aquatic and autotropic with all
56. Marchantia reproduces vegetatively by the possessing chlorophyll a,while two divi-
following except sion have chlorophyll a and c
(a) use of adventitious roots
(b) use of fibrous roots 3. Which of these features possessed by my-
(c) use of gemma cup cotinae makes them to placed among plants
(d) use of dead basal protion of thallus (a) absence of cell wall
Ans B (b) Engulfing or taking of solid food par-
ticles
57. The swollen basal portion of Dryopteris (c) formation of spores with cellulose cell
archegonium is called walls
(a) neck (b) venter (d) Amoeboid movement
(c) protective jacket (d) oospore Ans C
Ans B
4. Which of these pteridophytes has a stem
58. The type of fertilization that occurs only with distinct nodes and internodes
in Angiosperm is called (a) selaginella (b) equisetum
(a) triplold fertilization (b) single fertil- (c) Dryopteris (d) psilotum
ization Ans B
0.3. BIOL 111 2010/2011 11

5. An example of comma shaped bacteria is vegetative hypha lacks cross-walls (pl.septa,singular


(a) staphylococcus aureus septum)
(b) vibrio cholera
13. The most important feature of mosses
(c) agrobacterium tumefaciens
showing advancement over other Bryophytes
(d) rhizobium tumefaciens
is
Ans B
(a) spore formation (b) central stand
6. A bacteria which is known to fix nitrogen (c) rhizoid formation (d) seta formation
for its host plant is Ans C
(a) Agrobacterium turnefaciens 14. Epiphytic species in tropical forest are
(b) rhizobium tumefaciens common in
(c) esherichia coli (d) vibrio cholerae (a) psilophyta (b) Arthrophyta
Ans B (c) Pterophyta (d) lycophyta
Ans A
7. The pteridophytes are restricted to damp
environment because they need water for 15. Which of the following is an example of
(a) germination of their spores a broad leaved Gymnosperm ?
(b) growth of the sporophytes (a) Pinus spp (b) Lemna spp
(c) development of the zygote (c) mnium spp (d) cycas spp
(d) the movement of the antherozoids to Ans C
the archegonia for fertilization to take
place 16. Anthophyta are subdivided into 2 main
Ans D classes based on
(a) seed (b) cotyledous
8. New system of classification place bacte- (c) habit (d) flower structure
ria in Ans B
(a) proctista (b) monera
17. The stage at which a flower bud opens is
(c) protozoa (d) phycotinae
known as
Ans A
(a) Anthesis (b) fertilization
9. A nutrient-rich algae cultivated for the (c) Flower budding (d) Pollination
food is Ans A
(a) spirulina (b) diatom 18. Which of the following is required for the
(c) euglena (d) fucus process of cell enlargement ?
Ans D (a) Marcromolecules (b) Mineral elements
(c) stored food reserves (d) water
10. An example of specific name based on the
Ans D
colour of plant parts
(a) Acacia albida (b) Baphia 19. The complex ageing process that eventu-
(c) esculentum (d) maxima ally leads to death in plant is known as
Ans A (a) Senescence (b) Deceleration
(c) Morphogenesis (d) Development
11. Agaricus species are edible mushrooms Ans A
and belongs to
(a) water moulds (b) club-fungi 20. The dividing tissue at the tip of the plant
(c) sac-fungi (d) none of the above which gives rise to the primary plant body
Ans B is
(a) Lateral meristem (b) Determinate meris-
12. Lower fungi are coenocytic and tem
(a) septate (b) uninucleated (c) Apical meristem (d) slerenchymatous
(c) multinucleated (d) binucleated cells
Ans C-Coenocytic (non- septate ) i.e the Ans C
12

21. The corpus layer gives rise to .......... 28. Which of these commercial fertilizers in
(a) stem primordal (b) Parenchymatous highly toxic ?
cells (a) SSP (b) NPK (c) DNP (d) urea
(c) leaf primordial (d) slerenchymatous Ans D
cells
Ans B 29. Photosynthesis stops when the tempera-
ture in above 400 C because
22. which of the following is not a quantita- (a) Water becomes vapour
tive expression of growth ? (b) Matabolism is too high
(a) increase in leaf areas (c) Carbohydrates are denatured
(b) increase in root length (d) Enzymes are denatured
(c) increase in fresh weight Ans D
(d) increase in photosynthesis
30. The formation of ATP during photosyn-
Ans D
thesis is called ?
23. Deficiency of potassium in plant can cause (a) calvin cycle/ photorespiration
........ (b) photophosphorylation
(a) Yellowing of leaves (b) Mottling of (c) Kreb’s cycle / photoreactivation
lower leaves (d) phototransduction
(c) Poor root development (d) discoloura- Ans C
tion of leaves to orange
31. The dark stage of photosynthesis cannot
Ans D
occur
24. One of these phytohormones brings about (a) Without the light stage taking place
senescence of the flower parts at the on- (b) without light
set of fruiting ? (c) without the light-independent stage
(a) Ethylene (b) Gibberellins (d) without darkness
(c) Auxins (d) cytokinnins Ans D
Ans A 32. The product of photolysis are
(a) Electron (e− ) and proton (H + )
25. Which of the following elements are nec-
(b) Electron (e− ) and oxygen (O2 )
essary for the synthesis of chlorophyll ?
(c) protons (H + ) and oxygen (O2 )
(a) Nitrogen,phosphorus and potassium
(D) Protons,electrons and oxygen
(b) sulphur,iron and potassium
Ans D
(c) Nitrogen, iron and magnesium
(d) Nitrogen,calcium and sulphur 33. The process of the formation of triose
Ans C phosphate from phosphoglyceric acid is
a/an
26. Which of the following hormones are in-
(a) Oxidation process
volved in promoting cell division in plants
(b) reduction process
?
(c) condensation process
(a) Auxins and Gibberellins
(d) Transfer process
(b) Auxins and Cytokinins
Ans B
(c) Cytokinis and gibberellins
(d) Auxins and Ethylene 34. Movement of curvature occur in which of
Ans B the following plants
(a) Oscillatoria sp (b) Euglena sp
27. Which of the following stimulate the flow- (c) Zea sp (d) All of the above
ering of long day plant Ans C
(a) Gibberellins (b) Auxins
(c) Ethylene (d) Cytokinis 35. The response of Mimosa pudica leaves to
Ans A touch in an example of
0.3. BIOL 111 2010/2011 13

(a) Seismonasty (b) Haptotropism 42. The significance of electron carrier molecule
(c) Nyctinasty (d) Thigmotropism is to
Ans D (a) pass electron into the next molecule
(b) Transport hydrogen ions into the sys-
36. The sensitive reaction of the protoplasm tem generating a proton motive force
to various external agents is (c) Shuttle protons to ATP synthesis
(a) plant direction movement (d) All of the above
(b) plant non-direction movement Ans D
(c) plant movement .
(d) irritability 43. The chemical structure of chlorophyll b
Ans D is
(a) C55 H73 O5 N4 M g
37. Movements that are determined by the (b) C55 H70 O6 N4 M g
anatomy of the responding organ are called (c) C55 H71 O6 N4 M g
(a) Tropical movement (b) Directional move- (d) C55 H72 O5 N4 M g
ment Ans B
(c) Nastic movement (d) None of the above
44. The pigment Bacterioviridin is possessed
Ans A
by
(a) iron bacteria (b) Nitric bacteria
38. Which of the following is true about tran-
(c) purple sulphur bacteria (d) Green sul-
spiration in plants ?
phur bacteria
(a) it helps get rid of excess water
Ans D
(b) it aids the descends of water in trees
(c) High leaf surface area to volume ratio 45. Deciduous trees transpire mostly through
reduces the rate of transpiration (a) stomata (b) cuticle
(d) All of the above (c) lenticle (d) All of the above
Ans D Ans C

39. The site of photosynthesis in plants are 46. The colour of the reflected wavelength in
(a) Palisade mesophyll cell/tissues photosynthesis is
(b) spongy mesophyll cells (a) Green (b) Blue
(c) Palisade spongy tissues (c) Red (d) None of the above
(d) None of the above Ans A
Ans A
47. Movement that are obligate to the direc-
40. The product(s) of pyruvate conversion to tion of stimuli are
Acety-Co A include (a) diatropic (b) Orthotropic
(a) carbon dioxide and ATP (c) plagiotropic (d) Galvanotropic
(b) Carboxyl group attached to coenzyme Ans C
(c) One turn of the Kreb’s cycle com- 48. The by-product of photosynthesis in both
pleted photosynthetic and chemosynthetic bac-
(d) Both anaerobic and aerobic teria is
Ans A (a) water (b) carbon dioxide
(c) carbohydrate (d) sulphide
41. Respiration can best be defined as Ans A
(a) Breathing in and out
(b) The release of energy in the form of 49. One of the statement below is a good rep-
ATP . resentation of the characteristics of fungi
(c) A catabolic process (a) All fungi bear spores that germinate
(d) both anaerobic and aerobic into strands of hyphae
Ans B (b) The microscopic lower fungi mainly
14

live within a host reproduce mostly by (c) zygote (d) Mitotic


asexual spores . Ans A
(c) Higher fungi have elaborate fruiting
bodies,composed of hyphae,in which spores 57. ....... is the type of plant that has covered
are produced seeds
(d) All of the above (a) Angiosperm (b)Coniferophyte
Ans D (c) Gnetophyte (d) Bryophyte
Ans A
50. The shoot is the portion of the plant above
the roots and is composed of 58. ....... is the part of the angiosperm where
(a) stems,leaves, flowers and fruit the embryo develops
(b) The stem bark and cork (a) ovule (b) fruit
(c) stem lateral meristems (c) Endosperm (d) Seed
(d) None of the above Ans D
Ans A
59. ...... is the structure that has a vegetative
51. The root apical meristems produces cell and a generative cell .
(a) New root cap cells ahead of the root (a) Gametophyte (b) Antheridium
apex . (c) Archegonium (c) Embryo
(b) Cells of the protoderm (young epi- Ans B
dermis)
60. ,,,,,,,,, is a plant without vascular tissue
(c) Ground meristem and procambium
(a) Angiosperm (b) Gymnosperm
back of the root apex
(c) Bryophyte (d) Thallophyte
(d) All of the above
Ans D
Ans D

52. ....... is the group of plants with vascular


tissues and motile sperm . 0.4 BIOL 111 2014/2015
(a) fern (b) Antheridium
1. Plants are named according to rules con-
(c) sporophyte (d) Zygote
tain in
Ans A
a. International standard for botanical
53. ....... is the plant structure that has four nomenclature
whorls of modified leaves b. International circle for botanical nomen-
(a) Moss (b) Cycadophyta clature
(c) sepals (d) fruit c. International authority for botanical
Ans C nomenclature
d. International code for botanical nomen-
54. ...... is a group of gymnosperms that re- clature
semble palm trees Ans D”ICBN”
(a) Cycadophyta (b) Mycorrhizae
(c) sporophyte (d) pterophyte 2. Algae ans fungi original constitute the
Ans A a. Pteridophyte b. thallophyte
c. bryophyte d. spermatophyte
55. ....... is a plant that produces a pro- Ans B
tonema
(a) pterophyte (b) gametophyte 3. Cyanobacteria was formerly known as
(c) Angiosperms (d) sporophyte a. Blue algea b. red algea
Ans D c. blue-green algea d. green algea
Ans C
56. ...... is the stage of the life cycle where
all the cells are haploid 4. Sexual organs in algea, gametes are dif-
(a) Gametophyte (b) sporophyte ferentiated into
0.4. BIOL 111 2014/2015 15

a. Gametangia b. Heterogamy d. A complete internal transport system


c. Conjugation d. Scalariform Ans A
Ans A
11. The two principal types of organs respon-
5. In advance forms of algae, gametes are sible for tissue specialization in higher
differentiated into plants are
a. Microgamete and mega gamete a. Vegetatives and reproductive organs
b. Maga sporangia and microsporangia b. Reproduction and respiratory organs
c. Antheridia and archegonia c. Vegetative and excretory organs
d. Gametophyte and sporophyte d. vegetative and respiratory organs
Ans A Ans A

6. The forms of gamete fusion in algae are 12. Absorption of water and nutrients in lower
a. heterogemy, isogamy and anisogamy plants in through
b. Isogamy, anisogamy and microsporan- a. Rhizoids and Cells b. Roots and tis-
gia sues
c. Scalariform, heterogemy and monogamy c. Cells and roots d. Rhizoids and tis-
d. microgamy, megagamy and Isogamy sues
Ans A Ans A

7. The following are economic importance 13. Algae and fungi are
of fungi except a. Thallophytes b. Bryophytes
a. primodial agents b. Antibiotics c. Pteridophtes d. Anthocerophytes
c. bakers and brewers d. Cheese produc- Ans A
tions
14. Gametangia of bryophytes and pterodophytes
Ans A
always have a protective jacket of .....cells
8. Penicillin is form from a. Fertile b. sterile
a. Penicillium notatum b. Penicillium- c. mobile d. stable
chysopgenum Ans B
c. Penicilliumcillie d. penicilliumageri-
15. A narrow through canal through which
tum
the antheridium of bryophytes communi-
Ans A
cates with the exterior is known as ......
9. The types of vascular tissue arrangement (a) venter (b) neck
in pteridophytes are (c) strobilus (d) ostiole
a. protostele, dictyostele and siphonos- Ans D
tele 16. A sexual reproduction in pteridophytes
b. Actinostele, dictoyoatele and sophonos- is accomplished by ........
tele a. conidia b. gonidia
c. prostela, actostele and sophonostele c. spores d. basidia
c. Siphonostele, actinostele and prostostele Ans C
Ans A
17. The production of two kinds of spores is
10. The attributes of higher plants that most known as ...... a. dispory b. monospory
advanced and complex algae lack include c. heterospery d. trispory
the following Ans C
a. A protective layer ao a sterile jacket
of cells around the reproductive organs 18. A collection of small stalked sporangia
b. Multicellular embryos that are retained on the undersurface of a fern is known as
within the archegonia ......
c. The absence of cyticle on aerial parts a. Sori b. indusium
16

c. prothallus d. endosperm c. Adenosine triphosphate


Ans A d. Nucleotide tetraphosphate
Ans C
19. Pinus is a .......... Plant
a. Monoecious b. dioecious 27. ..... in the plant cell is sensitive to var-
c. trioecious d. tetraecious ious external stimulus such as light and
Ans A chemicals
a. protoplasm b. Nucleus
20. Pollination in pinus is by ......... c. Mitochondria d. Endoplasm
a. Animals b. Wind Ans A
c. Water d. man
Ans B 28. Which of the options below best describes
a non directional movement of part or en-
21. Double fertilization is a phenomenon that
tire organism induced by light, chemical
is found in .......
and heat
a. Angiosperm b. Gymnosperm
a. Phototropism, chemonasty and ther-
c. Bryophyte d. pteridophyte
motropism
Ans A
b. Photonasty, chemonasty and thigmotropism
22. In pinus, the ...... cell divides to form a c. Photonasty, Chemonasty and ther-
generative cell and a tube cell monasty
a. Pollen grain b. micropyle d. Photonasty, Chemonasty and ther-
c. archegonia d. antheridial motropism
Ans D Ans C

23. The seed of angiosperm consists of the 29. The major difference between nastic and
following except Tropic movement an plants is based on
a. Seed coat b. endosperm a. Direction (b) responding organ
c. embryo d. ventor (c) Curvature (d) Stimuli source
Ans D Ans A

24. Each molecule of pyruvic acid that enters 30. Which of the following is not true about
into the krebs cycle releases movement in plants
a. 3 NADH, 4 FADH2 and 1 ATP molecule (a) movement of entire organism is pos-
b. 4 NADH,1FADH3 and 1ATP molecule sible in lower limb
c. FADH molecules only (b) In higher plants movement of whole
d. Oxaloacetic acid is released organism is possible
Ans B (c) Higher plants move only by move-
ment of curvature
25. Which of the following is not an adapta- (d) All movement in plants are either spon-
tion by plants to reduce rate of transpi- taneous or induced
ration Ans B
a. Presence of thick cuticles
b. Reduction of leaves to spine 31. Ninety percent of water lost in plant re-
c. Presence of trichomes sult from
d. Absence of sunken stomata (A) lenticular transpiration
Ans D (b ) Cuticular transpiration
(c) Leaf surface transpiration
26. The energy produce by respiration is con- (d) stomatal transpiration
vert into molecules of a chemical nucleotide Ans D
known as
a. Nucleotide triphosthate 32. The chemical pathway for respiration in
b. Adenosine tetrephosphate plants include
0.4. BIOL 111 2014/2015 17

(a) Glycolysis and kreb cycle 39. Chlorophyll is a pigment with a core made
(b) Light reaction and calvin cycle up of .........
(c) Calvin cycle alone (a)Fe (b) Mg (c) N (d)O2
(d) Phloem pathway Ans B
Ans A
40. The first product of the dark phase is
33. ........ is responsible for the transporta- .......
tion of photosynthates from sight of pro- (a) phosphoglyceric acid
duction to other parts of the plant (b) triose phosphate
(a) Xylem (b) Vessels (c) glyceraldehyde phosphate
(c) phloem (d) Tracheids (d) phosphoglyceraldehyde
Ans C Ans A

34. Photosynthates are translocated from ...... 41. The light dependent phase of photosyn-
(a) Source to sink (b) sink to source thesis takes place in ......
(c) source to source (d) sink to sink (a) stroma (b) stoma
Ans A (c) thylakiod (d) thykaloid
Ans C
35. Opening of the stomata is occurs when
(a) the guard cells are flaccid 42. Purple sulphur bacteria contains .........
(b) The guard cells are turgid pigment
(c) The guard cells are recuperating (a) bacteriochlorophyll
(d) The epidermal cells are turgid (b) Bacterioviridin
Ans A (c) chloroviridin
(d) sulphur
36. Which of the following statement is in- Ans A
correct
43. One of the following converts nitrite to
(a) In the presence of sunlight the chloro-
nitrate
plast in the guard cells start to photosyn-
(a) Nitrosococcus (b) Nitrosolubus
thesize
(c) Nitrococcus (d) Ectothirhodospirac-
(b) Transpiration takes place more dur-
cae
ing the day time than at night
Ans C
(c) At higher temperature water evapo-
rates more freely than at lower tempera- 44. Iron bacteria are usually found in
ture (a) air (b) food
(d) Water moves into the root hair be- (c) lakes (d) sea
cause it contains a weaker solution than Ans C
the soil
Ans D 45. High light intensity damages the ........
(a) ATP (b) chlorophyll
37. Anaerobic respiration resulting in the pro- (c) leaves (d) chloroplast
duction of alcohol is otherwise known as Ans B
(a) glycolysis (b) Alcoholic fermentation
(c) industrial fermentation (d) Anaerobic 46. Potassium is important in photosynthe-
fermentation sis because it ......
Ans B (a) activates the enzymes
(b) regulate the opening and closing of
38. Electrons move from one carrier to an- stomata
other in a ...... reaction (c) it is pumped in and out of the guard
(a) redox (b) oxidation cells as the situation demends
(c) reduction (d) carboxylation (d) All of the above
Ans A Ans A
18

47. All the following are required for chemosyn- (a) Permanent cells (b) initial cells
thesis except (c) Temporarily cells (d) None of the above
(a) H2 S (b) F e2 O3 Ans B
(d) N H3 (d) light
Ans A 56. The outermost part of a shoot ape is called
(a) primordia (b) periblem
48. The optimum temperature for enzyme ac- (c) plerome (d) tunica
tivities during photosynthesis is Ans D
(a) 30 0 C (b) 250 C (c) 400 C (d) 230 C
Ans A 57. The meritem located at the base of nodes
and leaf blade is
49. Energy used during chemosynthesis is got- (a) determinate (b) intercalary
ten from (c) Apical (d) indeterminate
(a) sunlight (b) oxidation of inorganic Ans A
compounds
(c) reduction of inorganic compounds 58. An example of annual plant is
(d) splitting water (a) onion (b) Maize
Ans B (c) cassava (d) mango
Ans B
50. All of the following are required for pho-
tosynthesis except 59. Plant that grow for several years,flower
(a) chlorophyll (b) light and die is called
(c) water (d) carbon monoxide (a) Biannual (b) perennial
Ans D (c) perennial annual (d) Biannual annual
Ans C
51. The chloroplast is refer to as a free living
organism millions of years ago because of 60. An example of plant in question 15 above
the presence of is
(a) DNA and ribosome (b) DNA and stroma (a) Corn (b) Agave
(c) stroma and thylakoid (d) ribosome (c) Yam (d ) none of the above
and thylakoid Ans B
Ans A

52. Growth is initiated in certain tissues called


(a) collenchymas (b) parenchyma
(c) Meristems (d) sclerenchyma
Ans C

53. Beginning of shape is a literal meaning


for
(a) morphogenesis (b) Anthesis
(c) Pollination (d) germination
Ans A

54. Growth is classified into two


(a) vegetative and germination
(b) Reproductive and germination
(c) Vegetative and reproductive
(d) reproductive and seedling emergence
Ans C

55. Quiescent center is surrounding by group


of
0.5. CHEM 101 2010/2011 19

0.5 CHEM 101 2010/2011 (c) Atoms remind unchange at the end
of a chemical reaction
1. Bohr’s Atomic model cannot explain the (d) Atoms of different elements are like
line spectra of , in every aspect
(a) He+ (b) Li2+ (c) Be3+ (d) Ca2+ Solution
Solution Dalton’s first atomic theory sates ;
Bohr’s atomic model explained the spec- - All element are made up of tiny indi-
tra of very light element (Hydrogen like vidual particles called atom .
element) - Atomic are neither created nor destroyed
Ans D .
- Atom of the same element are exactly
2. The basis for the relative Atomic Mass is alike in every aspect and differs from atoms
an atom of carbon 12 which was assigned of other element .
a mass of - During chemical reaction,atoms com-
(a) 1 amu (b) 6 amu bine in simple whole number ratio .
(c) 12 amu (d) 24 amu Ans D
Solution
The basis for relative atomic mass is an 7. What is the mass of 0.25 mole of K2 CH2 (SO4 )2 ?,
atom of carbon-12 having a mass of 12 K = 39, C = 12, S = 32, O = 16, H = 1
amu (a) 126g (b) 63g (c)106.5g (d)252g
Ans C Solution
Molar mass of K2 CH2 (SO3 )2 = (39 ×
3. How many Na atoms are in 22.9g of Na 2) + 12 + (1 × 2) + 2[32 + 16(3)]
(a) 6.0223 × 1023 (b) 6.0223 × 1024 = 252gmol
(c) 1.12 × 1023 (d) 1.12 × 1024 1 mol of K2 CH2 (SO3 )2 contains 252g
Solution 0.25 mol of K2 CH2 (SO3 )2 will contain x
1 mol of Na contain 23g 252 × 0.25
x= = 63g(B)
23g of Na contain 6.02 × 1023 1
22.9g of Na will contain x
6.02 × 1023 × 22.9 8. How many moles are in 6.8g of N H3 (N=14,H=1)
x= (a)0.1 moles (b) 0.2moles
23
= 5.99 × 1023 (b) 0.4 moles (d) 0.5moles
Solution
4. Plum pudding model of atom was disap- Molar mass of N H3 = 14 + (1 × 3) =
proved by 17gmol−1
(a) Thomoson j.j (b) Rutherford 1 mol of N H3 contains 17g
(c) Dalton j (d) Boyle .R x mol of N H3 will contain 6.8 g
Solution 6.8 × 1
x= = 0.4mol(C)
Ans A 17
9. An element Z contain two isotopes 16 Z and 17 Z
5. Oil drop experiment was carried out by
in the ratio 9:1.The relative atomic mass
(a) Milikan.R.A (b) Geiger and Marsden
of Z is
(c) Rutherford E (d) Thomos j.j
(a)6.0 (b) 8.0 (c) 16.1 (d) 12.1
Solution
Solution
Ans A
Given that,element Z has two isotopes
16
6. Which of the following was not part of Z and 17 Z with ratio 9 : 1 it means,
16
Dalton’s Atomic Theory Z and 17 Z = 1
(a) All element are made up of tiny indi- Sum of ratio = 9+1=10
(16 × 9) + (17 × 1)
visible particles called atom R.A.M= = 16.1gmol−1 (C)
(b) Atoms can neither be created nor de- 10
stroyed 10. In the reaction CaC2 (s) + 2H2 O(l) →
20

Ca(OH)2 (l)+C2 H2 (g),What mass of solid 27.0 g of aluminium contain 6.02 × 1023
calcium carbide in needed to produced 1.0g of Aluminium will contain x
23 ×1
3
1000cm of acetylene at S.T.P ? x = 6.02×1027
= 2.23 × 1022 (B)
(a) 2.9g (b) 3.9g (c)2900g (d)3900g
Solution 15. James Chadwick discovered neutron atom
CaC2 + 2H2 O → Ca(OH)2 + C2 H2 by bombarding beryllium with
BUT C2 H2 is a gas,and 1mol of every gas (a) Alpha particles (b) Beta particle
occupies a space of 22.4dm3 = 22400cm3 .Also (c) Gamma rays (d) All of the above
1 mol of CaC2 = 64g Solution
3 Ans A
1 mol of CaC2 Produce 1 mol of C2 H2 (22400cm )
64g of CaC2 produce 1000 cm3
3 16. Which of the following are Radioisotopes
x = 64g×1000cm
2400cm3
= 2.9(A)
?
11. A metal Y form two oxides A and B,6g (a) 37 Cl and 38 Cl (b) 2 H and 3 H
of each A and B contain 1.5g and 1.0g of (c) 16 O and 17 O (d) None of the above
oxygen respectively .Calculate the masses Solution
of Y which combine with 1g of oxygen in
each case
17. Analysis shows that 84.2g of pitchblende
(a) 3g,5g (b) 5g,3g
contains 71.4g of Uranuim with oxygen
(c)4g,5g (d) 5g,4g
as the other element.How many grams of
Solution
Uranium can be obtained from 102g of
pitchblende ?
12. Which law of chemical combination was (a) 8.65 × 101 g (b) 8.65 × 102 g
illustrated by the masses in 11 above ? (c) 8.65 × 103 g (d) 8.65 × 104 g
(a) Law of conservation of mass Solution
(b) law of multiple proportion if 84.2g of pitch blende produce 71.4g of
(c) law of definite proportion uranium
(d) All of the above then 102g of pitch blende will produce x
Solution of uranium
Ans B x = 102×71.4
84.2
= 86.5 = 8.65 × 101 g(A)
State the mass law(s) demonstrated
13. The nucleus of Uranuim 238 contains 92 by the following observation 18-20
positive particles.How many neutral par-
ticles are there ? 18. A sample of potassium chloride from Zaria
(a) 144 (b) 92 (c)146 (d)94 contain the same percent by mas of potas-
Solution sium as one from yenagoa
Mass of nucleus = mass of proton + mass (a) Law of multiple proportion
of neutron (b) Law of conservation of mass
289 = 92 (proton is positive )+ mass of (c) Law of definite proportion
neutron (d) Daltons atomic theory
mass of neutron = 283 − 29 = 146(C) Solution
Ans C
14. How many atoms are in a small piece of
Aluminium foil with a mass of 1.0g?The 19. Arsenic and oxygen form one compound
mass of an atom of Aluminium is 27.0 that is 65.2 mass % arsenic and another
amu that is 75.8 mass % arsenic
(a)2.23 ×1021 (b) 2.23 ×1022 (a) Law of definite proportion
22 23
(c) 2.23 ×10 (d) 2.23 ×10 (b) Law of conservation of mass
Solution (c) Avogadro’s hypothesis
1 mole of aluminium contains 27.0g and (d) Law of multiple proportion
23
also contains 6.02 × 10 Solution
0.5. CHEM 101 2010/2011 21

since Arsenic and oxygen are two differ- (c) Bohr (d) Planck
ent elements that react to form two dif- Solution
ferent product containing different per- Ans. B
centage by mass of Arsenic ,by the defi-
nition of law of multiple proportion 24. Which scientist proposed a quantized model
Ans D for the atom
(a) Thomson (b) Rutherford
20. A flash bulb contain magnesium and oxy- (c) Bohr (d) Planck
gen before being used and magneseium Solution
oxide afterwards but its mass does not Ans (C)
change
(a) Gay lussacs’s law 25. An electronic transition occurred which
(b) Law of multiple proportion gave frequencies that fell in the visible
(c) Avogadro’s hypothesis region of the EM spectrum.The transi-
(d) Law of conservation of mass tion is most likely to be in which series
Solution (a) Lyman (b) Balmer
Ans D (c) Paschen (d) Pfund
Solution
21. Calculate the de Broglie’s wavelength for Ans: B
a 50mg weight moving with a velocity
1ms−1 26. The angular momentum of an electron
(a) 1.33 × 10−24 m (b) 1.33 × 10−34 m can be represented by the formula
(c) 1.33 × 10−29 m (d) 1.33 × 10−30 m (a) nh/2π (b) n2 h/2π
Solution (c) n2 h2 /2π 2 (d) n2 h2 /4π 2
De blogie wa velenght (λ) = mv h Solution
Where h = Plan constant =6.63 × 10−34 The angular Momentum of an electron is
nh
M= mass of specie= 50mg=50 × 10−6 kg represented by

V=velocity of specie= 1ms−1 Ans: A
6.63 × 10−34
= = 1.326 × 10−29 m
50 × 10−6 kg × 1 27. A photon has a wavelength of 400 Arm-
Ans: C strong.Calculate the energy of the pho-
ton in the electron volts.
22. Which of the following species will not (a) 3.10eV (b) 0.310eV
give spectra that will fit into the equa- (c) 31.0eV (d) 310eV
tion 1/λ = RH [1/n21 − 1/n22 ] Solution
(a) He+ (b) Li+ (c) Li2+ (d) Be3+ Energy of photon E= hv but v= frequency
Solution =c/λ
The equation 1/λ = RH(1/n21 −1/n22 Ap- Where c= speed of light =3 × 108
plies to elements with only one electron ∴ E = hc/λ = 6.63×10
−34 ×3×108
= 4.973×
400×10−10
like Hydrogen. 10 −18
He has two electron but H+ has only one. But lev = 1.6 × 10−19
Li has three electrons but Li+ has only X= 4.973×10−18
two. 1ev × 4.973 × 10−18
Li has three electrons but Li2+ has only X=
1.6 × 1019
one. = 31.0lev Ans: C
Be has Four electrons but Be3+ has only
one. 28. The energy of a photo-electron was 3.10eV.What
Ans. B is the energy in joules ?
(a) 4.97 × 10−19 j (b) 4.97 × 10−18 j
23. Which scientist proposed an atomic model (c) 4.97 × 10−17 j (d) 4.97 × 10−16 j
that was eventually meant to collapse Solution
(a) Thomson (b) Rutherford Since energy=3.10ev But lev = 1.6×1019
22

Then 3.1ev = x Paul’s exclusion principle state that ; no


3.lev × 1.6 × 10−19 two electrons in an action can have the
x= = 4.96 × 10−19 j
1ev same set of values for all the four quan-
Ans: A
tum number. Ans: B
29. When an electron descends from a higher 34. According to the definition of chemical
to a lower energy level,the spectral ob- bound,the following agree with the defi-
serverd is called nition except
(a) Hydrogen spectral (a)ionic bound (b) covalent bound
(b) Absorbtion spectral (c) dative bound (d) hydrogen bound
(c) Emission spectral Solution
(d) Transportation spectral Hydrogen bond is only between hydrogen
Solution and a strongly electronegative elements.
When an electron descend from a higher It does not occur for all elements. Ans:
to a lower energy level, it given out en- D
ergy in form of emission spectra. Ans:
C 35. The number of electron transferred from
one atom to the another to achieve a no-
30. Dimerisation occurs in some compounds ble gas configuration is called
due to the presence of (a) electro valency (b) electrocovalency
(a) Ionic bounds (b) Covalent bonds (c) electronegativity (d) electron affinity
(c) Hydrogen bonds (d) Dative bonds Solution
Solution Eloctrovalency is the number of electrons
Molecular dimes often occurs by the reac- loss or gained by an atom of an element
tion of two identical molecules e.g. 2A A- during the farmetion of an electrovalent
A. The element A is said to dimenise to bond. Ans: A
give dimer A-A since only covalent bond
exist between identical molecules; Ans: 36. Which quantum number is responsible
B for the geometric orientation of orbitals
(a) Principal (b) Azimuthal
31. Which quantum number determines the (c) Magnetic (d) Spin
angular momentum of an electron Solution
(a) Principal (b) Azimuthal Magnetic quantum number describes the
(c) Magnetic (d) Spin orientation of orbitals in space. Ans: C
Solution
The angular momentum of an electron is 37. Which of the following theory accurately
determined by the Azimutual Quantum account for the geometric orientation of
member. Ans: B molecules
(a) valence electron pair repulsion the-
32. Which of the following orbital is spheri- ory
cally symmetrical (b) Valence pair electron repulsion the-
(a) s-orbital (b) p-orbital ory
(c) d-orbital (d) f-orbital (c) Valence shell electron pair repulsion
Solution theory
S- orbital is spherical in shape (d) valence electron pair theory
Solution
33. In the filling of electron into the available Valence shell electron pair repulsion the-
orbitals,which principle limits the num- ory accounts for the geometric orienta-
ber of electron to two per orbital tion of molecules. Ans: C
(a) Aufbua principle (b) Pauli’s principle
(c) Hunds rule (d) Screen rule 38. In a vessel containing chlorine molecule,which
Solution intermolecular interaction is most likely
0.5. CHEM 101 2010/2011 23

to occur the two half cell


(a) dipole-dipole (b) dipole-induced dipole Solution
(c) induced dipole-induced dipole A salt bridge performs the following func-
(d) valence electron theory tions; - It prevent direct mixing of the
Solution two solutions - As a semi-permeable mem-
Since the chlorine molecules have electro brane , it allows the movement of charges
negativity, one chlorine molecule has to between the solutions thereby preventing
be induce before it can induce dipole to charge imbalance in each half cell.
a neighboring molecule. Ans: C
44. Which of the following compound is the
39. The expected shape for methane is strongest acid
(a) linear (b) trigonal planar (a) HClO (b) CH3 COOH
(c) regular tetrahedron (d) octahedron (c) (CH2 )2 CHCOOH (d) H2 SeO4
Solution Solution
The shape of methane is a regular tetra- Oxyacids are acids containing one or more
hedron. Ans: C oxygen atoms in their molecules. For
such acids , the strengths of the acid in-
40. The scientist that postulate that it is im-
creases since cl is more eletronnegative
possible to determine simultaneously the
than c and se.
position and velocity of the electron is
Ans: A
(a) de Brogile (b) Planck
(c) Heisenberg (d) Einstein 45. In the reaction H2 S + N H3 HS − +
Solution N H4+ a Bronsted-lowry conjugate acid-
Heisenberg postulated that; it is impos- base pair can be represented as
sible to determine both momentum and (a) H2 S and N H3
position of an electon simultaneously with (b) H2 S and HS −
great accuracy. Ans: C (c) N H3 and HS −
(d) HS − and N H4−
41. What is the oxidation number of the cen-
Solution
tral atom in Si2 O7 molecule
Bronsted lowry defined acid as a proton
(a) +14 (b) −14 (c) +7 (d) −7
(Hydrogen ion) donor while a base as a
Solution
proton acceptor. H2 S + N H3 HS − +
The central atom in Si2 O7 molecule is Si
N H +4
2Si + 7(−2) = 0 ∴ 2si = 14
H2 S donated one Hydrogen ion and be-
and Si =+ 7 (C)
come Hs−
42. In a galvanic cel,oxidation is said to occur N H3 accept one hydrogen ion and be-
in comes N H +4
(a) the anode (b) the cathode ∴ Has is the acid and N H3 the base.
(c) the electrode (d) the cell Ans: A
Solution
In a galvanic cell, oxidation occurs at the 46. According to lewis concept a reaction be-
anode. tween an acid and a base is the transfer
of
43. A salt bridge in an electrochemical cell (a) a proton from acid to base
does not prevent (b) OH − ion from base to acid
(a) The direct mixing of the two elec- (c) electron pair from acid to base
trolyte (d) electron pair from base to acid
(b) the building up of charges imbalance Solution
in each half cell According lewis , an acid is a specie that
(c) the conduction of electrical energy accept electron pair while a base is the
(d) A spontaneous redox reaction between specie that transfer electron pair.
24

Ans: D Solution

47. The strength of an acid depend on


(a) the concentration of H + ion in aque- 52. Hydrogen ion associates with polar water
ous solution molecules form the
(b) the concentration of electron pairs in (a) hydroxyl ion (b) hydronuim ion
aqueous solution (c) hydrated ion (d) All of the above
(c) The tendency to release the electron Solution
pair HA + H2 O H3 O + A−
(d) the tendency to gain the electron pair Where A could be any Halogen. Ans: D
Solution
The strength of an acid depends upon 53. Ethanoic acid in a solution of toluene
the concentration of H+ in aqeous (a) A strong electrolyte
Ans A (b) A weak electrolyte
(c) A non-electrolyte
48. All nucleophiles are (d) None of the above
(a) Arrhenius acid (b) Bronsted bases Solution
(c) lewis acids (d) lewis bases Ethanoic acid is a polar molecule , it can
Solution only dissolved in water. Toluene is a non
A nucleophile is an electron rich specie. polar solvent, it can only dissolve non po-
By our definition of No 40, it is a lewis lar compounds. Ans: D
base.
Ans: D 54. Bronsted-lowry referred to an acid as
(a) A proton acceptor
49. In anionic hydrolysis,the pH of the re- (b) a proton donor
sulting solution is (c) A conjugate pair
(a) greater then 7 (b) equal to 7 (d) A reactant in water
(c) less than 7 (d) less than 0 Solution
Solution Reference to quention No 39 Ans: B
In an ionic hydrolysis, the PH is greater
than 7. Because less than 7 is acidic 55. An acid-base reaction is a competition
(H+) between bases for
Ans: A (a) Hydroxyl ions (b) Hydronium ions
(c) Acid components (d) Hydrogen ions
50. A salt of weak base and strong acid on Solution
hydrolysis gives a solution which has when acid react with a base, the OH-
(a) more H + ion then OH − ions from the base complete for a Hydrogen
(b) more OH − ions than H + ions (H+) ion from the acid Ans: D
(c) equal H + ion and OH − ions
(d) no H + ions 56. Who defined an acid as electron pair donor
Solution (a) Lewis (b) Bronsted-lowry
Since a weak base will only dissociate (c) Arrheneius (d) None of the above
partially releasing small amount of OH− Solution
and strong acid will dissociate completely Lewis define acid as an electron pair ac-
given large amount of H+ ceptor NOT donor. Ans: D
Ans: A

51. The weaker the acid,the greater is the 0.6 CHEM 101 2011/2012
(a) ionic product
(b) dissociation constant 1. 16g and 32g of oxygen reacts with 12g
(c) hydrolysis constant of carbon each to form carbone dioxide
(d) degree of ionization and carbon monoxide with oxygen in the
0.6. CHEM 101 2011/2012 25

ratio 2:1 respectively.This statement jus- Given that 69 Ga(68.957) = 60.4% and
71
tifies the Ga(70.9248) = 39.6%
(a) Law of definite proportion (68.9257 × 60.4) + (70.9248 × 39.6)
A.A.W=
(b) Law of multiple proportion 100
(c) Law of conservation of mass = 69.7173(A)
(d) Law of constant mass
Solution 6. A certain atom has an atomic weight of
The law of multiple proportion states that; 52g/mol.If it contains 28 neutrons,how
when two elements say A and B react many protons does it have
with the fix MASS of B are in a simple (a) 24 (b) 25 (c) 26 (d) 27
multiple ratio . Solution
Ans B Since atomic weight=mass of proton +
mass of neutron
2. Which of the following accounts for the 52= mass of proton + 28
fractional atomic masses of some element mass of proton = 52 − 28 = 24(A)
(a) Mole concept (b) Isotopy
(c) Hybridization (d) Nuclear particles 7. Which of the following laws will enable us
Solution to make use of mass fraction and find the
Ans B actual mass of an element in any given
compound ?
3. How many molecules are in 25.5gof am- (a) Law of constant composition
monia ?(N H3 = 17,Advogadro’s constant (b) Law of multiple proportion
= 6.02 × 1023 ) (c) Law of conservation of mass
(a) 9.03 × 1023 (b) 9.03 × 1024 (d) Law of constant mass
((c) 9.03 × 1021 (d) 9.03 × 1022 Solution
Solution
Molar Mass of N H3 = 14+3(1) = 17gmol=
mass in 1 mole of N H3 8. If an element in Group IV losses an alpha
1 mole of N H3 contain 17g particle,to which group will the product
17g of NH3 is contains in 6.02 × 1023 belong
25.5g of N H3 Will contain x (a) Group I (b) Group II
6.02 × 1023 × 2.5 (c) Group III (d) Group IV
x= = 9.03 × 1023
17 Solution
Ans A
When an element in group foue losses
4. How many particle are in 12.0g of carbon an alpha particle (42 He),since the valence
atom ? electrons of group IV element is four,its
(a) 6.023 × 1023 (b) 12.0 (c) 1 (d) 24 valence electron becomes II
Solution Ans B
Molar mass of carbon = 12gmol= mass
of carbon in 1mol 9. Which of the following statement is /are
12g of carbon contain 6.02 × 1023 true of radioactivity
12g of carbon will contain x i. it involves the transformation of a nu-
23 ×12g
x = 6.02×10 = 6.02 × 1023 cleus into another nucleus
12g
Ans A ii. it gives out radiation
iii. Energy is given out in the process
5. Gallium has two natual occurring isotopes iv.The rate of radioactive decay in not
60.4% is 69 Ga(mass = 68.9257 amu) and affected by temperature and pressure .
71
39.6% is Ga (mass = 70.9248 amu).Calculate (a) i and ii only (b) i and iii only
the average weight of Gallium (c) i,ii and iii only (d) i,ii,iii and iv
(a) 69.8 (b) 59.5 (c) 69.5 (d) 55.6 Solution
Solution Ans D
26

10. How many grams of nitrogen are present (a) 241 (b) 251 (c) 261 (d) 259
in a 0.1g sample of caffeine (C8 H10 N4 O2 ) Solution
11 251 x 1
a stimulant in coffee and tea (C=12,H=1,O=16 5 B +78 Cf →103 Cr + 30 n
and N=14) Going by the equation
(a) 0.01g (b) 0.02g (c) 0.03g (d) 0.04g 11 + 251 = x + 3(1)
Solution x = 262 − 3 = 259(D)
Molar mass of caffeine (C8 H10 N4 O2 ) =
12(8) + 1(10) + 14(4) + 16(2)
= 194gmol−1 15. Two radioactive elements X and Y have
since 1 mole of caffiene contains 4 nitro- half-lives of 20 and 10 hours respectively.Therefore
gen = 14 × 4 = 56g (a) X decays faster than Y
∴ 194g of caffeine contains 56g of N (b) Y is twice as stable as X
0.1g of caffeine will contain xg of N (c) X is more stable than Y
56 × 0.1 (d) Y emits fewer particle than X
x= = 0.02887g(C)
194 Solution
X is more stable than Y
11. Spontaneous emission of radiation by an
Y decays faster than X
unstable atomic nucleus is known as
Ans C
(a) Radioactivity (b) Beta emission
(c) Gamma emission (d) Nuclear fission 16. Which of the following compounds con-
Solution tain hydrogen in a negative oxidation state
Ans A ?
(a) H2 S (b) CH4 (c) CaH2 (d) N H3
12. Which of the following can be used to
Solution
detect radiation
When H is bounded to a metal ,it carries
(a) Geiger muller counter
a negative oxidation state
(b) Scintillation counter
Ans C
(c) Photographic plate
(d) All of the above 17. The arrangement of the oxidation state
Solution of carbon atom for the listed compounds
Ans D /atoms in increasing order is ?
(a) CH4 < CH3 CH < C < CO2
13. When a sheet of paper is placed in the (b) C < CH8 OH < CH4 < CO2
path of a radioactive source,the radiation (c) CO2 < C < CH3 OH < CH4
emitted which passes through the paper (d) CO2 < CH4 < CH3 OH < C
consist Solution
(a) Alpha and beta particles The oxidation state of carbon in CH4 =
(b) Alpha and gamma rays C + 4(1) = 0 ∴ C = −4
(c) Beta and gamma rays The oxidation state of carbon in CH3 OH =
(d) Alpha,beta and gamma rays C + 3(1) + 1(−2) + 1(1) = 0
Solution C = −4 + 2 = −2
Alpha particles are stopped by a sheet of The oxidation state of carbon in C is 0
paper beta particle can only be stopped The oxidation state of carbon in CO2 =
by a metallic sheet ,gamma particles are C + 2(−2) = 0 C = +4
stopped by a lead block It increases from −4 to −2 through 0 and
so the rays which passes through the pa- to +4
per are beta and gamma Ans A
Ans C
USE THE REACTION BELOW TO
14. what is the value of x in the following ANSWER QUESTION 18 AND 19
nuclear reaction 3Sn2+ (aq)+Cr2 O72− +XH + (aq) → 3Sn4+ (aq)+
11 251 x 1
5 B +98 Cf →103 Lr + 30 n 2Cr3+ (aq) + Y H2 O(l)
0.6. CHEM 101 2011/2012 27

18. The letters X and Y respectively repre- oxidation number


sent (b) A loss of electron and an increase in
(a) 10 and 5 (b) 5 and 10 oxidation number
(c) 7 and 14 (c) 14 and 7 (c) A gain in electron and increase in ox-
Solution idation number
From the above equation it can be seen (d) A gain in electron and a decrease in
that, there are 7 oxygen molecules at the oxidation number
left hand side of the equation.This im- Solution
plies ,Y is 7.With this,we can see that When an atom is reduced in a chemical
the number of Hydrogen atoms have be- reaction
come 14,thus implies X is 14 - It gains electrons and its oxidation num-
The letter X and Y are 14 and 7. ber increases .
Ans D - It accepts Hydrogen
Ans D
19. For the reaction above ,which of these
statements is correct ? 23. A space that is simultaneously oxidized
(a) Both Sn2+ and H + ions are oxidizing and reduced in a reaction is said to be
agents (a) double displace (b)disproportionated
(b) Cr2 O72− is the reductant (c) redoxed (d) neutralized
(c) Sn+ is oxidized from +4 to + 2 oxi- Solution
dation state
(d) Cr2 O72− is reduced to Cr3+
Solution 24. The redox reaction below is balanced in
From the reaction,the following can be basic medium
seen ; 8Al+3N O3 +QOH − +18H2 O → 8Al(OH)4 +
2−
- Cr changes from +6 in Cr2 O7 to +3 in 3N H3
Cr3+ ,This is gain of electron(Reduction).Therefore
The letter Q represents
,it is the oxiding agent . (a) 1 (b) 3 (c) 5 (d) 9
- Sn Changes from +2 to +4.This is loss Solution
of electron (oxidation). Balancing the number of either oxygen
Therefore it is the reductant.Ans D or hydrogen from the equation
Balancing for oxygen
20. The oxidation number of Cl in N aClO3
9 + Q + 18 = 8 × 4
is
Q + 27 = 32
(a) −5 (b) −3 (c) −2 (d) −1
Q = 5(C)
Solution
N aClO3 = 1(1) + Cl + 3(−2) = 0 25. In the redox reaction: 3Ag + +Al → 3Ag+
Cl = +5 Al3+
21. which of the following compounds would (a) Ag+ is the reductant,Al is the oxidant
have oxygen in +1 oxidation state (b) Ag+ in the oxidant,Al is the reduc-
(a) H2 O2 (b) KO2 (c) O2 F2 (d) OF2 tant
Solution (c) Ag+ is the reductant,Al3+ is the oxi-
H2 O2 ⇒ 2x + 2(1) = 0 ∴ x = −1 dant
KO2 ⇒ 2x + 1(1) = 0 ∴ x = − 21 (d) Ag is the oxidant,Al3+ is the reduc-
O2 F2 ⇒ 2x + 2(−1) = 0 ∴ x = +1 tant
OF2 ⇒ x + 2(−1) = 0 ∴ x = +2 Solution
Ans C 3Ag + + Al → 3Ag + Al3+
Ag gains electrons and change +1 and
22. What occur when an atom is reduced in goes to 0 (Reduction) .This is an oxidiz-
a chemical reaction ing agent.
(a) A loss of electron and a decrease in Al losses electrons and change from 0 to
28

+3 (oxidation).This is reduction 29. The oxidation numbers of N atom in NH3


Ans B and N2 H4 are..... and ..... respectively
(a) −3 and −2 (b) +3 and +2
26. The oxidation numbers of carbon atoms (c) −5 and −2 (d) −2 and −3
in CH3 CH2 OH are ....... and .... respec- Solution
tively . Oxidation number of N in N H3 N +
(a) +2 and +2 (b) −3 and −1 3(1) = 0 N = −3
(c) −1 and −3 (d) −2 and −2 Oxidation number of N in N2 H4 ; 2N +
Solution 4(1) = 0 ; N = −2
The oxidation number of carbon in CH3 CH2 OH Ans A
is ;
for the first carbon ,CH3 = 0 C+3(1) = 30. For the reaction below the spectator ion
0 is
C = −3 K2 M nO4 + H2 O → M nO2 + KM nO4 +
For the second carbon CH2 OH = 0 ⇒ KOH
C + 2(1) + 1(−2) + 1(1) = 0 (a) K + (b) M n+4 (c) OH − (d) H +
C +2−2+1 Solution
C = −1 .Ans B

27. The oxidation numbers of Cr and Cl atoms 31. An electron can be said to be
in Cr2 Cl93− are ...... and .... respectively (a) a particle
(a) +3 and −1 (b) +6 and −1 (b) a substomic particle
(c) +6 and −3 (d) +6 and −1 (c) a negatively charged subatomic par-
Solution ticle
The oxidation number of Cr and Cl atoms (d) An atom
in CrCl3− for Cl, Solution
9Cl + 2(6) = −3 Ans C
Cl = −1
32. An atom can be appropriately identified
For Cr
as
2Cr + 9(−1) = −3
(a) a particle
Cr = +3
(b) an indivisible particle of an element
Ans A
(c) A spherical electrically neutral cen-
tral core with negative electron(s) revolv-
28. The balanced ionic equation for CrO42− →
ing around the orbitals
Cr3+ in acidic medium is
(d) A subatomic particle
(a) CrO42− + 8H + + 3e− → Cr3+ + 4H2 O
Solution
(b) CrO42− + 8H + + 6e− → Cr3+ + 4H2 O
An atom can be defined as a particle
(c) CrO42− + 6H + + 3e− → Cr3+ + 6H2 O
(d) CrO42− + 8H + + 3e− → Cr3+ + 8H2 O 33. Which scientist gave an equation that re-
Solution lates the wavelength of a body to its mo-
CrO42− → Cr3+ mentum
Cr + 4(−2) = −2 (a) planck (b) De Brogile
Cr = +6 (c) Heisenberg (d) Schrodinger
CrO4−2 + 3e− → Cr3+ Solution
In acidic medium,for every excess oxy- De broglie’s equation relates the wave-
gen.balance with 2 moles of H + on the length of a body and its momentum .
same side and balanced with 1 moles of x = mvh
= hp
H2 O on the opposite side .
CrO42− + 3e− + 8H + → Cr3+ + 4H2 O 34. Which scientist gave an equation that states
Solution that it is imposible to determine the po-
Ans A sition and momentum of an electron si-
0.6. CHEM 101 2011/2012 29

multaneously tween their molecule


(a) planck (b) De Broglie (a) Electrovalent (b) Co valent
(c) Heisenberg (d) Schrondinger (c) Dative (d) Hydrogen
Solution Solution
Heisenberg states that; it is impossible Ans B
to determine simultaneously the position
and momentum of an electron at the same 40. The type of intermolecular force of at-
time with great accuracy . traction that may exist when a symmet-
rical molecule comes close to a polarized
35. The wave function (Ψ) in the schrodinger molecule
wave equation is a mathematical expres- (a) dipole-dipole (b) dipole-induced dipole
sion that seeks to give (c) induced dipole-induced dipole
(a) the momentum of an electron (d) Hydrogen bounding
(b) the coordinates of an electron charge Solution
(c) the density of an electron space since v = f λ
(d) the shape of an electron 3 × 108 = 100000 × λ
3×108
Solution λ = 100000 = 3000m (C)
Ans D
41. An FM station broadcast rock music at
36. The Aufbau’s principle states that elec- 100 kilohertz (Khz).find the wavelength
tron will fill into orbitals starting from of the radio waves
the orbital with the lowest available en- (a) 3 × 106 m (b) 3 × 104 m
ergy before moving to the next;however (c) 3 × 103 m (d) 3 × 102 m
this rule is violated at some point .The Solution
rule that explain this discrepancy is h
De broglie equation = λ = mv
(a) Pauli’s rule (b) Hund’s rule h=plank constant ,m= 50mg = 50×10−3 g =
(c) Screen rule (d) none of the above 50 × 10−6 kg
Solution 6.63 × 10−34
v = 1ms−1 ∴=
Hund’s rule states that; when filling elec- 50 × 10−6 × 1
−29
tron into the degenerative orbital,the elec- = 1.33 × 10 (C)
tron goes in singly before pairing take
place 42. Calculate the de Brogile’s wavelength for
Ans C a 50mg weigth moving with a velocity of
1ms−1
37. In the filling of orbitals by electrons,which (a) 1.33 × 10−24 m (b) 1.33 × 10−34 m
rule advocates the single filling of degen- (c) 1.33 × 10−29 m (d) 1.33 × 10−30 m
erate orbitals Solution
(a) Aufbau’s principle (b) Pauli’s rule Ans D
(c) Hund’s rule (d) Screen rule
Solution 43. Which of the following species will not
Ans D give spectra that will fit into the equation
1/λ = RH [1/n21 − 1/n2 ]
38. Which electronic quantum number gives (a) He+ (b) Li+ (c) Li2+ (d) N a10+
an idea of the direction of rotation of an Solution
electron Since 1 ev = 1.6 × 10−19 j
(a) Principal (b) Azimuthal then 3.310eV = x
−19
(c) Magnetic (d) Spin x = 3.10eV ×1.6×10
1ev
= 4.96 × 10−19 j
Solution Ans A

44. The energy of a photo-electron was 3.10eV.What


39. Water and Alcohol are miscible because is the energy in joules
of their ability to form ..... bonds be- (a) 4.97 × 10−19 j (b) 4.97 × 10−18 j
30

(c) 4.97 × 10−17 j (d) 4.97 × 10−16 j (c) Is directly proportional to the square
of its concentration
Solution (d) Does not depend on temperature
Solution
The degree of hydrolysis of a salt depends
45. An electronic transition occurred which on its concentration
gave frequencies that fell in the visible
region of the EM spectrum.The transi- 50. When a weak base undergoes hydrolysis
tion is most likely to be in which series in water its dissociation constant Kb is
(a) Lyman (b) Balmer represented as :
(c) Paschen (d) pfund (a) [BH + ][OH − ]/[B]
Solution (b) [BH + ][OH − ]/[B][H2 O]
Ans B (c) [BH + ][H2 O]/[B]
(d) [BH + ][OH − ]/[H2 O]
46. In the reaction H2 S + N H3 HS − +
Solution When a weak base e.g BOH
N H 4+ a Bronsted-Lowry conjugate acid-
hydrolysis in water
base pair can be represented as
B + H2 O BH + + OH −
(a) H2 S − N H3 (b) N H3 − HS − [BH + ][OH − ]
(c)H2 S − HS − (d) HS − − N H 4+ Kb =
[B][H2 O]
Solution [BH + ][OH − ]
H2 S + N H3 HS − + N H 4+ = (since the concentration
[B]
In Bronsted-lowry ,an acid is a proton of water is constant)
donor while a base is a proton acceptor.so Ans A
from the equation H2 S donated proton
to N H3 .
An acid-base conjugate = H2 S − N H3 0.7 CHEM 101 2012/2013
Ans A
1. 16 and 32g of oxygen reacts with 12g
47. Which of the following is the limitation
of carbon each to form carbon dioxide
of Arrhenius concept of acid and bases
and carbon monoxide with oxygen in the
(a) This concept is limited to water only
ratio 2:1 respectively.This statement jus-
(b) Free H + and OH − ions do not exist
tify
in water
(a) Law of definite proportion
(c) Some bases do not contain OH − ions
(b) Law of multiple proportion
.
(c) Law of conservation of mass
(d) All of the following
(d) Law of constant mass
Solution
Solution
Ans D
The law of multiple proportion state that
48. A salt of a weak acid and a strong base ” when two elements (A & B) react to
on hydrolysis givs a solution which has form more than one product ,the MASS
(a) More H + ions than OH − ions of A which separately come to join with
(b) More OH − ions than H + ions the fix MASS of B are in a simple multi-
(c) Equal number of H + and oh− ple ratio
(d) No H + ions Ans B
Solution
Ans B 2. Which of the following account for the
fractional atomic masses of some element
49. The Degree of hydrolysis of ammonium (a) mole concept (b) Isotopy
acetate (c) Hybridization (d) Nuclear Particles
(a) Depends on its concentration Solution
(b) Does not depend on its concentration Ans B
0.7. CHEM 101 2012/2013 31

3. Atoms of the same element with same 8. A certain atom has an atomic weight of
number of protons but different number 52g/mol .if it contains 28 neutrons,how
of neutrons are termed as many protons does it have
(a) Isotopes (b) Nuclear particles (a) 24 (b) 25 (c) 26 (d) 27
(c) Hybrids (d) None of the above Solution
Solution Atomic weight= proton + neutron
Ans A ∴ 52=proton + 28
proton =52-28=24 (A)
4. How many molecules are in 25.5g of am-
monia (N H3 = 17,Avogadros constant 9. Which of the following laws enable us to
=6.02 × 1023 ) make use of mass fraction and find the
(a) 9.03 × 1023 (b) 9.03 × 1024 actual mass of an element in any given
(c) 9.03 × 1021 (d) 9.03 × 1022 compound ?
Solution (a) Law of constant composition
Given that mass of N H3 = 25.5g (b) Law of multiple proportion
Avogadro’s number = 6.02 × 1023 (c) Law of conservation of mass
but molar mass of N H3 = 17gmol−1 (d) Law of constant mass
17g of N H3 is contain in 6.02 × 1023 Solution
25.5 g of M H3 will be contained in x Ans C
6.02×1023 ×25.5
17
= 9.03 × 1023 Ans A
10. The molar mass of C18 H37 O2 N is (C=12,H=1,O=16,
5. Nuclear particles are and N=14)
(a) Nuetrons and protons (a) 296g/mol (b) 299g/mol
(b) Neutrons and electrons (c) 297g/mol (d) 294g/mol
(c) Protons and electrons Solution
(d) Neutrons,Protons and Electrons Molar mass of C18 H37 O2 N = 12(18) +
Solution 1(37) + 2(16) + 14 = 299gmol−1 (B)
Ans A
11. If an element in group IV loses an alpha
6. How many particle are in 12.0g of carbon particle ,to which group would the prod-
atom ? uct belong
(a) 6.023 ×1023 (b) 12.0 (c) 1 (d) 24 (a) group I (b) group II
Solution (c) group III (d) group IV
Given that : mass of carbon = 12g Solution
12 1 4 9
Avogadro’s number = 6.02 × 1023 6 C +0 n →2 H +4 Be
but molar mass of carbon= 12gmol−1 it produce Be which is a group II element
12g of carbon is contain in 6.02 × 1023 Ans B
12g of carbon will be contain in x 12. Which of the following is/are true of Ra-
23 ×12
x = 6.02×10
12
= 6.02 × 1023 (D) dioactivity
7. Gallium has two naturally occuring isotope,60.4%I it involves the transformation of nucleus
is 69 Ga (mass = 68.9257 amu) and 39.6% into another nucleus
is 71 Ga (mass =60.9284 amu).Calculate II it gives out radiation
pthe avarage atomic weight of Gallium III energy is given out in the process
(a) 69.8 (b) 59.5 (c) 69.5 (d) 55.6 IV The rate of radioactivity decay is not
Solution affected by temperature and pressure
since 69 Ga(68.9257) has an abundance of (a) I and II only (b) I and III only
60.4% and (c) I,II and III only (d) I,II,III and IV
71
Ga(70.9248) has an abundance of 39.65 Solution
(68.9257 × 60.4) + (70.9248 × 39.6) In radioactivity
R.A.M= - One nucleus can be transform to an-
100
= 69.717 other
32

- Radiation is emitted (given out) (c) Photographic plate


- Energy is released . (d) All of the above
But temperature and pressure have no Solution
effect on the rate of radioactivity Instrument that can be used to detect
Ans C radiation are ;
NOTE-: It should be known that,it is - scintillation counter
the bombardment of carbon with a neu- - Geigermuller counter
tron that causes it to split - Photographic plate
∴ The addition of 10 n should not cause Ans D
confusion
17. When a sheet of paper is placed in the
path of a radioactive source,the radiation
13. How many grams of nitrogen are present emitted which passes through the paper
in a 0.1g sample of caffeine (C8 H10 N4 O2 ) consist
a stimulant in coffee and tea (C = 12, H = (a) Alpha and Beta particles
1, O = 16 and N = 14) (b) Alpha and Gamma rays
(a) 0.01g (b) 0.02g (c) 0.03g (d) 0.04g (c) beta and gamma rays
Solution (d) Alpha,beta and gamma rays
Given that ; C8 H10 N4 O2 Solution
Mass of nitrogen in the sample = 0.1g Ans C
molar mass of sample = 12(8)+ 1(10)
18. What is the value of x in the following
+14(4)+16(2)=194gmol−1
nuclear reaction
But mass of nitrogen in the compound = 11 251 X 1
14 × 4 = 56g 5 B +98 Cf →103 Lr + 30 n
(a) 241 (b)251 (c) 261 (d)259
56g of nitrogen is contain in 194g of C8 H10 N4 O2
Solution
Xg of nitrogen will be contained in 0.1g 11 251 x 1
of C8 H10 N4 O2 5 B +98 Cf →103 Lr + 30 n
56 × 0.1 11 + 251 = x + 3(1)
X= = 0.02886 ≈ 0.03(C) x = 262 − 3 = 259(D)
194
14. Spontaneous emission of radiation by un- 19. Two radioactive elements X and Y have
stable atomic nucleus is known as half-lives of 20 and 10 hours respectively.Therefore
(a) Radioactivity (b) Bate emission (a) X decays faster than Y
(c) Gamma emission (d) Nuclear Fission (b) Y is twice as stable as X
Solution (c) X is more stable than Y
Ans A (d) Y emits fewer particles than X
Solution
15. The mass of 2.61 mol sample of carbon Half life of X= 20 years
monoxide is Half life of Y = 10 years
(a) 10.73g (b) 73.08g This means Y decayed faster then Y ,X
(c) 21.46g (d) 83.08g is more stable than Y
Solution Ans C
Given that ;
mass of 1 mole of CO = 12+16=28gmol−1 20. Which of the following move with a ve-
1 mol of CO contain 28g locity of about one tenth that of light
∴ 2.61mol of CO will contain x (a) Alpha Particles (b) gamma rays
x = 28×2.61 = 73.08g(B) (c) Beta and Gamma rays (d) Beta par-
1
ticles
16. Which of the following can be used to Solution
detect radiation - Gamma rays have same speed as light .
(a) Geiger muller counter - Beta rays ranges from being small to
(b) Scintillation counter being as large as speed of light
0.7. CHEM 101 2012/2013 33

- Alpha rays is about one tenth,the speed 25. What is the maximum number of double
of light bonds that a hydrogen atom can form?
Ans A (a) 0 (b) 1 (c) 2 (d) 3
Solution
21. For a given arrangement of ions,the lat- A hydrogen atom has only one electron
tice energy increases as ionic radius ...... ,therefore ,it cannot form double bound
and as ionic charge ........ Ans A
(a) decreases,increases (b) increases,decreases
(c) increases,increases (d) decreases,decreases26. In the molecule below,which atom has
Solution the largest partial negative charge ..... :
lattice energy is directly proportional to
the charge on the ion and inversely pro-
portional to the distance between the nu-
cleus of the ions (radius)
Ans A

22. What is the energy of a photon of light


with a wavelength of 400 Armstrong ?
(a) 3.100 (b) 0.310 (c) 31.00 (d) 310.00
Solution (A) Cl (b) F (c) Br (d) I
Solution
Flourine is the highest with the highest
Given that wavelength (λ = 400A0 ) = electro negativity in the compound.Therefore
400 × 10−10 m ,it attract bounding electrons towards it-
c
Energy (E)= hv but v = self than any other atoms.That gives it
λ
hc 6.63 × 3 × 108 the largest partial negative charge
∴ E= = = 4.9725 × Ans B
λ 400 × 10−10
10−18 j
27. The ability of an atom in a molecule to
23. A ....... covalent bond between two iden- attract electrons is best quantified by the
tical atoms is the longest ,.....
(a) single (b) double (a) paramagnetism (b) diamagnetism
(c) triple (d) They are all the same length (c) electro negativity (d) first ionization
Solution potential
In covalent bond Solution
- single bound is the longest followed by Ans C
double bound and tripple bound is the
shortest 28. What is the wavenumber of the radiation
Ans A corresponding to the spactral line of the
lowest frequency in lyman series in the
24. Electronic radiation of wavelength 242 spectrum of hydrogen atom?
nm is sufficient to ionize an atom,what (a) 83458.50−1 (b) 83358.50−1
pis the ionization energy of the atom (c) 811258.50−1 (d) 82258.50m−1
(a) 0.082 × 10−17 j (b) 1.082 × 10−17 j Solution
(c) 0.095j (d) 1.095j
Solution
wavelength (λ) = 242nm = 242 × 10−9 m 29. Electro negativity ...... from left to right
−34 8
hc 6.63 × 10 × 3 × 10 × 3 × 108 a period and ..... from top to bot-
within
Energy = hv = = =
tom within a group
λ 242 × 10−9
8.219 × 10−19 (a) decreases,increases (b) increases,increases
0.082 × 10−17 m(A) (c) increases,decreases (d) remains the same,increases
34

Solution components in an ionic solid


Ans C Solution
Ans B
30. A nonpolar bound will form between two
....... atoms of ........ electronagativity.34. The type of compound that is most likely
(a) identical,equal (b) identical,different to contain a covalent bond is .....
(c) different,different (c) similar,different (a) one that is composed of a metal from
Solution the far left of the periodic table and a
A non polar bond will form between two non metal from the far right of the peri-
identical atoms with equal electro nega- odic table
tivity (b) A solid metal
Ans A (c) one that is composed of only non metal
(d) held together by the electrostatic forces
31. The formal charge on carbon in the molecule between oppositely charged ions
below is ....... Solution
Ans C

35. Of the molecule below,the bond in ......


is the most polar
(a) HBr (B) HI (c) HCl (D) HF
Solution
Since electro negativity decreases down
the group, and polarity is a function of
electro negativity ,polarity decrease down
(a) 0 (b) +1 (c) +3 (d) +3 the group too.Therefore HF is polar and
Solution HI is the least polar
Carbon can not have any charge in the Ans D
molecule because the net dipole moment
cancel out .(The same amount of charge 36. Which scientist proposed a quantized model
withdrawn by the first oxygen is equally of atom ?
withdrawn by the second oxygen). (a) thompson (b) Rutherford
Ans A (c) Bohr (d) Planck
Solution
32. Which ion below has a noble gas elec- Ans C
tronic configuration ?
(a) Li2+ (b) Be2+ (C) B 2+ (d)C 2+ 37. An electronic transition occurred which
Solution gave frequncies that stopped in the vis-
Berrylium has four electron,when it losses ible region of the electromagnetic spec-
2 ,it obtain tthe helium configuration trum.The transition is most likely to be
Ans B in which series
(a) Lyman series (b) Balmer series
33. Lattice energy is ........ (c) Paschen series (d) pfund
(a) the energy required to convert a mole Solution
of ionic solid into its constituent ions in Ans B
the gas phase
(b) the energy given off when gaseous 38. Given the reaction
ions conbine to form ane mole of an ionic 4HCl(aq) + M nO2 (s)  M nCl2 (aq) +
solid 2H2 O(l) + Cl2
(c) the element required to produce one The manganese is
mole of ionic compound from its constituents (a) Reduced and its oxidation number
element in their standard states changes from +4 to +2
(d) the sum of ionization energies of the (b) reduced and its oxidation number changes
0.7. CHEM 101 2012/2013 35

from +2 to +4 Cr2 O7−2 + C2 H5 OH  Cr3+ + C2 H4 O2


(c) oxidized and its oxidation number changes Which substance is a reducing agent and
from +4 to +2 which is an oxidizing agent
(d) oxidized and its oxidation number changes (a) C2 H5 OH,reducing agent; no oxidiz-
from +2 to +4 ing agent
Solution (b) C2 H5 OH,reducing agent ; CrO7−2 , ox-
4HCl + M nO2 M nCl2 + 2H2 O + Cl2 idizing agent
M n + 2(−2) = 0 (c) C2 H5 OH,oxidizing agent ;Cr3+ ,reducing
M n = +4 agent .
(d) C2 H5 OH,reducing agent; Cr3+ ,oxidizing
M n + 2(−1) = 0 agent
M n = +2 Solution
Mn is reduced from +4 to +2 (A) Cr2 O72− + C2 H5 OH Cr3+ + C2 H4 O2
2Cr + 7(−2) = −2
39. What occurs when an atoom is oxidized Cr = +6
in a chemical oxidation Cr2 O72− accept there electron and the ox-
(a) loss of electron and decrease in oxi- idation number of Cr changes from +6 to
dation number +3
(b) loss of electron and increase in oxida- Reduction is given of electron and the
tion number specie that caused reduction is an oxida-
(c) Gain of electron and decrease in oxi- tion agent .
dation number Therefore CrO72− is an oxidizing agent
(d) Gain of electron and increase in oxi- and C2 H5 OH the reducing agent
dation number Ans B
Solution
Ans B 43. Which of the following shows a metal be-
ing oxidized
40. Standard cell potential is I 2N a + 2H2 O  2N aOH + H2
(a) Measured at a temperature of 250 C II Cu  Cu2+ + 2e−
(b) Measured when iron concentration of III Cu2+ + 2e−  Cu
acqueous reaction are 1.00M (a) I and II (b) I and III
(c) Measured under the condition of 1.00atm (c) II and III (d) All of the above
for gaseous reactant Solution
(d) All of the above 2N a + 2H2 O 2N aOH + H2
Solution At the reactant side N a = 0
Ans D At the product side 2N a+2(−2)+2(1) =
41. The Standard reduction potential in volts 0
2+ +
for Pb and Ag are −0.13 and 0.80 re- N a = +1
0
spectively .Calculate E in volts for a cell Na moves from 0 to +1,this is an increase
in which the overall reaction is in oxidation number (oxidation)
+ 2+
P b + 2Ag  P b + 2Ag Cu Cu2+ + 2e−
(a) 1.73 (b) 0.67 (c) 0.93 (d) 1.47 Cu moves from +2 to 0,this is a decrease
Solution in oxidation number (oxidation)
2+
Given standard potential of P b and Ag + Cu2+ + 2e− Cu
to be −0.13 and 0.80 respectively,standard Cu moves from +2 to 0 .this is a decrease
cell potential is , in oxidation number (reduction)
E 0 = 0.80 − (−0.13) = 0.80 + 0.13 = Ans A
0.93(C)
44. It is possible to generate an electrical po-
42. Breath Analyzer determined alcohol con- tential by inserting two strips of different
tent via the redox reaction metals into an acidic citrus fruit such as a
36

lemon .Among other function,the lemon Talking a look at the final eqaution we
serves as a salt bridge.Which short hand see that,equ iii was reversed .By revers-
notation would best described a lemon ing,the sign of the cell changes as well.
into which has been inserted a strip of 3M n 3M n2+ + 6e− E 0 = 3.54
zinc and a strip of copper,with the two 2AuCl4− +6e− 2Au+8Cl− E 0 = 2.00
metals strips connected by a wire 3M n+2AuCl4− 3M n2+ +2Au+8Cl−
2+
(a) zn(s)/zn (aq)//O2 (g)/H2 O(l)/Cu(s) E 0 = 3.54 − 2.00 = 1.54
(b) zn(s)/zn2+ (aq)/H + (aq)/H2 (g)/Cu(s)
(c)Cu(s)/Cu2+ (aq)//O2 (g)/H2 O(l)/Zn(s) 47. Which of the following is FALSE regard-
(d) zn(s)/zn2+ (aq)//Cu2+ (aq)/Cu(s) ing the salt bridge used in voltaic cells?.The
Solution salt bridge
Ans D (a) Allows for the two half-cells to be
kept separated
45. Which of these statement about a gal- (b) Maintains the electrical neutrality in
vanic cell are NOT TRUE each half-cell
I The cathode carries a positive charge (c) Allows mixing of the two electrode
II The anions migrate towards the cath- solutions
ode (d) Is made of a medium through which
III The electron are releases through the ions slowly pass
anode Solution
IV Reduction occurs at the anode The salt bridge serve as follows
(a) I and III (B) I and II - Keeping the two half cell separated.
(c) II and III (d) II and IV - It maintains electrical neutrality in each
Solution half cell .
During electrolysis chemical processes ; - Is a semi permeable membrane,only ions
- The cathode carries a negative charge can pass through .
- The anions migrate to the anode - It does not allow the mixing of the two
- The electron are released through the solution .
anode . Ans C
- Reduction occurs at the anode .
48. Based on the following information, which
it can be seen that I and II are not true ,
will be the most effective oxidizing agent
Ans B
?
46. From a consideration of the following two N a + e−  N a E 0 = −2.71
half reactions O2 + 4e− + 2H2 O  4OH − E 0 = +0.40
(i) M n2+ + 2e−  M n E 0 = −1.18 Cl2 + 2e−  2Cl− E 0 = +1.36
(ii) Aucl+ + 3e−  Au + 4Cl− E 0 = 1.00 (a) Na (b) N a+ (c) O2 (D)Cl2
what is the standard cell potential for the Solution
reaction Ans D
3M n + 2AuCl4−  3M n2+ + 2Au + 8Cl− 49. The salt bridge in the electrochemical cell
(a) −2.18v (b) −0.18v (c) 0.18v (d) 2.18v serves to
Solution (a) increase the rate at which equilibrium
M n2+ +2e− M n E 0 = −1.18−−−(i) is attained
AuCl4− + 3e− Au + 4Cl− E 0 = (b) increase the voltage of the cell
1.00 − − − (ii) (c) Maintain electrical neutrality
Multiply equ 1 by 3 and equ 2 by 2 to (d) increase the oxidation /reduction rate
cancel out charges Solution
3M n2+ + 6e− 3M n E 0 = −3(1.18) − Ans C
− − (iii)
2AuCl4− + 6e− 2Au + 8Cl− E 0 − 50. Complete the following equation (All sto-
−(IV ) ichiometric coefficient must be integers)
0.8. CHEM 101 2013/2014 A 37

M nO4− (aq) + Cl− (aq)  M n2+ (aq) + (a) Absorption (b) Emission
Cl2 (g) (acidic solution) (c) Both absorption and emission
How many hydrogen ion are needed and (d) None of the above
on what side of the equation must they Solution
appear When an electron jumps from a lower en-
(a) 16,on the left (b) 8,on the left ergy level to a higher level such transition
(c) 16,on the right (d) 4,on the right is accomplanied by absorption of energy
Solution Talking the half cell which it gives out energy (in the form of
M nO4− → M n2+ radiation) when descending from higher
+7 +2 energy level to a lower energy level
Balancing charges Ans A
M nO4− + 5e− → M n2+ (Reduction)
Balancing O2 3. Given that 1/λ = RH(1/n21 −1/n22 ).Calculate
M nO4− + 5e− + 8H + → M n2+ + 4H2 O the wavelength (λ) of the first line in the
Taking the other half cell Balmer series of the spectrum of hydro-
Cl− → cl2 gen atom (RH = 109677CM −1 ).
Balancing atoms (a) 6565cm−1 (b) 6.565 × 10−10 cm−1
2Cl− → Cl2 (c) 6.565×10−5 cm−1 (d) 1.523×10−4 cm−1
−2 0 Solution
Balancing charges
2Cl− → Cl2 + 2e−
4. The possible value of the azimuthal quan-
putting equ 1 and 2 together
tum number for the principal shell n=4
M nO4− + 5e− + 8H + → M n2+ + 4H2 O
are
2Cl− → Cl2 + 2e−
(a) 0,+1 (b) 0,+1,+2
Multiplying equ 1 by 2 and equ by 5 to
(c) 0,+1,+2,+3 (d) 0,+1,+2,+3,+4
cancel out charges
Solution
2M nO4− + 10e− + 16H + → 2M n2+ +
Ans C
8H2 O
10Cl− → 5Cl2 + 10e− 5. In the radioactive reaction 235 1 142
92 U +0 n →56
putting the two equation as one Ba +90 36 Kr + Y , Y represents ,
2M nO4− + 10Cl− + 16H + 2M n2+ + (a) 2 He (b) 20−1 e
4

5Cl2 + 8H2 O (c) 410 n (d) 211 P


it can be seen that ,16H + Appears on the Solution
left . From the above equation
Ans A 251 + 1 = 142 + 90 + a
236 = 232 + a
a = 236 − 232 = 4
0.8 CHEM 101 2013/2014 Also 92 + 0 = 56 + 36 + b
A b=0
∴ 40 Y = 4(10 n) Ans C
1. The s,p,d and f orbitals can accommo-
date a maximum of ....... electrons in 6. The splitting of a heavy isotope by a neu-
each case tron into light nuclei with emission of two
(a) 2n2 (b) 2(2l+1) (c) 2+n2 (d) (2l+1)2 or more fast moving neutrons is called
Solution (a) Nuclear fusion (b) Nuclear fission
(c) Radioactivity (d) Nuclear chemistry
Solution
2. When an electron jumps from a lower en- Ans B
ergy level to a higher level ,such a tran-
sition is accomplished by ............ of ra- 7. During beta-decay a neutron splits up
diant energy . forming a
38

(a) Proton and an electron (b) Proton Solution


and a position
(c) Gamma particle and an electron
(d) Gamma particle and a proton 13. In the nuclear fission of Uranium -235 ,if
Solution there is a mass loss of 0.20 g per mole
During Beta decay,a nuetron splits into ,what is the quantity of heat associated
a proton and an electron with the mass loss in kilojoules ?(velocity
1 1 0 of light = 3.0 × 108 ms−1 ).
0 n →1 P +−1 e Ans A
(a) 3.0 × 1010 (b) 1.0 × 1010
8. Isotopes with an n/p ratio below the sta- (c) 6.0 × 1010 (d) 1.8 × 1010
ble value try to stabilize themselves by Solution
(a) Electron capture (b) Beta-decay mass loss = 0.20g= 0.2 ×10−3 kg
(c) Alpha-decay (d) Gamma emission quantity of heat = N l2 = 0.2 × 10−3 (3 ×
Solution 108 )2
Beta Decay Ans B = 18 × 1012 j = 1.8 × 1010 j Ans D

9. The region in which an electron is most 14. The central atom in XeF4 is surrounded
probably found is commonly referred to by
as (a) 3 single bonds,1 double bond ,and no
(a) Orbital (b) shell lone pair of electrons
(c) electrostatic field (d) principal quan- (b) 3 single bounds, 1 double bound ,and
tum number 1 lone pair of electrons
Solution (c) 4 single bounds ,no double bounds,and
Ans A no lone pair of electrons
(d) 4 single bounds,no double bonds ,and
10. The rods of graphite in a nuclear reactor 2 lone pair of electrons
act as Solution
(a)Accelerators (b) Shields
(c)Moderator (d) Coolants
Solution 15. Which one of the following molecules has
Ans C a dipole moment ?
(a) Cl4 (b) P F5 (c) N Cl3 (d) SO3
11. In medical research ,radioactive Radon is Solution
used as a tracer to
(a) Monitor cancer growth
(b) Detect body defects 16. Which of the following compounds has
(c) Detects the presence of bacteria the greatest number of electron trans-
(d) Study absorption of iodine by thyriod ferred to form ionic bonds ?
gland (a) Carbon dioxide (b) Copper (II) oxide
Solution (c) potassium fluoride (d) Iron (III) chlo-
ride
Solution
12. The Use of radioactive C-14 includes the
following except
(a) To follow the paths of chemical reac- 17. Which one of the following atoms A,B,C
tion or D would readily form an ion with a
(b) To determine the equilibrium posi- charge of 1+
tion in reversible reaction Element Atomic Number
(c) To deduce the age of the remains of A 6
plant B 8
(d) To study the rate at which thyroid C 11
gland aborbs iodine D 15
0.8. CHEM 101 2013/2014 A 39

Element C having 11 electrons can easily The bond in MgCl2 is electrovalent due
loss one electron and because stable with to the large size of Mg and also their large
a charge of +1 difference in their electro negativity .
Ans C Ans B

18. The table below gives the atomic number 22. What are the three bound angle in the
of four elements : trigonal bipyramidal structure
Element Atomic Number (a) 900 , 1200 , 1800
W 8 (b) 1100 , 1200 , 1800
X 10 (c) 1070 , 1200 , 1800
Y 11 (d) 1090 , 1200 , 1800
Z 17 Solution
Which of the following pair of atoms can Ans A
combine to form a covalent compound
(a) Two atoms of W (b) Two atoms of X 23. Using the VSERPR model ,the electron-
(c) An atom of Y and an atom of Z domain geometry of the central atom in
(d) An atom of W and an atom of X SF2 is .......
Solution (a) Linear (b) Trigonal planer
-Two atoms of W can form a covalent (c) Tetrahedral (d) Trigonal bipyramidal
bond with each other to become stable . Solution
-The atom X is already stable and can- Since only two flourine atoms bounded
not bond with itself . with the sulphur atom making a total
- A bind between Y and Z is electrova- of four electron pair around the sulphur
lent . atom.it will ave a linear shape
- The element X cannoy=t bond with any Ans A
other element because it is already un re-
active . 24. Th electron-domain geometry of the cen-
Ans A tral atom in BrF4− is .......
(a) Trigonal planar (b) Tetrahedral
use the following Lewis diagram to an- (c) Trigonal bipyramidal (d) Octehedral
swer question 19 and 20 Solution
DIAGRAM

19. How many bounding pairs of electrons 25. The molecule geometry of the central atom
are present in the molecule in N Cl3 is .......
(a) 1 (b) 3 (c) 4 (d) 7 (a) Trigonal planar (b) Tetrahedral
Solution (c) Bent (d) Trigonal Pyramidal
The only bonding pair of electrons is the Solution
one between H and Cl Ans D
Ans A
26. Of the molecule below ,only .......... is
20. How many lone pairs of electrons are present nonpolar
in chlorine atom ? (a) BF3 (b) N F3 (c) IF3 (d) P Br3
(a) 1 (b) 2 (c) 3 (d) 4 Solution
Solution Ans A
Ans C
27. In Scattering of fast alpha particles by a
21. Which type of bound is in M gCl2 thin foil of gold,rutherford and his col-
(a) covalent bound (b) Ionic bound laborators observed one of the following
(c) coordinate covalent bound (d) Metal- (a) many alpha particles scattered through
lic bound large angles
Solution (b) Very few alpha particles scattered through
40

large angles Solution


(c) alpha particles only scattered through Since number of electron in a shell is given
small angles by 2n2 ,when n=1
(d) alpha particles are not scattered at number of electron = 2(1)2 =2 (D)
all
Solution 33. For most elements ,an atom has
Ans A (a) no neutrons in the nucleus
(b) more protons than electrons
28. Which of the following radioactive rays (c) less neutrons than electrons
will not be deflected by a magnetic field (d) just as many electrons as protons
(a) Alpha rays (b) beta rays Solution
(c) gamma rays (d) electrons Ans D
Solution
Gamma rays are electrically neutral.Therefore34. How many molecules of H2 are in 0.001
,it can not be detected by either electric mole of H2
or magnetic field. (A) 3.01 × 1036 (b) 3.01 × 1023
Ans C (c) 6.02 × 1020 (d) 6.02 × 1021
Solution
29. The scientist whose Law Of Definite Pro- Since 1 mole of H2 contains 6.02 × 1023
portion provode strong evidence for the then 0.001mol of H2 will contain x
atomic theory is : x = 6.02×10
23
= 6.02 × 1020 Ans C
0.001M
(a) Faraday (b) Bohr
(c) Dalton (d) Rutherford 35. Which of the following statement is not correct
Solution (a) according to Faraday’s 1st law of elec-
Ans C trolysis ,1 mole of any substance require
1F for complete deposition
30. A sodium atom basically has :
(b) according to Dalton atomic theory,atoms
(a) no electron charge in it
can neither be created nor destroyed in a
(b) more neutrons than protons
chemical reaction
(c) an equal number of electron and pro-
(c) equal volume of gases at STP contain
tons
equal number of particles
(d) a net positive charge
(d) All of the above
Solution
Solution
A sodium atom 23 11 N a ,it can be observed
According to the faraday’s first law of
that,number of proton is 11 and that of
electrolysis,the amount of a substance de-
neutron is 23 − 11 = 12.
posited during electrolysis id directly pro-
it has more neutron than proton Ans B
portional to the quantity of heat applied
31. The principal characteristics that distin- and not to IF .
guish one element from another is the Ans A
(A) number of protons in the nucleus
(b) number of neutrons in the nucleus 36. In a given compound ,the constituent el-
(c) combined number of protons and neu- ement are always combined in the same
trons in the nucleus proportion by mass,regardless of the ori-
(d) number of variation in the electron gin or made of preparation of the com-
configuration pound.This is a statement of
Solution (a) the law of multiple proportion
Ans C (b) the law of constant composition
(c) the law of conservation of mass
32. The maximum number of electron allowed (d) the law of reciprocal
in the n=1 shell of potassium is Solution
(a) 8 (b) 6 (c) 4 (d) 2 Ans B
0.8. CHEM 101 2013/2014 A 41

37. Cathode ray produced in a gas discharge 41. What is the number of moles of proton
tube are present in the balanced equation
(a) x-ray (b) alpha particle (a) 52 (b) 15 (c) 22 (d) 32
(c) electrons (d) gamma rays Solution
Solution CrI3 + Cl2 → CrO42− + IO3− + Cl−
Ans C using the ion-electron method ;
CrI3 → CrO42− + IO3−
38. Which of the following is not correct Cr + 3(−1) = 0
(a) The oxidized substance is always the Cr = +3
reducing agent
(b) The reduced substance is always the
Cr + 4(−2) = −2
oxidizing agent
Cr = +6
(c) The oxidized substance is always the
Cr changes from +3 to +6.This is oxida-
oxidant
tion
(d) The reduced substance is always the
Balancing iodine atom;
oxidant
CrI3 → CrO42− + 3IO3− + 3e−
Solution
Balancing for excess oxygen
Ans C
CrI3 + 13H2 O → CrO42− + 3IO3− + 3e− +
26H +
39. Which of the following statement is correct
balancing for electrons ;
with respect to the redox equation below
CrI3 +13H2 O → CrO42− +3IO3− +26H + +
CH4 + O2 → CO2 + H2 O
21e− ——-(i)
(a) carbon is oxidized and oxygen is re-
Cl2 → Cl−
duced
Balancing chlorine atom
(b) Carbon is reduced and hydrogen in
Cl2 → 2Cl−
oxidized
chlorine changes from 0 to −2.this is re-
(c) Oxygen is oxidized and hydrogen is
duction
unchange
Cl2 + e− → 2Cl−
(d) Oxygen is reduced while hydrogen
Balancing for electrons ;
and carbon were oxidized
Cl2 + 2e− → 2Cl− − − − −(ii)
Solution
By comparing equation i and ii,there are
Ans B
21e− in equ 1 but only 2e− in equ (ii)
The oxidation-reduction equation for a .multiplying equ 1 by 2 and equ 2 by 21
reaction in acid medium is given below.Balance 2CrI3 + 26H2 O → 2CrO42− + 6IO3− +
the equation by the ion-electron method 52H + + 42e−
and use it to answer 40 to 43 21Cl2 + 42e− → 42Cl−
CrI 2CrI3 + 26H2 O + 21Cl2 → 2CrO42− +
6IO3− + 52H + + 42Cl−
40. The change in oxidation number of io- Ans A
dine is from :
(a) 0 to +3 (b) −1 to − 3 42. What is the ratio of electron transfer in-
(c) +2 to −3 (d) −1 to + 5 volved in the redox reaction ?
Solution (a) 7:12 (b) 2:12
2−
CrI3 + Cl2 → CrO4 + IO3 + cl− − (c) 2:11 (d) 5:6
Change in oxidation number of I Solution
3I + 1(3) = 0 Ans B
I = −1
43. The oxidation number of ce and P in
Ce(ClO3 ) and N a2 HP O4 are
I + 3(−2) = −1 (a) +5 and -2 respectively
I = −1 + 6 = +5 Ans D (b) +7 and + 2 respectively
42

(c) +2 and +5 respectively rect ?


(d) +3 and −5 respectively (a) H2 T e > H2 Se > H2 O > H2 S
Solution (b) H2 O > H2 S > H2 Se > H2 T e
Ce + 2(−1) + 2(3)(−2) = 0 (c) H2 T e > H2 Se > H2 S > H2 O
Ce = 14 (d) H2 Se > H2 T e > H2 S > H2 O
for N a2 HP O4 Solution
2(+1) + 1 + P + 4(−2) = 0 Since acidic strength increases down the
p = +5 group.Group 6 elements are oxygen,sulphur
selenium etc
44. The E 0 cell for the redox reaction N i/N i2+ Ans C
// Sn2+ /Sn is 0.114v.If the standard elec-
trode potential of nickel is −0.250v,what 49. All electrophiles are
is the potantial of tin half-cell ? (a) Arrhenius bases (b) Arrhenius acids
(a0 −0.136v (b) −0.364v (c) Lewis acids (d) Lewis base
(c) 0.364v (d) +0.1130 v Solution
Solution Ans D

50. Given that the half-cell potential of Ag + /Ag


45. Which of the following is a lewis acid ?
and Al3+ /Al are +0.799v and −1.662 re-
(a) BF3 (b) CN − (c) Cl− (d) N H2−
spectively .what is the standard cell po-
Solution
tential for the redox reaction shown be-
A lewis Acid is a lone pair acceptor.it
low
must have a low lying d-orbital
Al + 3Ag + → 3Ag + Al3+
Ans A
(a) 2.461 v (b) −0.613v
46. The Bronsted-lowery concept referred to (c) −0.261v (d) +0.665v
a base as Solution
(a) a proton donor (b) a proton acceptor
(c) an electron pair acceptor (d) an elec-
tron pair donor 0.9 CHEM 101 2013/2014
Solution
Ans B
B
47. Calculate the pH of a 0.02M Ba(OH)2 1. Which of the following laws will enable
solution the use of mass fraction to find the ac-
(a) 10.4 (b) 10.8 (c) 11.5 (d) 12.6 tual mass of an element in any given com-
Solution pound ?
Ba(OH)2 When ionizes I law of constant composition .
Ba(OH)2 Ba2 + 2OH − II law of multiple proportion .
Since molar conentration is 0.02moldm−3 . III Law of conservation of mass .
since there are 2 moles of OH − ,the con- IV Law of constant mass .
centration becomes 0.02 × 2 = 0.04 (A) I and II (b) I and III
pOH = −Log(0.04) = 1.3979 (c) II and IV (d) I only
but pH + pOH= 14 Solution
pH = 14 − pOH = 14 − 1.3979 = 12.60
Ans D
NOTE:for bases,the pOH must be deter- 2. Which of the following is NOT TRUE
mined first.The equation pH +pOH = 14 about radioactivity ?
is then used to find the pH (a) It involves the transformation of a nu-
cleus to another nucleus .
48. Which of the following arrangement in (b) It gives out radiation
order of decreasing acid strength is cor- (c) Energy is given out in the process .
0.9. CHEM 101 2013/2014 B 43

(d) The rate radioactive decay can be ac- with atomic mass unit of 106.90509 and
celerated by increase in temperature and 108.90476 respectively.Calculate the rel-
pressure ative atomic mass of silver .
Solution (a) 108.905 (b) 107.905
During radioactivity ; (c) 107.90 (d) 107.89
I one nucleus can be converted to another Solution
. 106.90509 + 108.90476
R.A.M = = 107.90
II particles and radiations are given out 2
Ans C
.
III very large amount of energy is emit- 6. Boron has two naturally occurring iso-
ted . topes.Calculate the percentage abundances
IV Temperature and pressure have no ef- of 10 B and 11 B,given that the relative
fect on the rate of radioactivity atomic mass of Boron is 10.81amu;10 B =
Ans D 10.0129amu and 11 B = 11.009amu
(a) 10 B = 10%,11 B = 90%
3. The atomic mass of a certain element
(b) 10 B = 20%,11 B = 80%
is 63.63g,if it contains 32 electrons how
(c) 10 B = 30%,11 B = 70%
many neutrons does it have ?
(d) 11 B = 20%,10 B = 80%
(a) 30 (b) 31 (c) 32 (d) 33
Solution
Solution
Given that 10 B = 10.0129 and 11 B =
Given that ,atomic mass is 63.63g ≈ 64g
11.009 , since the sum of the percentage
Since it contains 32 electrons
abundance of the two isotopes is 100.
and number of electron = number of pro-
Then,if the percentage abundance of 10 B =
tons
x
∴ number of protons = 32
The percentage abundance of 11 B = 100−
number of neutrons = 64-32= 32
x
Ans C (10.0129 × x) + 11.009(100 −
Relative atomic mass =
100
4. The isotopic masses of two isotopes of an 10.0129x + 1100.9 − 11.009x
element measured by a mass spectrome- 10.81 =
100
ter are 34.969 and 36.968 atomic mass 1081 = −0.9961x + 11009
1100.9 − 10811
respectively.What is the relative mass of x= = 19.9779 = 20
the element if the percentage abundance 0.9961
but 10 B = x = 20
of the heavier isotope is 24.50? 11
B = 100 − x = 100 − 20 = 80
(a) 34.97 (b) 36.97 (c) 35.97 (d) 35.97 Ans B
Solution
Since the isotopes are 34.969 and 36.968 7. Cathode rays are deflected by
, (a) electric field only
The heavier isotopes (36.968 ) has a % (b) magnetic field only
abundance of 24.50 but the sum of the (c) electric and magnetic field
percentage abundance of the two isotopes (d) none of these
is 100 . Solution
∴ The smaller isotopes (34.969) has an Ans C
abundance of 100 − 24.50 = 75.50
(36.968 × 24.50) + (34.969
8. The× e/m
75.50)
value for the particle constitut-
Relative atomic mass =
100 ing the cathode rays is the same regard-
905.716 + 2791.1595
= = 36.968755 ≈ less of
100
36.97 Ans B (a) the gas present in the cathode ray
tube
5. Silver has 46 known isotopes,but the nat- (b) the metal of which cathode was made
107 109
urally occurring ones are Ag and Ag (c) All of the above
44

(d) none of the above 13. Which of the following processes does not
Solution change the atomic number ?
The e/m value as determined by J.J thomp- (a) Gamma emission (b) Alpha emission
son ,is independent of ; the gas present (c) Positron emission (d) Beta emission
in the cathode ray tube and the metal of Solution
which the cathode tube was made from . A beta emittion (0−1 e) increases the atomic
Ans C number by 1
An Alpha emittion (42 He) reduces the atomic
9. A sub-atomic particle which has one unit number by 2
mass and one unit positive charge known But a Gamma emittion is just a ray which
as has no mass nor charge .
(a) hydrogen atom (b) neutron Ans A
(c) electron (d) proton
Solution 14. Which of the following statement is NOT
A sub-atomic particle with one unit mass true about orbitals
and one unit positive charge is proton (a) Orbitals with the same principal quan-
(11 H) tum number constitute a shell .
Ans D (b) Orbital with the same n and 1 but
different m values are of the same en-
10. Which is true about isobars ?
ergy (degenerate) in the absence of ap-
(a) they have same mass number and same
plied field .
atomic number
(c) Orbitals are regions in which elec-
(b) they have the same mass number but
trons are most probably found.
different atomic number
(d) Orbitals of different 1 values in the
(c) they have different mass number and
same n are the same in energy .
same atomic number
Solution
(d) they have different mass number and
(a) Orbitals with the same principal qum-
different atomic number .
tum number constitute a shell -True
Solution
(b) Orbitals with the same n and l but
Isobars are atoms of different, having the
different m values are of the same energy
same mass number but different atomic
(degenerate ) in the absence of external
number .
magnetic field - False .
Ans B
Take for example when n=2 it implies
27 3+ l=0,1 which means s and p orbital 2s,2px 2py and 2p2
11. What is the relationship between 13 Al
23 + but pX , py and pz are not on the same de-
and 11 N a ?.They are
(a) isobars (b) isotopes generate orbital
(c) isoatoms (d) isoelectric . Ans B
Solution
27 3+ 15. The s,p,d and f orbitals can accommo-
13 Al ; This element losses 3 electron and
is left with only 10 electrons . date a maximum of ........ electron in
23 + each case
11 N a ; This element loss 1 electron but
is also left with 10 electrons (a) 2n2 (b) 2(2l + 1)
∴ 27 3+
and 23 + (c) 2 + n2 (d) (2l + 1)2
13 Al 11 Na are isoelectric
Ans D Solution
Number of orbitals is given by 2n and
12. Which of the following is most damaging number of electron is given by 2n2 .
when ingested ? Ans A
(a) Beta emitters (b) Alpha emitters
(c) Gamma emitters (d) All of the above 16. When an electron jumps from a lower en-
Solution ergy level to a higher energy level, such a
Ans C transition is accompanied by ........... of
0.9. CHEM 101 2013/2014 B 45

radiant energy . (c) radioactivity (d) nuclear chemistry


(a) absorption (b) emission Solution
(c) both absorption and emission (d) none Nuclear fission is the splitting of a heavy
of the above . isotope by a neutron into light nuclei while
Solution ;
Ans A Nuclear fussion is the joining of two or
more light nuclei to form a heavy isotope
17. Given that 1/λ = RH(1/n21 −1/n22 ).Calculate Ans B
the wavelength (λ) of the first line in the
Balmer series of the spectrum of hydro- 21. During beta-decay a neutron splits up
gen atom. forming a
(a) 6565 cm−1 (b) 6.565 × 10−10 cm−1 (a) proton and an electron
(c) 6.565×10−5 cm−1 (d) 1.523×10−4 cm−1 (b) Proton and a positron
Solution (c) gamma particle and electron .
Given 1/λ = RH( n12 − n12 ),RH = 1.097 × (d) gamma particle and a proton .
1 2
107 cm−1 Solution
for a Balmer series,electron can only drop During beta decay ,a neutron splits into
to n=2 a proton and an electron e.g
14 14 −
∴ λ1 = 1.097 × 107 ( 111 − 212 ) 6 C →7 N + e

= 1.097 × 107 (1 − 14 ) = 1.097 × 107 ( 43 ) Ans A


λ = 1.2 × 10−7 cm 22. Isotopes with an n/p ratio below the sta-
ble value try to stabilize themselves by
18. The possible values for the azimuthal (l) (a) electron capture (b) beta-decay
for the principal shell n=4 are (c) alpha-decay (d) gamma emission
(a) 0,+1 (b) 0,+1,+2 Solution
(c) 0,+1,+2,+3 (d) 0,+1,+2,+3,+4 For an n/p ratio below stable value,an
Solution inner electron is absorbed into the nu-
For the principal shell n=4 clues.This is incorporated with a proton
Azimuthal = 0,1,2,3 . to form a neutron thereby increases n/p
Ans C ratio.This process is called electron cap-
ture
19. In an artificial radioactivity 238 1 239
92 U +0 n →94
Ans A
P u + X, X represents
(a) 42 He (b) 20−1 e 23. The temperature of the pile in nuclear re-
(c) 00 γ (d) 211 P actor can be controlled by moveable rods
Solution of
Given the equation ; (a) copper (b) zinc
238 1 239
92 U +0 n →94 P u + X (c) iron (d) cadmium
Taking a look at the equation 238 + 1 = Solution
239 also 92 + 0 = 94 control rods for a nuclear reaction includes
it means,the only difference is the sub- ; cadmium ,boron silver,indium etc.These
script (2) element are capable of absorbing many
for a beta decay , 0−1 e.this means 2(0−1 e) neutron without fissining themselves
was emitted Ans D
Ans B
24. The rods of graphite in a nuclear reactor
20. The splitting of a heavy isotope by a neu- act as
tron into light nuclei with the emission (a) accelerators (b) shields
of two or more fast moving neutrons is (c) moderators (d) coolants .
called Solution
(a) nuclear fusion (b) nuclear fission Rod of Graphite serves as a moderator
46

in a nuclear reaction . Balancing for excess H2


Ans C P4 + 3e− + 12H2 O → 4P H3 + 12OH −
multiplying equ i by 3 and ii by 2 to bal-
25. In the nuclear fission of Uranium-235,if ance charges
there is a mass loss of 0.20g per mole.What 3P4 + 48OH − + 24H2 O → 12H2 P O2− +
will be the quantity of heat associated 24H2 O + 24OH − + 3e−
with the mass loss in kilojoules P4 + 3e− + 12H2 O → 4P H3 + 12OH −
(a) 3.0 × 1010 (b) 1.0 × 1010 4P4 + 36OH − + 12H2 O → 12H2 P O2− +
(c) 6.0 × 1010 (d) 1.8 × 1010 4P H3 + 24OH − + 12H2 O
Solution dividing both sides by 4 to reduce the
Given that ; mass loss = 0.20g = 0.20 × whole numbers
10−3 kg P4 +9OH − +3H2 O → 3H2 P O2− +P H3 +
Quantity of heat = M C 2 = (0.20×10−3 )× 6OH − + 3H2 O
(3 × 108 )2 it can be seen that phosphorous changes
= 1.8 × 1010 kj Ans D from 0 to +1 and -3 respectively .
Ans D
26. What is the oxidation number of Mo in
H2 M oO10 28. What is the least number number of moles
(a) +5 (b) +7 (c) +6 (d) +8 of hydroxide ion (OH − ) present in the
Solution balanced equation ?
Oxidation number of MO in H2 M o3 O10 (a) 3 (b) 5 (c) 6 (d) 12
is Solution
2(1) + 3M0 + 10(−2) = 0 Least number of moles of OH − From the
M0 = +6 Ans C final equation is 6
The oxidation-reduction equation fro Ans C
a reaction in alkaline medium is given
29. What is the ratio of electron transfer in-
below.Balance the equation by ion-
volved i the redox reaction ?
electron method and use the bal-
(a) 4:12 (b) 3:5
anced equation to answer question
(c) 6:12 (d) 5:6
27-30:
Solution
P4 → H2 P O2− + P H3
ignoring the sign,phosphorous first changes
to 1 and then to 3,therefore,the ratio is
27. The change in oxidation number of phos- 1 : 3 = 4 : 12
phorous is from zero to Ans A
(a) +1 and +3 (b) −2 and −3 30. Which of the following is the correct stoi-
(c) +2 and −3 (d) +1 and −3 chiometry of P4 , P H3 and H2 P O2− in the
Solution balanced chemical equation ?
P4 → H2 P O2− + P H3 (alkaline medium ) (a) 1:5: 6 (b) 2:3:6
Taking the half equation . (c) 1:1:6 (d) 1:1:3
P4 → H2 P O2− Solution
Balancing the atoms The correct stoichiometry for P4 , P H3 and H2 P O2−
P4 → 4H2 P O2− from the final equation is 1,1,3
Balancing for excess O2 Ans D
P4 + 16OH − → 4H2 P O2− + e− + 8H2 O
Balancing for excess H2 31. which of the following is the reduntant in
P4 + 16OH − + 8H2 O → 4H2 P O2− + e− + the unbalanced redox equation below ?
8H2 O + 8OH − Zn + N O3− → Zn(OH)2− 4 + N H3

taking the other half equation (a) N O3 (b) Zn
P4 → P H3 (c) N H3 (d) Zn(OH)2− 4
0 −3 Solution
0.9. CHEM 101 2013/2014 B 47

Zn + N O3− → Zn(OH)2− 4 + N H3 but S.C.P= Ecathode − Eanode


For Zn =0 = 0.114 = Esn − (−0.250)
For N O3− : N + 3(−2) = −1 Esn = 0.114 − 0.25 = −0.136 (A)
N − 6 = −1
N = +5 35. Which of the following statement is in-
For Zn(OH)2− correct ?
4 : Zn+4(−2)+4(1) = −2
Zn − 8 + 4 = −2 (a) The stronger a base the weaker is its
Zn = +2 conjugate acid .
For N H3 : N + 3(1) = 0 (b) The stronger an acid the stronger is
N = −3 its conjugate base.
It can be seen that Zn changes from 0 to (c) The stronger an acid the weaker is its
+2,nitrogen changes from +5 to +3 . conjugate base .
A reductant is the specie that causes ox- (d) The weaker a base the stronger is its
idation .But oxidation is an increase in conjugate acid .
oxidation number . Solution
Since the oxidation number of Zn is in Ans B
the increase Zn is the reductant .
36. Which of the following is not an acidic
Ans B
oxide ?
(a) SO3 (b) P4 O10
32. Given that the half-cell potential of Ag+ /Ag
(c) BeO (D) Cl2 O7
and Al3+ /Al are +0.799v and −1.662v
Solution
respectively.What is the standard cell po-
Oxides of metals are BASIC oxides while
tential for the redox reaction shown be-
that of non-metals are ACIDIC oxides
low
Ans C
Al + 3Ag + → 3Ag + Al3+
(a) 2.461v (b) -0.863v (c) -2.461 (d) +0.863v 37. Which of the following is a Lewis acid ?
Solution (a) Cl− (b) BF3
Standard cell potential = Ecathode −Eanode (c) CN − (D) N H2−
= 0.799 − (−1.662) = 2.441v Ans A Solution
HINT : The most negative always at the A lewis acid has an empty low-lying d-
right hand side orbital which it uses to accept lone pair
of electrons .
33. Which of the following reagents can ox-
Ans B
idize H2 O to O2 under standard -state
condition 38. Which of the following arrangement in
(a) Cl− (aq) (b) Cl2 (g) order of decreasing acid strength is cor-
(c) Cu2+ (d) P b2+ (aq) rect ?
Solution (a) HBrO4 > HBrO3 > HBrO2 > HBrO
Since P b2+ is above hydrogen on the elec- (b) HBrO2 > HBrO3 > HBrO > HBrO4
trochemical series,it follows that it can (c) HBrO > HBrO2 > HBrO3 > HBrO4
displace H in any of its compound . (d) HBrO > HBrO3 > HBrO2 > HBrO4
Ans D Solution
In oxyacidic compounds, the higher the
34. The E0cell for the redox reaction N i/N i2+ //Sn2+ /Sn
number of O atoms ,the stronger acid .
is 0.114v.if the standard electrode poten- Therefore : HBrO4 > HBrO3 > HBrO2 >
tial of nickel is -0.250v,what is the poten- HBrO
tial of the tin half cell ? Ans A
(a) 0.364 v (b) −0.364v
(c) −0.136v (d) +0.136v 39. Which of the following causes the boiling
Solution point of HF to be much higher thab that
Given that S.C.P= 0.14v of HCl or HBr ?
48

(a) coordinate covalent bounds is most likely to form an ionic compound


(b) hydrogen bound ?
(c) covalent bonds (a) magnesium and fluorine (b) nitrogen
(d) van der waals forces and sulphur
Solution (c) oxygen and chlorine (d) sodium and
The Hydrogen bounding in HF accounts aluminium
for its high melting and boiling points Solution
than other hydrogen halides Among the pair of elements listed,only
Ans B magnesium and flourine involve a metal
and non metal pair.The rest are either all
40. Water molecules have a bent shape,based metals or all non metal ,ionic is always
on VSEPR theory.This is because of between a metal and non metal ;
(a) ionic attraction and repulsion Ans A
(b) the ususual location of the free elec-
trons 44. For a molecule with the formula AB2 the
(c) interaction between the fixed orbitals molecule shape is ......
of the unshared pairs of oxygen (a) linear or bent (b) linear or trigonal
(d) repulsive forces between specific elec- (c) linear or T-shape (d) T-shaped
trons Solution
Solution Ans A
Water molecule has a bent shape.According
to the VSERR theory,it is as a result of 45. According to VSEPR theoty,if there are
the repulsive forces between the lone pair five electron domains in the valence shell
and bond pair electrons surrounding the of an atom,they will be arranged in a(n).......
oxygen molecule . geometry
Ans D (a)octehedral (b) trigonal bipyramidal
(c) tetrahedral (d) trigonal planar
41. Which of the following atoms acquires
Solution
the most negative charge in a covalent
According to the VSEVPR theory,if there
bond with hydrogen ?
are 5 electron domain in the valence sheel
(a) Carbon (b) Oxygen
of an atom,they will be arranged in a
(c) Sodium (d) Sulphur .
trigonal bi pyramidal geometry.
Solution
Ans B
Among the elements listed in the option
(carbon,oxygen,sodium and sulphur), oxy- 46. Which of the following has a square pla-
gen has the highest electronegativity value nar molecule geometry ?
hence,it will withdrawn more bonding elec- (a) CCl4 (b) XeF4
trons towards itself . (C) P H3 (d) XeF2
Ans B Solution
42. Which of the following covalent bonds is Ans B
the most polar
(a) C-C (b) C-H (c)C-Cl (D) C-S 47. When H + forms a bond with H2 O to
Solution form the hydronium ion H3 O+ ,this bond
Taking a look at the pair of elements is called a coordinate covalent bond be-
once again ,chlorine is the most electro cause .....
negative element.This means it will with- (a) both bonding electrons come from the
draw more bonding electron from carbon oxygen atom
hence,the higher the polarity . (b) it form an especially strong bond
Ans C (c) the electron are equally shared
(d) the oxygen no longer has eight va-
43. Which of the following pairs of elements lence electrons
0.9. CHEM 101 2013/2014 B 49

Solution
Ans A

48. Why is hydrogen bonding only possible


with hydrogen?
(a) Hydrogen’s nucleus is electron defi-
cient when it bound with an electroneg-
ative atom
(b) Hydrogen is the only atom that is the
same size as an oxygen atom .
(c) Hydrogen is the the most electroneg-
ative element .
(d) Hydrogen tends to form covalent bonds
.
Solution
Hydrogen nucleus is electron deficient when
ever it forms a bond with an electroneg-
ative element hence ,its ability to form
Hydrogen bond
Ans A

49. The Cl-Si-Cl bond angle on the SiCl2 F2


Molecule is approximately ,,,,,,,
(a) 900 (b) 109,50 (c) 1200 (d) 1800
Solution
Ans B

50. The central iodine in IF5 has ....... un-


bonded electron pairs and ....... bonded
electron pairs inits valence shell .
(a)1,5 (b)0,5 (c) 5,1 (d) 4,1
Solution
The central iodine atom in IF5 has i un-
bounded electron pair and 5 boundec elec-
tron pair in its valence shell .
Ans A
50

0.10 GENS 103 2013/2014


General instruction: answer all questions
PART1: Reading Comprehension 10 marks: 1mk for each question
Read all the following passage and answer the questions that follow

No human being can be happy without friends. The human heart is formed for love and cannot
be happy without the opportunity of giving and receiving affection. But you cannot receive
affection unless you give it also. Love can only be obtained by giving love in return. If your
friends do not love you; it is your own fault.
However, we must not forget that there are many types of friends. There are fair-weather
friends who enter into friendship simply because of what they can get from other people. This
category of friends would do anything for their friends for as long as the going is good. They
are ready to call it quits when the tide turns.
There are those people who enter into friendship genuinely . They are quite prepared even to
stake their lives to ensure the continuity of the friendship. They are the friends in need who
are friends indeed. I remember quite vividly a crucial period in my life when one of my friends
came to my assistance. I had just lost my spouse who was to me an indispensable ally . Then,
I was all at sea as to what to do next. This angel of a friend gave me the necessary succor and
encouragement and restored hope to a seemingly hopeless situation. Since that time, I have
learnt my lesson on what true friendship is.
There are thousands of friends who have helped their friends in diverse ways to ascend economic,
business, social and educational ladders. Yet, there are other shameless pretenders who lure
other people to enter into friendship with them when in fact they have some hidden agenda.
Many people have got into various ignoble groups through the clever tricks of such friends.
Indeed, many students have fallen victim to the seemingly innocuous tricks of friends who get
them initiated into cults. So, my friends,watch it!That innocent looking and seemingly gentle
and harmless person may be a devil incarnate. Don’t push into friendship. You better look
carefully before you leap so that you will not regret eternally.

Questions

a Why is it that human beings cannot be happy without friends ?


Answer: human beings cannot be happy without friends because love and affection
needed to be happy can only be given by friends.

b Why does the writer say ”it is your fault if your friends do not love you” ?
Answer: the writer says this because it is only those that show love and affection to
others that get love in return

c What are the characteristics of fair-weather friends?


Answer: fair-weather friends go into friendship for their own selfish interest. And also,
they break up with friends when the friends starts facing challenges.

d What lesson did the writer learn about true friendship?


Answer: the writer learnt that a true friend is he who stood by you during challenges
and problems.

e How do people join ignoble groups?


Answer: people join ignoble groups by trusting tricky friends who deceive them with
lies.
0.10. GENS 103 2013/2014 51

f .....all at sea....

i What does this mean as used in the passage?


Answer: it means the writer was confused

g ” ....who was to me an indispensable ally ”

i What is the grammatical name of the above expression as used in the passage?
Answer: adjectival clause

ii What is its function?


Answer: it qualifies the noun ’spouse’

h Provide a suitable title for the passage


Answer: happiness comes from friends

i State the main advice from the passage in your own words .
Answer: we should be vigilant when it comes to choosing our friends.

PART 2: SUMMARY
Read the following passage and tick the most appropriate option after each
question 10 marks

The family has five basic functions. The first of these is sexual expression, which is con-
sidered an important family function. I America society, as in many others, marriage is
not considered legally binding until it is consummated by the sexual union of the two
marriage partners. A try marriage is not regarded as having existed if the two partners
involved have not had sexual expression.
The second function of the family is child bearing or procreation. In some societies, this
function is considered essential. In many cases, if the wife does not bear a son, the sta-
bility of the family is destroyed. In most cases, it is assumed that the wife is at fault
and may be divorced for not fulfilling her function. Such conclusions may be scientifically
unwarranted but are very real in their social consequences.
Once a child has been born, the child must be brought up. So child rearing is the third
function of the family and it is a prolonged and arduous task. The new born child is to
be reared into a mature and responsible member of the society. Child rearing includes
the tasks of providing for the child’s material needs such as food, shelter and clothing,
teaching it to establish relationships with adults and others of its own age, leading it to
see its position in society and helping it to develop emotionally.
The fourth function of the family is economic. Often the work that has to be done is
divided up according to sex, so that certain tasks are done be men and others by women.
In some societies, the family itself provides most of its own needs as a productive unit. It
is best seen in hunting, gathering and agricultural societies. Through the careful division
of labour, the necessities of food, clothing, shelter and protection are provided within the
context of the family. In the larger technological society the family tends to be more of
a consumption unit. The division of labour is less rigorous. It is not unusual for both
parents to work productively outside the home. As resources are pooled and the members
of the home share in consuming the products and services bought by the members of the
household, the economic function of the family is apparent.
The fifth function of the family is exchange of affection. In many societies where marriages
52

are arranged by parents or others, the exchange of affection follows marriage. Neverthe-
less, it forms an important part of the marriage relationship. At its most desirable level,
the exchange of affection is voluntary, reciprocal, consistent and constant.

Questions

(a) The writer states that in some societies, the sexual union of the two partners ....
A. must come before marriage, b. must not come before marriage,
c. is necessary for the procreation of children, d. makes the marriage illegal, e.
makes the marriage really a marriage.
Answer: E- makes the marriage really a marriage.
(b) In some societies, what (according to the writer) might happen if the wife does not
bear a son?
A. the marriage may break up, b. she may be punished,
c. she will be comfortable, d. she will become unstable, e. she will be treated by a
doctor.
Answer: a - the marriage may break up
(c) Which of the following is not mentioned as part of child rearing?
A. intellectual development, b. social development,
c. emotional development, d. the provision of material needs, e. teaching children
their place in the world.
Answer: a - intellectual development
(d) Where is the family best seen as a productive unit?
A. in America society, b. in hunting, gathering and agricultural societies
c. in the larger technological society, d. in the modern economy, e. in the home
Answer: b- in hunting, gathering and agricultural societies
(e) What makes it clear that the family has an economic function?
A. the fact that goods and services have to be paid for, b. the cost of bearing up
children,
c. the need of parents to work, d. the sharing of goods and services bought by the
family, e. the need for the children to obtain educational qualifications.
Answer: d - the sharing of goods and services bought by the family.
(f) In simple sentences and in your own words as much as possible, state five basic
functions of a family.
Answer: the five basic functions of the family are:
a Family is a medium of satisfying sexual need.
b Family is a legal way of reproduction and bearing of young ones
c Family is also a medium through which a child is reared.
d Family provides the need and wants of its members
e Family in conclusion, provides love and affection for a child.

PART 3: writing 10marks

1. In simply sentences, write out five features of an informal letter below: Answer: they
are:
a. An informal letter has only the address of the writer
b. The language is interactive and chatty c. It starts with ’dear fatima ’ (if that is the
0.10. GENS 103 2013/2014 53

receiver’s name) d. An informal letter is written to friends and some close relations. e.
The complimentary close depends on the relationship between the writer and the receiver.
E.g; two friends can use ’yours sincerely’ or ’your friend ’, with only the first name.

2. In simple sentences, writ out five features of a narrative essay below: Answer: a narrative
essay has the following features:
a. It is a recount of a past experience, real or fictional. b. The tenses are in past forms
c. It has introduction, body and conclusion. d. The narration is divided into paragraphs
e. The name of the writer comes at the right hand side of the narration.

Part 4: extensive reading: newer and never. Text: never and never

1. What genre of literature does the text Never and Never belong?
Answer: drama

2. Where is the setting of the text?


Answer: a university campus

3. What is the central thematic preoccupation of the text?


Answer: cultism in our universities

4. ”finally, I give glory to the Supreme Artist for allowing the inspiration to keep flowing ”
who is referred to as Supreme Artist?
Answer: Almighty God

5. What is the name of the cult group in the text?

6. ”those students who want short cuts to everything. Short cuts to reading, short cuts to
learning, short cuts to passing examinations.” From the text, this is an attribute of the
......

7. What Is the punishment for ” those who have not brought one before now, the next
meeting is your very last chance or else, you lose the ....”

8. Why was Shola told that he was at a point of no return?

9. Any one against the cult is not allowed to ......

10. What did the Dog do to Abu that prompts: ” Never I never and Never ”?

11. The expression ”Capon, this one is gone ” means .......

12. What cause shola’s mental derangement?...........

13. ”we also have other specimen in that bag” what does specimen mean as used in the text?
Answer: human parts

14. What is the total number of cultists discovered in the school as stated in the text?............

15. Who wrote the text ”Never and Never ”?.............. Text: the beggers’ strike

16. ” the Beggers’ Strike” was translated from French to English by ....

17. The title of the novel is either ” the Beggers’ Strike ” or.....

18. ” this morning, there has been another article about it in the newspaper ” what is the
article about?.............
54

19. ” the capital is crying out to be cleared of them.” Identify the figure of speech in this
sentence ....
Answer: personification

20. Who in the novel is the director of the Department of Public Health and Hygiene ....

21. ” this year the number of tourists has fallen considerably, in comparison with last year’s
figure.” What is responsible for this ?.........

22. The whole population of keur Gallo scarcely exceeds......

23. ”Cheiyalla” means ......

24. Who in the novel is Lolli’s husband?.........

25. What is the central theme of this novel?..........

26. ”after those years spent sowing his wild oats, he had learned to appreciate his wife’s good
points.” Who does this describe?

27. ” and just tell me what contract am I tied by that prevents me from taking a second wife,
if I so desire ”. State the response given for this question ......

28. What is the motto of the metal firm in the novel?

29. Did the beggars return to their positions as requested in the text?................
Write out one moral lesson learnt from the text ........

0.11 GENS 103 CA TEST 2013/2014


Answer all questions

1. Write out the sentence patterns of the following sentences e.g the man died (S + V)
(10marks)

(a) He hailed the girl on the stage in faded jeans. ( SVOA )


(b) The audience elected the man president with a wide margin. ( SVOC )
(c) Birds fly (SV)
(d) The hunter killed a lion (SVO)
(e) River Niger is a big river. (SVC)
(f) His father gave him all the money. (SVOI Od)
(g) My husband bought me a brand new car (SVOI Od)
(h) My car consumes fuel a lot (SVOA)
(i) Most drivers drive recklessly. (SVA)
(j) I love Nigeria. (SVO).

2. Group the following nouns under these headings in the box below: love, Ike, Jury, Hassan,
Abuja, Shirt, guidance, family, clock, goodness, army, honesty, goat, pot, club, friendship,
sokoto, Femi, class, book (5marks)

ANSWERS
0.12. GENS 103 2011/2012 55

Common Noun Proper Noun Abstract Noun Collective Noun


Shirt, clock, goat Ike, Jury, Hassan, Abuja, Love, guidance, goodness, Family, army,
pot, book Sokoto, Femi. honesty, friendship club, class
3. Supply the plural form of each of the following nouns (5marks)
Sheep − sheep W olf − wolves Shoe − shoes T ooth − teeth
Japanese − Japanese Goose − geese Knif e − knives Ox − oxen
Stand − by Ox − oxen Half − halves Deer − deers
V alley − vallies P antry − pantries Belief − belief s P asser − by − passers − by
Roof − roof s Alumna − alumnae Lady − ladies Close − up − close − ups
4. Provide the full form of each of the following abbreviations and acronyms (10marks)
(a) NUC - Nigeria Universities Commission
(b) UBEC - Universal Basic Education
(c) NTI - National Teachers’ Institute
(d) OPEC - Organization of Petroleum Exporting Countries
(e) ASUU - Academic Staff Union of Universities
(f) NMA - Nigerian Medical Association
(g) NANS - National Association of Nigerian Students
(h) NAN - News Agency of Nigeria
(i) CNN - Cable Network News
(j) ETC - Etcetera
5. State the grammatical name of the word/group of words in bold in the following sentences
(10marks)
(a) Dr. Akale, the coordinator is my sister........ Answer: noun phrase
(b) The student whose work won the national award schools in A.B.U Zaria....
Answer: adjectival clause
(c) My wife gave me what to eat ........ Answer: noun phrase
(d) I enjoyed teaching my students because they are hard working ......
Answer: Adverbial clause
(e) I will dance as soon as I become a graduate ...... Answer: Adverbial Phrase
(f) Introductory technology is my favourite course ........ Answer: adjectival phrase
(g) The man who married an Indian is a medical doctor .........
Answer: relative pronoun
(h) The thief was arrested yesterday ......... Answer: verb phrase
(i) The dark young lady is very sonsy ........ Answer: Adjectival phrase
(j) My mother cooks well .......... Answer adverb

0.12 GENS 103 2011/2012


SESSION PAST QUESTIONS AND ANSWERS
General instruction: answer all questions
PART 1: Reading Comprehension
Read all the following passage and answer the questions that follow
56

Nagongo is not a large town. The population is probably under 20,000 inhabitants. This small
town is located not far away from the well - known town of Okene also in kogi state. One of
the Magongo’s unique features is the location. A border settlement, close to, between Kogi and
Edo, Magongo is boundary bordered on two sides by rocky hills. It is said that centuries ago
Magongo people usually escaped into the hills when slave raiders struck.

An incident occurred in 1968 during the Nigeria - Biafran Civil War which will always be
remembered in Magongo. There were movements of troops from all over the North to Bendel
and the Eastern States. Magongo a border settlement was naturally a stopping post. When
the company of Nigerian troops arrived, you could see tiredness and terror in their faces. The
major approached the Magongo Chief.

Major: chief. Good evening to you. My men are tired and hungy. We hope we can count
on your full cooperation. We’ve heard much about your place. And I expect good care from
you in ALL respects. I repeat - ALL respect.

Magongo Chief: welcome to my land. Is hall make every effort to attend to your request.
I shall ensure your troops are will - provided for. They will have plenty to eat and enough to
drink.

Major: but don’t forget our other needs. Soldiers cannot live on food and drink alone. Soldiers
too need to relax. Happily even around your place we can see beautiful faces.

Magongo Chief: I promise to take care of you. It is our duty. You soldiers are fighting to
keep Nigeria one. As for the beautiful faces you see I know they are all my subjects. But that
is not to say that I own them all. These beautiful faces have their own soldiers to attend to.
After all, our people say, ”Every man is a soldier”.

Major: well Chief, you have to try. If I may borrow from your wise saying ”Every soldier
is a man”. So make adequate arrangement for troops.

Magongo Chief: officer, I shall do all that is within my power.

That night, the Magongo Chief quietly summoned ”Iya - Egbe” that is, the leader of the
women to his palace. He gave a stern order that until further notice, no woman was to be seen
outside her house. The women were to cook and brew the local beer but all indoors. Young
boys in Magongo, most of them, school boys were rounded up. They were assigned to carry
food and drinks to the ”Ebariki” or ”Soldiers Camp”.

That night is always rememebered in Magongo. It was the night the chief proved that, ”Every
man is a soldier”.

1. Can we say that Magongo had never witnessed anything resembling troop’s movement?
Answer: No. magongo had witnessed troop’s movement in the past, when the slave
raiders used to strike.

2. What is significant about the location of Magongo?


Answer: Magongo’s location is significant in the sense that is a board between kogi and
Edo states.

3. What request did the major make of the chief?


0.12. GENS 103 2011/2012 57

Answer: the major requested for a place to rest, eat and drink in order to regain lost
energy

4. What was the Chief’s immediate response?


Answer: the chief responded in the affirmative that the major’s request will be attended
to by assuring him (the major) of full cooperation.

5. Why was the Major not quite satisfied with the Chief’s assurance?
Answer: the major was not satisfied with the chief’s response because he noticed sarcasm
in the chief’s response. To the major, food and drinks are not enough.

6. What was the chief’s main argument?


Answer: The chief’s main argument is that the beautiful faces around his palace or
under his administration are not totally under his complete control.

7. What was the Major’s reaction tot Chief’s final stand?


Answer: the major’s reaction to the chief’s final stand was that of anger and disappoint-
ment.

8. How did the Chief resolve the imminent crisis?


Answer: the chief commanded that all women in the land should stay indoors. This will
keep them away from the wandering eyes of the soldiers.

9. What was likely to be remembered most in the whole episode?


Answer: the stern order given by the chief the night he summoned ’Iya-Egbe’ for women
to stay indoors was remembered most in the episode.

10. What had brought the soldiers to Magongo in the first place?
Answer: they were on their way to the eastern states since Magongo is a border town,
they had to pass through it.

What do you observe about the following?

11. The magongo Chief’s use of the word ”soldier”


Answer : The chief’s use of the word ’solder’ means husbands of women with beautiful
faces,who might also be warriors that will do anything to defend their wives and children

12. The Major’s use of the word ”man”.


Answer : The major’s use of word ’man’ means that they too (the soldier) need women
to keep them company

13. Provide a suitable title for the passage


Answer : Every man is a soldier

14. ”But don’t forget our other needs ”what is this other need as inferred from the passage?
Answer : Women

15. What literary device is ”Every man is a soldier” as used in the passage?”
Answer : Metaphor

PART 2: GRAMMAR AND USAGE


Read the following short passage, write out the sentences and identify them by their structure.

I’m frequently asked if virtual learning and technologies will eliminate the need for teachers
58

.quite the contrary, I say we need teachers !, We need teachers who can successfully navigate
from the traditional to a transformed venue. We need teachers who can rigorously set the stage
for students’ self-directed, personalized learning - exploiting the power of technology.

ANSWERS

1. I’m frequently asked if virtual learning and technologies will eliminate the need for teach-
ers (complex sentence).

2. Quite the contrary, I say. (simple sentence)

3. We need teachers who can successfully navigate from the traditional to a transformed
venue. (complex sentence).

4. We need teachers who can rigorously set the stage for students’ self-directed, personalized
learning-exploiting the power of the technology. (complex sentence)

PART 3: WRITING
Write out what you think would be your concluding paragraph if you were to write a full blown
essay on the topic; THE DANA CRASH in not more than TEN LINES.

ANSWER

Finally, it is high time we faced the realities and accept our mistakes and do everything within
our power to avoid future re-occurrence. As a matter of urgency, the Dana airline operations
in Nigeria should be immediately stopped and their license withdrawn. All other airline opera-
tors Nigeria should be made and mandated to import only aircraft with international approval
and standards. Also, a thorough and comprehensive investigation should be carried out on all
agencies responsible for taking off and landing of air crafts in Nigeria, and the culprits legally
punished.

PART 4: spelling, the use of dictionary and abbreviations

A Write out five importance of a good Dictionary.

1. It gives the correct pronunciation of English words.

2. It shows the parts of speech words belong to

3. It shows the synonyms and various meanings of words.

4. It also shows how words are used in idiomatic expressions.

5. Finally, a good dictionary tells us the collocations and phrasal verbs of words.

B Write out the full meaning of the following abbreviations and acronyms. UNESCO -
United Nation Education, scientific and Cultural Organisation
CGPA -
ASUU - Academic Staff Union of Universities
CNN - Cable Network News
ASAP - As soon as possible
ETC - Etcetera
E.G. - for example
0.13. GENS 103 CA TEST 2011/2012 59

USAID - United States of America International Development.


NANS - National Association of Nigerian students
NATO - North Atlantic Treaty Organization.

C SPELL OUT THESE BADLY SPELT WORDS CORRECTLY IN BOLD PRONOUN-


CIATION - PRONUNCIATION
BIBLOGRAPHY - BIBLIOGRAPHY
BUSSINESS - BUSINESS
CERTIPICATE - CERTIFICATE
CONTINIOUSLY - CONTINUOUSLY
EXPANTIATE - EXPATIATE
GREATFUL - GRATEFUL
MAINTAINENCE - MAINTENANCE
GUIDIANCE - GUIDANCE
FORIEGNER - FOREIGNER

D USE OF LIBRARY: MATCH THE FOLLOWING WORDS WITH THEIR


MEANING
Shelf - guide, Entry card, Academic library, Library, Catalogue card, Catalogue, gazette
, Annotation, Reserve books and Reference book.

(a) Card on which details about a book are recorded ......


Answer: catalogue card
(b) Instructions especially numbers that show which books can be found
Answer: shelf guide
(c) A building or part which contains books that may be borrowed
Answer: library
(d) Including school, university and research libraries ....
Answer: academic library
(e) A book that is looked at for information Answer: Reference book
(f) Books that are not to be borrowed from the library ..... Answer: reserved books
(g) An official newspaper, especially one from government ......
Answer: gazette
(h) A written note in summary form about a book ..... Answer: annotation
(i) A card that contains the full entry of a book .....
Answer: entry card
(j) A list of books, title of books, author of books put in a special order
Answer: catalogue

0.13 GENS 103 CA TEST 2011/2012


ANSWER ALL QUESTION IN THE SPACE PROVIDED
PART 1: GENERAL INTRODUCTION TO LITERATURE

1. A word used to refer to the choice of words or effective use of words as used in literature
is .......
Answer: diction
60

2. A serious play with an unhappy ending is ..... Answer: tragedy


3. Connotation means ........
a. Literary meaning of a word b. Historical meaning of a word
c. Figurative meaning of a word d. Commercial meaning of a word
Answer: A
4. Literature is a creative work of imagination that affects the human feeling. True/false?
Answer: true.
The three major genres of literature are:
5. .......... Answer: drama
6. ......... Answer: poetry
7. .......... Answer: prose
8. When a writer basically writes about himself, such work is referred to as .........
Answer: autobiography
9. ” A play is a dramatic piece of work drawing upon human activities and presenting them
on stage ” True or False .......
Answer: true
10. Characters are the fictional persons whose stories a novel tells. True or False?
Answer: true
11. Actors and actresses who take part in a play are collectively known as .........
Answer: cast
List four benefits you derive from the study of the two texts in this course
12. .................................
Answer: one derives pleasure from the entertainment
13. ..............................
Answer: one’s linguistic competence is improved
14. ..............................
Answer: moral lessons such as the evil of wickedness is learnt
15. ...............................
Answer: no sinner goes unpunished.
16. A good novelist often employs the appropriate .......... technique in recounting his story.
Answer: Narrative
17. An overstatement deliberately made for artistic impact is called........
Answer: hyperbole or exaggeration
18. The atmosphere of a poem derives directly from its ...........
Answer: dictions
19. The creative writer who writes a play is called ........... Answer :Playwright
20. An author creates and sustains the curiosity of his reader through .......
a. Conflict b. climax c. flashback d. suspense
Answer: suspense
0.13. GENS 103 CA TEST 2011/2012 61

PART 2: SIEGED. NOVEL BY ADAMU KYUKA USMAN

21. What part of speech is the word sieged?...................

22. ”swords are never far from the camp of ...........”

23. PLM as used in the text means ...........

24. ”........... and

25. ..........had over the years worn out the trains until there was no train left. ”

26. ” .......many people liked the idea more by sheer force of its novelity than by any visible
benefits of such means of mobilizing a campaign.” What idea is being described?...............

27. According to Myth, politics in Bivan’s house originated from a society of ..........

28. From the text, eating too much of groundnut oil takes away a ........

29. In the writers view .......... And

30. ......... are cults preying on the society.

31. Why was Jamimi not retained to lecture at the university (his alma mater) despite his
grade?...............

32. Merima was the prime head candidate for the ruling .............

33. ”Siko mee du” was the most popular political slogan used by supporters for different
candidates seeking elections. What does it mean?...................

34. What figure of speech is ” the more the people are given power, the more impotenet they
become” ..........

35. What act of support from the people moved Jamimi to tears?......................

36. What literary device is ”Who has ever eaten iron and survived?.............

37. Which state stood out as a state where politics was based on issues and not sentiments
of tribe and religion?...............

38. When it is raining and the sun is shining from the text, this means ......

39. What caused the rail crash .........

40. Why was Jamimi not on the crashed train?...................

41. The intention of those who caused the crash to happen in Okunno forest was to .......

42. ........ was with jamimi in the hotel room watching the news when they heard about the
crash.

43. Jamimi could not sleep after the news of the crash. Why?.............................

44. ”at this moment of national grief, we must put politics behind us and give the victims of
the crash and their families the sympathy they deserve.” Who said this?................

45. The chief protagonist of the novel SIEGED is ...........


62

46. Boyama collided with an electric pole. Why?.............................


To succeed in politics according to the novel, one needs four things. List them:

47. .............

48. ..................

49. ...................

50. ...............

51. There were two industries in Bivan’s house that at all times were always in business:
politics and .................
PART 3: A MYSTICAL RING ;BY IDRIS MOHAMMED

52. The expression ”you refused to let me in on it” simply means .........

53. The central them of the novel ”a mystical ring” is that of ..........

54. The novel has its setting in the .............. part of Nigeria.

55. Queen Amra’s mother died of .............

56. Who is Amra’s husband ..........

57. The mystical ring is made up of ............ and

58. ........

59. The ring is also a symbol of ..............

60. ...........never liked Amra from her birth.

61. Name the council members requirement always at the time of enthronment of a new king
........

62. From the time the king gave Amra the ring and his eventual death was a span of .......

63. Who was the old farmer that first found mystical ring .........

64. Afra’s favourite bird is ........

65. Abulkhair is a very wealthy arab .........

66. According to the novel, one can only be responsible when one has .......

67. ............ is Ila’s wife.

68. Ila’s letter was delivered to Nabil at Kagu in ......... days

69. The most trusted of all the personal servants of the Queen also turned against the Queen.
Who is this?..........................

70. Kind Abdussalaam’s most trusted friend is .........

71. What in the novel was referred to as a miraculous performance ........

72. What makes the ring mystical?................................


0.14. GENS 103 LVT CONTACT 2012 63

73. From the novel, what does this expression mean in english ” masoyi ka tashi lafiya” ........

74. The mystical ring represents ............

75. .......... is the protagonist of the novel

76. Kogi is a hunter with a reputation for curing illnesses with the use of ........
In two simple sentences, state the moral lessons that can be gleaned from the study of
the novel ...................................

77. .......................

78. .................

0.14 GENS 103 LVT CONTACT 2012


SECTION 1: carefully read the passage below and answer the questions that follow:

a. Researchers say that watching too much TV is as dangerous as smoking or being over-
weight. Anyone who spends six hours a day in front of the box is at risk of dying five
years sooner than those who enjoy more active pastimes, it is claimed. Researchers say
that watching too much TV is as dangerous as smoking or being overweight and that the
’ubiquitous sedentary behaviour’ should be seen as a ’public health problem’.

b. Experts from the university of Queensland, Australia, write: ”TV viewing time may
have adverse health consequences that rival those of lack of physical activity, obesity and
smoking: every single hour of TV viewed may shorten life by as much as 22 minutes”.
Referring to Australian and American guidelines that suggest children should spend no
more than two hours a day in front of a screen, the academics conclude: ”with further
corroborative evidence, a public health case could be made that adults also need to limit
the time spent watching TV.” Although health campaigners - and parents - have long
warned of the dangers of watching too much television, its effects on life expectancy have
never before been calculated.

c. In a paper published in the British Journal of Sports Medicine, Dr. J. Lennert Veerman
and colleagues looked at the results of a survey of 11,247 Australians taken in 1999-2000,
which asked about time spent watching TV, and also mortality figures for the country.
They constructed a model in which they compared life expectancy for adults who watch
TV to those in the top 1 per cent of the population who watch six hours of programmes
a day, they ”can expect to live 4.8 years less than a person who does not watch TV”.

d. The researchers say that watching TV is among the most common forms of sedentary
behaviour, along with sitting in cars. ”because TV viewing is a ubiquitous behaviour
that occupies significant portions of adults’ leisure time, its effects are significant for
overall population health.”

e. England’s Chief Medical Officer, Sally Davies, said: ”physical activity offers huge benefits
and these studies back what we already know - that doing a little bit physical activity
each day brings health benefits and a sedentary lifestyle carries additional risks. That’s
why the UK’s Chief Medical Officers recently updated there advice on physical activity
to be more flexible, right from babyhood to adult life. Adults, for example, can get their
150 minutes of activity a week in sessions of 10 minutes or more and for the first time
we have provided guidelines on reducing sedentary time. We hope these studies will help
64

more people realize that there are many ways to get exercise-activities like walking at a
good pace or digging the garden over can count too.”

f. Maureen Talbot, senior cardiac nurse at the British Heart Foundation, said: ” sedentary
behaviour succeed verging in front of the TV is practically a cultural institution these
days and it’s good to relax for a while, this study supports the view that too much of it
can be bad for our health. ”many of us make a consciouse decision not to smoke because
we know it’s bad for us, and this study suggests that more of use should make the same
kind of pledge about lounging around and watching lots of TV introducing more activity
to our daily lives, whether it’s walking to the shops instead of taking the bus, using the
stairs instead of the lift or taking up active hobbies like sport or gardening mean we won’t
spend as much time in front of the TV where we are likely to pile on the pounds”.

g. Too much time sitting and watching TV is taking years off your life expectancy, another
study says, researchers arrived at the conclusion from the results of five studies that
explored the effects on nearly 167, 000 people of sitting and watching television.

h. Peter Katzmarzyk, the study’s lead author and a professor at the Pennington Biomedi-
cal Research Centre in Baton Rouge, La., said ”......the study doesn’t establish a direct
cause-and-effect relationship between sitting, TV watching and death. But the evidence
suggesting an association between shortened lives and sedentary activities, like TV watch-
ing and driving, is piling up.” There also seems to be something about sitting itself that
is bad for one’s health. Studies in both animals and humans have found that sitting leads
to changes in resting glucose levels and blood pressure, and that lots of sitting bumps up
levels of certain biomarkers of cardiovascular disease and cancer.

i. ”the take-home message is clear: we may not know exactly why sitting is bad for you,
but if you reduce the amount of time spent sitting, there are real health benefits,” the
researchers said, Katzmarzyk said since many people spent at least eight hours each day
sitting at a computer, the workplace is an ideal place t start looking for ways to reform
behaviour. People can get up to talk to colleagues instead of emailing them, or spend
a few minutes of their lunch breaks taking a short walk. And of course, a good place
to start making changes is by squeezing the recommended 30 minutes of exercise into
everyday.

Attempt the questions below (1-20) 20 mks

According to the passage, the two other conditions that are equated with spending to much
time watching the TV are (1).......... Answer: smoking and (2).......... Answer: overweight
the ”ubiquitous sedentary behaviour” as used in the passage means (3)....... Answer: wasting
too much time watching TV or sitting in one position. and (4)........... Answer: becoming
overweight in the processes due to lack of exercise .
Four activities, as seen in the passage, that can keep us away from spending too much time in
front of TV are (5)........ Answer: taking a walk (6)........ Answer: gardening (7)......... exer-
cising (8)........Answer: go straight and tell the people instead of emailing them. (9) watching
TV for five hours in a day, according to the research, may shorten life by .........minutes. An-
swer: 110 minutes (10) one major factor that links driving and watching TV is ......... Answer:
they both involve sitting for long hours (11) how many minutes of activity are recommended for
a week?......................... answer: (12) ........ Answer:sitting for too long leads to changes in
glucose level and blood pressure. and (13) ........ Answer: Leads to diseases like cardiovascular
arrest and cancer , are the negative effects of being on a seat for too long, according to the
passage. 14) one bad/negative habit that we decide not to engage in is ......... In paragraph f,
0.14. GENS 103 LVT CONTACT 2012 65

(15)........ Answer: smoking (16) ...... Answer: walking (17) ......... Climbing the stairs and
(18) ......... Answer: gardening are suggested as forms of remedy that can distract us from
watching too many TV.
(19) what lesson can we learn from this study?........................ Answer: sitting in one position
fro long has health hazards. (20) suggest an appropriate title for the passage ......... Answer:
sitting for long is dangerous as smoking.
SECTION 2: Time indication (21-32) 12mks
Study the table below very carefully and provide answers to the time indication
Past Present Future
(E.g) Morning (E.g) yesterday morning (E.g) this morning (E.g) Tomorrow Morning
Afternoon (21) yesterday (22) this afternoon (23) next afternoon
Night (24) last night (25) tonight (26) tomorrow night
Day (27) yesterday (28) day (29) tomorrow night
Week (30) last week (31) this week (32) next week
SECTION 3: spelling (33-40) 8mks
Below are some words that are spelt wrongly. Rewrite them appropriately to answer questions
33-38.

33 Acomodation ⇐⇒ accommodation

34 Previledge ⇐⇒ privilege

35 Begining ⇐⇒ beginning

36 Reciept ⇐⇒ receipt

37 Aknowledgment ⇐⇒ acknowledgement

38 Mosquetos ⇐⇒ mosquitoes

39 Commite ⇐⇒ committee

40 Pronounciation ⇐⇒ pronunciation
SECTION 4: Essay Type (41 - 44) 4mks
Read the following extracts carefully and answer the questions that follow (Question 38
- 41)

i. if asked what the best benefit of the military is, a soldier and his family will probably
answer that it’s the job security. A soldier does not have to worry about getting fired:
they bring home a guaranteed pay check that does not depend on hours. In the civilian
world, even garnering a job doesn’t guarantee a paycheck. However, a soldier cannot quit
his job if he doesn’t like it. whereas a civilian is free to leave their job at will.
Answer: expository

ii. He was a typical African man in look and structure. His house was built of bricks, painted
green with the only one door at the side leading to the narrow path beside. The space
between the house and the main road was so narrow that one car had to park for the
other to pass. The passers-byes looked at the small but beautiful house with lots of
admirations.
Answer: descriptive
66

iii. She was so happy that she found if difficult to smile, let alone, laugh out loud. Yet that
was what the turn of events would have led her to execute. If it had happened, the
execution would have been the first of its kind she had ever experienced
Answer: narrative

iv. in writing a formal letter, the address of the writer is always at the top right hand side,
followed by that of the receiver at the left hand side, and then the title comes after
the salutation. The body of a letter is generally divided into three parts which are the
introduction. The body and the conclusion. The complimentary close that contains the
signature of the writer comes last.
Answer: expository/descriptive

41 passage i is an instance of ........essay. Answer: argumentative

42 passage ii is an instance of .........essay. Answer: descriptive essay.

43 Passage iii is an instance of ....... essay. Answer: narrative essay

44 passage iv is an instance of .........essay. Answer: expository


SECTION 5: Question tag (45-50) 5mks
write out the correct question tag for the following sentences.
e.g Question: it is a big rat. Answer: isn’t it?

45 Dr. jonathan is the Nigerian President isn’t he?

46 They were accused of big fraud weren’t they?

47 She will not write the examination will she?

48 You eat a kilo of meat daily don’t you?

49 I have not travelled outside Zaria this year have I?

50 Gladys must not travel tomorrow must she ?


SECTION 6: Grammar (51 - 55) 5mks
Write out the past tense and past particle of the following words
Present tense Past tense Past participle
(51) Arise Arose Arisen
(52) Bleed Bled Bled
(53) Go Went Gone
(54) Cut Cut Cut
(55) Drive Drove Driven
SECTION 7: Abbreviations (56-60) 5mks
Write out the full meaning of the following abbreviations and acronyms.

56 I.E ......... ans: that is

57 NNPC ......... answer: Nigerian National Petroleum Corporation

58 GSM........ Global System for mobile communication

59 SMS ........ Short Message Service

60 S/N ........ Serial Number


0.15. GENS 103 2008/2009 67

0.15 GENS 103 2008/2009


General Instruction: Answer all questions
SECTION A: passage A: read the passage below and answer the questions that follow

1. children have become much more interested in cartoons over many years and it has become
a primary action to some lives. Typically, children begin watching cartoons on television
at an early age of six months, and by the age of two or three, children become enthusiastic
viewers. This has become a problem because too many children are watching too much
television and the shows that they are watching (Even if they are cartoons) have become
violent and addictive.

2. The marketing of cartoons has become overpowering all over and so has the subliminal
messaging. The marketing is targeted towards the children to cause them to want to
view the cartoons on a regular basis, but the subliminal messaging is for the adults’ to
target them into enjoying the ”cartoons”. This is unfortunate because children watch
the cartoons on the television and they see material that is not appropriate for their age
group. The children who watch too much cartoons on television are more likely to have
mental and emotional problems, along with brain and eye injuries and unexpectedly, the
risk of the physical problem increases.

3. From the time children start school to the time that they graduate they are averaged to
spend around 13,000 hours in school. This may seem like an awful lot of hours to attend
school unless it is compared to the hours a child television, which is nearly 18,000 hours
(from the time school is started to the time of graduation). This comparison is an outrage
because of the amount of television that is watched by a child will have an effect on their
brain, emotions and their sense to feel pain. In a 2000 report on adolescent violence, the
U.S. surgeon General David Satcher stated that more aggressive behaviour in a young
child’s life is caused by frequently watched entertainment that incorporates violence in it.

4. This has become a public health issue and because of the research findings; the American
Psychological Association passed a resolution in February of 1985, informing broadcasters
and the public about the dangers violence on the television has on children. Three major
effects have been proven by psychological research caused by children seeing violence on
television are that the child may become less sensitive to the pain and suffering of others;
children who watch violence do not fear violence nor are they bothered by violence in
general and the children are more likely to become aggressive or use harmful actions
towards others.

5. When we are born , we have the capacity for motivation, experience, and training, and
because of this, our minds re very impressionable. Therefore, our brains’ development is
a dynamic mix of nature and nurture, so it is important to choose a healthy environment
for the children. This means cartoons with violence will be unhealthy for child because in
general, being interactive with any environment enhances the development of a successful
brain. As a result, a tremendous amount of childhood involvement with electronic media
can limit social interaction and may obstruct the development of a brains’ social systems.

6. In December 1997, an episode of the Japanese carton ”pocket Monster” (later rename
”Pokemon” for international distribution) drew world wide attention after multiple cases
of children suffering seizures after watching the episode were reported (Warner, 2004).
Parents began to wonder how the cartoons their children watched affected their mental
development. While no former study specifically relating to cartoons has taken place,
68

multiple studies over the years have charted the impact of television on the minds and
eyes of developing children.

7. Most eye specialists agree that watching television is not a danger to the eyes, as long
as the children watch in the right conditions. The room should not be pitch black, and
children should not sit closer than five feet away from the screen. Sitting in a dark room
or closer than five feet will not damage the eye but will result in eye fatigue. (Adams,
1992).

8. As for the brain, there is scientific evidence that too much television can be detrimental
to children. The April 2004 issue of the medical journal Pediatrics published a study
done by Children’s Hospital and Regional Medical Center of Scattle, Washington. The
study revealed that children who watched three to four hours of television daily had a
30 to 40 percent greater risk of developing attention deficit disorder than children who
did not watch television. While no specific program is directly responsible, Dr. Dimitri
Christakis, leader of the study, speculates that the speed of the images displayed could
affect children’s brains (Today’s Chiropractic, 2004).

9. But does watching television give young children seizures? Yes, and no. a study released
by the New England Journal of Medicine in july 2004 found that most children who
suffered seizures from that December 1997 episode of ”pocket Monsters” had epilepsy,
or some other underlying condition that would have caused development of seizures,
regardless of whether or not they saw that program (Warner, 2004).

10. Today in many children’s cartoons you see cartoon characters jumping, diving and falling
from very high heights, then landing without being harmed. Parents seem to be happy
with this as long as the cartoon doesn’t promote sex or any kind of violence. But, are
these types of cartoons really ok for your children to watch? On every episode of Loony
Tunes you will probably see a short clip of Wile E. coyote trying to catch the Road
Runner. And in every clip , Wile E. Coyote in some shape or form seems to fall off a
cliff or simply have one of his inventions back fire and cause a mass explosion. Yet, he
manages to collect his teeth, dust himself off and get up for another attempt to catch the
Road Runner.

11. Another example of this sort of cartoon would have to be Bugs Bunny and Elmer Fudd.
In these cartoons it always seems to be rabbit hunting season and Elmer Fudd can never
seem to shoot that ”dang ol’ rabbit.” When he tries, Bungs usually finds a way to either
bend the gun to aim at Elmer or to plug it up with some kind of object. But either
way, the guns always fire into Elmer’s face. I could go on about numerous cartoons that
display these kinds of characteristics. But, what I’m trying to get at is that these sorts of
cartoons that are displaying false sense of reality upon children. Being able to fall off a
tall cliff or being shot with a gun in the face and walking from these incidents with barely
a scratch.

12. At a young age, this false sense of reality can really affect them. It has been proven that
children on average watch 4 hours of television a day. And because of this false sense of
reality ”for every hour of TV viewed per day, the risk of injury rose by about 34% in the
children studied ” (Website) as a child growing up, I know I looked up to and wanted
to be like the super heroes in the cartoons I watched. I would imagine that it would be
the same today. That’s why the false sense of reality that cartoons show may encourage
children to try things that they see their favourite super hero do.
0.15. GENS 103 2008/2009 69

13. In some cases, TV has also been linked to causing seizures. ”either high- speed flashes of
light or rapid color changes are thought able to induce seizures in vulnerable individuals”
(Website). They found this to be the reason for so many seizures in Japan. ”Rapid
changing stimuli can play havoc with the special cells in the retina called rods and cones
that help the eye transmit visual information to the brain.”(Website) so just because
the cartoons your children are watching don’t show sex or violence doesn’t mean they
are innocent. Because of this false sense of reality children at young age can’t tell the
difference between cartoons and realism. An increase of 34% isn’t a small jump when
talking about injuries to children.

14. Television has long been criticized for influencing our children. People complain that
certain TV shows are having negative effects on their children. The American Academy
of Pediatrics (AAP) and the American Academy of Child and Adolescent Psychiatry
(AACAP) both feel that TV does influence the behaviour of children as young as one
year old.

15. From their studies, the AACAP states, ”children who view shows in which violence is
very realistic, frequently repeated or unpunished, are more likely to imitate what they
see. This speaks to the impressionable mindsets of young children, who are still learning
control of their minds and bodies, and are likely to mimic what they see, as it seems quite
normal to them. The AACAP also stresses the need for parents to keep a close eye on
what their children watch. They must be there, but would actually be seriously injured,
or die in real life. They should also work to tell their children that violent behaviour is
not the best course of action to resolve a conflict.

16. The APP states ”Neuroscientists have shown that environmental experiences significantly
shape the developing brain”. This again adds to the idea that young children are very im-
pressionable. They paid more attention to the effect of TV on children in their daily lives.
”Higher Levels” the AAP states, ” of television viewing correlate with lowered academic
performance, especially reading scores. This may be because television substitutes for
reading practice, partially because the compellingly visual nature of the stimulus blocks
development of left hemisphere language circuitry. A young brain manipulated by jazzy
visual effects cannot divide attention to listen carefully to language”.

17. TV is a very quick medium. Messages are shot at the viewer as if by an automatic rifle.
Their minds must be equally as quick to interpret the messages, and with such a ”two -
minute mind”, many messages are misinterpreted or confused. When the child becomes
used to receiving information at so fast a rate, they lose interest in information that Is
more detailed and methodical, such as information received in day-to-day schooling.

18. Television certainly does affect our children, who find themselves mesmerised by the bright
flashing objects, and rapid assault of messages. It is good to know that the leader of our
medical professions feel that parents and supervisors of children are able to help slow
the information down, and explain what the messages really mean, so s to have a more
positive effect on our children.

19. Children’s cartoons are packed with many controversial topics such as violence and sex-
uality. These messages are not always purposely placed in the cartoon, but instead are
sometimes the result of an oversensitive parent or misunderstanding. However, many
parents would be shocked to learn that cartoon makers are intentionally brainwashing
their children secretly placing messages never to be deciphered by the conscious mind,
but instead propagating the subconscious, into behaving a certain way.
70

(source: an article entitled wait a minute, do you know what cartoons do to your kids?
Written by C.J Choma and published at page 48 of the spectator a weekly Nigerian News-
paper of august 7-13 2009)
Now answer the following questions

1 According to paragraph 8, the effect of television on the brain is......


Answer : The images cause attention deficit disorder

2 True or false? Watching cartoons could cause seizures ......... Answer : True

3 Watching TV in a dark room cause....... Answer : Eye fatigue

4 True or false? Children who watched ’Pokemon’ suffered seizures .....Answer True

5 According to the passage, the two conditions under which children should watch are ......

6 In a light room

7 a distance of not less than 5 feets from the tv


according to the passage, the two effects on children of to much contact with the electronic
media are: .......

9 Motivation

10 ’Nature’ and ’nurture’ in the passage means ......... Answers : Motivation


The three main effects of violence shown on TV on children are:

11 They become less sensitive to the pain and suffering of the others

12 They do not fear violent situations

13 They may become more aggressive

14 Summarise the U.S Surgeon General David Satcher’s report in not more than ten words
........
Answer : The speed of images can cause brain problems in children.

15 Summarize the import of paragraph 2 in one sentence of not more than ten words .........
Answer :The selling of cartoons has overpowered the violent messages therein

16 Why, according to paragraph 1 is watching of cartoons by children a problem? (not more


than ten words) .........
Answer : Children have developed too much interest for cartoons

17 Suggest an alternative title for the passage ..........Answer Children and cartoons

18 ”that program ” the last phrase of paragraph 9 refers to .......... Answer Pocket monster

19 The main gist of paragraph 12 is ........(not more than ten words) Answer : Children
imitating their favourite character in real life
0.15. GENS 103 2008/2009 71

20. Which of ”nature or nurture” dichotomy does the argument in paragraph 15 support?
Answer :Birth and training

21. According to paragraph 16, the effect of television viewing on reading is ........
Answer :It impedes their understanding of language 22. True or false? The messages
packed in cartoons are deliberate on the part of the producers ..........
Answer :True

22. Summarize the passage in one sentence


Answer : Cartoons are detrimental to the psychological and emotional development of
children.
SECTION B TASK 1: Read the following passages carefully and answer the questions
on them.

A It was two weeks after Gladys had told her husband of her plan to nail Lati Baba. During
those two weeks, she and Peter had made numerous visits to Abata before they finally
decided on a suitable accommodation for the hotel.

B He was well-dressed up and shaven to look as young as possible, his suit exotically per-
fumed. Even without his B.M.W, anyone would swear he was a money bag. His authentic
Omega watch was exposed enough to get quick attention .

C To make Adire cloth requires a number of items like dye, thread, pot and water

D Polygamy unlike one man one wife encourages strife within the family; while children
born into monogamous families experience relative harmony, it seems that the norm for
children of polygamous parents is constant unease.

23. Passage A is an instance of .........essay type.Answer : Narrative

24. One word or phrase that supports the answer in question 24 above is ........
Answer :During those two weeks

25. Passage B is an instance of ........essay type Answer : Descriptive

26. One word or phrase that supports the answer in question 26 above is ...... Answer :
He was well dressed - up

27. Passage C is an instance of ........essay type. Answer : Expository .

28. One word or phrase that supports the answer in question 28 above is ....... Answer :
To make attire ...

29. Passage D is an instance of ........type of essay. Answer : Argumentative

30. One word or phrase that supports the answer in question 28 above is ....... Answer :
Polygamy unlike one man one wife
SECTION C: Acknowledgement of sources
For all the tasks in sections A and B above, compile a bibliography entry using the MLA
style sheet. The entries should be provided in the spaces below.

31. ............................

32. Wale Okediran.The Boys at the Border.Ibadan : spectrum Books Limited .1991
72

33. Joseph Wanshe.Presidential Candidate (p.79) Makordi:vally fiction.2009 .

34. ......................

35. Taiwo Abioye. et al.The Marriage Practices of Nigeria Ethnic Groups .Ikeja,Lagos:Go
Upper Books Limited.2007.reprinted :Selling poiny Books .2008
List ten abbreviation symbols for documentation and give their full meanings

36. .................................

37. ........................................

38. ..........................

39. .............................

40. ................................

41. .......................

42. ..........................

43. ..........................

44. .........................

45. .........................
SECTION D: Spelling
Below are groups of words with only one word correctly spelt in each group. Underline
the word that is correctly spelt:
A B C D
47.onomatoepia onomatopoeia onomatopia onomatopie
48.ambigous ambigouse ambiguous ambigouss
49.nuemonia pnewmonia pneumonear pneumonia
50.adolescence adolesense adolecense adoliscence
51.pronounciation prononciation pronounsiation pronunciation
SECTION E: Literature: the boys at the border
A: ”Doc, please...can.....can ......i see him?”
B: ”Sure, come along”

52 Speaker A in the discourse above is ..........

53 ’Him’ in A’s utterance refers to ............

54 Lati Baba is the head of a formally registered trade organization named .......

55 Emeka Emodi ........ years older than his wife, Gladys

56 .......... is the name of Lati Baba’s medicine man ”NO, no this can’t be true,” Adepoju
said shaking his head as soon as that bit of news sank in.”

57 ..............

58 Adepoju was in the company o ........... ” This is a shame,” he said as he soon revealed
the contents of the sac.”
0.15. GENS 103 2008/2009 73

59 .........was the content of the said sac.

60 The content was that of .......the son of .........

61 Prof. Baki is a ........ By specialization.

62 Prof. Baki compared the sound of Chi Chi the medical student’s name to a ........

63 ........is the anaesthetist who attended to Mr. Emodi

64 Alhaji Jibo gave the sume of .........counterfeit cash to Mrs. Emodi

65 .is the name of the hotel set-up by Gladys.


74

0.16 MATHS 103 2010/2011 Solution


Recall,l2 = (x + y)2 + (y + f )2 − r2
1. Evaluate√tan 150 to √
get √ (x, y) = (−3, 3); r = 5; (g, f ) = (−2, +3)
(a) 1√+ 2 (b) 1√− 3 (c) 2 − 3 l2 = (−3 − 2)2 + (3 − 3)2 − 25
(d) 3 − 3 (e)2 3 = 25√+ 36 − 25
Solution √ l = 36
tan 15 is 2 − 3(C) using calculator l = 6(E)

2. Find the value of 7. The tangent line to the circle at the point
tan(1200 − θ) + tan(600 + 0) (−2, 0) has slope :
(a) 20 (b) 0 (c) 1 (d) 2 (e) 1.5 (a) 4/3 (b) 2/3 (c) 2 (d) 1/3 (e) −3
Solution Solution
tan(120 − θ) + tan(60 + θ) Let m,be the gradient between the centre
N.B tan(2α − θ)+tan(α + θ is 0 (2, −3) and the given point (−2, 0)
∴ tan(120 − θ) + tan(60 + θ) is 0(B) 0 − (−3) −3
M1 = =
−2 − 2 4
3. A straight line makes equal intercept with −1 4
for tangent m2 = = (A)
the coordinate axes and passes through m1 3
the point (1, 1/2),what is its equation
8. The equation to the tangent line to the
(a) 2x + 2y = 3 (b) x + y = 10 (c) x+y=8
circle at the point (6,0) is
(d) 2x − 3y = 10 (e) x − y = 5
(a) 4x + 2y − 23 = 0 (b) 2x + 4y − 21 = 0
Solution
(c) 3x − 4y + 1 = 0 (d) 3x + 4y − 18 = 0
The formulae for a straight line with equal
(e) None of the above
intercept is
x y 1 Solution
+ = 1 where (1, ) = (a, b) Recall equation of tangent is
a b 2
x y xx1 + yy1 + g(x + x1 ) + f (y + y1 ) + c = 0
+ 1 =1
1 2
(x1 , y1 ) = (6, 0),g=-2 f=3 c=-12
x + 2y = 1 No correct option. 6x − 2(x + 6) + 3(y) − 12 = 0
6x − 2x − 12 + 3y − 12 + 0
Given the circle x2 + y 2 − 4x + 6y = 12 4x + 3y − 24 = 0(E)
answer question 4-10 9. The circle touches x2 + y 2 − 2x − 4y = 36
at the point :
4. The center of the circle is the point
(a) (7,3) (b) (−3, −3) (c) (7, −3)
(a) (3,2) (b) (−4, 6) (c) (2, −3)
(d) (1,2) (e) (4,3)
(d) (1, −2) (e) (1, 3)
Solution
Solution
Given x2 + y 2 − 2x − 4y − 36 = 0
x2 + y 2 − 4x + 6y = 12
By inspection i.e substituting all the op-
x2 + (−2)2 + y 2 + (3)2 = 12 + (−2) + (3)2
tion into the above equation only option
(x − 2)2 + (y + 3)2 = 12 + 4 + 9
(B) (-3,-3) will give 0
(x − 2)2 + (y + 3)2 = 25
i.e (−3)3 +(−3)2 −2(−3)−4(−3)−36 = 0
∴ (x − a) + (y − b)2 = r2
or from
(a = 2, b = −3) = (2, 3)(C)
x2 + y 2 − 4x + 6y = 12 − − − −i and
5. The radius of the circle is x2 + y 2 − 2x − 4y = 36 − − − ii ,
(a) 8 (b) 10 (c) 15 (d) 12 (e) 4 eqni − eqnii = −2x − 10y = −24
Solution √ x = 5y + 12
Radius r = 25 = ±5(C) sub x = 5y + 12 into eqni
(5y + 12)2 + y 2 − 4(5y + 12) + 6y = 12
6. The tangent drawn from point (−3, 3) to 25y 2 + 106y + 84 = 0
the circle is ...... units y = −1.1 ; or y = −3
(a) 8 (b) 13 (c) 4 (d) 12 (e) −3 when y = −3
0.16. MATHS 103 2010/2011 75
 
x = 5(−3) + 12 = −3 x1 x2 x3
1
= (−3, −3)(B) Area =  y1 y2 y3 
2
1 1 1
1
10. The sum of squares of the radii of the area = (1(−1 + 3) + 3(−3 − 3) + (−3)(3 + 1))
given circle and x2 + y 2 − 2x − 4y = 36 is 2
1 −32
(a) 36 (b) 66 (c) 41 (d) 46 (e) 54 = (−2 − 18 − 12) = = 16.
2 2
Solution No correct option
From x2 + y 2 − 4x + 6y = 12 ; r1 = 25 0
from x2 + y 2 − 2x − 4y = 36 14. The value
√ of cos √75 − r cos 150 is
2 3 3 3
(x − 1)2 + (y − 2)2 = 41 r2 = 41 (a) − (b) (c)
r1 + r2 = 41 + 25 = 66(B) √3 2
√ 2
2 2
(d) − (e) −
3 2
11. The angle between the lines 3x − 3xy + Solution
17 = 0 and x − y − 3 = 0 is cos750 − cos15
π π π A+B A−B
(a) 0 (b) (c) (d) (e) π using cosA−cosB = −2sin sin
6 2 3 2 2
Solution cos75 − cos15 = −2sin45sin30
3x − 3y + 17 = 0 1 1
= −2 × √ ×
3y = 3x + 17 2√ 2
17 −1 − 2
y =x+ m1 = 1 =√ = (E)
3 2 2
x−y−3=0
y = x − 3 m2 = 1 15. What is the value of the angle between
Recall Angle between two line is the pair of lines x2 + xy − 6y 2 = 0?
m2 − m1 π 2π π
tanθ = (a) ((b) (c) (d) π (e) 0
1 + m1 m2 4 3 2
tanθ = 1−1
2
=0 Solution
−1
θ = tan 0 = 0(A)

16. Find the value of x if,


π
tan−1 (1) + tan−1 3x =
12. The length of a perpendicular√drawn from 2
the origin to the line l is 5 2 unit.The (a) 1 (b) −1/6 (c) 0
perpendicular makes an angle of 450 with (d) 1/3 (e) Not defined
the axis.The equation of l is Solution
√ π
(a) x−y = 6 (b) x+y = 50 (c) x+y = 2 T an−1 1 + tan−1 3x =
2
(d) 2x + y = 1 (e) x+y= 10 Let α = tan−1 1 ∴ tan α = 1
Solution and β = tan−1 3x
The formulae is xcosθ√ + ysinθ = l tanβ = 3x
xcos45 + ysin45 = 5 2 tanα + tanβ π
x y √ tan(α + β) = =
√ +√ = 2 1 − tanαtanβ 2
2 2 1 + 3x π
x + y = 10 =
1 − 3x 2
1 + 3x
= 90
13. The area in square units of a triangle 1 − 3x
whose vertices (1, 3), (3, −1), (−3, −3) is 1 + 3x = 90 − 270x
(a) 36 (b) 19 (c) 11 (d) 14 (e) 13 273x = 89
273 1
Solution x= = (D)
89 3
Given(x1 , y1 ) = (1, 3)
(x2 , y2 ) = (3, −1)(x3 , y3 ) = (−3, −3) 0

17. if tan(A
√ + 60 ) = 3 then cot A is
Area of a triangle is given by area =
1 (a) −√ 3 (b) 2/3 (c) 4/3
(x1 (y2 −y3 )+x2 (y3 −y1 )+x3 (y1 −y2 ))Or (d) 3 (e) Undefined
2
76

Solution √ (a) tan θ (b) − cot θ (c) − tan 2θ


T an(A + 60) = √ 3 (d) − tan 2θ (e) sin 21 θ
A + 60 = tan−1 3 Solution
A + 60 = 60 sin3θ + sinθ
A = 60 − 60 = 0 cos3θ − cosθ
A+B A−B
∴ cotA = cot0 using SinA+SinB = 2Sin cos
1 2 2
= sin3θ + sinθ = 2sin(2θ)cosθ
tan0
1 similarly,
cotA = , undef ined(E) cos3θ − cosθ = −2sin(2θ)sinθ
0
sin3θ + sinθ −2sin2θcosθ
then, = = −cotθ(B)
18. If A(1,3),B(3, −1) and C(−3, −3) are ver- cos3θ − cosθ 2sin2θsinθ
tices of a triangle ,then tan A is 7π
(a) 7/4 (b) 15/4 (c) 2/3 (d) −3/4 (e) 5/3 21. radians in degree is equal to
3
Solutionp (a) −1200 500 (b) 3000 (c)4200
/AB/ = (−1 2
√− 3) + (3 − 1)
2 (d) 3300 (e) −1300 220

= 16 +p 4 = 20 Solution
7 × 180
/BC/ = (−3√− 1)2 + (−3 − 3)2 = 4200 (C)
√ 3
= 16 +p 36 = 52

/AC/ = (3 +√3)2 + (1 + 3)2 −1 5
√ −1
22. The value of tan 3 + cosec is
= 36 + 16 = 52 2
Using cosine Rule π π
(a)π (b) (c)
b2 + c2 − a2 2 3
CosA = 3π
2bc (d) (e) none of the above
52 + 20 − 52 4
cosA = √ √ Solution √
2 52 × 20 5
10 10 5 tan−1 3 + cosec−1
=√ = √ = √ 2
1040 4 65√ 2 65 α = tan−1 3 √∴ tanα = 3
1 2 65 5
secA = = β = cosec−1 ∴
cosA 5 v √ 2
u √ !2 5
√ u 2 65 cosecβ =
tanA = sec2 A − 1 = t −1 2
5 2
sinβ = √ ∴ tanβ = 2
5
r r
52 47
−1= tanα + tanβ
5 5 tan(∝ +β) =
No correct option. 1 − tanαtanβ
3+2 −6
cos x tan(∝ +β) = =
19. tan x + simplifies to 1−6 5
1 + sin x −1 −6
(a) cot x (b) sin 2x (c) sec x α + β = tan = 50.19(E)
5
(d) cosecx (e) cos 2x
Solution
sinx cosx 23. The length of the diameter of the circle
+ 4x2 + 4y 2 − 4x + 8y − 31 = 0 is
cosx 1 + sinx
sinx(1 + sinx) + cos2 x (a) 11 (b) 8 (c) 6 (d) 9 (e) 12
cosx(1 + sinx) Solution
sinx + sin2 x + cos2 x 4x2 + 4y 2 − 4x + 8y = 31
31
cosx(1 + sinx) x2 + y 2 − x + 2y =
sinx + 1 1 4
= = secx(C) (x − 21 )2 + (y + 1)2 = 31
4
+ 14 + 1
cosx(1 + sinx) cosx 1 2 2
(x − 2 ) + (y + 1) = 9
sin 3θ + sin θ r2 = 9
20. simplifies to r = ±3
cos 3θ − cos θ
0.16. MATHS 103 2010/2011 77

diameter = 2r Solution
2 × 3 = 6(C)

7 28. The least non-zero value for 0 ≤ x ≤ 180


24. Given that tan θ = , θ is in the first
24 in the solution of sin 3x − sin x = 0 is
quadrant then 5 cos θ − 10 sin θ = (a) 1800 (b) 450 (c) 22.50 (d) 300 (e) 900
(a) −3 (b) 2 (c) −2 (d) 4 (e) 3 Solution
Solution Recall sin3x = 3sinx − 4sin3 x
7 24
If tan θ = then cos θ = F rom question sin3x − sinx = 0
24 25
7 ∴ 3sinx − 4sin3 x − sinx = 0
also sin θ =
25 2sinx − 4sin3 x = 0
24 7 2sin = 4sin3 x
5 cos θ − 10 sin θ = 5 × − 10 × 1
25 25 sin2 x =
120 70 50 2
= − = = 2(B) 1
25 25 25 sinx = ± √
2
25. The area in squares units of the quadri- 1
lateral whose angular points are x = sin−1 ( √ ) = 450 (B)
2
(0, 1), (1, 3)(2, −3) and (3,1) is
(a) 6 (b) 9 (c) 10 (d) 7 (e)8 29. The perpendicular distance from the point
Solution (5, −2) to the line 24x + 7y + 19 = 0 is
A(0,1)=(x1 , y1 ), B(1,3)=(x2 , y2 ) (a) 1 (b) 2 (c) 3 (d) 4 (e) 5
C(2,-3)=(x3 , y3 ) D(3,1)=(x4 , y4 ) Solution
ax1 + by1 + c
Area of quadrilateral is given by: Using the formulae d = √
1 a2 + b 2
[y1 (x3 − x2 ) + y2 (x1 − x3 ) + y3 (x2 − x1 ] From the point(5, −2) = (x1 , y1 )
2
1 24(5) + 7(−2) + 19
+ [y1 (x3 − x4 ) + y3 (x4 − x1 ) + y4 (x1 − xy ] d= √
2 242 + 72
1 125
AREA = [1(2 − 1) + 3(0 − 2) + (−3)(1 − 0)] = = 5(E)
2 25
1
+ [1(2 − 3) + (−3)(3 − 0) + 1(0 − 2)]
2 30. An equation to a line bisecting the angle
1 1 between 2x−3y+2 = 0 & 3x−2y+5 = 0
= [1 − 6 − 3] + [−1 − 9 − 2]
2 2 is
8 12
= + = 10(C) (a) x+y = −3 (b) x−y = 5 (c) 5x+2y =
2 2
1 (d) 2x − 5y = 0 (e) x − y = 0
26. The equation of a straight passing through Solution
(1,1) and (−1, −1) is
(a) x − y = 0 (b) x+y=0 (c) x − 2y = 0 sin 6α cos 6α
(d) 2x+2y=1 (e) x + y + 1 = 0 31. − simplifies to
sin 2α cos 2α
Solution (a) 4α (b) 2 (c) tan 2α (d) 4 (e) 1/2
y − y1 y2 − y1
Using the formulae = Solution
x − x1 x2 − x1 sin 6α cos 6α
y−1 −1 − 1 −
= sin 2α cos 2α
x−1 −1 − 1 sin 6α cos 2α − cos 6α sin 2α
y−1 =
=1 sin 2α cos 2α
x−1 sin 4α 2 sin 2α cos 2α
y−1=x−1 = =
sin 2α cos 2α sin 2α cos 2α
x − y = 0(A)
= 2(B)
sin(B − A)
27. + tan A simplifies to 32. The value of cos 3000 sin 390+cos 660 sin 570
sin A cos B
(a) sin A (b) tan B (c) cot B (d) cos B (e) is
tan A (a) 2 (b) 3/2 (c) 1 (d) 12 (e) 0
78
r
Solution 1 1 + cosθ
Also cos 2
x =
cos 300 sin 390 + cos 660 sin 570 2
1 1 1 −1 1 − cos x 1
using calculator = × + × 1
∴ tan 2 x = = − cot x
2 2 2 2 sin x sin x
1 1 5 3 2 1
− = 0(E) = − = = (A)
4 4 4 4 4 2
33. The points (−1, −1) and (1,1) are ends
of a diameter of a circle whose equation 1
is 37. The value of sec y is :
√ √2 √
(a) x2 + y 2 = 2 (b) x2 + y 2 + 2x = 0 (c) 13 5 26 25 26
x2 + y 2 − 4x = 0 (d) x2 + y 2 = 16 (e) (a) (b) (c) (d) (e)
5 26 26 5
x2 − y 2 = 0 −2
Solution Solution
2 2
The midpoint
q of the diameter is (−1, −1)and(1, 1)Recall,sec x = 1 + tan x
√ ∴ sec2 21 y=1 + tan2 21 y
and r = (1 − 0)2 + (1 − 0)2 = 2 q
Using (x − a)2 + (y − b) 2
= r2 sec 2 y = 1 + tan2 21 y
1

(x − 0)2 + (y − 0)2 = ( 2)2 1 13 12 1
But tan 12 y= − cot y = − =
r sin y r 5 5 5
2 2
x + y = 2(A)
1 26
4 5 sec 21 y= 1 + =
Given that sin x = , tan y = x and y √ 25 25
5 12 26
are acute angles.Answer question 34-40 = (D)
5
34. The value of cos 2x is
1 4 7 9
(a)− (b) (c) − (d) (e) −2 sin 2y
2 7 25 25 38. The value of is
Solution sin 2x
4 5 8 4 120 24 125
sin x = , tan y = (a) (b) (c) (d) (e)
5 12 65 13 169 25 169
using Pythagoras theorem Solution
5
3 5 sin 2y 2 sin y cos y × 12
cosx = , siny = , = = 13 13
5 13 sin 2x 2 sin x cos x 4
× 3
4 12 5 5
tanx = , cosy = 60 25 125
3 13 = × = (E)
4 169 12 169
Recall,cos2x = 1 − 2sin2 x = 1 − 2( )2
5
32 −7
=1− = (C) 39. The value of tan(x + y) is
25 25 23 7 63 9 21
(a) (b) (c) (d) (e)
35. The value of cos(x − y) is 4 4 16 4 16
3 7 4 21 56 Solution
4
(a) − (b) (c) (d) (e) tan x + tan y + 5
13 13 65 25 65 tan(x+y) = = 3 4 12 5 =
Solution 1 − tan x tan y 1 − 3 × 12
cos(x − y) = cos x cos y + sin x sin y 16 + 5 16 21 36 63
3 12 4 5 36 20 56 ÷ = × = (C)
= × + × + = (E) 12 36 12 16 16
5 13 5 13 65 65 65
1 40. The value of sin 3x + sin x is
36. The value of tan x is 36 20 9 36 16
2 (a) (b) (c) (d) − (e)
1 1 2 5 125 125 25 25
(a) (b) −1 (c) − (d) (e) 0 Solution
2 5 3
Solution sin 3x + sin x = 2 sin 2x cos x
1 sin 12 x
tan x = = 2 cos x(2 sin x cos x) = 4 sin x cos2 x
1
2 cos 2 x 4 9 144
r =4× × =
1 − cosx 5 25 125
But sin 12 x = No correct option
2
0.17. MATHS 103 2012-2013 79

0.17 MATHS 103 2012-2013


r
θ 1 − cos θ
sin =
2
1 2 r
Given that cot x = 2, and tan y = , x,y are 1 + cos θ
3 Similarly cos 2θ =
acute angles.Answer question 1-12 2
Therefore tan x2 = sin x2 ÷ cos x2
r
1. The value of cos 2x is q 1 − cos x
3 4 1 9 = 1−cos x
÷
(a) (b) (c) − (d) (e) −2 2
2
5 7 5 25 r
Solution 1 − cos x (1 − cos x)
= ×
To answer 1 − 12 r 1 + cos x 1 − cos x
1 (1 − cos x)2
cot x = 2 & tan y = =
3 2
1 p 1 − cos x
tan x = (1 − cos x)2
2 = √
Applying both SOH CAH TOA and Pythago- sin2 x
ras theorem 1 − cos x 1
opp Adj opp = = − cot x
sin θ = , cos θ = , tan θ = sin x√ sin x
x
hyp √ Hyp
√ Adj tan 2 = 5 − 2(E)
For x Hyp = √4 + 1 = 5
1
For y hYP = 3 + 1 = 2 ∴ 4. The value of tan y is
1 1 2

sin x = √ & sin y = √ 10 2 1
5 2 (a) 2 10 − 3 (b) (c) (d) (e) −2
2 3 2 3 6
cos x = √ & cos y = Solution
5 2 1 − cos y 1
cos 2x = cos2 x − sin2 x = 2 cos2 x − 1 tan 12 y = = − cot y
sin y sin y
= 1 − 2 sin2 x = 2 − 3 = −1 No correct option
Any of the above equation will work sin 2y
cos 2x = 1 − 2 sin2 x = 1 − 2 × ( √15 )2 5. The value of is
sin 2x
2 3 5 3 2 4 2
= 1 − = (A) (a) (b) (c) (d) (e)
5 5 6 2 3 5 5
Solution
2. The value of tan(x − y) is sin 2y 2 sin y cos y
2 1 4 7 =
(a) − (b) (c) (d) −1 (e) sin 2x sin
 x cos x 
7 7 3 6 1 3 1 2 3 2
Solution = × ÷ √ ×√ = ÷
tan x − tan y 2 2 5 5 4 5
tan(x − y) = 3 5 15
1 + tan 
x tany = × =

1 1 1
 4 2 8
tan(x − y) = − ÷ 1+ No correct option
2 3 6
1 7 1 6 1 6. The value of tan(x + y) is
= ÷ = × = (B) 1 1 3
6 6 6 7 7 (a) 1 (b) − (c) (d) (e) 2
4 2 4
1 Solution
3. The value of tan x is tan x − tan y
2 tan(x + y) =
√ 1
(a) 1 (b) −1 + 2 (c) − √  1 − tan
 x tan y
2 1 1 1
1 √ = + ÷ 1−
(d) √ (e) 5 − 2 2 3 6
5 5 5
Solution = ÷
6 6
sin 12 x 5 6
tan 12 x = = × = 1(A)
cos 12 x 6 5
2
From cos 2θr = 1 − 2 sin θ 7. The value of√sin 3x − sin x is
1 − cos 2θ 3 10 9 36 6
∴ sin θ = (a) (b) − (c) (d) − (e) −
2 5 25 125 25 5
Let θ = 12 θ Solution
80

sin 3x − sin x using Solution


sin A − sin B = 2 cos (A+B) sin (A−B) cos 4y (cos 2x)2 − (sin 2x)2
2 2 =
A = 3x B = x sin 2x 2 sin x cos x
= 1 − 2(sin 2y)2 ÷ 2 sin x cos x
sin 3x − sin x = 2 cos( 4x ) sin( 2x )
2 2
= 1 − 2( 32 )2 ÷ (2 √15 × √25 )
= 2 cos 2x sin x = 2 sin x(1 − 2 sin2 x)
2 1 = 1 − 2( 32 )2 ÷ (2 × √15 × √25 )
= 2 sin x − 4 sin3 x = √ − 4( √ )3 9 4 7 4 14
5 5 1− ÷ =− × =−
2 4 50 − 4 46 2 5 2 5 5
=√ − √ = √ = √ No correct option
5√ 25 5 25 5 25 5
46 5 12. The value of cot(2y − x) is
=
125 11 33 22 3 11
No correct option (a) (b) (c) (d) (e)
7 7 3 2 2
Solution
8. The √ value of cos√ 3x + cos x √ √ 1 tan 2y − tan x
3 10 12 5 −2 5 2 5 cot(2y−x) = =
(a) (b) (c) (d) tan(2y − x) 1+tan 2ytan x
5√ 25 5 5 2 tan y 1 1
6 5 tan 2y = 2
=2× ÷ 1−
(e) − 1 − tan y 3 9
25 2 8 3
Solution = ÷ =
3 9 4
cos 3x + cos x Using 3 1

3 1

cos A + cos B = 2 cos( A+B ) cos( A−B ) tan(2y − x) = − ÷ 1 − ×
2 2 4 2 4 2
= 2 cos 2x cos x = 2 cos x(2 cos2 x − 1) 6−4 3 2 5 2
3 = ÷ (1 − ) = ÷ =
 x−3 2 cos x
= 4 cos 8 8 8 8 5
2 2 No correct option
=4 √ −2× √
3 5
32 4 68 The angular point of a quadrilateral ABCD
= √ −√ =− √
25 √ 5 5 25 5 are (1, 5), (4, 1), (−1, −4), (−4, 3) respec-
68 5 tively.Use this to answer question 13-22
=− No correct option
125
13. The area of the quadrilateral in square
9. The value of sin 4x is √ units is :
4 7 3 10 (a) 28 (b) 36 (c) 38 (d) 72 (e) 34
(a) − (b) − (c)
25 25 5 Solution
2 6
(d) − (e) − Area of Quadrilateral is given by
5 5 1
Solution Area = [y1 (x3 − x2 ) + y2 (x1 − x3 ) + y3 (x2 − x1 )]+
sin 4x = sin(2x + 2x) 2
[y1 (x3 − x4 ) + y3 (x4 −  x1 ) +y4 (x1 − x3 )] 
OR
= sin 2x cos 2x+cos 2x sin 2x = 2 sin 2x cos 2x
4 3 24 1 x1 y 1 1 x1 y 1
=2× × = No correct option 1 1
5 5 25 Area = 1 x 2 y 2
+ 1 x4 y 4 
2 2
1 x3 y 3 1 x3 y 3
10. The value of cos 4x √ is
4 7 3 10 2 6 x1 y 1 x2 y 2
(a) (b) (c) (d) − (e) Given , ,
A(1 , 5) A(4 , 1)
25 25 10 5 25
Solution x3 y3 x4 y4
,
cos 4x = cos(2x + 2x) = cos2 2x − sin2 2x A(−1 , −4) A(−4 , 3)
= (cos 2x)2 − (sin 2x)2 1
9 16 Area = [5(−1 − 4) + 1(1 − (−1)) + (−4)(4 − 1)]
= ( 35 )2 − ( 45 )2 = − 2
25 25 1
7 + [5(−1 + 4) + (−4)(−4 − 1) + 3(1 + 1)]
=− No correct option 2
25 1 1
= [−25 + 2 + (−12)]+ [15 + 20 + 6] =
cos 4y 2 2
11. The value of | − 35| |41|
sin 2x √ + = 38(B)
2 2
4 7 3 10 3 11
(a) (b) (c) (d) (e) 14. The area of the triangle ABD in squares
25 25 10 2 2
0.17. MATHS 103 2012-2013 81

units is : = (−2, 25 ) No correct option


(a) 20 (b) 16 (c) 19 (d) 13 (e) 14
Solution 17. The equation to the line AD is
Area of triangle is given by (a) 2x − 5y + 23 = 0 (b) x + 5y + 25 = 0
1 (c) 2x − 3y + 9 = 0 (d) 3x − 2y − 1 = 0
Area = [x1 (y2 − y3 ) + x2 (y3 − y1 ) + x3 (y1 − y2 )]
2 (e) x + y + 1 = 0
Or   Solution
x x x
1 1 2 3  AD= (1, 5)(−4, 3)
Area = y1 y2 y3 y − y1 y2 − y1
2 Using =
1 1 1 x − x1 x2 − x11
x1 y 1 x2 y2 x3 y3 y−5 3−5
Given , , =
A(1 , 5) A(4 , 1) A(−1 , −4) x−1 −4 − 1
1 2x − 5y + 23 = 0(A)
Area = [1(1 − 3) + 4(3 − 5) + (−4)(5 − 1)]
2
1 −26 18. The equation to the line perpendicular to
= (−2 − 8 − 16) = = 13(D)
2 2 AC through A is
15. The equation to the bisector of the line (a) 2x − 5y + 23 = 0 (b) 2x + 9y − 17 = 0
DC is (c) 2x + 3y − 9 = 0 (d) 3x − 2y − 1 = 0
(a) 3x − 5y + 23 = 0 (b) 7x + 5y + 25 = 0 (e) x + y − 11 = 0
(c) 3x − 5y + 9 = 0 (d) 7x − 2y − 1 = 0 Solution
(e) 3x − 7y + 4 = 0 AC (1, 5)(−1, −4)
−4 − 5 9
Solution Gradient of AC = m1 = =
−1 − 1 2
Line DC = (−4, 3)(−1, −4) Gradient of perpendicular line m2
−4 − 3 −7 1 2
Gradient of DC =M = = m2 = − =−
−1 + 4 3 m1 9
Gradient of the perpendicular is m2 = y − y1
1 3 = m2 at B(4, 1)
− = x − x1
m1 7 y−1 2
=−
 
−4 − 1 −4 + 3
Midpoint of DC is , = x−4 9
  2 2 2x + 9y − 17 = 0(B)
−5 −1
, 19. The equation to the line parallel BC through
2 2
y − y1 A is
The equation of the line is given by =
x−1 (a) 2x − 5y + 2 = 0 (b) x + 5y + 25 = 0
m2 (c) x − 3y + 9 = 0
y + 12 3
= (d) 3x − y − 1 = 0 (e) x + y − 6 = 0
x + 52 7 Solution
7 15 Gradient of BC at (4, 1)(−1, −4) is
7y − 3x + − =0
2 2 −4 − 1
3x − 7y + 4 = 0(C) m1 = =1
−1 − 4
m2 is the gradient of the equation paral-
16. The coordinate of a point dividing the
lel to equation of m1 i.e m1 = m2 = 1
join BD in the ratio 3:1 is
The equation of the line through A is
(a) (2, 52 ) (b) (6, 14 ) (c) (3, 12 ) (d) (1,2) y − y1
(e)(3, 41 ) =m
x − x1
Solution y−5
Since m : n = 3 : 1 and m > n ⇒ inter- =1
x−1
nally divided
  x − y + 4 = 0 No correct option
mx2 + nx1 my2 + ny1
(x, y) = , 20. The lines AC and BD intersects at the
m+n m+n
BD = (4, point
1)(−4, 3) 1 9 8 6
(a) (2, 17 ) (b) (1, 17 ) (c) ( 19 , 19 )

3(−4) + 1(4) 3(3) + 1(1)
(x, y) = , 6 73 77 3
(d)( 19 , 38 ) (e) ( 38 , 19 )
3+1 3+1
82

Solution 5
5 cot c = = 7.86 ≈ 8 No cor-
The line AC at (1, 5)(−1, −4) is tan 32.47
−4 − 5 y−5 rect option
=
−1 − 1 x−1 23. The least non-zero value for 0 ≤ x ≤
2y − 9x − 1 = 0 − − − − − −(1) 1800 in the solution of cos 3x + cos x = 0
The line BD at (4,1) (−4, 3) is is
3−1 y−1
= (a) 1800 (b) 450 (c) 22.50 (d) 300 (e) 900
−4 − 4 x−4
2 y−1 Solution
= Recall cos 3x = 4 cos3 x − 3 cos x then
8 x−4
4y + x − 8 = 0 − − − − − − − (2) substitute into the question
solving equ (2) and equ (1) simultane- i.e 4 cos3 x − 3 cos x + cos x = 0
ously to get the point of intersection 4 cos3 x − 2 cos x = 0
73 6 4 cos3 x = 2 cos x
y= x= 1
38  19 cos = √
6 73 2
(x, y) = , (D)
19 38 x = cos−1 ( √12 ) = 450 (B)

21. The value of 7 tan A is cos(A + B)


24. + tan B simplifies to
(a) 14 (b) 24 (c) 26 (d) 13 (e) 12 sin A cos B
(a) sin A (b) tan B (c) cot B
Solution From the 4 ABC
(d) cos B (e) tan B
Solution
cos(A + B)
+ tan B
sin A cos B
cos A cos B − sin A sin B sin B
= +
sin A cos B cos B
cos A cos B sin A sin B sin B
= − +
sin A cos B sin A cos B cos B
cos A sin B sin B
= − +
sin A cos B cos B
p √ cos A
|AB| = (5 − 1)2 +√ (1 − 4)2 = 25 = c = = cot A(C)
√ sin A
|BC| = √ 25 + 25 =√ 50 = a
|AC| = 4 + 81 = 85 = b sin x
25. cot x + simplifies to
Using Chain 2rule 2 1 + cos x
b + c − a2 (a) cot x (b) sin 2x (c) sec x (d) cosecx

−1
A= cos (e) cos 2x
2bc

85 + 25 − 50
 Solution
= cos √ √ sin x cos x sin x
2 × 85 × 25 cot x + = +
1 + cos x sin x 1 + cos x
= cos−1 ( 92.195
60
) = 49.400 cos x(1 + sin x) + sin2 x
∴ 7 tan A = 7 × tan 49.40 = 8.167 =
sin x(1 + cos x)
No correct option cos x + cos2 x + sin2 x
=
sin x(1 + cos x)
22. The value of 5 cot C cos x + 1 1
(a) −2 (b) 2 (c) 3 (d) 4 (e)−4 = = = cosecx(D)
sin x(1 + cos x) sin x
Solution
5 26. The angle of depression of a boat 120m
5 cot C =
tan C from the top of a mast is 300 .if the mast
Using Sine Rule
a c is standing on the cliff of 50m high,the
= height of the mast is
sin A sin C
C sin A 5 × sin 49.40 (a) 12m (b) 5m (c) 10m (d) 20m (e) 15m
sin C = = √
a 50 Solution
c = sin−1 (0.54) = 32.470
0.17. MATHS 103 2012-2013 83

27. The√value√of cos 15√0 is √ √ & (0, 8) use the information to answer
6+ 2 6− 2 6 question 31-38
(a) (b) (c)
√ 4 √ √4 2
2− 3 2 31. The equation of the circle is
(d) (e) − (a) x2 + y 2 − 3x − 4y = 4 (b) x2 + y 2 = 25
4 2
Solution (c) x2 + y 2 − 6x − 8y = 0
cos 150 = cos(60 − 45) using (d) x2 + y 2 − 2x − 5y = 0
cos(A − B) = cos A cos B + sin A sin B Solution
∴ cos(60−45) √= cos 60 cos 45+sin 60
√ sin 45 (3, −1) = (x1 , y1 ) (0, 8) = (x3 , y3 )
1 1 3 1 1 3 (6, 0) = (x2 , y2 )
= ×√ + ×√ = √ + √
2 √ 2 √2 √2 2 2 2 2 Using the formula
1+ 3 2+ 6 x21 + y12 + 2gx1 + 2f y1 + c = 0
= √ = (A) @ (3, −1)
2 2 4
9 + 1 + 6g − 2f + c = 0
28. The length of a perpendicular √ drawn from 6g − 2f + c = −10 − − − − − − − (i)
the origin to the line l is 48 units .The at (0, 8)
perpendicular makes an angle of 600 with 0 + 64 + 0 + 16f + c = 0
the x-axis.The
√ equation of l is
√ 16f + c = −64 − − − − − − − (ii)
(a) x − √3y = 6 (b) x + y√= 48 √ at (6, 0)
(c) 2x
√ + 3y = 2 (d) x + 3y = 8 3 36 + 12g + c = 0
(e) 3x + y = 10 12g + c = 0 − − − − − − − (iii)
Solution Solving the 3 equation simultaneously g =
Recall that if a perpendicular make an −3, f = −4, c = 0
angle of θ with x-axis the equation of the i.e x2 + y 2 + 2gx + 2f y + c = 0
length of the perpendicular is x2 + y 2 − 6x − 8y = 0(C)
l = x cos θ + y sin θ i.e√
x cos√ 60 + y sin 60 = 48 32. The coordinate of the centre is
x 3 √ (a) (4,3) (b) (2,4) (c) (0,4)
+ y = 48 (d) (3,4) (e) (4,2)
2 √2 √
x + 3y = 8 3(D) Solution
x2 + y 2 − 6x − 8y = 0
sin 5θ − sin 3θ x2 + (−3)2 + y 2 + (−4)2 = 0 + 9 + 16
29. simplifies to
cos 5θ + cos 3θ (x − 3)2 + (y − 4)2 = 25
(a) tan θ (b) − cot θ (c) cos 2θ (d) − tan 2θ
compare with (x − a)2 + (y − b)2 = r2
((e) sin 12 θ
a = 3, b = 4, r = 5
Solution
center (3,4) (D)
Using sin A − sin B = 2 cos A+B 2
sin A−B2
I.E 33. The equation of its tangent at the point
sin 5θ − sin 3θ = 2 cos(4θ) sin θ (3,9) is
sin 5θ − sin 3θ 2 cos 4θ sin θ (a) x + y = 3 (b) x − y = 3 (c) x + 9 = 0
∴ =
sin 5θ + cos 3θ 2 cos 4θ cos θ (d) y = 9 (e) x + y = 9
= tan θ(A)
Solution
30. if tan(A + 60) = 2 then cot A is Using the formula
1 1
(a) 3 (b) 2 (c) 3 (d) 4 3 xx1 + yy1 + g(x + x1 ) + f (y + y1 ) + c = 0
(e) none of the above (x1 , y1 ) = (3, 9) , g = −3 , c = 0, f = −4
Solution 3x + 9y − 3(x + 3) − 4(y + 9) + 0 = 0
tan(A + 60) = 2 5y = 45
A + 60 = tan−1 2 y = 9(D)
A + 60 = 63.43 34. The units of length of tangent drawn from
A = 663.43 − 60 = 3.34 ≈ 3(C) the point (8, −6) is
A circle passes through the points (3, 1−), (6, 0) (a) 12 (b) 6 (c) 8 (d) 14 (e)10
84

Solution tan(2400 − θ) + tan(120 0


√ + θ) √
(a) 2 (b) 1 (c) 0 (d) − 3 (e) 3
Solution
tan(240 − θ) + tan(120 + θ)
tan 240 − tan θ
tan(240 − θ) =
√ 1 + tan 240 tan θ
3 − tan θ
= √
1 + 3 tan θ
tan 120 + tan θ
tan(120 + θ) =
√ √ √ 1 − tan 120 tan θ
AC = 102 + 52 = q125 − 3 + tan θ
√ = √
By Pythagoras x = ( 25)2 + 25 = 12.25 ≈ 1 + 3 tan θ √
12(A) 3 − tan θ
∴ tan(240−θ)+tan(120+θ) = √ +
35. The unit length of radius of the circle is √ 1 + 3 tan θ
− 3 + tan θ
(a) 4 (b)3 (c) 5 (d) 7 (e) 10 √ = 0(C)
Solution 1 + 3 tan θ

From question 32 solution
39. The value of tan−1 ( 13 ) + sec−1 ( 25 ) is
r = ±5 π π π
(a) (b) (c) (d) 2π
36. The circle touches x2 + y 2 − 4x + 6y = 12 4 2 3
(e) none of the above
at the point Solution √
(a) (6,7) (b) (−1, 4) (c) (5,7) (d) (−1, 7) tan−1 ( 13 ) + sec−1 ( 25 )
(e) (0,6) Let α = tan−1 ( 31√) ∴ tan α = 13 √
Solution
Also β = sec−1 ( 35 ) ∴ sec β = 35
from question 31 solution the equation of p
the circle is Recall
q tan β = sec2 β − 1
5 1
x2 + y 2 − 6x − 8y = 0 − − − − − −(i) = 4
−1= 2
the given circle is tan α + tan β
x2 + y 2 − 4x + 6y = 12 − − − (ii) tan(α + β) =
1 − tan α tan β
equ (i)-equ (ii) 2x + 14y = 12 1 1 1 5 5
= + ÷ (1 − ) = ÷ = 1
∴ x = 6 − 7y substitute into equ (ii) 3 2 6 6 6
(6 − 7y)2 + y 2 − 4(6 − 7y) − 6y = 12 α + β = tan−1 (1) = π4 (A)
36 − 84y + 49y 2 + y 2 − 24 − 28y + 6y = 12
sin 3α cos 3α
50y 2 − 50y = 0 40. − simplifies to
sin α cos α
∴ y = 0 or 1 α α
(a) (b) (c) 2 (d) 2α
at y = 0 ;x= 6 − 0 = 6 i.e (6,0) 4 2
at y=1 ; 6 − 7 = −1 i.e (−1, 1) No cor- (e) none of the above
rect option Solution
sin 3α cos 3α sin 3α cos α − cos 3α sin α
− =
37. The equation of the common chord of the sin α cos α sin α cos α
sin(3α − α) sin 2α 2 sin 2α
circle and x2 + y 2 − 4x + 6y = 12 is = = =
(a) 2x − y = 7 (b) x + y = 6 (c) x − y = 6 sin α cos α sin α cos α 2 sin α cos α
= 2(C)
(d) x + y = 7 (e) x − y = 7
Solution
The equation to the common chord is 0.18 MATHS 103 2013-2014
x2 +y 2 −6x−8y−(x2 +y 2 −4x+6y−12) =
0 1. The general solution to the equation sin 3θ+
2x − 14y + 12 = 0 sin θ = 0 is
nπ π nπ π
x − 7y = −6 No correct option (a) (b) 2nπ − (c) or 2nπ +
2 2 2 2
π
38. Find the value of (d) 2nπ or nπ +
2
0.18. MATHS 103 2013-2014 85

Solution Solution
sin 3θ + sin θ = 0 y−x=4
Recall sin 3θ = 3 sin θ − 4 sin3 θ From the point(1, 2) using the formula
∴ sin 3θ + sin θ = 0 ax1 + by1 + c 1(1) + (−1)2 + 4
d=± √ = √
⇒ 3 sin θ − 4 sin3 θ + sin θ = 0 a2 + b 2 2
4 sin θ − 4 sin3 θ = 0 3
= √ (B)
sin θ(1 − sin2 θ) = 0 2
θ = 0 or θ = ± π2
6. If a line intercept x and y axes at equal
i.e θ = nπ or 2πn + π2 (C)
2 2
point and passes through ( 12 , −1) ,its equa-
2. The radius of the circle x2 + y 2 + 2y = 8 tion will be
is √ (a) 2x+2y=3 (b) 2x+y = 3 (c) 2x+2y +

(a) 2 2 (b) 2 (c) 3 (d) 4 1 = 0 (d) 2x + 2y = 1
Solution Solution
x2 + y 2 + 2y = 8
x2 + y 2 + 2y + (1)2 = 8 + (1)2
(x + 0)2 + (y + 1)2 = 9 compare with 7. The angle 220 1600 expressed in degree only
(x − a)2 + (y − b)2 = r2 is
r2 = 9 ; r = ±3(C) (a) 18.6260 (b) 0.37710 (c) 8.3740 (d) 0.37110
Solution
cos x 22 16
3. The expression tan x + simpli- 220 1600 = + = 0.3711(D)
1 + sin x 60 3600
fies to
(a) tan x (b) sec x (c) − cos x (d) cos x 8. The equivalent of 18.6260 expressed in
Solution degree,minute and seconds is
cos x sin x cos x (a) 180 300 1200 (b) 180 120 2300 (c) 180 370 3300
tan x + = +
1 + cos x cos x 1 + sin x (d) 180 370 1200
sin x(1 + sin x) + cos2 x Solution
=
cos x(1 + sin x) 18.6260 = 180 + (0.626 × 60)0
sin x + sin2 x cos2 x = 180 + 37.560 = 180 + 370 + (0.56 × 60)0
=
cos x(1 + sin x) = 180 370 3300 (C)
sin x + 1 1
= = = sec x(B) 9. The center of the circle x2 + y 2 − 6x +
cos x(1 + sin x) cos x
4y + 9 = 0 is at
4. The value of x that satisfy 2 cos 3x−1 = 0 (a) (2,3) (b) (3, −2) (c) (−3, 2) (d) (−6, 4)
in the range (00 , 3600 ) are Solution
(a) 600 , 3000 (b) 200 , 1000 (c) −600 , 1000 x2 + y 2 − 6x + 4y + 9 = 0
(d) 200 , 3000 x2 + y 2 − 6x + 4y = −9
Solution x2 +(−3)2 +y 2 +(2)2 = −9+(−3)2 +(2)2
2 cos 3x − 1 = 0 recall (x − 3)2 + (y + 2)2 = −9 + 9 + 4
cos 3x = 4 cos3 x − 3 cos x (x − 3)2 + (y + 2)2 = 4 compare with
2(4 cos3 x − 3 cos x) − 1 = 0 (x − a)2 + (y − b)2 = r2
8 cos3 x − 6 cos x − 1 = 0 ∴ (a, b) = (3, −2) r = 2(B)
let p= cos x
8p3 − 6p − 1 = 0 9π
10. The angle in degree measure is
∴ p = −0.766, 0.936, −0.1736 20
∴ x = cos−1 p (a) 1400 (b) 810 (c) 360 (d) 900
x = 1400 , 200 , 1000 (B) Solution
9π 9 × 180
= = 810 (B)
5. The distance of (1, 2) from the line y − 20 20
x = 4 is √ 11. The value of tan 150√is
3 3 2 √ 1− 3
(a) 3 (b) √ (c) (d) (a) 1 − 3 (b)
2 2 3 2
86
√ √
(c) 2 − 3 (d) 1 + 3 15. Equation of a circle whose ends of diam-
Solution eter are (1,1) and (−1, −1) is
tan(15) = tan(60 − 45) (a) (x − 1)2 + (y + 1)2 = 4 (b) (x − 1)2 +
tan 60 − tan 45 (y−1)2 = 4 (c) x2 +y 2 = 4 (d) x2 +y 2 = 2
tan(60 − 45) =
√ 1 + tan 60 tan 45 Solution
3−1 Diameter (1, 1) and (−1, 1−) ,find the
= √ by rationalizing
1+ 3 √
√ midpoint  
3−1 3−1 x 1 + x2 y 1 + y 2
=√ ×√ (x1 , y1 ) = , = (0, 0)
3 +√1 3−1 2 2
4−2 3 √ using (0,0) as centre and (1,1) as point
= = 2 − 3(C) ,using the formula
2
(x − x1 )2 + (y − y1 )2 =
√ r2
12. The equation of tangent to x2 + y 2 − 4y + (x − 0)2 + (y − 0)2 = 2
1 = 0 is x2 + y 2 = 2(D)
(a) y−x = 2 (b) y−x = 1 (c) x−y+1 = 0
(d) y = 2x − 1 16. The polar form of the point (1,1) is
Solution (a) (2, π4 ) (b) (2, 450 ) (c) (1, π4 ) (d) ( 4,2
π
)
x2 + y 2 − 4y + 1 = 0 compare with Solution
x2 + y 2 + 2gx + 2f y + c = 0
∴ g = 0, f = −2 and c = 1 0
The equation of tangent to a circle is 17. The√value of cos
√ 75 in√surd form √ is
given by (a) 23−1

2
(b) 3+1
2
(c) 3+1

2
(d) 3−1

2
xx1 + yy1 + g(x + x1 ) + f (y + y1 ) + c = 0 Solution
at what point cos 75 = cos(30 + 45)
(x1 , y1 ) is not given Question not given cos(30
√ + 45) cos 45 −√sin 30√
√ = cos 30 √ sin 45
3 2 1 2 6 2
= × − × = − =
13. The line y = mx + c tangent to x2 + y 2 = √ 2 √ 2 2 2 4 4
r2 if 6− 2
(A)
(a) r2 (1 − c2 ) = m2 (b) c2 (1 + r2 ) = m2 4
(c) m2 (1 + r2 ) = c2 (d) r2 (1 + m2 ) = c2
Solution 18. Which of the following is a trigonometry
y = mx + c is tangent to x2 + y 2 = r2 if identity
c2 = r2 (1 + m2 )(D) (a) sin2 x = sin x − 1 (b) sin x + cos x = 1
(c) sin x = sin x cos2 x+sin3 x (d) sin x =
14. Which of the following does not intersect cos2 x + 1
the circle x2 + y 2 =1 Solution
(a) y = 2x (b) y = 2x + 3 (c) y = 3x + 3 sin x = sin x cos2 x + sin3 x
(d) y = 4x − 1 = sin x(cos2 x + sin2 x) = sin x(1)
Solution sin x = sin x(C)
x2 + y 2 = 1 √
we will put the value of y in the above 19. The distance between (cos x, sin 2x) and
equation (sin x, 0) is
x2 + (2x)2 − 1 = 0 (a) 0 (b) 2 (c) 1 (d) 3
x2 + 4x2 − 1 = 0 Solution √
5x2 − 1√= 0 (sin x, sin 2x) and √ (sin x, 0)
5 (x1 , y1 ) = (cos x, sin 2x) and (x2 , y2 ) =
x=±
5 (sin x,
p0)
x2 + (2x + 3)2 − 1 = 0 d= − y1 )2 + (x2 − x1 )2
x2 + 4x2 + 12x + 9 − 1 = 0 q (y2 √
5x2 + 12x + 8 = 0 = (0 − sin 2x)2 + (sin x − cos x)2
p
x = − 56 ± 25 i complex (B) = (sin 2x) + sin2 x − 2 sin x cos x + cos2 x
0.18. MATHS 103 2013-2014 87

= √sin 2x + 1 − 2 sin x cos√x (a) 23 (b) 31 (c) 14 (d) 15
= sin 2x + 1 − sin 2x = 1 = 1(C) Solution
x1 y1 x y2 2
, 2 has a gradient
20. If √
the distance between (a, 2) and (3, 1) (x , 13) (5 , 7) 3
is 5 ,the value of a is y2 − y1
m=
(a) 1 (b) 2 (c) 3 (d) 4 x2 − x1
Solution 2 7 − 13
=
(x1 , yp
1 ) = (a, 2) and (x2 , y2 ) = 3 5−x
√ (3, 1) 10 − 2x = −18
2 2
√= (1 − 2) + (3 −
d √9) = 5 x = 14(C)
2
1 + 9 − 6a + a = 5
a2 − 6a + 5 = 0
25. If the distance between (1, a) and (4, 1)
a = 5 or 1(A)
is 5.The possible value of a are
21. if t= tan 21 θ ,then sin θ equals (a) 5, −3 (b) 5, 2 (c) 3, −5 (d) 2, −5
2 2 2t 2t Solution
(a) 1−t (b) 1−t (c) (d) between
2t 1+t2
1+t 2 1 − t2 p (1, a) and (4, 1) ,d is 5
Solution d = (1 − a)2 + (4 − 1)2
2t √
t = tan 2θ then sin θ = 1 − 2a + a2 + 9 = 5
1 + t2 2
a − 2a − 15 = 0
Proof ,from sin 2θ = 2 sin θ cos θ
cos θ sin θ a = 5 or − 3(A)
= 2 sin θ cos θ × =2 × cos2 x
cos θ cos θ
2 tan θ 26. The angle 87.50 in radian measure is
=
sec2 θ (a) 0.435π (b) 0.486π (c) 2.299π (d) 22π
2 tan θ
sin 2θ = Solution
1 + tan2 θ 87.50 in radians is
θ
2 tan 2θ 87.5π
let θ = 2 , sin θ = = 0.486π(B)
1 + tan2 2θ 180
but t = tan 2θ
2t 27. If the cos x = 53 and x is in the first quad-
sin θ = (C) rant .The value of 1 − sin 2x is
1 + t2
(a) 54 (b) 25
24
(c) 24
25
1
(d) 25
22. Find the value of tan(1200 −θ)+tan(600 + Solution
θ) cos x = 35 by Pythagoras sin x = 45
√ 1 1
(a) 0 (b) 3 (c) (d) √ Recall sin 2x = 2 sin x cos x
2 3 ∴ 1 − sin 2x = 1 − 2 sin x cos x
Solution
= 1 − 2 × 54 × 35 = 1 − 25 24
tan(120 − θ) + tan(60 + θ) 1
= 25 (D)
let α = 120 − θ and β = 60 + θ
α + β = 120 − θ + 60 + θ = 180
28. Which of the following line is perpendic-
tan(α + β) = tan 180 = 0(A)
ular to 3x − 2y = 1
23. The slope of the line 3x − 3y + 17 = 0 is (a) 2x − 3y + 4 = 0 (b) y = 4x − 3 (c)
(a) −3 (b) 31 (c) −1 (d) 1 3y = 1 − 2x (d) y = −2x + 3
Solution Solution
3x − 3y + 17 = 0 Perpendicular to 3x − 2y = 1
17 y = 23 x − 12
y =x+ compare with 1
3 for perpendicular line m2 = −
y = mx + c m1
∴ m = 1(D) 2
m2 = −
3
24. If a line passes through the point (5, 7) 2
and has slope 23 .The x-coordinate on the option c has the gradient of − i.e
3
line when y = 13 is y = 21 − 23 x(C)
88

29. Which of the following line is not parallel 33. For any angle θ ,the expression
to x+2y=0 cos θ cos θ
− is equivalent to
(a) x = 2y (b) 2x = 3 − 4y (c) y= 5- 21 x 1 − sin θ 1 + sin θ
(d) 1 − x − 2y = 0 1
(a) (b) 2 tan θ
Solution 1 − sin2 θ
2 sin θ
x + 2y = 0 (c) (d) tan2 θ
1 cos2 θ
y = − x ∴ m = − 12 Solution
2 cos θ cosθ
for parallel line m1 = m2 = − 21 −
1 − sin θ 1 + sin θ
option A has a gradient of 12 i.e x = 2y cos θ(1 + sin θ) − cos θ(1 − sin θ)
is not parallel to y = − 12 x (A) =
1 − sin2 θ
cos θ + cos θ sin θ − cos θ + cos θ sin θ
30. If (3, 6) are coordinate of midpoint of the =
1 − sin2 θ
line joining (−1, 1) and (x,y).The value 2 cos θsinθ sin θ
of x and y are = 2
=2 = 2 tan θ(B)
cos θ cos θ
(a) 1,2 (b) 2,3 (c) 7,11 (d) 7,6
34. The distance between (2, −1) and (3, 4)
Solution
is √ √
(−1, 1) and (x,y) has a midpoint of (3,6)
(a) 25 (b) 26 (c) 5 (d) 2 5
i.e
Solution √
( x−1 , y+1 ) = (3, 6) p
2 2 d = (4 + 1)2 + (3 − 2)2 = 26(B)
x−1
2
= 3 and y+1 2
=6
x − 1 = 6 and y + 1 = 12 35. The value
√ of sin√120 in surd
√ form
√ is
x = 7 and y = 17 i.e (7, 11)(C) (a) 1+2 3 (b) 1−√ 3 (c) − 3 (d)
2 2 2 2
3
(B)
Solution √
31. Which of the following is not a trigonom- sin 120 = 23 using calculator
etry identity
(a) cot cot θ
= sin θ (b) sin θ(1−cos2 θ) = 36. The line whose slope is a2 passes through
θ+tan θ
sin3 θ (c) 2 sin θ+1 = cscθ (d) cot2 θ+1 = the intersection of x − y + 17 = 0 and
cscθ x + y − 3 = 0.Find its equation
Solution (a) x − y = 10a (b) 2x − y + 10a = 7
(C) sin θ + 1 = cosecθ (c) 2x − ay + 2(5a + 7) = 0
(d) x − ay + 10a = 14
cscθ cos θ Solution
32. The expression can be sim-
tan θ − cot θ To get point of intersection ,solve simul-
sin2 taneously
plified to (a) cot θ (b) (c) cos2 θ
cos 2θ x − y = −7 and x + y = 3
cos2 θ ∴ x = −7 and y = 10
(d) − y − y1
cos 2θ m=
Solution x − x1
cosecθ cos θ 2 y − 10
tan θ − cot θ =
sec θ cos θ ÷ (tan θ − cot a x+7
θ) 2x + 14 = ya − 10a
cos θ sin θ cos θ 2x − ay + 2(5a + 7) = 0(C)
÷ −
sin θ  cos θ sin θ 
cos θ 2
sin θ − cos2 θ 37. If θ in an angle in the 2nd quadrant .the
÷ reference and for θ is given by
sin θ cos θ sin θ
sin θ sin θ cos θ (a) 1800 − θ (b) 1800 + θ (c) θ − 1800 (d)
× 3600 − θ
sin θ sin2 θ − cos2 θ
cos2 θ cos2 θ Solution
= = − 1800 + θ(B)
sin2 − cos2 θ cos2 θ − sin2 θ
2 2
but cos θ − sin θ = cos 2θ
cos2 θ 38. The reference angle for 36400 is
=− (D) (a) 1400 (b) 400 (c) −400 (d) 750
cos 2θ
0.19. MATHS 101 2010-2011 89

Solution 0.19 MATHS 101 2010-2011


Reference angle α = 2nπ + θ
1+i
for 3640 = π × 90 + 40 1. The complex number , written in
1−i
θ = 40(B) 1+i
the form x + i y, is (a) (b) 1− i (c) i
39. The value 0 2
√ of csc2460 is √ (d) − i (e) i − 1
(a) − 3 (b) − 2 (c) 1 (d) − 23
1
Solution
Solution 1+i 1+i 1 + i + i + i2 1 + 2i − 1
1 1 × = 2 =
cosec2460 = = 1−i 1+i 1+i−i−i 1+1
sin 2460 sin(2160 + 300) 2i
1 2 = = i (c)
√ = −√ 2
− 23 3
No correct option 2. For any three sets A, B and C. Let D =
A\ (B\C) and E = (A\B) \C. Which
40. The value of sec 34650√is of the following must be true:
(a) −1 (b) − √12 (c) − 2 (d) √1
2
(a) D ⊆ E (b) E * D (c) E ⊆ D (d) E =
Solution φ (e) D = φ
1 Solution
sec 2465 =
cos 3465
Using calculator

sec 3465 = − 2(C) 3. If A is a set with n elements, then, the
number of subsets of subsets of A is
n
(a) 2n (b) n2 (c) nn (d) n! (e)
2
solution
The number of subsets of any set is equal
2
to (A)
n
4. Let R is a relation on Z, the set of all
integers, defined by xRy if x < y. Which
of the following statement is true?
(a) R is reflexive (b) R is transitive
(c) R is symmetric (d) R−1 does not exist
(e) R is equivalence
Solution
R = {xRy, x < y} ∴ R is transitive (B)

5. Let A be given set. The power set of A


is defined as
(a) P (A) = {x : x is an element of A}
(b) P (A) = {B :B ⊆ A}
(c) P (A) = {x :x ∈ A}
(d) P (A) = {B :B ⊃ A}
(e) P (A) = the number of subsets of A
Solution
if A is a given set of the power of A is
defined as: P(A) = the number of subsets
of A (E)

6. If A = (−1, 2] and B = (1,T3) are inter-


vals on the real
Sline, then A B equalTto:
(a) (−∞, −1] S(2, ∞) (b) (−∞, −1) [2, ∞)
(c) (−∞, −1) [2, ∞) (d) (−∞, −1]∪(2, ∞)
90

(e) none of the above but θ is in clockwise direction hence, θ =


Solution -450
if A= (−1, 2] and B= (1, 3) OR
θ in anti-clockwise direction is = 360 - 45
= 315o
π −π
in radian θ = -45 × =
180 4
OR
π 7π
θ = 315 × =
180 4 √  π   π 
:- 1 - ı in polar form = 2 cos − ısin
4 4
OR

   
7π 7π
2 cos − ısin (C)
4 4

∴ = (1, 2](E) Let E, E + , E − , P, P + and P − be the set


of even integers ,positive even integers,negative
7. A set is define as even integers ,prime integers,positive prime
(a) A collection of distinct objects integers,negative prime integers respec-
(b) A family of well-defined number tively .
(c) A collection of elements
(d) A collection of identical objects which
T
9. E P equal to:
is well-defined (a) {2} (b) {−2, 2} (c) {−2} (d) φ (e) E
(e) A well-defined collection of distinct Solution
objects if E = {...4, −2, 0, 2, 4...}
Solution E+ = {2, 4, 6...}
A set is defined as a well-defined collec- E− = {−2, −4, −6...}
tion of distinct objects P = {−7, −5, −3, −2, 2, 3, 5...}
P+ = {2, 3, 5...}
8. The polar form of the complex number 1- − 
√  π √ P7πT= {−2, −3, −5, −7...}

π 7π
i is (a) 2 cos + isin (b) 2 cos − isinE P = {−2, 2} (B)
4 4 4 4


7π 7π  π π
(c) 2 cos + isin (d) 2 cos + isin
4 4 4 4
10. E+ P + equal to:
T

7π 7π
(e) 2 cos − isin (a) {2} (b) {−2, 2} (c) {−2} (d) φ (e) E+
4 4
Solution
Solution
E+ P = {2}
T
the polar form
p of 1 - i r(cosθ + isinθ)
where 2 2
√r = x +y 11. E− P − equal to:
T
2
r = √1 + 1 2
(a) {2} (b) {−2, 2} (c) {−2} (d) φ (e) E−
r= 2  
1 Solution
−1
:- θ = tan = 450 E − ∩ P − = {−2}
1
12. The intersection of two sets A and B is
definedTas
(a) A T B= {x : x ∈ Aorx....B}
(b) A T B = {x : x ∈ A and x ∈ B}
(c) A T B = {x : x ∈ Aorx ∈ B}
(d) A B = {x : x is not an element of Aand x ∈
B} T
(e) A B = {x : x is not an element of A and x
is not an element of B}
0.19. MATHS 101 2010-2011 91

(a)S(A B)0 =
S 0
T 0 T 0 0
Solution A B and (A
T B) =
A0 BS0 (b) A SA = A andTA A =TA
T T S
A B = {x : x ∈ A and x B} (B)
(c) A S B = B A S and A B = BT A
13. A relation R on X is said to be symmet-
(d) A ∅ = ∅ (e) A A0 = µandA A0 =
ric if for all x, y ∈ X we have

(a) (x, y) ∈ R ⇒ (x, x) ∈ R
solution
(b) (y, x) ∈ R ⇒ (x, y) ...R S 0
De-Morgans theorem states that: (A B) =
(c) (x, y) ∈ R ⇒ (y, x) ∈ R 0
T 0 T 0 0
S 0
A B and (A B) = A B (A)
(d) (x, y) is not an element of R ⇒ (y, x) ∈ √
R 16. 1 − ı  3 in polar form is 
(e) (x, x) is not an element of R ⇒ (y, y) ∈ π π π π
(a) 2 cos + ısin (b) 2 cos − ısin
R √  3π 3 
π  3 π 3
π
Solution (c) 3 cos + ısin (d) 2 −cos + ısin
(x, y) ∈ R ⇒ (y, x) ∈ R (C) √  3 π 3 
π
3 3
(e) 3 −cos + ısin
3 3
14. The modulus√and argument of the com- Solution
plex number 2 (1 + ı) are
π π π
(a) 1 and (b) 1 and (c) 2 and
2 3 2
π π
(d) 2 and (e) 1 and
4 4
Solution
The
√ modulus√and argument√ of
2 (1 + ı) = 2 + q √ 2ı
p 2 √ 2
r = x2 + y 2 = 2 + 2
√ √
r = 2+2= ! 4=2

2
θ = tan−1 √ = tan−1 1
2 √
o 1 − ı 3 in polarq form r(cosθ − ısinθ)
θ = 45 p √ 2
OR r = x2 + y 2 = ı2 + 3
√ √
= 1+3= 4=2
y √ !
3
θ = tan−1 = tan−1
x 1
o
θ = 60
θ = −60o {clockwise direction}
in anti-clockwise direction
θ = 360 − 60 = 300o
in radian
π π π 5π
−60o × = − or 300 × =
180 3 180 3
inpolar πform
  π 
θ = −360+45 = −315{ in clockwise direction} 2 cos − ısin
in radian 3 3
π −7π OR    
−315 × = 5π 5π
180 4 2 cos + ısin (B)
OR 3 3
π π
45 × = 17. Let R be a relation on Z define by xRy
180 4
∴ the modulus and arg. are: if y − x2 = 1. Then (a) (−10, 3) ∈ R
−7π π (b) (2, 3) ∈ R (c) (3, −10) ∈ R (d) (5, −2) ∈
2 and or 2 and (D)
4 4 R (e) (5, 2) ∈ R
15. Let A and B be any sets. Then De- Solution
Morgan’s theorem states that R = {xRy if y − x2 = 1}
92

1+ı 3
(d) − 1 + 0ı (e)
2
Solution
18. if z is a complex number, z̄ its complex
conjugate, Re (z) the real part of z and
Im (z) the imaginary part of z. Then
(a) z − z̄ = 2Im (z) (b) z z̄ = Re (z) +
Im (z) (c) |z| = z z̄ (d) z z̄ = Re (z)2 +
Im (z)
(e) z + z̄ = 2Re (z)
solution

19. Let z = r (cosθ + ısinθ) be a complex


number and n a rational number. then
Dé-Moivre’s theorem states that
(a) z n = rn (cosn θ + ısinn θ) √
To find the cube root of unity i.e 3 1 =
(b) z n = rn (cosnθ + ısin nθ) √3
1
1 + 0ı = (1 + 0ı)  from
3.
  de-moivre’s
(c) zn = rn (cos nθ + ısin nθ) 1 1 θ
 
θ
1 1 theorem, Z 2 = r 2 cos + ısin
√ n √ n
(d) z n = r n (cos nθ + ısin nθ) p
2 2 2 2
r = x + y = 1 + 0 = 1 = 1
(e) z n = rn (cos nθ + ısin nθ) y 0
Solution θ = tan−1 = tan−1 =0
The DE-MOIVRE’S theorem states that x 1
one of the rootis 
Z n = rn (cos nθ + ısin nθ) (E)
 
1 1 0 0
Z 3 = 1 3 cos + ısin = 1 (1 + 0ı) =
3 3
Let U = {x/x is a non-negative integer 1
less than 20}. If B and C are subsets of U since for a cubic root, three roots are re-
defined by B = {x/x is a relatively prime quired
to 15}, and C = {x/3 divides x }, ∴ on the argand diagram we have:

20. The number of elements in U is


(a) 15 (b) 9 (c) 20 (d) 21 (e) 22
solution
U = {0, 1, 2, 3, 4, 5, 6, 7, 8, 9, 10, 11, 12, 13, 14,
15, 16, 17, 18, 19}
∴ the number of elements = 20 (C)

21. The number of elements in B is


(a) 7 (b) 11 (c) 5 (d) 10 (e) 15
Solution
B = {2, 4, 7, 8, 11, 13, 14, 16, 17, 19}
360
∴ the number of elements in B is 10 (B) = {where 3 is the number of roots}
3
=1200 apart
22. The number of elements in C is
hence the three roots are 1200 apart, since
(a) 7 (b) 12 (c) 10 (d) 9 (e) 6
the first root is at 00 , the others will be
Solution
at 1200 and 2400 away from the first root
C = {0, 3, 6, 9, 12, 15, 18}
having the same modulus
∴ the number of elements in C = 7 (A)
Z = 1 (cos (0) + ısin (0)) and
23. Which of the following is a cube root√of Z = 1 (cos (120) + ısin (120)) and
√ Z = 1 (cos (240) + ısin (240))
−1 − ı 3 1−ı 3
unity? (a) (b) 0+ı (c) ∴ Z = 1 and
2 2
0.19. MATHS 101 2010-2011 93
√ √
1 3 −1 + 3ı (a) {x|x ∈ A| : andx is not an element of B}
Z = - +ı = and
2 √2 2√ (b) {x|x is not an element of A| : and x ∈
1 3 −1 + 3ı B} (c) {x|x ∈ A| : or x is not an element of B}
Z = - −ı = (A)
2 2 2 (d) {x|x is not an element A| : or x ∈ B}
1 (e) {x|x is not an element of A| : or x is not an eleme
24. The complex number z = - − ı lies in Solution
2
(a) 1st quadrant (b) 2nd quadrant A\B is defined as {x|x ∈ A x is not an element of B
(c) 3rd quadrant (d) 4th quadrant (A)
(e) 1st and 3rd quadrant.
Solution 28. Let ε = {10, 11, 12, ...30} and A = {x ∈
ε| the sum of the digits of x is a prime
number}. Which of the following gives
the elements of A?
(a) {11, 13, 17, 19, 23, 29}
(b) {10, 12, 15, 18, 20, 21, 24, 25, 27, 30}
(c) {11, 12, 13, 14, 16, 20, 21, 23, 25, 29, 30}
(d) {11, 12, 14, 17, 20, 21, 23, 25, 29, 30}
(e) {11, 12, 14, 16, 20, 21, 23, 25, 29, 30}
Solution
ε = {10, 11, 12, 13, 14, 15, 16, 17, 18,
19, 20, 21, 22, 23, 24, 25, 27, 28, 29, 30} and
A = {x ∈ ε| the sum of the digits of x is
1 a prime number} (e.g. 11 i.e. 1 + 1 = 2)
Given Z = - −ı
2 A = {11, 12, 14, 16, 20, 21, 23, 25, 29, 30}
∴ Z lies on the 3rd quadrant (C)
(E)
25. Let A = {1, 2, 3}, which of the following
relation on A is reflexive 29. Let A = {2, 3} and B = {1, 2, 3, 4}. Let
(a) {(1, 2) , (2, 2) , (2, 3) , (1, 1) , (3, 3)} R be a relation defined as xRy; for x∈A,
(b) {(1, 2) , (2, 2) , (2, 3) , (3, 3)} y∈B, if and only if x is relatively prime
(c) {(1, 2) , (2, 1) , (2, 3) , (3, 3)} to y. Then R =
(d) {(2, 1) , (2, 3) , (3, 3)} (a) {(2, 1) , (2, 3) , (3, 1) , (3, 2) , (3, 4)}
(e) {(2, 1) , (2, 2) , (3, 3)} (b) {(2, 1) , (2, 2) , (2, 3) , (3, 1) , (3, 2) , (3, 3)}
Solution (c) {(2, 2) , (2, 4) , (3, 3)}
if A {1, 2, 3} (d) {(2, 1) , (2, 2) , (2, 3) , (2, 4) , (3, 1) , (3, 2) ,
and R is reflexive (3, 3) , (3, 4)}
∴ {(1, 2) , (2, 2) , (2, 3) , (1, 1) , (3, 3)} (e) {(1, 2) , (1, 3) , (2, 2) , (2, 3) ,
(3, 2) , (3, 3) , (4, 2) , (4, 3)}
26. which of the following is not true? Solution
(a) The empty set is a set with no el- if A = {2, 3} and B = {1, 2, 3 i.e. R =
ement (b) the empty set is a subset of {xRy, for x∈A, y∈B if and only if is rel-
every set atively prime to y
(c) the empty set is denoted by {∅} ∴ R = {(2, 1) , (2, 3) , (3, 1) , (3, 2) (A)
(d) ∅ is a member of P(∅)
(e) P (∅) is non-empty 30. Let A = {1, 2, 3}and B = {1, 2, 3, 4, 9},
Solution Let function, f assign to each element of
an empty set is not denoted as {∅} rather A its square that is f :A −→ B. Then the
it is denoted as either ∅ or {} range of f is
(a) B (b) {1, 2, 3, 4} (c) {1, 4, 9} (d) ∅
27. If A and B are any sets, then the differ- (e) None of the option
ence of A and B denoted A \ B is defined Solution
as if A = {1, 2, 3} and B = {1, 2, 3, 4, 9}
94

∴ f : A−→B, the f : A −→B = {(1, 1) , (2, 4) , (2, 9)}


∴ the range of f is {1, 4, 9} (C)
√ 6
31. if ı = −1, then ı10 = 36. let C be the real set of complex numbers,
(a) ı (b) − ı (c) 1 (d) − 1 (e) 106 N the set of natural numbers, Q the set of
Solution
√ rational numbers, R the set of real num-
6
if ı = −1 then ı10 bers and Z the set of integers. Which of
250000
ı1000000 = (ı4 ) and ı4 = 1 the following is true?
∴ (1)250000 (C) (a) Q ⊂ R ⊂ C ⊂ Z ⊂ N
(b) N ⊂ R ⊂ Q ⊂ Z ⊂ C
32. Let A = {1, 2, 3}. which of the following
(c) Q ⊂ N ⊂ C ⊂ Z ⊂ R
relation on A is an identity relation?
(d) N ⊂ R ⊂ C ⊂ Z ⊂ Q
(a) {(1, 1)} (b) {(1, 2) , (1, 1) , (3, 3)}
(e) N ⊂ N ⊂ Q ⊂ R ⊂
(c) {(1, 2) , (2, 1)} (d) {(2, 2) , (1, 1) , (3, 3)}
Solution
(e) {(1, 1) , (3, 3)}
since N ⊂ N ⊂ Q ⊂ R ⊂ C(E)
Solution
If A = {1, 2, 3} where an identity relation
37. The modulus of the complex number Z =
is also on equivalence relation
x + ıy is
Hence R = {(2, 2) , (1, 2) , (3, 3)} (D) p p
(a) x2 + y 2 (b) x2 + y 2 (c) x2 − y 2

33. if |X| = m and |Y | = n then |P (X × Y ) | (d) x2 − y 2 (e) x + y
m Solution
= (a) mn (b) 2mn (c) 2m+n (d) m+n (e)
n The
p modulus of Z ∴
Solution x2 + y 2 (B)
if |X| = m and |Y | = n
T
34. The interval (2, 4] [1, 4) = 38. Let A = {1, 2, 3}. Which of the following
(a) (2, 4) (b) [3, 5) (c) [2, 4] (d) (3, 5] (e) [2, 4) relation on A is a universal relation?
Solution (a) {(1, 1) , (2, 3) , (1, 2) , (2, 1) , (1, 3) , (3, 2) ,
(3, 1) , (2, 2) , (3, 1)}
(b) {(1, 1) , (2, 3)
(c) {(1, 1) , (2, 3) , (1, 2) , (2, 1) , (1, 3) , (3, 2) ,
(3, 1) , (2, 2) , (3, 3)}
(d) {(1, 1) , (2, 3) , (1, 2) , (2, 1) , (1, 3) , (3, 2)}
(e) {(1, 2, 3)}
Solution
if A = {1, 2, 3} then its universal relation
is R = {(1, 1) , (1, 2) , (1, 3) , (2, 1) , (2, 2) , (2, 3) ,
(3, 1) , (3, 2) , (3, 3)} (C)

T T 39. The unionS of two sets A and B is defined


(2, 5] [1, 4) ∴ (2, 5] [1, 4) = (2, 4) (A) as (a)SA B = {x : x ∈ A or x@B
(b) A S B = {x : x ∈ A or x ∈ B
35. Let f be a function such that f (x) = (c) A S B = {x : x ∈ A andx ∈ A
x2 − 4
, then f (−2) = (d) A S B = {x@A and x ∈ B
3−x (e) A B = {x : x@A or x @ B
−8 8
(a) 0 (b) (c) − 8 (d) − 2 (a) Solution
5 5 S
Solution A B = {x : x ∈ A or x ∈ B} (B)
x2 − 4 √
if f (x) = then f (−2) is:
3−x 40. The argument of 2 3 + 2ı is
(−2)2 − 4 4−4 0 (a) 300 (b) 600 (c) 900 (d) 450 (e) 1800
∴ x = −2 = = =0
3 − (−2) 3+2 5 Solution √
(A) The argument of 2 3 + 2ı is
0.20. MATHS 101 2011/2012 95
 
−1 2 (c) {x : x ∈ C or x @ }
θ = tan √
 2 3 (d) {x : x @C orx E} (e) {x : x @C and x @ E}
1 Solution
θ = tan−1 √ ;θ = 300 (A)
3 C/E is defined as {x : x ∈ C and x @ E}
(A)
0.20 MATHS 101 2011/2012 6. Let A = {x : x ∈ Z : 10 6 x 6
25 and the
Let N be the set of Natural Numbers, Z denote
sum of the digits of x is a prime number }
the set of integers, Q denote the set of Rational
Which of the following set gives the ele-
numbers, Q denote the set of Irrational num-
ment of A ?
bers, R denote the set of Real numbers and C
(a){11, 13, 17, 19, 23, 24}
denote the set of Complex numbers, then use
(b){10, 12, 15, 18, 20, 21, 24, 25}
of the information to answer questions 1-4
(c){11, 12, 14, 16, 20, 21, 23, 25}
1. Which of the following is not true (d){11, 12, 14, 17, 20, 21, 23, 25}
(a)N ⊆ Z ⊆ Q ⊂ R (e){11, 12, 15, 16, 20, 21, 23, 25}
(b) Q ∪ Q0 (c) Q ∩ Q0 = ∞ (d) N ∪ Q0 6= φ Solution
(e) Z ⊂ Q ⊂ C if A = {x ∈ Z : 10 6 x 6 25 and the
solution sum of the digits of x is a prime number}
Z ⊂ Q ⊂ R which is false (A) {e.g.11 i.e 1 + 1 = 2}
∴ A = {11, 12, 14, 16, 20, 21, 23, 25}
2. which Tof theSfollowing Tsets isTempty T S
(a) (Z T R) Q (b)S(Z TQ0 ) R (c) N (Z 7. R) Which of the following is true about any
(d) (C N) (e) (R Q) C subset A of a universal setTU
Solution (a)S∅ ⊆ A ⊆ U (b) ∅ A0 = U (c)
T 0
if Q Q = ∅ A ∅ = U (d) A = U (e) x ∈A for all
and Z ⊂ 0 x∈U
TQ,0 Z * Q Solution
thenTZ Q = ∅
∴ ∅ R = ∅ (B) x∈ A for all x ∈ U (E)

8. Let A = {0, 1, 2, 3} and B = {1, 2, 3, 5, 7, 9, 10}


S T
3. N (RT C) is
let f : A−→ B and define f(a) = a2 + 1,
S 0
(a)(Q N) (b) Z (c) C (d) ∅ (e)Q Q
Solution for all a∈A. Then the range of f is
(a) B (b) {1, 2, 3, 5, 7, 9, 10} (c) {1, 2, 3, 5, 7}
S T
N (R C)
sinceTR ⊂ C (d) {1, 2, 3, 7, 9, 10} (e) {1, 2, 5, 9, 10}
∴S R C=R Solution
N R if A = {0, 1, 2, 3} and B = {1, 2, 3, 5, 7, 9, 10}
S −−−−−→
since ∴ N SR = R if f : a2 + 1 B
but R = Q Q0 (E) then f :A−→B = {(0, 1) , (1, 2) , (2, 5)}
∴the range of f is: {1, 2, 5, 10} (C)
4. All the following are true except
x √
(a) Q = { : x, y ∈ Z} 9. 1 − ı 3 in polar form is 
y π π π π
(b) C = {a + ıb : √
S 0
a, b ∈ R} (c) ∅ Q = (a)2 cos + ısin (b)2 cos − ısin
Q0 (d) Q ⊇ R (e) x @ N for all x∈ Z √  3π 3 
π  3 π 3
π
(c) 3 cos + ısin (d)2 −cos + ısin
Solution √  3 3  3 3
0 0
π π
Q ⊂ R while Q ⊇ R (D) (e) 3 −cos + ısin
3 3
Solution

5. If C and E are any sets, then the differ- 1 - ı 3 in polar form − ısinθ)
ence of C and denoted by C\E is denoted p q r (cosθ
√ 2
as r = x2 + y 2 = ı2 + 3
√ √
(a) {x : x ∈ Candx @ E} (b){x : x @Candx E} = 1 + 3 = 4 = 2
96
√ ! S
y 3 The interval (a,b) (a,d) where b < d i.e
θ = tan−1 = tan
x 1
θ = −60 - clockwise
in anti-clockwise direction

S
∴ (a,b) (a,d) = (a,d) (A)
Z2
13. Suppose Z1 = 1 + ı and Z2 = 2 then
Z1
θ = 360 − 60 = 300o is
in radian (a)1 (b)1 + ı (c)1 - ı (d)ı3 (e)2+1
π π π 5π
−60 × = or 300 × = Solution
180 3 180 3 if Z1 = 1 + ı and Z2 = 2 then
∴in polar
π  form  π  Z2 2 2 1−ı 2 − 2ı
2 cos − ısin = = × =
 3  3  Z1 1+ı 1+ı 1−ı 1 + ı − ı + (ı)2
5π 5π (C)
2 cos + ısin (B)
3 3
14. if Z = 2 − 3ı2 + ı then Z is
10. Let R be a relation on A = {2, 3, 6} de- (a)2-ı (b)3-ı (c) 5-ı (d)2+ı (e)2+ı
fined by xRy if x is relatively prime to y. Solution
Then if Z = 2 − 3ı + ı
(a) R = {(2, 2), (2, 3)} (b) R = {(2, 3), (3, 2)} Z = 2 − 3(−1) + ı
(c) R = ∅ (d) R = {(2, 3)}(e) R = A Z=2+3+ı
Solution Z=5+ı
if A = {2, 3, 6} ∴ Z = 5 − ı (C)
then xRy(if x is relatively prime to y. i.e.
R = {(2, 3), (3, 2)} (B) 15. if A is a set with n elements, then, the
number of subsets of A is
n
11. let z = r(cosθ+ısinθ) be a complex num- (a)2n (b)n 3 (c) n n (d)n! (e)
2
ber and n a rational number. Then De- Solutions
Moivre’s theorem states that the number of subset of any set is equal
(a)z2 = r2 (cosn θ + ısinn θ) to 2n (A)
(b) z n = rn(cosnθ + ısin nθ)
π
(c) zn =r2 (cosnθ + ısinn θ) 16. Let f (t)= sin t + . Then the value of
2
(d) z 1/2 = r1/2 (cosnθ + ısinnθ) π π π
n n f (0) is (a) (b) (c)30 (d)t2 (e)
(e) z = r (cosnθ + ısinnθ) 4 7 2
Solution Solution
π
if Z = r(cosθ + ısinθ) if f (t)= sin(t)+
2
the De-Moivre’s states that π
then f (0) = sin(0)+
Zn = rn (cos(nθ) + ısin(nθ)) (E) 2
π π
∴f (0)= 0 + = (E)
S
12. The interval (a,b) (a,d) where b <d is 2 2
(a) (a,d) (b) (a,b) (c) (a,a) (d)∅ (e) (1,2) 17. Let A be a given set. The Cardinality of
solution A is defined as
0.20. MATHS 101 2011/2012 97

(a) Possible numbers of subsets of A


(b) number of elements of A
(c) number of elements of the product set
of A T
(d) number of elements of A ∅ (e) ∅
Solution
if A is a given set: the cardinality of A is
defined as number of element in A (A)

18. The intersection of two sets A and B is π −7π


−315 × = OR
defined 180 4
T as 45 ×
π
=
π
(a) AT B = {x : x ∈ A and x@ B} 180 4
(b)A TB = {x : x ∈ A and x ∈ B} ∴ The modulus and argument are 2 and
(c) A T B = {x : x ∈ A or x ∈ B} −7π π
or 2 and (D)
(d) AT B = {x : x @ A and x ∈ B} 4 4
(e) A B = {x : x @ A and x ∈ B}
21. Let A = {1, 3, 5} which of the following
Solution
T relation on A is an identity relation?
A B is defined as {x : x ∈ A and x ∈
(a){(1, 1), (2, 2), (1, 2), (2, 1)}
B} (B)
(b){(1, 1), (2, 1)} (c){(3, 3)} (d){(1, 4), (2, 2)}
(e) none of the options
Solutions
19. A relation R on X is said to be anti-
if A= {1, 3, 5} and it identity relation is
symmetric if for all x,y ∈ X. we have
given as R = {(1, 1), (3, 3), (5, 5)} (E)
(a) (x,y)∈ R=⇒ (y, x) ∈ R
(b) (x,y)∈ R and (y,x)∈ R =⇒ x = y
22. Let A and B be any sets. Then De-
(c) (x,y) ∈ R =⇒ (x, x) ∈ R
Morgan’s S theorem states that T
(d) (x,y)∈ Rand (y,x)∈ R =⇒ x 6= y 0
B’ and (A B)0 =
T
(a)S(A B) = A’
(e)none of the options 0 0
A B
Solutions S T
(b) AS A = ASand A AT= A T
A relationR is said to be anti-symmetric
(c) A SB = B A andTA B = B A
if (x,y)∈ R and (y,x)∈ R =⇒ x = y (B)
(d) AS ∅ = A and A T∅ = ∅
(e) A A0 = U and A A0 = ∅
Solution
20. The modulus√and argument of the com- S 0
De-Morgan’s theorem states that (A B)
plex number 2(1 + ı) are 0
S 0 0 0 0
π π π = A B and (A ∩ B) = A ∪ B (A)
(a) 1 and (b) 1 and (c) 2 and
2 3 2
π π 23. The argument of the complex number
(d) 2 and (e) 1 and
4 4 Z = 3 + 4ı is
Solution √ (a) 36.880 (b)15.40 (c) 30.00 (d)450 (e)53.130
The √modulus
√ and argument of 2(1 +
Solution
ı) = 2 + 2ı q
√ 2 √ √ The argument of Z = 3 + 4ı is:
2
r = x + y = ( 2) + ( 2) 2 2 −1 y
√ √ ∴ θ = tan ( )
x
r = 2+2= 4=2 √ 4
x θ = tan−1 ( )
−1 √2 3
tanθ = ( ) ∴ θ = tan ( )
y 2 θ = 53.130 or
∴ θ = tan−1 (1) = 45o in clock-wise direction
OR θ = −360 + 53.13 = −306.87
θ = 360+45 = −315{in clockwise direction} ∴ the argument are:
in radian 53.130 or −306.870 (E)
98

24. The union of two sets A and B is defined C = {c : c ∈ U and c divides 15}, use the
as S information to answer questions 28-31
(a) A S B = {x : x ∈ A or x @ B} S
(b) AS B = {x : x ∈ A or x ∈ B} 28. The cardinality of A B is
(c)A S B = {x : x ∈ A and x ∈ B} (a)10 (b)8 (c)13 (d)15 (e)20
(d)A SB = {x :x @ A and x ∈ B} Solution
(e) A B = {x : x @ A or x @ B} if U = {1, 2, 3, 4, 5, 6, 7, 8, 9, 10, 11, 12, 13, 14,
Solution 15, 16, 17, 18, 19, 20}
A= {1, 2, 4, 5, 7, 8, 10, 11, 13, 14, 16, 17, 18, 19, 20}
S S
A B is defined as A B = {x : x ∈ A
or x∈B (B) B = {2, 3, 5, 7, 11, 13, 17, 19}
C =S {1, 3, 5, 15}
25. A set is defined as ∴ A B = {1, 2, 3, 4, 5,S7, 8, 10, 11, 13, 14, 16, 17}
(a) A collection of distinct objects ∴ The cardinality of A B = 15 (D)
(b) A family of well-defined number T
(c) A collection of elements 29. A B is (a){2, 5, 7, 11, 13, 17, 19} (b){1, 3, 5, 7}
(d) A collection of identical objects which (c)∅ (d){7, 9, 15, 17} (e){2, 5, 7, 11, 13, 15}
is well-defined Solution
T
(e) A well-defined collection of distinct A B = {2, 5, 7, 11, 13, 17, 19} (A)
objects T
Solutions 30. B C is (a){3, 5, 7, 9} (b){1, 3, 5, 15} (c){2, 3, 5, 7}
A set is defined as a well defined collec- (d){3, 5} (c)∅
tion of distinct objects (E) Solution
T
B C = {3, 5} (D)
26. The modulus of the complex number Z = T T
31. The cardinality of the power set of A B C
x +pıy is p p is (a)8 (b)16 (c)32 (d)1 (e)2
(a) px3 + y 2 (b)p x2 + y 2 (c) x2 − y 2
solution
(d) x2 + y 3 (e) x0 + y 0 T T
A T B T C = {5}
Solution
p(A B C) = 2n (where n = number
The
p modulus of Z = x + ıy is: of elements)
x2 + y 2 (B)
= 21T= 2T
p(A B C) = {5}
27. Let A,B⊆ Z, if A = {3a, 6b}, B = {6, 9}
∴ the cardinality is 2 (E)
and A = B then the possible values of a
are
(a) 2 or 3 (b)2 or 4 (c)3 or 4 1
32. The complex number z = − − i lies in
(d)0 or 2 (e)3 or 1 2
Solution (a)1st quadrant (b)2nd quadrant
if A,B ⊆ Z (c)3rd quadrant (d)4th quadrant
then A = {3a, 6b} B = {6, 9} (e)1st and 3rd quadrant
since A = B the possible values of a are: Solution
3a = 6 or 3a = 9 { since arrangement
does not matter in set}
6 9
a = or a =
3 3
a = 2 or 3 (A)

Let U = {x : 16 x 6 20 and x ∈ Z}
if A,B,C are subsets of U defined by:
A = {a : a ∈ U and a is relatively prime
to 3},
B = {b : b ∈ Z and b is prime},
0.20. MATHS 101 2011/2012 99

1
Given Z = − − ı
2
∴ Z lies on the 3rd quadrant.

33. Let A = {1, 2, 3}. Which of the following


relation on A is reflexive
(a){(1, 2), (2, 2), (2, 3), (1, 1), (3, 3)}
(b){(1, 2), (2, 2), (2, 3), (3, 3)}
(c){(1, 2), (2, 2), (2, 3), (3, 3)}
(d){(2, 1), (2, 3), (3, 3)} S 1
(e){(2, 1), (2, 2), (3, 3)} ∴ A2 A8 = (0, ) = A2 (E)
2
Solution
if A = {1, 2, 3} 38. For each n ∈ Z consider the open interval
R is reflexive if {(1, 2), (2, 2), (2, 3), (1, 1), (3, 3)} An = (0, 1 ) then, A3 A6 is
T
(A) n
(a)A3 (b)A2 (c)A 1 (d)A6 (e)A18
2
34. Question not clear Solution
1
if n ∈ N and An = (0, )
T n
35. Let A b any non empty set and B = ∅, then A3 A6 is:
then A×B is (a){(a, b) : a, b ∈ A} (b)∅ 1
if An = (0, )
(c) none of the options (d){(a, b) : a, b ∈ n
1 1
B} (e) it doesn’t exist A3 = (0, ) and A6 = (0, )
Solution 3 6
T 1
∴ A3 A6 = (0, ) = A6
6

36. If X = {1, 2, 3, 4} determine which of the


following relations is a function on X
(a)f = {(2, 3), (1, 4), (2, 1), (3, 2), (4, 4)}
(b)g = {(3, 1), (4, 2), (1, 1)}
(c)h = {(2, 1), (3, 4), (1, 4), (2, 1), (4, 4)}
(d) j = {(1, 1), (2, 4), (1, 3)}
(e)k = {(1, 2), (3, 1), (4, 4)}
Solution
(D)

37. Each n ∈ N consider the open interval 39. Let A,B be non empty sets such that
1 S A ∩ B = φ ,then which of the following
An = (0, ) then, A2 A8 is
n is not true
A2 (a) n(A∪B) = n(A)+n(B) (b) A∪B 6= φ
(a) A3 (b)A8 (c)A4 (d) (e) A2
2 (c) A-B=A (D) A ∪ B = B (e) A v B
Solution
1 Solution
if n ∈ Z and An = (0, )
n
then A2 UA8 is:
1 40. Let A and B be any sets,then which of
if An = (0, )
n the following is false
1 1
A2 = (0, ) and A8 =(0, ) (a) A is the disjoint of union A/B and
2 8
A ∩ B (b) A is a subset of B if and only
100

if B 0 ⊂ A0 (c) A is a subset if and only if Solution


A0 ⊂ B 0 (d) A ⊆ B if and only if A∩B 0 = If D={... − 3, −4, −2} and
φ (e) A∩B 0 is the disjoint union of A∩B E={−5, −4, −3, −2, −1, ...}
and B/A D \ E = {..., −7, −6}
Solution E \ D = {−2, −1, 0, 1, 2, 3, ...}
(D \ E) ∩ (E \ D) = φ (D)

0.21 MATHS 101 2012/2013 6. Which of the following best define an anti-
symmetric relation?
1. A ll the following relationships between (a)If a=b, then(a,b)∈ R
sets of real(R), integers(Z), natural(N), (b)If a6= b then(a,b)∈ R and(b,a)∈ R
rational(Q), irrationalQ1 and complex num- (c)(a,b)∈ R if a=b
bers are not true except (d)if a6= b, then either (a,b)∈ R or (b,a)
(a)Z ⊂ R ⊂ N ⊂ C(b)N ⊂ Z ⊂ R ⊂ ∈ R, but never both
Q(c)Q ⊂ Q1 andR ⊂ N (d)Z ⊂ Q ⊂ R Solution
Solution Anti-Symmetric relation is best defined
Since N ⊂ Z ⊂ Q ⊂ Q1 ⊂ R ⊂ C as
∴ Z ⊂ Q ⊂ R(D) (a,b)∈R if a=b (C)
√ 6
2. If i= −1, then i10 =
7. Let A=x|1 <x≤ 3 and B=x|1 ≤ x<3 be
(a)i (b)-i (c)1 (d)106
two subsets of real numbers. Which of
Solution√ 6 the following defines the set A∩ B?
If i = −1 then i10
(a){x|1 ≤ x ≤ 3}(b){x|1 < x < 3}
i1000000 = (i4 )250000 and i4 = 1
(c){x|1 < x ≤ 3}(d){x|1 > x ≤ 3}
∴ (i)250000 = 1 (C)
Solution
3. Which of the following is NOT true? if A={x|1 < x ≤ 3} = {2, 3}
(a)The empty set is not an element of ev- B={x|1 ≤ x < 3} = {1, 2}
ery set A ∩ B = {2}
(b) the empty set is a subset of every set A ∩ B {x|1 < x < 3}(B)
(c)the empty set is denoted by φ
(d)φ is a member of P (φ) 8. If Z is any complex number and z̄ is the
Solution complex conjugate of Z. Then z¯−1 =
An empty set is not denoted as {φ} rather (a)(z̄)−1 (b) z̄ (c)z (d)z −1
it is denoted either φ or {} (C) Solution
Z is any complex number.Z̄ is the com-
4. |x| < 2 means that plex conjugate. Then Z¯−1 = (Z̄)−1 (A)
(a) the distance between x and 2 on R is
less than zero 9. Let A and B be 2 sets, which of the fol-
(b) the distance between x and 2 on ”(a+b)≥ lowing is not true
1” is less than 2 (a)If A=∩ B=φ ,A\ B=A
(c) the distance between x and the origin (b)If B⊂ A, B\ A 6= φ
on Z is less than 1 (c)A1 ∪ B 1 =(B∩ A)
(d) x must lie between -2 and 2 on the (d)If A=B,(B \ A)=U
real line. Solution
Solution If B⊂ A B\A = φ (B)
|x| < 2 ∴ x must lie between -2 and 2 on
the real line (D) 10. Let A,B and C be sets, (A1 ∪ B 1 ∩ C 1 ) =
(A1 ∩ C 1 ∪ (B 1 ∩ C 1 ) ) is justified by
5. Given the following sets. D = (−∞, −2), (a)De Morgan’s law (b)Associative law
E=[−5, ∞). Then. (D \ E)∩(E \ D)= (c)Distributive law (d)Idempotent law
(a) [2,5) (b) (2,5) (c) 2,3,4 (d) φ Solution
0.21. MATHS 101 2012/2013 101

(A1 ∪ B 1 ) ∩ C 1 = (A1 ∩ C 1 ) ∪ (B 1 ∩ C 1 if and only if x−y = 3k, for some integer


is justified by distributive (C) k. Then R−1 =
(a){(1, 1), (1, 4), (2, 2), (3, 3), (4, 1), (4, 4), (3, 3)}
11. Let {H = −2, 0, 2}.let R be a relation on (b){(1, 1), (4, 1), (2, 2), (3, 3), (1, 4), (4, 4)}
A defined by a Rb if”(a+b)≥ 1” then R is (c){(1, 1), (2, 2), (3, 3), (4, 4)}
(a)Reflexive (b)Symmetric (c)Anti-Symmetric (d){(1, 2), (2, 1), (1, 3), (3, 1), (1, 4), (4, 1)}
(d)Equivalence Relation Solution
Solution If x = {1, 2, 3, 4} R={x − y = 3k}where
If H={−2, 0, 2} a Rb if”(a+b)≥ 1 k
2+2≥1 is any integer
0+2≥1 If k=0.
2+0≥1 x-y=0 {(1, 1), (2, 2), (3, 3), (4, 4)}.
R={(2, 2)(0, 2)(2, 0) If k=-1 x-y=-3{(1, 4)}.
Hence the relation is symmetric (B) If k=1 x-y=3{(4, 1)}
12. Let A = {1, 2, 3} and B = {1, 2, 3, 4, 9}.Let R={(1, 1), (2, 2), (3, 3), (4, 4), (1, 4), (4, 1)}
function, f assign to each element of A its R−1 ={(1, 1), (2, 2), (3, 3), (4, 4), (4, 1), (1, 4)}
−−−−−→ (B)
square that is f : A x − x2 B..Then
the range of f is 16. Given the following sets
(a)B (b)1, 2, 3, 4 (c) 1, 4, 9 (d)φ 1 1 1
A[− , ), C = [ , 1)(A ∪ C)1 =
Solution 2 2 2
If A = {1, 2, 3}andB = {1, 2, 3, 4, 9} ∴f: 1 1
−−−−→2 (a)(−∞, − )∪(1, ∞) (b)(−∞, − ]∪(1, ∞)
2 2
f : Ax − x B then 1 1
f: A −→ B ={(1, 1)(2, 4)(3, 9) (c) (1, ∞) (d) {− , 0, }
2 2
∴ the range of f is {1, 4, 9} (C) Solution
1 1 1
If A=[− , ) and C=[ , 1)
13. Let A and B be 2 sets, such that A∩B = 2 2 2
φ, all are true except
(a) (A B)1 ⊂ A (b) (A B)1 ⊂ B
(c) B A = B (d) A1 ∪ B 1 = U
Solution
If A ∩ B = φ A \ B 1 = A and (A)1 = B
Hence B* A (A)
14. Let ε = {12, ..., 30} and A={x ∈ ε| the
sum of the digits of x is a prime number}.
Then set A is?
(a){11, 13, 17, 19, 23, 19}
1
(b) {10, 12, 15, 18, 20, 21, 24, 25, 27, 30} ∴ A ∪ C = [− , 1) (A ∪ C)1
(c){11, 12, 13, 14, 16, 20, 21, 23, 25, 29, 30} 2
1
(d){12, 14, 16, 20, 21, 23, 25, 29, 30} =(−∞, − ) ∪ [1, ∞)
2
Solution
ε = {12, 13, 14, 15, 16, 17, 18, 19, 20, 21, 22, 17. Let K = {−1, 1, 0} and let R = {(0, −1), (0, 0),
23, 24, 25, 26, 27, 28, 29, 30} (−1, −1), (1, −1)(1, 1)},then
A ={x ∈ ε| the sum of the digits of x is (a)R is transitive and anti-symmetric
a (b)R is anti-symmetric and reflexive
prime number } (c)R is reflexive and transitive
e.g 12=1+2 =3 (d)R is reflexive and symmetric
A={12, 14, 16, 20, 21, 23, 25, 29, 30} (D) Solution
If k={−1, 1, 0} and R={(0, −1), (0, 0), (−1, −1),
15. Let X = {1, 2, 3, 4}. Define a relation (1, −1), (1, 1)} Hence R is reflexive and
R on the set of integers such that xRy antisymmetric (B)
102

18. |x| means that Solution


(a)the distance between x and the ori- The set of rational number is closed with
gin on R (b)the distance between x and respect to multiplication (C)
one on Z (c)the distance between x and
the origin on N (d)the distance between 23. If y ∈ (A B)1 then
x and two on Z. (a)y ∈ (A B) (b)y ∈ (A ∪ B)1
Solution (c)y ∈ (B 1 A) (d)y ∈ (A1 B)
Solution
if y &(A/B)0 then y(A/B)(A)
19. Let A = {1, 2, 3} and R on the relation
on A defined by a Rb if ”(a-b)=0” which 24. Which of the following is not true?
of the following is not true about R (a)R (a)ifa, b ∈ Z, (a − b) ∈ Z
a a
is reflexive (b)R is transitive (c)R is a set (b)if a, b ∈ R, ∈ R (c)if a, b ∈ Q, ∈ Q
(d)R is anti-symmetric b b
(d)if a, b ∈ N, (a − b) ∈ N
Solution Solution
If A={1, 2, 3} and a Rb ∈ R if (a-b)=0. If a,b∈N (a-b)is not element in N if a<b
Then R={(1, 1), (2, 2), (3, 3)} hence Natural number is not closed un-
Hence R is transitive, reflexive and anti- der subtraction (D)
symmetric but not a set (C)
25. Let z = r(cosnθ + isinθ) be a complex
20. If A and B are any sets, then the dif- number and n a rational number. Then
ference of A and B denoted by A/B is De-Moivre’s theorem states that
defined as (a)Z n = rn (cosn θ + isinn θ)
(a){x|x ∈ A or x ∈ B} (b)Z n = rn (cosnθ + isin nθ)
(b){x|x ∈ A and x ∈ B} (c)Zn = rn (cosnθ + isin nθ)
(c){x ∈ Ax B or x is not an element in 1 1
(d)Z n = r n (cosnθ + isin nθ)
B}
Solution
(d){x ∈ A and x is not an element in B}
If z = r(cosnθ + isinθ) then De-Moivre’s
Solution
theorem states that Z n = rn (cosnθ +
A \ B is defined as {x|x ∈ A and x is not
isin nθ) (B)
an element in B} (D)
26. Which of the following is a cube root of
21. If y ∈ A1 ∩ B 1 this implies that
unity? √ √
(a)y ∈ A, andy ∈ B (b)y ∈ A1 ∪ B 1
−1 − i 3 1−i 3
(c)y ∈ (A ∪ B) (d)y ∈ (A1 ∪ B 1 ) (a) (b)0+i (c)
Solution 2 √ 2
1+i 3
y ∈ (A1 ∩ B 1 ) this implies that y ∈ A1 (c)-1+0i (d)
and y ∈ B 1 , y is not an element in A and 2
Solution
not an element in B, y is not an element To find the cube root of unity i.e
in A ∩ B = y ∈ (A ∩ B)1 1 1
(1) 3 =(1 + 0i) 3 From De-moivre’s theo-
From De-Morgan’s law (A∩B)1 =A1 ∪B 1 1 1 θ θ
∴ y ∈ A1 ∪ B 1 (B) rem Z n = r 2 (cos + i sin ),
p √n n√
r = x2 + y 2 = 12 + 02 = 1 = 1,
22. Which of the following is true? y 0
(a) The set of odd numbers is closed with θ=tan−1 ( ) = tan−1 ( ).
x 1
respect to addition One of the root is
(b) The set of even numbers is closed 1 1 0 0
Z 3 = 1 3 (cos( ) + i sin( ))=1(1+0i)=1
with respect to division 3 3
Since for a cube root, three root are re-
(c)the set of rational numbers is closed
quired. ∴On the argand diagram we have
with respect to multiplication 360
(d) the set of natural numbers is closed = = 120o apart(where 3 is the num-
3
with respect to addition ber of root). Hence the three root are
0.22. MATHS 101 2013/2014 103

1200 apart. Since the first root is at 0o 30. Let f:R→R be a function defined by
the others will be at 1200 and 2400 away f (x) = {−x,x≥0
x,x<0 then the range of f is
from the first having the same modulus. (a)(−∞, 0) (b) (−5, 0) (c){1, 2, 3, 4, 5} (d)(−∞, ∞)
∴ Z = 1(cos 0 + i sin 0) and Solution
Z = 1(cos(120) + i sin(120) and −→ R then
If f : R 
Z = 1(cos(240) + i sin(240).
√ √ −x, x > 0
f (x) = the the range of f
1 i 3 −1 + 3i x, x < 0
Z = 1 or Z = − + = or is (−∞, 0) (A)
√ 2 2 √ 2
1 i 3 −1 − 3i
Z=− − = (B)
2 2 2
1 1
0.22 MATHS 101 2013/2014
27. Given the following sets A = [− , ),
2 2 Let A=(0, ∞) and B=(−∞, 3},then
1 1
B = [0, ), (A ∩ B) =
2 1. B \ A=
1
(a)(−∞, 0] (b){0, } (a)(−∞, 0) (b)(−∞, ∞) (c)(−∞, 3) (d)(−∞, 0]
2
1 1 Solution
(c)(−∞, 0] ∪ [ , ∞) (d)(−∞, 0) ∪ [ , ∞)
2 2 If A = (0, ∞) and B = (−∞, 3]
Solution hence A = {1, 2, 3, 4, 5, ...}
1 1 1
If A=[− , ), B=[0, ) B = {..., −4, −3, −2, −1, 0, 1, 2, 3}
2 2 2 ∴ B \ A = {..., −3, −2, −1, 0}
1
diagram ∴ (A ∩ B) = [0, ) B \ A=(−∞, 0] (D)
2
1
∴ (A ∩ B)1 = (−∞, 0) ∪ [ , ∞) (D) 2. (A \ B)1 =
2
(a)(−∞, 0) (b)(−∞, 3) (c)(−∞, 3] (d)(3, ∞)
28. The modulus of the complex number −2+ Solution
2i is (A \ B) = {4, 5, 6, 7, 8, 9, ....)
π π π
(a)3 (b) (c)1 (d) = (A \ B)0 = (−∞, 3] (C)
4 4 2
Solution
−2 3. Which of the following is true about a
∴ θ = tan−1 ( ) = tan−1 (−1)=-45
2 Universal relation?
(a)it is reflexive, symmetric but not tran-
sitive (b)it is an equivalence (c)it is sym-
metric only
(d)it is an identity relation, but not re-
flexive
Solution
A universal relation is an equivalence re-
lation (B)

4. A function f : N −→ R defined by
2
f(x)= , has a range
∴ α=180-45 = 1350 OR β= -180-45 =−2250 x
3π 5π 3π 5π 2 2 2
In radian =α or β = − Argument= or− (a)[0,1) (b){1, 2, , , } (c){0, 1, 2, 3} (d)[0,2]
4 4 4 4 3 4 5
Solution
(A) 2 2 2 2 2 2
If N = {1, 2, 3, 4, 5, ...} f(x)= = { , , , , , ...}
x 1 2 3 4 5
29. If z is a complex number, z̄ its complex 2 1 2
conjugate, Re(z) the real part of z and = f(x)={2, 1, , , , ...}
3 2 5
lm(z) the imaginary part of z. Then ∴f(x)=[0,2] since for any value of x it will
(a)z-z̄ = 2Im(z) (b)z z̄ = Re(z) + Im(z) always lies between this range Therefore
2
(c)z+z̄ = 2Re(z) (d)z z̄ = Re(z )+Im(z) Answer (D)
104

5. Given that Y={a, b, c},and R1 = {(a, a), (b, b), 10.(c,Let


c), (a, {10, 11, 12, ..., 30} and {x ∈ ε|the
ε =b)},then
R1 is sum of the digits of x is a prime number}.Which
(a)equivalence relation of the following gives the element of A?
(b)reflexive and symmetric (a){11, 13, 17, 19, 23, 29}
(c)identity but not reflexive (b){10, 12, 15, 18, 20, 21, 24, 25, 27, 30}
(d)both identity and reflexive (c){10, 13, 15, 17, 18, 19, 22, 24, 26, 27, 28}
(e)an reflexive and anti-symmetric (d){11, 12, 14, 17, 20, 21, 23, 25, 29, 30}
Solution (e){11, 12, 13, 14, 16, 20, 21, 23, 25, 29, 30}
Given that Y = {a, b, c} and R1 = {(a, a), (b, b), (c, c), (a, b)}.Hence
Solution
R1 is an reflexive and anti-symmetric re- ε = {10, 11, 12, 13, 14, 15, 16, 17, 18, 19, 20, 21, 22,
lation (E) 23, 24, 25, 26, 27, 28, 29, 30 and
A = {11, 12, 14, 16, 20, 21, 23, 25, 29} ∴
6. Given X={1, 2, 3, 4} and R={(1, 1), (2, 2), (3, 3)},A0 = {10, 13, 15, 17, 18, 19, 22, 24, 26, 27, 28}
which of the following is true? (C)
(a)R is reflexive (b)none of the above
(c)R is both reflexive and identity 11. Let A={x|1 < x ≤ 3} and B={x|1 ≤
(d)R is an identity relation x < 3} be two sets of real numbers. which
Solution of the following defines the set A ∩ B?
If X={1, 2, 3, 4} and R = {(1, 1), (2, 2), (3, 3)}R (a){x|1 > x ≤ 3} (b){x|1 < x < 3}
is not equivalence relation (B) (c){x|1 < x ≤ 3} (d){x|1 ≤ x ≤ 3}
(e){x|1 ≤ x < 3}
7. Given A={1, 2, 3} if R={(1, 1), (2, 3)} which Solution
of the following is true about R If A = {x|1 < x ≤ 3}={2, 3}
(a)R is reflexive (b)R is symmetric A = {x|1 ≤ x < 3}={1, 2}
(c)R is a universal relation (d)R is tran- ∴ A ∪ B = {1, 2, 3}
sitive (e)R is anti-symmetric A ∪ B = {x|1 ≤ x ≤ 3} (D)
Solution
Given that A={1, 2, 3} and R=(1, 1), (2, 3)} 12. Let A={2, 3} and B={1, 2, 3, 4}. Let R
then R is anti-symmetric relation (D) be a relation defined as xRy, for x ∈ A
x ∈ B, if and only if x is not relatively
8. Which of the following is not true? prime to y. Then R=
(a)an empty set is a set with no element (a){(2, 2), (2, 4), (3, 3), (3, 1)}
(b)an empty set is a subset of every set (b){(2, 1), (2, 2), (3, 3)}
(c)φ is a member of P (φ) (c){(2, 2), (2, 4), (3, 3)}
(d)0 is a member of {} (e)P (φ) is non- (d){(2, 1), (2, 2), (2, 3), (2, 4), (3, 1)}
empty (e){(1, 2), (1, 3), (2, 2), (3, 3), (4, 2), (4, 3)}
Solution Solution
0 is a member of ∅ which is not true (D) If A = {2, 3} and {1, 2, 3, 4}.If R = {xRy, f or x ∈
A and y ∈ Bif f x is not relatively prime to y}
9. If A and B are any sets, then the inter- R = (2, 2), (2, 4), (3, 3) (C)
section of A and B denoted by A ∩ B is
defined as 13. Let X={1, 2, 3, 4, 5}.Define a relation R
(a){x|xis not element of A and x is not on X such that xRy if and only if x-y=3k,
element of B} for some integer k. Then R−1 =
(b){x|x ∈ A or ∈ B} (a){(1, 1), (4, 1), (2, 2), (3, 3), (1, 4), (4, 4), (2, 5),
(c){x|x ∈ A and x is not element of B} (5, 2), (5, 5)}
(d){x|xis not element of A and x ∈ B} (b){(1, 1), (1, 4), (2, 2), (3, 3), (4, 1), (4, 4), (2, 5),
(e)x|x ∈ A andx ∈ B} (5, 2), (5, 5)}
Solution (c){(1, 1), (2, 2), (3, 3), (4, 4), (2, 5), (5, 2), (5, 5)}
A ∩ B is defined as {x|x ∈ A andx ∈ B} (d){(1, 2), (2, 1), (1, 3), (3, 1), (1, 4), (4, 1), (2, 5),
(E) (5, 2), (5, 5)}
0.22. MATHS 101 2013/2014 105

(e){(1, 1), (1, 4), (2, 2), (3, 3), (4, 4), bers and Z the set of integers, which of
(2, 5), (5, 2), (5, 5)} the following is true?
Solution (a)C ⊂ N ⊂ Q ⊂ R ⊂ Z
If X = {1, 2, 3, 4, 5} ∴ R = {x − y = 3k} (b)N ⊂ Z ⊂ R ⊂ C
where k is any integer.If k=0 (c)C ⊂ N ⊃ Q ⊂ R ⊂ Z
x-y=0{(1, 1), (2, 2), (3, 3), (4, 4), (5, 5)} If (d)N ⊂ Z ⊂ C ⊂ R ⊂ Q
k=-1 x-y=-3{(1, 4), (2, 5)} If k=1 (e)(a)N ⊂ Z ⊂ Q ⊂ R * C
x-y=3{(4, 1), (5, 2)} Solution
∴ R = (1, 1), (2, 2), (3, 3), (4, 4), (5, 5), (1, 4), If N ⊂ Z ⊂ R ⊂ C (C)
(2, 5), (4, 1), (5, 2)}
R−1 = {(1, 1), (2, 2), (3, 3), (4, 4), (5, 5), (4, 1), Use the following information to answer
(5, 2), (1, 4), (2, 5)} Therefore (A) or (B) questions 18-19:suppose z is any complex
number and z̄ is the complex conjugate
14. Let A={1, 2, 3, 4} and B={1, 2, 3, 4, 9, 16}.Let of Z.
a function f, assign to each element of A
−−−−−−−→ 18. z = z̄ if and only if
its square is f: A x 7→ x2 B.Then the
range of f is (a)B (b){1, 2, 3, 4} (c)None (a)z + z̄ = 0 (b)z is purely imaginary
of the option (d)φ (e){1, 4, 9, 16} (c)z is purely real (d)z − z̄
Solution (e)none of the option
If A={1, 2, 3, 4} and B={1, 2, 3, 4, 9, 16} Solution
−−−−−−→ Let z̄ = (1 + 0i) = 1 z̄ = (1 − 0i) = 1
∴f:A x 7→ x2 B then
∴ z = z̄(if z is purely real) (C)
f:A −→ B = {(1, 1), (2, 2), (3, 9), (4, 16)}
∴ the range of f is {1, 4, 9, 16} (E) 19. z = −(z)¯ if and only if
(a)z is purely real (b)none of the option
15. Let A and B be any sets. Then De- (c)z + z̄ = 0 (d)z − z̄ = 1 (e)z is purely
Morgan’s law states that imaginary
(a)A ∪ A1 = U, A ∩ A1 = φ Solution
(b)A ∪ A = A, A ∩ A = A z=(2+2i) z̄ = (2 − 2i) ∴ ∴ z̄ = −(2 −
(c)(A∪B)1 = A1 ∩B 1 , (A∩B)1 = A1 ∪B 1 2i)=-2+2i ∴ z = −z̄
(d)(A ∪ B)1 = A1 ∪ B 1 , A ∩ B =, B ∩ A 2+2i = -2+2i (hence z = z̄ if and only if,
(e) A ∪ φ = A, A ∩ φ = φ it is purely imaginary)(E)
Solution
The De-Morgan’s law states that (A ∪ 4 + 7i
20. The simplification of gives
B)0 = A0 ∩ B 0 (C) 2 + 3i
29 + 2i −17 + 2i
(a) (b) (c)1 + 3i
16. Which of the following is not a property 13 13
of the set of integers? (d)−1 + 2i (e)−1 − 2i
(a)every integer has a multiplicative in- Solution
4 + 7i 2 − 3i 8 − 12i + 14i + 21
verse (b)integers are closed under the op- × =
eration of addition. 2 + 3i 2 − 3i 4+9
29 + 2i
(c)the set of integers is associative = (A)
13
(d)the set of integers is commutative √
(e)integers are closed under the opera- 21. The complex number −1 + i 3 in polar
tion of subtraction formis 
Solution 2π 2π
(a)2 cos − i sin
Every integer has a multiplicative inverse 3 3
 π π
is not true for a set of integers (A) (b)2 − cos + i sin
√  3 3
π π
17. Let C be the set of complex numbers, N (c) 3 cos − i sin
3 3 
the set of rational numbers, Q the set of

2π 2π
rational numbers, R the set of real num- (d)2 cos + i sin
3 3
106
√  π π
(e) 3 − cos − i sin From De-moivers theorem
3 3 1 1 θ
Solution z n = r n (cos + i sin nθ )
√ pn √
Given −1 + i 3 where r = x2 + y 2 = 12 + 02 = 1
diagram θ = tan−1 ( xy ) = tan−1 ( 01 ) = 0
p q √
modulus= x2 + y 2 = (−1)2 + ( 3)2 ∴ one of the root is
√ √ 1 1
z 3 = 1 3 (cos 0 + i sin 0)
= 1 + 3 = 4 ∴ r=2
√ √ ! z=1
y 3 3
tan α = = =α = tan−1 = Since for a cube root,three root are re-
x 1 1
quired ,in the argand diagram we have
600 ∴ 180 − 60 = θ(angle on a straight 360
line equal to 180o ) θ = 120 = = 120
3
π 2π Hence the three root are 1200 apart since
In radian 120 × =
√ 180 3 the first root is at 00 the others will be
∴ (−1 + i 3) in polar form is at 1200 and .........from the
r(cos θ)+i sin θ) = 2{(cos(2π/3)+i sin(2π/3)} ∴ z = 1(cos 0 + i sin 0) or
(D) z = 1(cos(120) + i sin(120)) or
z = 1(cos(240) + i sin(240)) √ √
22. The argument of the complex number
π π π 1 3 −1 + 3i
−2 − 2i is (a)-1 (b)− (c)−3 (d)− ∴ z = 1 or z = − + i =
4 4 2 2 2 2
π or √ √
(e)3 1 i 3 −1 − 3i
2 z=− − = (B)
Solution 2 2 2

24. The square roots of the complex number


2 −√2i are π √ −7π
(a) 4 8ei(− 8 ) and 4 8ei( 8 )
√ π √ π
(b) 8ei(− 8 ) and 8ei( 8 )
√ −3π √ 5π
(c) 4 8ei( 8 ) and 4 8ei( 8 )
√ π √ π
(d) 8ei(− 8 ) and 8ei(− 8 )
√ π √ 7π
(e) 4 8ei(− 8 ) and 4 8ei( 8 )
Solution
√ find the square 1root of 2 − 2i i.e
To
2 − 2i = (2 − 2i) 2
x −2 √ 1
tan α = = =1 let 2 − 2i = z ∴ Z 2
y −2 from de-movies theorem
α = tan−1 1 = 45 1 1
Z n = r n (cos nθ + i sin nθ )
Argβ = 180 + α = 180 + 45 = 2250 p √
Argγ = −1350 (since it is in clock-wise Where r = 22 + (−2)2 = 8
direction) θ = tan−1 ( −2 2
) = tan−1 (−1) = −450
π 5π for square root,two root are required ,there-
in radian 225 × = or 360
180 4 fore in the argand diagram we have =
π 3π 2
−135 × = − (C) 0
180 apart ,hence the two root are 180
180 4 1 √ 1 45
apart ,for the first root Z 2 = ( 8) 2 (cos +
23. Which of the following is√a cube root of 2
−1 − i 3 45
unity? (a)0+i (b) i sin )
√ 2 √ √2
1−i 3 1+i 3 = 4 8(cos 22.5 − i sin 22.5)
(c) (d)-1+0i (e) since α = 22.5 ; β = (180 − 22.5) =
2 2 0
Solution 157.5√ Hence the two root are
4
To
√ find√the cube root of unity i.e z=√ 8(cos(22.5) − i sin(22.5)) and
3 1 4
1 = 3 1 + 0i = (1 + 0i) 3 z = 8(cos(157.5) − i sin(157.5)).
0.22. MATHS 101 2013/2014 107

In exponential form z=reiθ (where θ is in answer)


radian)
π π 28. Let A and B 2 sets, such that A ∩ B = φ,
22.50 in rad=22.5 × =−
180 8 and A ∪ B = U . All are true except
0 π 7π (a)(A \ B)1 ⊂ A (b)(A \ B)1 = B
157.5 in rad=157.5 × =
√ 180
√ 8 (c)(B \ A = B) (d)A1 ∪ B 1 = B
∴ z = 8ei(−π/8) and 8ei(−7π/8) (E)
4 4

(e)A ∩ B = B ∩ A
Solution
25. Let Z = r(cos nθ + i sin nθ be a com-
If A ∩ B = φ, A ∪ B = U A \ B = A and
plex number and n a rational number,
(A)0 = B Hence B* A (A)
De-Moivre’s theorem states that
(a)Z n = rn (cos nθ + i sin nθ) 29. Let A and B be 2 sets, and A ∪ B = U ,
(b)Z n = rn(cos nθ + i sin nθ) which of the following is not true
(c)Zn = rn (cos nθ + i sin nθ) (a)A1 \ B 1 = (B ∪ A)1 (b)A \ B = φ
1 1
(d)Z n = r n (cos nθ + i sin nθ ) if A=B (c)A ∩ B = φ,A \ B = A (d)if
(e)Z n = rn (cosn θ + i sinn θ) A=B,(B \ A)1 = U
Solution Solution
De-Moivre’s theorem states that If A ∪ B = U A0 \ B 0 = (B ∪ A)0
Z n = rn (cos nθ + i sin nθ) (A) A0 \B 0 = (U )0 A0 \B 0 = φ Hence A0 \B 0 6=
φ Since A 6= B (A)
26. In the complex plane, the set of points
satisfying the equation z 2 = |z|2 is a 30. Let A,B and C be sets, (A1 ∪ B 1 ) ∩ C 1 =
(a)pair of points (b)line (A1 ∩ C 1 ) ∪ (B 1 ∩ C 1 ) is justified by
(c)set of intersecting lines (d)circle (a)Distributive law (b)Associative law
(e)union of infinitely many different lines (c)Idempotent law (d)De Morgan’s law
Solution Solution
(A0 ∪ B 0 ) ∩ C 0 = (A0 ∩ C 0 ) ∪ (B 0 ∩ C 0 ) is

justified by distributive (A)
27. If z = ei 5 ,then
1+z+z 2 +z 3 +5z 4 +4z 5 +4z 6 +4z 7 +4z 8 +5z 9 = 31. If A and B are any sets, then the dif-
3π 3π 4π 2π
(a)−5ei 5 (b)4ei 5 (c)5ei 5 (d)−4ei 5 ference of A1 B 1 denoted by A1 \ B 1 is
(d)0 (e)1 defined as
Solution (a){x|x ∈ A1 and x ∈ B 1 }

If z = ei 5 ,then (b){x|x ∈ A and x ∈ B}
1+z+z 2 +z 3 +5z 4 +4z 5 +4z 6 +4z 7 +4z 8 +5z 9 (c){x|x ∈ A1 and x is not element of B 1 }
using De-Moivre’s theorem ∴ (d){x|x ∈ A1 and x is not element of B}
1 + (1 < 72) + (1 < 72)2 + (1 < 72)3 + (e)none of the options
5(1 < 72)4 + 4(1 < 72)5 + 4(1 < 72)6 + Solution
4(1 < 72)7 + 5(1 < 72)8 + 6(1 < 72)9 A0 \ B 0 ={x|x ∈ A1 and x is not element
∴ 1 + (1 < 72) + (12 < 2 × 72) + of B 1 } (C)
(13 < 3 × 72) + 5(14 < 4 × 72) +
4(15 < 5 × 72) + 4(16 < 6 × 72) + 32. Which of the following is true?
7 8
4(1 < 7 × 72) + 4(1 < 8 × 72) + (a)an empty set is denoted by {φ}
5(19 < 9 × 72) (b)φ is a member of P{φ}
=1 + (1 < 72) + (1 < 144) + (1 < 216) + (c)an empty set is an element of every set
5(1 < 288) + 4(1 < 360) + 4(1 < 432) + (d)an empty set is a singleton set
4(1 < 504) + 4(1 < 576) + (1 < 648) (e)φ is a singleton
1+(1 < 72)+(1 < 144)+(1 < 216)+(5 < Solution
288) + (4 < 360) + (4 < 504) + (4 < φ is a member of P(φ) (B)
576) + (5 < 648)=5 < −72=-72 in ra-
π 2π 33. If A = {1, 2, 3} and
dian −72× = − =∴ 5e(−2π/5)i (no R1 = {(a, b) : a−b < 0 and a, b ∈ A}.then,
180 5
108

the number of elements in R1 is options


(a)4 (b)2 (c)0 (d)5 (e)3 Solution
Solution Anti-symmetric relation is best defined
If A={1, 2, 3} R1 = {(a, b) : a − b < as (a,b)∈R if a=b (A)
0 and a, b ∈ A}
∴1−2<0 38. All the following are examples of sets ex-
: 1−3<0 cept
: 2−3<0 (a){a} (b)∅ (c)φ
∴ R1 = {(1, 2), (1, 3), (2, 3)} (d){{a}, φ} (e){a, φ}
Therefore number of element in R1 = 3 Solution
(E) {{a}, φ} is not a set, it is called a family
of set (D)
a−b
34. If A = {1, 2, 3} and R2 = {(a, b) : =
2 39. All the following are examples of family
1 and a, b ∈ A}.then, the number of ele-
of sets except ?
ments in R2 is
(a){∅} (b){∅, a} (c){{a}, {b}{ (d){φ, {b}{
(a)1 (b)7 (c)3 (d)0 (e)2
(e){φ}
Solution
Solution
If A={1, 2, 3}
a−b {∅, a} is not a family of set,it is called a
R2 = {(a, b) : = 1 and a, b ∈ A} set since it can be represented as {a} (B)
2
3−1
=1 40. Let S be a space consisting of all straight
2
R1 = {(3, 1)} lines. Let R be a relation on S defined
∴ The number of element in R2 = 1(A) by ”x is perpendicular to y”, R is
(a)Reflexive (b)An equivalence relation
35. (A ∩ B) \ C could be expressed as (c)Anti-Symmetric (d)Transitive (e)Symmetric
(a){x|x ∈ A and x ∈ B 1 } and x ∈ C 0 Solution
(b){x|x ∈ A or x ∈ C} If S is a space consisting of all straight
(c){x|x ∈ A1 or x is not element of C} lines and S defined by ”x is a perpen-
(d){x|x ∈ C and x ∈ B andA0 } dicular to y. Hence R is said to be an
(e){x|x ∈ A0 and B 0 orx ∈ C} anti-symmetric relation (C)
Solution
(A∩B)\B could be expressed and {x|x ∈
A and x ∈ B 1 } and x ∈ C 0 (A)
0.23 MATHS 101 2014/2015
36. Which of the following is not true?
(a)if a, b ∈ N, (a − b) ∈ N 1. Which of the following is incorrect ?
(b)if a, b ∈ Z, (a − b) ∈ Z (a) empty set is a subset of every set
a (b) |P (A)| = 2|A|
(c)if a, b ∈ R, ∈ N
b (c) |A × B| = |A||B|
a
(d)if a, b ∈ Q, ∈ Q (d) empty set contains no element
b (e) 0 ∈ φ
(e)if a, b ∈ N, (a + b) ∈ N
Solution Solution
If a,b∈ N, (a − b) is not an element N if Since an empty set is a set with no ele-
b>a hence natural number is not closed ment i.e 0 @ φ (E)
under subtraction (A)
2. Given that√Z = x + 2i ,for what value of
37. Which of the following best define an anti- x is |Z| = 10 √ √ √
symmetric relation? (a)if a 6= b, theneither (a, b)(a)
∈ ±8 (b) ±6 (c) ± 14 (d) ± 6 (e) 6
Ror(b, a) ∈ R,never both (b)if a 6= b, then (a, b) ∈Solution
√ Given that z = x + 2i |z| =
Rand(b, a) ∈ R (c)(a, b) ∈ R if a = b √ 10 p
(d)if a=b, then (a,b)∈R (e)none of the 10 = x2 + y 2
0.23. MATHS 101 2014/2015 109

2
10 = x√ +4 the above
x = ± 6(D) Solution

3. Let Z1 and Z2 be two complex numbers,which


of the following is not true
(a) Z1 + Z2 = Z1 + Z2
(b) Z1 Z2 = Z1 Z2
(c) |Z1 | = |Z1 | (d) |Z1 | = |Z2 | If Z1 = Z2
(e) |Z1 + Z2 | > |Z1 | + |Z2 |
Solution
|z1 + z2 | > |z1 | + |z2 | is correct rather it
is stated as If x is the number of student who offer
|z1 + z2 | < |z1 | + |z2 |(E) statistics only .
∴ x+ 50 +100 = 200
4. Let f be a function from the set of real x = 150(A)
1
numbers to itself, defined by f(x) = ,then
|x| 8. Which of the following is true
the range of f is (a) power of a set is the set of all set
(a) set of non-positive real numbers (b) every set is an element of its power
(b) set of negative real numbers set
(c) set of positive real numbers (c) the set of all set exist
(d) set of non-negative real numbers (d) the cardinality of every set is greater
(e) none of the above than that of its power sets
Solution (e) none of the above
1
If f(x) = ,then the range of f is a set Solution
|x|
of non-negative real numbers (D) Every set is an element of its power set
(B)
5. Let A and B be any two sets .Then De-
Morgen’s law state that 9. Let A = (0, ∞) and B = (−∞, 5] then
(a) A ∪ A = A, A ∩ A = A (A/B)0 =
(b) (A ∪ B)0 = A0 ∩ B 0 (a) (5, ∞) (b) (−∞, 5) (c) (−∞, 5] (d)
(C) A ∪ A0 = ∪, A ∩ A0 = φ (−∞, 0) (e) none of the above
(D) (A ∪ B)0 = A0 ∪ B 0 Solution
(e)none of the above If A = (0, ∞) and B = (−∞, 5]
Solution
The De-morgan’s law state that
(A ∪ B)0 = A0 ∩ B 0
(A ∩ B)0 = A0 ∪ B 0 (B)

6. If X,Y and Z are any three finite sets then


|P (X × Y × Z)| =
(a) |X| + |Y | + |Z| (b) 2|X|+|Y |+|Z| (c)
|X||Y ||Z|
(d) 2|X||Y ||Z| (e) none of the above ∴ A/B = (5, ∞)
Solution (A/B)0 = (−∞, 5](C)
|p(X × Y × Z)| = 2|x|+|y|+|z| (B)
10. The modulus and argument of Z = −2 +
7. In a class of 200 students ,50 students 2i are√.......and....... √
take both statistics and mathematics,100 (a) 2 2 and 3π (b) 4 and π
(c) 2 2 and
π
√ 4
−3π
4
students take mathematics only.How many 4
(d) 2 2 and 4 (e) none of the above
student(s) take statistics only Solution
(a) 50 (b) 180 (c) 200 (d) 150 (e) none of Given z = −2 + 2i
110

(3, 1) : 3−1
2
=1
(4, 2) : 4−2
2
=1
∴ R1 = {(3, 1), (4, 2)}
∴ the number of element in R1 = 2(B)
14. For any two sets finite A,B ⊂ N,if A ⊃ B
(a) |A| ≥ |B| (b) |B| < |A|
(c) |A| < |B| (d) |B| ≥ |A|
p p (e) none of the above
|Z|√= r = √ x2 + y 2 = (2)2 + (−2)2 Solution
= 8=2 2 if A,B ⊂ N and A ⊃ B
tan θ = xy = 22 = 1 ∴ A⊂B=B⊃A
θ = tan−1 1 = 45 B ⊂ A ≈ |B| < |A|(B)
argument = 180 − 45 = 135 OR −180 −
15. If A and B are any sets,then the inter-
45 = −225
section of A and B denoted by A ∩ B is
In radian 3π or −5π
4 4 √ 3π defined as
∴ the modulus and argument are 2 2 and (A)
4 (a) {x|x ∈ A and x@B}
11. Let A = {1, 2, 3} and B = {1, 2, 3, 4, 8, 9, 16, 27} (b) {x|x ∈ A or x ∈ B}
.Let a function,f,assign to each element of (c) {x|x ∈ A and x ∈ B}
−−−−−− → (d) {x|x@A and x ∈ B}
A its cube that is f : Ax 7→ x3 B .Then
(e) {x|x @A and x@ B}
the range of f is
Solution
(a) B (b) {1, 2, 3, 4} (c) {1, 8, 27} (d) φ
Ans C
(e) none of the above
Solution 16. Let C be the set of complex numbers,N
If A = {1, 2, 3} and B= {1, 2, 3, 4, 8, 9, 16, 27} the set of natural numbers, Q the set of
−−−−−−→
∴ f : A x → x3 B rational numbers ,R the set of real num-
then f : A → B = {(1, 1), (2, 8), (3, 27)} bers and Z the set of integer.Which of
∴ the range of f is {1, 8, 27}(C) the following is true
(a) C ⊂ N ⊂ Z ⊂ R (B)N ⊂ Z ⊂ R
12. The complex number 1 − i in an expo- (c) R ⊂ N ⊂ Q ⊂ Z
nential
√ form is ....
√ π (d) N ⊂ Q ⊂ Z ⊂ C
π π π
(a) 2e− 4 i (b) 2e 4 i (c) 2e 4 i (d) 2e− 4 i (e) none of the above
(e) none of the above Solution
Solution √ √ Since the range of number is from :
Given 1 − i |z| = 12 + 12 = 2 N⊂Z⊂Q⊂R⊂C
θ = tan−1 1 = 450 ∴ N ⊂ Z ⊂ R(B)
∴ argument = 360 − 45 = 315 or −90 +
45 = −45 17. If 2x + 0i = eπi then the value of x is
in radian 315 × 180 π
= 7π
4
or (a) − 12 (b) 21 (c) 2 (d) −1 (e) none of the
π
−45 × 180 = − π4 above
√ 7π i √ −π i
1−i in exponential form is 2e 4 or 2e 4 (A) Solution
if 2x + 0i = eπi change eπi back to stan-
13. If K= {1, 2, 3, 4} and R1 = {(m, n) : dard form we have |z| = 1
m−n
2
= 1 and m, n ∈ K},then the num- θ = π = π × 180 π
= 1800
ber of element in R1 is from de-moivers theorem
(a) 3 (b) 2 (c) 1 (d) 0 (e) none of the z = r(cos θ+isinθ) = 1(cos 100+i sin 100)
above = 1(−1 + 0i) = −1 + 0i
Solution ∴ 2x + 0i = −1 + 0i
if k = {1, 2, 3, 4} equating the real part 2x = −1
m−n
R1 = {(m, n) : 2 = 1 and m, n ∈ k} x = −12
(A)
0.23. MATHS 101 2014/2015 111

18. Let R be a relation on the set of non- the above


negative integers,defined as xRy ,if and Solution
only if x2 + y 2 = 25,for non-negative in- The Arg of −2 + 2i are θ = tan−1 1 = 450
tegers x and y.Then R= θ = 600
(a) {(0, 5), (−5, 0), (3, 4), (−4, 3)} Arg in clockwise direction is −180−45 =
(b) {(0, −5), (−5, 0), (−3, −3), (−4, −3)} −2250
(c) {(0, 5), (5, 0), (3, 4), (4, 3) Arg in Anti-clockwise direction is 180 −
(d) {(0, 5), (5, 0), (3, 4), (4, −3) 45 = 1350
(e) none of the above π
in radian −225 × 180 = −5π
4
π
Solution 135 × 180 = 3π
4
R= {xRy |x2 + y 2 = 25, f or any non − ∴ the Arg is either −5π
4
or 3π
4
(B)
negative integer}
(0, 5) = 02 + 52 = 25 21. Let R be a relation on the set of integer
(5, 0) = 52 + 02 = 25 defined as xRy ,if and only if x2 + y 2 =
(3, 4) = 32 + 42 = 25 4,for integer x and y.Then R is ?
(4, 3) = 42 + 52 = 25 (a) {(0, 4), (0, −2), (2, 0), (0, −4)}
∴ R = {(0, 5), (5, 0), (3, 4), (4, 3)}(C) (b) {(0, 0), (0, −2), (2, 0), (−1, 0)}
(c) {(0, 2), (0, −2), (2, 0), (−2, 0)}
√ (d) {(0, 2), (0, −2), (2, 0), ()0, 4} (e) none
19. −1 + i 3 in polar form is
(a) 2(cos 2π −i sin 2π ) (b) 2(cos 2π +i sin 2π ) of the above
√ 3 3 √ 3 3
Solution
(c) 3(− cos π3 + i sin π3 ) (d) 3(cos π3 +
R = {xRy|x2 +y 2 = 4, f or integer x and y}
i sin π3 )
(0, 2) = 02 + 22 = 4
(e) none of the above
(0, −2) = 02 + −22 = 4
Solution √ (2, 0) = 22 + 02 = 4
−1 + i 3 can be written in polar form as
q √ √ (−2, 0) = −22 + 02 = 4
2
∴ |z| = r = (−1) + ( 3) = 4 = 2 2

∴ R = {(0, 2), (2, −2), (2, 0), (−2, 0)}(C)
3
∴= 1 = 60 0 √
22. Given that Z1 = 1 + 3i, Z2 = 1 + i the
Arg(z1 z2 z2 ) and |z1 z2 z2 | are respectively
(a) π3 and 4 (b) √π3 and 4 (c) −π 3
and 3
π
(d) 6 and 4 (e) None
Solution √
if z1 = 1 + 3i and z2 = 1 + i
z2 = 1 − i
z2 z2 = (1 + i)(1 √ − i) = 2 √
z1 z2 z2 = (1q+ 3i)2 = (2 + 2 3i)

Arg in clockwise direction = −180−60 = |z1 z2 z2 | = 22 + (2 3)2 = 4

−240
θ = tan−1 2 2 3 = 600
Arg in Anti-clockwise direction = 180 −
Arg in clockwise direction is −360+60 =
60 = 1200
−300
∴ Arg = −240 or 1200
Arg in Anti-clockwise direction =60
in radian −240 × 180 π
= −4π or π
π
3 in radian −300 × 180 = − 5π
120 × 180 = 2π π
3
√ 3 60 × 60 = π3
∴ −1 + i 3 in polar form are Arg z1 z2 z2 and |z1 z2 z2 | are
2(cos 4π 3
− i sin 4π
3
)or π
and 4 respectively (A)
2π 2π 3
2(cos 3 + i sin 3 )(B)
23. Let A and B be any two sets such that
20. The argument of the complex number A ∩ B = φ and A ∪ B = U ,All of the
−2 + 2i is following are true except
3π 3π π
(a)− 4 (b) 4 (c) − 4 (d) 4 (e) none ofπ
(a) (A/B)0 = A0 (b) (A/B) = A (c)
112

(B/A)=B (d) (B/A) ⊆ B (e) (A/B) ⊂ A 29. If z1 = 1 + 2i and if z2 = −3 then the


Solution modulus
√ of z1 √ + z2 is √ √
(A/B) ⊂ Ais not true because (A/B) ⊆ (a)
√ 2 6 (b) 2 5 (c) 2 3 (d) −2 2 (e)
A since A ∩ B = φ(E) 2 2
Solution
24. A single set is a set with if z1 = 1 + 2i and z2 = −3 + 0i
(a) more than one element (b) no element then z1 + z2 = (1 + 2i) + (−3 + 0i)
(c) infinite element (d) one element (e) = −2 + 2i √
none of the options √
|z1 + z2 | = 22 + 22 = 2 2(E)
Solution
Ans D 30. If A and B are any sets,then the union of
A and B denoted by A ∪ B is defined as
25. φ is a set with (a) {x|x@A and x @B}
(a) more than one element (b) no element (b) {x|x ∈ A or@ B}
(c) infinite element (d) one element (e) (c) {x|x ∈ A or x ∈ B}
none of the above options (d) {x|x@A and B}
Solution (e) {x|x ∈ A and x ∈ B}
Ans B Solution
A ∪ B is defined as {x|x ∈ Aor x ∈
26. A family or class of sets has the property
B}(C)
that
(a) each of its element is also a set 31. Let z = r(cos θ + i sin θ) be a complex
(b) none of its element is a set number.Then De-movire
(c) some of its elements are sets and oth- (a) z n = rn (cosn θ + sinn θ) (b) z n =
ers are not rn (cosn θ − i sinn θ)
(d) one of its element is a set and others (c) z n = rn (cos nθ + i sin nθ) (d) Zn =
not rn (cos nθ + i sinn θ)
(e) none of the options (e) none of the above
Solution Solution
Ans A De-movivre’s theorem state that
z n = rn (cos(nθ) + i sin(nθ))(C)
27. If z1 = 1 + 2i and if z2 = −3 then the
modulus
√ of z1 + z√2 = ...... √ √ 32. In the complex plane,the set of points
(a) 2 10 (b) 13 2 (c) 10 2 (d) 2 13 satisfying the equation z 2 = |z|2 is a
(e) none of the option (a) set of intersecting lines (b) pair of
Solution points (c) straight line (d) circle (e) none
if z1 = 4 + 3i and z2 = 2 − i of the option
then z1 + z2 = (4 + 3i) + (2 − i) = (4 + Solution
2) + (3i − i)
= 6 + 2i √ √ √
∴ |z1 +z2 | = 62 + 22 = 40 = 2 10(A) 33. Which of the following is not a property
of the set of integers ?
28. A polygamous marriage from the set of (a) integer are closed under the operation
married men to the set of married woman of subtraction
is (b) integer are closed under the opera-
(a) a 1-1 function (b) an onto function tion of addition
(c) bijective function (d) not a function (c) every integer has a multiplicative in-
(e) an onto but not 1-1 function verse
Solution (d) the set of integer is associative
This is not a function since one element (e) the set of integer is commutative
of A is related to two or more element in Solution
B (D) Ans C
0.23. MATHS 101 2014/2015 113

34. Let A,B and C sets (A0 ∩ B 0 ) ∩ C 0 = since for a cube root ,three root are re-
(A0 ∩ C 0 ) ∪ (B 0 ∩ C 0 ) is justified by quired i.e on the argand diagram we have
(a) Associativity law (b) De morgan’s law = 360
3
(c) idempotent law (d) Distributive law hence the three root are 1200 apart.since
(e) none the root is at 300 ,the others will be at
Solution 150 and 270 away from the first having
Ans D the same modulus
z1 = 3(cos 30 + i sin 30) and
35. For any p,q,r,s ∈ N, [p, q] ∪ (q, s] is ...... z2 = 3(cos 130 + i sin 130) and
(a) [p,q] (b) [p,s] (c) [p,s) (d) {p, q, r, s} z3 = 3(cos 270 + i sin 270)

(e) No sufficient information to determine 3 3
z1 = 2 √+ i 23
the answer
z2 = − 3 2 3 + 32 i
Solution
z3 = 0 − 3i(D)
Ans B
39. Let z be a non-zero complex number.Then
36. Let a,b,c ∈ R : a ≤ b and b ≤ c then the argument of z lies between
(a) a < c (b) a ≥ c (c) a ≤ c (d) b ≤ a (a) [0, π] (b) [−π, 0] (c) [−2π, 2π] (d) (−π, π]
(e) none (e) none of the option
Solution Solution
Ans C

37. The interval (2, 5] ∩ [1, 4) = 40. A set can be defined as


(a) [3, 5) (b) [2, 4] (c) [2, 4) (d) (2, 4) (e) (a) a list of not well-defined and not dis-
(2, 5) tinct objects
Solution (b) a list of well -defined and distinct ob-
(2, 5] ∩ [1, 4) jects
(c) a list of not -well defined and distinct
objects
(d) a list of well-defined and not distinct
objects
(e) none of the option
Solution
Ans B

(2, 5] ∩ [1, 4) = (2, 4)(D)

38. Which of the following is not a complex


cube root √ of 27i √
3 3 3 3 3
(a)

− 2
+ 2
i (b) −3i (c) 2
+ 32 i (d)
3 3
2
− 23 i (e) none of the option
Solution √3
To
√ find the cube root of 27i i.e 27i =
3 1
0 + 27i = (0 + 27i) 3
from de-movres theorem
1
z n = r n1 (cos( nθ ) + i sin( nθ ))

r = 02 + 272 = 27
θ = tan−1 ( 27 0
) = 900
∴ one of the rootis
1 1
Z 3 =√27 3 (cos 90 3
+ i sin
√ 3
90
)
3 3 3
= 3( 2 + i sin 2 ) = 2 + i sin 32
1
114

0.24 MATHS 105 2009/2010


R 2
= R (1 + tan R x)2 dx
= dx + tan  xdx
x2 d y R sin2 x

1. If y = 3 then d x is R
    = dx + 2x
dx
2 2 2
A.2 3x ln 3x B.6 3x ln 3 C. 3x ln 3x
2 2
cos 
  R R sin x
2
D. 3 ln 3x E. 32x = dx + sin x 2x
dx
  cos
Solution R sin x
sin x dx
Here we are going to take the loge to both cos2 x
side. Using Integration by Part,
x2
loge y = loge 3  u = sin x du = cos xdx
d x2
 sin x R sin x 1
d (loge y) = d x loge 3 dv = 2
v = 2
=
cos x  cos x
 cos x
d 2 d 1 sin x
(x loge 3) because ⇒
dx dx cos x cos2 x
d 
sin x

d (loge y) = (x2 loge 3) R
= sin x dx
  dx cos2 x R
1 dx d
(x2 loge 3)
R
= ⇒ udv = uv − vdu
y dy dx  
1 R 1
but = sin x × − (cos x) dx
d 2 cos x  cos x 
(x loge 3) [P roductRule] 
sin x R sin x

dx = − dx ⇒ sin x
R
u = x2 v = loge 3 cos x cos2 x
dy du dv 
sin x

=v +u =
R
dx +
R
sin x dx
dx dx dx 2
2x loge 3 + x2 (0) R Rcos x
= dx + tan x − dx
2x log
 e3

1 dy = x + tan x − x
= 2x loge 3 ⇒ tan x + C
y dx
dy x
= y (2x loge 3) 4. If f (x) = then the inverse func-
dx x+1
but y = 3x
2
tion f −1 is given f −1 (x) =
dy 2 Solution
= 3x (2x loge 3) x
dx f (x) =
x+1
x
let y =
x2 − 9 x+1
2. If f (x) = then x 6= 3 then f (−2)is ⇒ We make x the subject of formula
x−3 x
Solution ⇒y=
Since x 6= 3 x+1
2 y (x + 1) = x
x −9 (x − 3) (x + 3)
⇒ = yx + y = x
x−3 (x − 3) Collecting like terms
(x + 3) then
y = x − yx
f (−2) ⇒ x = −2
y = x (1 − y)
= ((−2) + 3) y
=1 x=
(1 − y)
dy Since we have made x the subject of the
3. = sec2 x, then y =? formula, we then substitute x in place of
dx
Solution y and y in place of x.
2 y
Here we are asked to just integrate ”sec x, ⇒x=
dy (1 − y)
since = sec2 x, then to get y, we are x
dx y=
integrating. (1 − x)
R 2 −1 x
sec xdxR =2 tan x + c 2 f (x) = ⇒D
2 (1 − x)
Proof sec xdx V sec x = 1 + tan x
0.24. MATHS 105 2009/2010 115

dy c
5. If x = et and y = sin 2t, then = 1
dx = − cos (2x + 3) + c
Solution 2
Here we are given a parametric differen- Proof
R
tiation where we differentiate each term sin (2x + 3) dx
(i.e x and y) Given x = et and y = sin 2t letu = 2x + 3
d t d du
(e ) = et and (sin 2t) = 2 du = 2dx
dt dt dx
d du
(sin 2t) dx =
dt 2
du R R du
Let u = 2t ⇒ =2 ⇒ sin (2x + 3) dx = sin u ×
dt 2
y = sin 2t ⇒ y = sin u 1R
= sin udu
dy 2
= cos u 1
du = (− cos u) + c
d 2
(sin 2t) = cos u × 2 recall that u = 2x + 3
dt 1
= 2 cos u = − cos (2x + 3) + c
dy d t d 2
= (e ) ÷ (sin 2t)
dx dt dt
= et ÷ 2 cos 2t R 1
et 9. √ dx is
= ⇒E 25 − x2
2 cos 2t Solution
This is an integration by substitution [Trigono-
R2
6. (4x3 − 6x) dx is metric Substitution], we use the sharp
1
Solution sharp identity
WeRwill just integrate directly
R dx −1 x
 x
⇒ √ = sin arc sin
2
⇒ 1 (4x3 − 6x) dx 25 − x2 5 5
Because;
R2 R2 R dx −1 x
 x
= 1 4x3 dx − 1 6xdx √ = sin arc sin
R2 R2 a2 − x 2 a a
= 4 1x3 dx − 6 1 xdx Proof
x4 x2

R 1 R 1
=|21 4 − 6 |21 ⇒ √ dx = √ dx
4 2 25 − x2 52 − x2
=|21 x4 − |21 3x2  let x = 5 sin θ
=|21 (2)4 − 3 (2)2 − |21 (1)4 − 3 (1)2 dx = 5 cos θdθ (P roductRule)


= [16 − 12] − [1 − 3]
R dx R 5 cos θdθ
⇒ √ ⇒ q
= [4] − [−2] 25 − x2 (5)2 − (5 sin θ)2
= [4 + 2] R 5 cos θ
=6⇒C ⇒ p dθ
25 − 25 sin2 θ
R 5 cos θ
= q  dθ
7. Not Solved 2
25 1 − sin θ
R 5 cos θ
= p dθ
5 1 − sin2 θ
R
8. sin (2x + 3) dx
Note
Solution
1p− sin2 θ = cos√ 2
θ
This one we can use the sharp sharp iden- 2
1 − sin θ = cos2 θ = cos θ
tityR R
⇒ sin (2x + 3) dx ≡ sin (ax + b) R 5 cos θ
where ⇒ dθ
R 1 R 5 cos θ
sin (ax + b) = (− cos (ax + b)) + c = dθ
a =θ+C
R 1
⇒ sin (2x + 3) dx = (− cos (2x + 3))+ But
2
116

x = 5 sin θ ⇒ y = tan x + cot x


x dy d d
sin θ = = (tan x) + (cot x)
5  dx dx dx
−1 x
θ = sin but
5 d
dx x (tan x) = sec2 x
= sin−1 + c
R
⇒ √ dx
 25 −xx
2 5 and
d
= arc sin +c (cot x) = −cosec2 x
5 dx
=A dy d d
⇒ = (tan x) + (cot x)
dx dx dx
dy
10. If f (x) = x2 ln (x2 ) then f p (x) =? = sec2 x − cosec2 x ⇒ D
dx
Solution
We have to know that f p (x) means differ-
entiating f (x) . Whenever we are given a 13. Find the second derivatives of the func-
4
function with (y p ) or (f p (x)) all they need tion f (x) = (2x3 − 1)
is differentiation ( differentiating the given Solution
function ) Since f (x) = (2x3 − 1)
So f (x) = x2 ln (x2 ) The first differential (i.ef p (x))
f p (x) =? f p (x) =[Using Chain Rule]
d 4
Using Product Rule dx
(2x3 − 1)
let u = x2 v = ln (x2 ) let u = 2x3 − 1
du
du dx
= 6x
= 2x 4
Now, instead of (2x3 − 1) , we have u4
dx
[ChainRule] d
du
= 4u3
v = ln (x2 ) d d du
let u = x2 = ×
dx du dx
dv du = 4u3 × 6x2
⇒ = ln u ×
dx dx but u = 2x3 − 1
du 3
= 2x f p (x) = 4 (2x3 − 1) × 6x2
dx 3
⇒ 24x2 (2x3 − 1) (Chain Rule) 
dv 1
= × 2x 2 3
f q (x) = 24x2 3 (2x3 − 1) × 6x2 +(2x3 − 1) (24)
dx u
dv 1 [Product Rule]
= 2 × 2x
dx x 3
f q (x) = 24 (2x3 − 1) + 432x4 (2x3 − 1)
2
dv 2
=
dx x
dv du
⇒u×v =u +v 14. If f (x) = x3 − 2x + 1 then f (−3) =?
dx dx
2 du Solution
u=x = 2x
dx Here we are just to substitute f (−3) in
2
v = ln (x ) f (x)
dv 2 ⇒ f (x) = x3 − 2x + 1
=
dx  x f (−3) = (−3)3 − 2 (−3) + 1
x2 x2 + ln (x2 ) [2x]
= −27 + 6 + 1
2x + 2x ln (x2 )
= −27 + 7
2x [1 + ln (x2 )]
= −20
⇒C
11. Not Solved
dy 15. Evaluate the limit
12. If y = tan x + cot x, then is
dx
Solution x−1
We are differentiating each term by term lim
x→1 x2 + 2x − 3
0.24. MATHS 105 2009/2010 117

Solution = x sin x − [− cos x] + c


As we can see here we can factorise the =x R sin
π
x + cos +c R π
2
denominator (x + 2x − 3) ⇒ [(x − 1) (x + 3)] = h 0
2
xcos xdx = 02 xsini + cos x + c
π  π  π
x−1 = sin + sin −[(0) sin (0) + cos (0)]
= lim 2 h π 2 2i 2
x→1 x + 2x − 3 = (1) + 0 − [0 + 1]
2
limx→1 (x − 1) π
= = −1
limx→1 (x2 + 2x − 3) 2
⇒E
limx→1 (x − 1)
=
limx→1 (x − 1) (x + 3) x−1
18. If , for all x 6= −1, then f(x) is
1 x+1
= lim Solution
x→1 (x + 3)
x−1
since f (x) =
1 x+1
= f p
(x) =differential of f (x)
(1 + 3)
1 f (2) =differential at x = 2
p
= ⇒C f p (x) = using quotient rule
4
u
f p (x) =
x2 −16 v
du
16. If the function f (x) = {c x+4 if x 6= 4, dx
(V ) − dudx
(U )
f (x) =
p
x = −4 is continuous at x = −4, what is V2
the value of c? u = x − 1, v = x+1
du dv
Solution = 1, =1
x2 − 16 dx dx
f (x) = (1) (x + 1) − (x − 1) (1)
x+4 f p (x) =
We’ve seen that (x2 − 16) is a difference (x + 1)2
of two square. (x + 1) − (x − 1)
=
2
x − 16 = (x + 4) (x − 4) (x + 1)2
x2 − 16 (x + 1) − x + 1
= =
x+4 (x + 1)2
(x + 4) (x − 4) x+1−x−1
= =
(x + 4) (x + 1)2
= (x − 4) 2
x = −4 =
(x + 1)2
⇒ (x − 4) = (−4 − 4) f p (2) = f p (x) at x=2
= −8 ⇒ B 2
=
(2 + 1)2
Rπ 2
17. 02 x cos xdx =
(3)2
Solution
Rπ 2
2
x cos xdx is an integration part and = ⇒C
0 9
substitution. so we are goin to first inte-
grate before substituting the upper and R1 2x
lower on it 19. 0
2
√ dx
Rπ 1 − x 2
= 0 x cos xdx
2
Solution
let u = x, dv = R cos xdx
R Here we have to let u = 1−x2 x cause the
du = dx, v = dv = cos xdx ⇒ sinx differential of the denominator (1 − x2 )
Rπ R
= 0 x cos
2
R xdx = udv will give us the numerator
= uv − vduR R1 2x
2
√ dx
= x (sin x) − R sin xdx
0
1 − x2
= x sin x − sin xdx put u = (1 − x2 )
118

du dy −1 −3
= −2x = (u) 2
dx du 2
du dy dy du
dx = = ×
−2x dx du dx
R 1 2x du −1 −1
2
1 ×
(u) 2 × 1
0
u2 −2x 2
Note −1 −3
√ 1 (u) 2
u ≡ u2 2
1 but u = x
√ ≡u2
−1
−1 −3

u (x) 2
R1 1 2
R1 1 ⇒B
0
2
− 1 du = 02 −u 2 du
u2 using quotient rule
−1
1 −u 2 +1
1
−u 2 +1 1 u
|0
2
+c= +c y=√ ≡
−1 1 x v
+1 dy du
(V ) − du (U )
2 1
2 = dx dx
= −2 × u 2 dx √ V2
1 1 v = x, u = 1
= |02 − 2 (1 − x2 ) 2 dv 1 du
1
= √ ,
1
h 1
i
= |02 − 2 (1 − x2 ) 2 − |0 −2 (1 − x2 ) 2 =0
dx 2 x d
 2 ! 12 √ 1

1 1 h 1
i x (0) − 1 2 x √
= |02 −2 1 − −|0 −2 (1 − 02 ) 2 = √ 2
2 ( x)
  12 −1

3 =
2 x
= −2 − (−2) x
r4 −1 √
3 x×x
= −2 +2 2
4 −1 1
√ ! = √
3 2 ( x × x)
= −2 +2
2 recall indices a2 ×a = a(2+1)
√ −1  −1
=− 3+2 = x 2 × x−1
√ 2
√ 2 3 −1  −1 −1 
=2− 3≡ − = x2
√ 2 2 2 
3 −1 −3

1− = x2
2 2
⇒A ⇒B

R1
21. 0
(3x − 2)2 dx
1 Solution
20. If y = √ then y p =? R1
(3x − 2)2 dx
x 0
Solution let u = 3x − 2
Here we can use two methods either we du
=3
can use ”chain or quotient” rule dx
du = 3dx
using chain rule du 3dx
1 =
y=√ 3 3
x du
dx =
1 −1 3
y= 1 ≡ (x)
2
R1 2 R 1 2 du
(x) 2
0
(3x − 2) dx = 0
u ×
−1 3
y = (x) 2 1 1 2
R
du = u du
let u = x, =1 3 0
−1
dx 1 u2+1
y = (u) 2 = |10
3 2+1
0.24. MATHS 105 2009/2010 119

R3 x+1 1 R3 1
2+1 ⇒ 1 x2 +2x
dx ≡ du
1 u 2 1 u
= |10 +c 1
3 3 =|31 ln u
1 1 2
= × |10 u3 + c but u = x2 + 2x
3 3 1
=|31 ln (x2 + 2x)
1 2
= |10 u3 + c 1  1
= ln (3)2 + 2 (3) − ln (1)2 + 2 (1)

9
1 1 2 2
= (3 (1) − 2)3 − (3 (0) − 2)3 0 1
9 (ln (a + b) − ln (3))
1 2
= (3 − 2)3 − (−2)3
 1
9 (ln (15) − ln (3))
2
1 from law of logarithm
= (1) − (−8)
 
A
9 log A − log B = log
1 B
= (1 + 8) 1
9 (ln (15) − ln (3))
2  
9 1 15
= ln
9 2 3
1
=1⇒D ln 5
2
⇒A
d
22. (ln e2x ) =
dx 24. Not Solved
Solution
d 25. Not Solved
(ln e2x ) = using chain rule
dx
du 26.
let ue2x , = 2e2x 1 − cos2 x
dx lim
du 1 x→0 x2
y = ln u, =
dx u
dy dy du Solution
= ×
dx du dx Using ”L’Hospital Rule” i.e we differen-
1
= × 2e2x tiate each term.
u
but u = e2x 1 − cos2 x
d lim
= (lne2x ) x→0 x2
dx
1
= 2x × 2e2x
e d
=2⇒E limx→0 (1 − cos2 2x)
⇒ dx
d 2
limx→0 (x )
R3 x+1 dx
23. 1
dx
x2 + 2x NOTE:
Solution
R3 x+1 1 − cos2 2x = sin2 2x
1
dx
x2 + 2x
let u = x2 + 2x limx→0 sin2 x
du = (2x + 2) dx limx→0 x2
du
dx =
R 3 2x+1
(x + 1) d
sin2 x

⇒ 1 x2 +2x dx limx→0
dx
x+1 du d 2
= × limx→0 (x )
u 2 (x + 1) dx
120

2+x
28. y = then y p =
2 (2 sin 2x cos 2x) 3x + 1
⇒ Solution
2x
4 sin 2x cos 2x
= dy
2x yp =
Since if we substitute our limit, we differ- dx
d
(4 sin 2x cos 2x) Differential of y using quotient rule
entiate again ⇒ x → 0 dx d
2+x
(2x) y=
dx 3x + 1
Using Product Rule u=2+x
4 sin 2x (−2 sin 2x) + cos 2x (8 cos 2x) v = 3x + 1

2 du
2
=1
−8 sin 2x + 8 cos2 2x dx
lim dv
x→0 2 =3
dx
vdu
8 cos2 2x − 8 sin2 2x dy − udv
=x→0 = dx 2 dx
2 dx v
8 cos 2x − sin2 2x dy (3x + 1) (1) − (2 + x) (3)
2

lim =
x→0 2 dx (3x + 1)2

x → 0 4 cos2 2 (0) − sin2 2(0)



(3x + 1) − (6 + 3x)
⇒ 4 cos2 2 (0) − sin2 2 (0) =
(3x + 1)2
= 4 (1 − 0) (3x + 1) − 6 − 3x
=4 =
(3x + 1)2
1 − cos x
lim =4⇒E −5
x→0 x2 yp =
(3x + 1)2
Rk
27. If 0 (2kx − x2 ) dx = 18, then k =? 4
Solution 29. If f (x) = ,g (x) = 2x then f (g (x))
Rk 2
x−1
R 0 (2kx R− x )dx = 18
⇒ at x = 2
k k
2kx − 0 x2 dx = 18 Solution
0
Rk Rk 2 If we are given f(x) and f(a), they mean
0
2kxdx − 0
x dx = 180 we are going to substitute x with a but
2 3
2kx x here we are given g(x) and f(x), and ask
|k0 − |k0 + c = 18
2 3 to find f(g(x)) so everywhere we are deal-
x3 ing with (x) we substitute g(x).
|k0 kx2 − = 18
3
(k)3 4
k (k)2 − = 18 f (x) =
3 x−1
k3 f (x) = 2x at x = 2
k3 − = 18 4
 3 3 3 k f (g (x)) =
3k − k (2x) − 1
= 18 4
3 0 = at x = 2
2k 3 2x − 1
= 18 4
3 =
3
2k 2 (2) − 1
= 54
2 4
k√3 = 27√ =
3 4−1
k 3 = 3 27
4
k=3⇒C = ⇒A
3
0.24. MATHS 105 2009/2010 121

x + 2x3 dy 4
30. Evaluate limx→∞ =
1 + x3 dx −8
Solution −1
When limit is tending to infinity we di- ⇒C
2
vide with the highest power of ”x”.

x + 2x3 x3 − 8
= limx→∞ 32. Evaluate limx→2
1 + x3 x2 − 4
Solution
3
using our L-hospital rule
x
x3
+ 2x
x3
d
(x3 − 8)
= limx→∞ 1 3 i.e limx→2 dx
x3
+ xx3 d
(x2 − 4)
dx
(3x2 )
1
+2 = limx→2
= x2
limx→∞ 1 2x
+1 at x=2
x3

3 (2)2
limx→∞ = 2 ⇒ A =
2 (2)
3 (4)
=
4
dy
31. If 2x3 + 2xy − 3y 2 = 2, then at the =3⇒D
dx
point (1, -1) is
Solution R sin2 x
y = 2x3 + 2xy − 3y 2 = 2 (differentiate 33. Evaluate dx
1 + cos x
implicitly)     Solution
dy dy In this case when we are given a trig func-
⇒ 6x2 + 2x + 2y − 6y =c
dx dx tion with denominator having 1+ or 1-,
collect like term     something we can rationalise the denom-
2 dy dy
⇒ 6x + 2y + 2x − 6y =c inator as in the case
dx dx R sin2 x
dx
   
2 dy dy
6x + 2y = 6y − 2x 1 + cos x
dx dx on rationalising
dy R sin2 x 1 − cos x
6x2 + 2y = (6y − 2x) = ×
dx 1 + cos x 1 − cos x
divide through with ”6y − 2x” R sin2 x (1 − cos x)
= dx
(1 + cos x) (1 − cos x)
dy
6x2 + 2y (6y − 2x) R sin2 x (1 − cos x)
= dx =
(6y − 2x) (6y − 2x) 1 − cos x + cos x − cos2 x
dy 6x2 + 2y R sin2 x (1 − cos x)
= =
dx 6y − 2x 1 − cos2 x
at the point (1, -1) 1 − cos2 x = sin2 x
we then substitute the point (1, -1) in
place of (x and y) R sin2 x (1 − cos x)
i.e (1, −1) = (x, y) = dx
sin2 x
R
dy 6 (1)2 + 2 (−) = (1 − cos x) dx
=
dx 6 (−1) − 2 (1) R R
= dx − cos xdx
dy 6−2
=
dx −6 − 2
= [x − (sin x) + c]
122
 
1
= limx→0 x
= x − sin x + c ⇒ E (None of these) sin x
d
(x)
= d dx
√ dy dx
(sin x)
34. if x = t3 − t and y = 3t + 1 then at 1
dx =
t=1 cos x
Solution 1 1
limx→0 = =
We are given parametric equation where cos (0) 1
we just differentiate y and x differentiate =1⇒B
and divide the derivatives of x with y
x=√ t3 − t note
1
y = 3t + 1 ≡ (3t + 1) 2 we can use these L-hospital rule to any
d 3 limit that is not factorizable, and some
(t − t) = 3t2 − 1 trigonometric limits, where we just dif-
dt
dx ferentiate singly without following any rule.
= 3t2 − 1
dt
d
(3t + 1)
dt
using chain rule 2 sin 2x
36. limx→0
du sin 4x
=3 Solution
dt
As usual (L- hospital)
d 1 1 −1
U2 ⇒ U 2
du 2 d
(2 sin 2x)
dx
d 1 1 = limx→0 d
= U2 ×3 dx
(sin 4x)
dt 2 d
3 1 = (2 sin 2x) (chain rule)
⇒ dx
2 (3t + 1) 12 du
let u = 2x, =2
dy 3 dx
=√ d
dt 3t + 1 = 2 sin u, (2 sin u) = 2 cos u
d
dy dy dx = 2 (2 cos u)
= ÷
dx dt dt = 4 cos u
dy dt u = 2x
× 4 cos 2x
dt dx d
3 (2 sin 4x) = 4 cos 4x (chain rule)
⇒√ ÷ 3t2 − 1 dx
3t + 1 d
(2 sin 2x)
3 1 = limx→0 dxd
=√ × 2 (sin 4x)
3t + 1 3t − 1 dx
4 cos 2x
att = 1 = limx→0
3 1 4 cos 4x
=p × 2 cos 2x
3 (1) + 1 3 (1) − 1 = limx→0
3 1 cos 4x
=√ × cos 2 (0)
4 2 =
3 cos 4 (0)
= ⇒B cos (0)
8 =
cos (0)
35. limx→0 xcosecx 1
=
Solution 1
we use a hospital rule
d =1⇒E
= limx→0 (xcosecx)
dx
0.25. MATHS 105 2010/2011 123

|x| = −c2 (ac sin ct + bc cos ct)


37. limx→0
x | {z
y
}
Solution 2
dy
d = −c2 y ⇒ B
|x| dx2
limx→0 dxd
dx
x 39. If f (x) = x4 − 4x2 then f qp (2)equals
1 Solution
⇒ =1
1 f qp (2) = differentiatingf (x) three times
1
limx→0 = 1 f (x) = x4 − 4x2
x f p (x) = 4x3 − 8x
⇒C
f q (x) = 12x2 − 8
f qp (x) = 24x
38. if y = a sin ct + b cos ct where a,b and c
d2 y f qp (2) = 24 (2)
are constants, then 2 = 48 ⇒ A
dt
Solution
y = a sin ct + b cos ct R dx
d2 y d2 d2 40. Evaluate
= (a sin ct) + (b cos ct) 3 − 5x
dx2 dt2 dt2 Solution
d R dx R dx
(a sin ct) = chain rule i.e since a is con- Remember =
dt 3 − 5x ax + b
stant R 1
y = a sin ctletu = ct = log (ax + b)
a
du R dx R dx
= c ⇒ y = a sin u ⇒ =
dt 3 − 5x b − ax
dy −1
= a cos u ⇒ log (3 − 5x) + c
du 5
dy
= a cos u × c
dt ⇒B
d
(a sin ct) = ac cos ct
dt Proof
d2 d R dx
2
(a sin ct) = (ac cos ct)
dt dt 3 − 5x
d
⇒ (ac cos ct)
dt let u= 3 − 5x
⇒ −ac sin ct × c (chain rule)
d2 y du
= ac2 sin ct...............(1) = −5
dt2 dx
d2 d d du = −5dx
2
(b cos ct) = (b cos ct) × (b cos ct) −du
dt dt dt dx =
d 5
⇒ (b cos ct) −1
dt ⇒ logu + c
⇒ −bc sin ct (chain rule) 5
d2 d but u = 3 − 5x
(bc sin ct) = (−bc sin ct) −1
dt 2 dt ⇒ log (3 − 5x) + c
d 5
(−bc sin ct) = −bc cos ct×c(chain rule)
dt
d2
= −bc cos ct..................(2)
dt2 0.25 MATHS 105 2010/2011
d2 y d2 d2
= (a sin ct) + (b cos ct)
dx2 dt2 dt2 1. Find the derivative of ecos x with respect
to x
eqn ⇒ 1 + 2
(a) esin x cos x (b) ecos x sin x (c) esin x (− cos x)
d2 y
= −ac sin ct − bc2 cos ct (d) ecos x (− sin x) (e) e− sin x
dx2
Solution
124

Using chain rule let Y = ecos x df (x)


5. f(x) = log(cos 2x), f ind
du dx
put u = cos x , = − sec x 2(cos 2x) − sin 2x 2(sin 2x)
dx (a) (b) (c)
dy sin 2x cos 2x cos 2x
Y = eu , = eu cos 2x 2(− sin 2x)
du (d) (e)
dy dy du sin 2x cos 2x
= × = eu × sin x but u= tan x Solution
du du dx du
dy let u = cos 2x = 2(− sin x)
= ecos x × (− sin x)(D) dx
dx
df (x) 1
f (x) = logu , =
dy 1 1 du u
2. Find ( ) in terms of x and y for + = df (x) df (x) du 1
dx y x = × = × −2 sin 2x
1 dx du dx u
2 sin 2x
x2 −y 2 x2 y2 x =− but u= cos 2x
(a) 2
(b) 2
(c) − 2
(d) 2
(e) u
y x y x y df (x) 2 sin 2x 2(− sin 2x)
Solution =− = (E)
This is an implicit differentiation dx cos 2x cos 2x
1 1
+ ≡ y −1 + x−1 = 1 dy
y x 6. If y=x sin x + cos x then is
On differentiating dx
(a) x sin x (b) cos x − sin x
dy
−1(y −2 ) + (−x−2 ) = 0 (c) x sin x−cos x (d)sin x−cos x (e) 2x sin x
dx
Solution
−2 dy
−y ( ) − x−2 = 0 dy
dx = (x cos x + sin x) + (−sinx)
dy x−2 dx
= − −2 = x cos x + sin x − sinx
dx y = x cos x(E)
1
dy − 2
= x √ dy
dx 1 7. If y= 1 + x2 then Y ( ) is
dx √
y2 √ 1 2 x x
2
dy y2 (a) x (b) x x (c) x (d) x (e)
= − 2 (B) 2 2
dx x Solution
1

√ y = (1 + x2 ) 2 let u= 1 + x2
2 + 9x2 du
3. Find Limx→∞ = 2x
x dx
(a) ∞ (b)0 (c) 1 (d) 2 (e) 3 1 dy 1 1
∴ y = u2 ; = u− 2
Solution du 2
dy dy du 1 1
When a limit is tending to infinity (∞) = × = u 2 × 2x
we divide by the highest power pf x ,there- dx du dx 2
but u = 1 + x2
fore dy 1 x
√ = √
q
2 2
2 + 9x 2
x2
+ 9x
x2 dx 2 1 + x2
Limx→∞ = Limx→∞ x dy √

x

x x ∴ y( ) = 1 + x √ 2 = x(A)
q
r
2 dx 1 + x2
2
+9 +9
x2 ∞
= limx→∞ dy
√1 1 8. If x = αt2 and y = 2αt,find in term
= 9 = 3(E) dx
of t
(a) −2t (b) 2t (c) 1/t (d) 2/t2 (e) −1/t
4. Find the values of x for which the func- Solution
sin x
tion f (x) = 2 is not defined This is parametric differentiation
x −1 dx
(a) {1, 1} (b) {−1, 1} (c) {0, 0} (d) {−1, −1} = α(2t) + 0(t2 ) = 2αt
(e) {1, 0} dt
dy
= 2α(1) + t(0)
dt
0.25. MATHS 105 2010/2011 125

dy dy dx 2α 1 du
= ÷ = = (C) = 2 sin x cos x
dx dt dt 2αt t dx
du 1
9. Find the Second differential coefficient of ∴ y = tan−1 u =
du 1 + u2
y = x sin x dy dy du 1
= × = × 2 sin x cos x
(a) x cos x − 2 sin x (b) 2 cos x − sin x (c) dx du dx 1 + u2
2 cos x − x sin x ((d) x cos x (e) 2x sin x dy 2sinx cos x
= but u= sin2 x
Solution dx 1 + u2
dy dy 2 sin x cos x 2 sin x cos x
= x(cos x) + (1) sin x = x cos x + sin x = 2 = (A)
dx dx 1 + (sin x) 2 (1 + sin4 x)
Second differentiation
d2 y d 
1 1

2
= (x cos x + sin x) 13. lim x → 0 + /(ex − 1)
dx dx x sin x
d2 y
= [x(− sin x) + (1) cos x] + cos x (a) 0 (b) 2 (c) 1 (d) −1 (e) 4
dx2
= −x sin x + 2 cos x = 2 cos x − x sin x(C) Solution
Here we use L-Hospital rule (on differen-
dy tiating)
10. If y = (3x − 1)(x − 3) and = αx + 1

dx limx→0 1 + /ex − 0
β,find the value of α and β cos x
(a) 6 and 6 (b) 8 and −6 (c) 6 and −10 1+1
(d) 2 and −8 (e) 1 and 5 = = 2(B)
1
Solution
y = 3x2 − 10x + 3 (expanding) d
14. Determine (sin−1 x − cos−1 x)
dy dx
= 6x − 10 comparing with αx + β (a)0 (b) 1 (c) 2 (d) −2 (e) none of the
dx
α = 6 & β = −10(C) above Solution
d 1
sin−1 x = √
11. Evaluate (xx ) at x=2 dx 1 − x2
(a) 2(1 + log 4) (b) (1+log 2) d 1
cos−1 x = − √
(c) (4+ log2) (d) (1+log 2) dx 1 − x2  
(e) 4(1+log 2) d −1 −1 1 1
(sin x−cos x) = √ − −√
Solution dx 1 − x2 1 − x2
Let y = xx taking natural log (lin) 1 1 2
=√ +√ =√ (E)
logy = logxx 1 − x2 1 − x2 1 − x2
logy = xlogx √ √
d d 15. If y 1 − sin x = 1 + sin x,find which of
logy = (xlogx) the following about the equation is true
dx dx
1 dy 1 dy
( ) − x. + logx (a) (3 − sin x) =2
y dx x dx
dy dy
= y(1 + logx) but y=xx at x=2 (b) (2 − cos x) =3
dx dx
dy dy
= xx (1 + logx) = 22 (1 + log2) (c) (sin x) =2
dx dx
dy dy dy
= 4(1 + log2)(E) (d) (1 − sin x) = 1 (e) cos x =4
dx dx dx
R dx
12. Differentiate tan−1 (sin2 x) with respect to 16. Evaluate
x 9x2 + 4
2 sin x cos x 2 sin x cos x x cos x 1
(a) (b) (c) (a) tan−1 (3x/2) + c
4
(1 + sin x) 1 + cos x4 1 + cos2 x 6
1
2 sin x 3
(1 + sin x) (b) tan−1 (2x/3) + c tan−1 (3x/2) + c
(d) (e) 3
2
1 + sin x cos x sin x 1
Solution (c) tan−1 (2x/3) + c
6
y = tan−1 (sin2 x) let u = sin2 x (d) 6 tan−1 (3x/2) + c
126

(e) 6 tan−1 (2x/3) + c


R
23. logxdx
R dx 1 x (a) xlogx + x + c (b) −x + xlogx + c
Recall 2 2
= tan−1 + c
a +x a a (c) xlogx − x + c (d) xlogx+x+c
R dx dx (e) x − logx + c
R
So we are given =
9x2 + 4 (3x)2 + 22 Solution
1 3x Using integration
= tan−1 ( )(A)
6 2 1
let u = logx du = dx
17. Which of the following is false about the R x
 dv = dx v = dx = x
2x x 6= 1 R 1
function f (x) = ∴
R
logx = xlogx − x. dx
−1 x = 1
x R
(a) f(x) is continues for all x 6= 1
R
= xlogx − 1.dx = xlogx − dx
(b) limx→1 f (x) exist (c) limx→1 = 2 = xlogx − x + c(C)
(d) f(1)= −1 (e) limx→1 = −1 R
24. sin 2x sin 3xdx
18. Which of the following function is neither 1 1
(a) (sin x − sin 3x) + c
odd nor even function 2 3
1 1 2
1 1
(a) 1 (b) (c) x tan x (d) (sin x)x (b) (sin x − sin 5x) + c
x − 1 x−1 2 5
(e) e−x
2 1 1
(c) (sin x − sin 4x) + c
Solution 2 4
1 1
(d) (sin x − sin 2x) + c
5 2
dt 1 1
19. Find 0
R1 (e) (sin x − sin 2x) + c
(1 + t2 )2 3 2
Solution
(a) 2/3 (b) 1/4 (c) 4 (d) 1/2 (e) 3/4 R 1
(2 sin 2x sin 3x)dx
Solution 2
Recall that 2 sin A sin B = cos(A − B) −
R2 cos(A + B)
20. Find −2 |x + 1|dx 1R 1R
2 sin 2x sin 3x = cos(3x − 2x) −
(a) 5 (b) 6 (c) 7 (d) 8 (e) 9 2 2
Solution cos(3x + 2x)dx
1R
= (cos x − cos 5x) on integrating
2
R 10 1 1
21. Solve the equation 0 (1 + x)dx = 0.5x = (sin x − sin 5x) + c(B)
2 5
(a) 60 (b) 120 (c) 30 (d) 15 (e)90 R 1
Solution NB cos ax = sin ax
a
25. Z √3
dx
22. 2
−1 1 + x
Z  
1−x
1+x 2π π 7π
(a) (b) (c) −1 (d) (e)1
3 2 12
(a) −x + 2ln(1 + x) + c Solution
Z √3
(b) −x+2ln(1−x)+c (c) x+2ln(1+x)+c dx  −1 √3
(d) −x+ln(1+x)+c (e) x+2ln(1−x)+c 2
= tan x + c −1
−1 1 + x
Solution √ π π
−1
= (tan ( 3) − (tan−1 (−1))) = ( + )
Z 
1−x
 Z 
2
 3 4
= −1 + 7π
1+x x+1 = (D)
12
R R 2 26.
= −dx + dx Z 1
x+1 x4 dx
= −x + 2ln(x + 1) + c(A) −1
0.25. MATHS 105 2010/2011 127

(a) 2/3 (b) 0 (c) 4/5 (d) 2/5 (e) 1/5 1 1 √


α( √ ) + β( √ ) 2
Solution 2 2
But β = 1
1 1 α 1 √
x5
Z 
4 1 1 √ +√ = 2
x = = ( ) − (− ) 2 2
−1 5 −1 5 5 √
α + 1 = 2(multiply 2ru with 2)
1 1 2 α=2−1=1
= + = (D)
5 5 5 The value of α & β are (1,1) (C)
dy Rx
27. Find for y= 0 sec θdθ R xdx
dx 29. Determine the integral of
(a) sec θ (b) sec x (c) cosecx (d) cosecθ 2x − 3
3
(e) 0 (a)x + log(2x − 2) + c
Solution 4
x
(b) + log(2x − 3) + c
 
Rx Rx sec θ + tan θ
0
sec θdθ = 0 sec θ 2
sec θ + tan θ x 3
Z x 2
(c) − log(2x + 3) + c
sec θ + tanθ sec θ 2 4
= let u = sec θ+tan θ 3
0 sec θ + tan θ (d) x − log(2x − 3) + c
4
du x 3
dx = (e) + log(2x − 3) + c Solution
sec θ tan θ + sec2 θ 2 4
R x R 1 3R 1
Z
sec θ tan θ + sec2 θ du dx = dx + dx
× 2x − 3 2 2 2x − 3
u sec θ tan θ + sec2 θ 1 3 1
= x + ( log(2x − 3)) + c
2 2 2
R 1 x 3
= du = logu + c but u = sec θ + tan θ = + log(2x − 3) + c (E)
u 2 4
= [log(sec θ + tan θ)]x0
= log(sec x + tan x) − log(sec 0 + tan 0) dy
30. Find when xy 2 + cos 2y = 4
∴ y = log(sec x + tan x) dx
du y2 2(sin 2y − xy)
let u= sec x + tan x = sec x tan x + (a) (b)
dx 2(sin 2y − xy) y2
sec2 x (sin y − xy) (sin 2y − xy)
dy 1 (c) 2(sin 2y−xy) (d) (e)
y = log u ; = y y2
du u Solution
dy dy du d d d
= × (xy 2 ) + (cos 2y) = 4
dx du dx dx dy dx
dy 1 d dy
= × sec x tan x + sec2 (xy 2 ) = x(2y ) + (1)y 2
dx u dx dx
but u= sec x + tan x dy
dy 1 = (2xy) + y 2
= × sec x tan x + sec2 x dx
dx sec x + tan x d dy dy
dy sec x tan x + sec2 x (cos 2y) = 2 (− sin 2y) = (−2 sin 2y)
= = sec x(B) dy dx dx
dx sec x + tan x dy 2 dy
2
π = xy +cos 2y = (2xy) +y −2 sin 2y
28. If f (x) = α sin x+β cos x, f (0) = 1, f ( ) = dx dx
√ 4 dy
= (2 sin 2y − 2xy)
2,Find the value of the constants α and dx
β in that order . dy y2 y2
= = (A)
(a) 1, −2 (b) −2, 1 (c) 1,1 (d) 2,2 (e) 3,3 dx (2 sin 2y − 2xy) 2(sin 2y − xy)
Solution
31. WhichR of the following is false ?
f (x) = α sin x + β cos x
(a) sin xdx = − cos x + c
f (0) = α sin(0) + β cos(0) = 1 dx x
= sin−1 + c
R
α(0) + β(1) = 1 (b) √
a2 − x 2 a
∴ β=1 d 1
π π π √
f ( ) = α sin( + β cos( )) = 2 (c) (sin−1 x) = √
4 4 4 dx 1 − x2
128

d 1 Let y= 5sin x
(d) (tan−1 x) =
dx 1 + x2 loge Y = sin xloge 5 taking natural log of
d
(e) (cotx) = cosecx both side
dx d 1
Solution = cos x log 5 + 0 differentiating
d dx Y
(E) because (cotx) = −cosec2 x dy
dx = Y (cos xlog5) but y = 5sin x
dx
dy
32. Given that x = a cos3 θ and y = a sin3 θ = 5sin x (cos xlog5) at x=0
,find the derivative of y w.r.t x dx
dy
cos3 θ = 5sin x (cos 0log5)
(a) (b) − tan θ (c) − cot θ (d) tan θ dx
sin2 θ = 1((1) log 5) = log 5(A)
(e) cot θ
Solution 36. Integrate x2 e2x w.r.t x
y = a sin3 θ e2x 2
dy (a) e2x (x2 + 1) + c (b) (x − 1 + 1) + c
= a(3 sin2 θ cos θ) 2
dθ e2x 2
x = a cos3 θ (c) e2 (x2 − x + 1) + c (d) (x − x) + c
dx 2
= a(3 cos2 θ(− sin θ)) e2x 2
dθ (e) (x + x + 1) + c
dy dy dθ 2
= × Solution
dx dθ dx Using integration by part, let u= x2 du =
2
3a sin θ cos θ
= 2xdx
−3a cos2 θ sin θ 1
sin θ dv = e2x , v = e2x
=− = − tan θ(B) 2
2 1 2x
cos θ R 2 2x R 1 2x
x e =x ( e )− e .2xdx
2x x 2 2
e +e −2 1
33. Evaluate limx→0
R
x = x2 e2x − e2x dx
e −1 2
(a) −3 (b) 1/3 (c) 3 (d) 2/3 (e)0 Solu- R 1 R
but e2x xdx = xe2x − e2x dx
tion 2  
Using L-hospital rule (i.e differentiating) R 2 2x 1 2 2x 1 2x R 2x
x e dx = x e − xe − e dx
2e2x + ex 2e2(0) + e(0) 2 2
limx→0 x
= (0) 1 1 1

e e = xe − 2 2x 2x
xe − e dx 2x
2(1) + 1 2 2 2
= = 3(C)
1 1 1 1
= x2 e2x − xe2x + e2x
3x − 2x 2 2 2
34. limx→0 1
tan x = e2x (x2 − x + 1) + c(B)
2 2
(a) log (b) log 3 (c) log 2 1
3 37.
R
is
3 2x − x2
(d) log (e) 0
2 1 x
Solution (a) ln( )+c
2 2−x
Using
 L-Hospital  rule  x x
(b) ln( )+c
d 3x − 2x 3 log2 − 2x log2

2−x
= limx→0 1 2−x
dx tan x cos2 x (c) 2ln( )+c
limx→0 (30 log3 − 20 log2) (cos2 x) x
1
= 1(0+log3)−1(0+log2)(1) = log3−log2 (d) ln(2 − x) + lnx + c
log3 3 2
= = log (D)(law of logarithm) 1
log2 2 (e) ln(2 − x) + lnx + c
2
Solution
d sin x R 1 R 1
35. Evaluate (5 ) at x=0 =
dx 2x − x2 x(2 − x)
(a) log 5 (b) 5 (c) sin x (d) sin xlog5 (e)1 R 1 R A B
Solution = +
x(2 − x) x 2−x
0.26. MATHS 105 2011/2012 129

1 1 x2 1 R
by partial fraction A = and B = = logx − ( xdx)
2 2 2 2
1 R 1 R 1 1 x2 1 1
dx = xdx + dx = logx − ( x2 ) + c
x(2 − x) 2 22−x 2 2 2
1 1 x2
= lnx + ln(2 − x) + c(E) = (2logx − 1) + c(B)
2 2 4
38. Use the relation cos x = 1 − m sin2 x2 to
2 1
R
evaluate sin 2 xdx 0.26 MATHS 105 2011/2012
1
(a) (x − sin x) + c (b) (sin x) + c
2 dy
1 1. If Y = 3−2x then is
(c) 2(x − sin x) + c (d) (sin x − x) + c dx
2 2
2 (a) 3−2x (b) −2(3−x ln3−x ) (c) −2x(3−2x ln3)
1 2
(e) (x − sin x) + c (d) 3ln3−x (e) −2(3−2x ln3)
2
Solution Solution
1 Take natural log of both side
Using cos x = 1 − 2 sin2 x2 ∴ sin2 x =
2 lny = ln3−2x
1 − 2 cos x 1 dy
2 = −2ln3 differentiating w.r.t x
R 1 − cos x y dx
2 1
R
sin dx = dy
2 2 = y(−2ln3)
1R 1 dx
= (1 − cos x)dx = (x − sin x) + c(e) but y= 3−2x
2 2 dy
= −2(3−2x ln3)
d2 y 2 −x dx
39. Find (x e ) ALTERNATIVELY
dx2 dy d
(a) (2−4x+x2 )e2x (b) (2+4x−x2 )e−x (c) NOTE ; if y = akx ; = akx lna kx
(2−4x+x2 )e−x +c (d) (2e−x −4x+x2 )e−x dx dx
dy d
(e) (2 − 4x + x3 )xe−x So y = 3−2x ; = 3−2x ln3 × (−2x)
dx dx
Solution dy −2x −2x
d2 y = (3 ln3)(−2) = −2(3 ln3)
means differentiating it twice dx
dx2
y = x2 e−x x2 − 9
dy 2. If f(x)= ,when x 6= 3 ,then f (−4)
= 2xe−x + (x2 (−e−x )) = 2xe−x − x−3
dx is
x2 e−x (a) 6 (b) −4 (c) 0 (d) −1 (e) -2
d2 y
= 2e−x +(2x(−e−x )−(2xe−x −x2 e−x )) Solution
dx2 (−4)2 − 9 7
= 2e−x − 2xe−x − 2xe−x + x2 e−x f (−4) = = = 1(D)
= 2e−x − 4xe−x + x2 e−x = e−x (2 − 4x + −4 − 3 −7
x2 )(C) x
3. f(x) = then the inverse function
R
40. Use integration by parts to evaluate xlogxdx x+2
f is given by f −1 (x)
−1
x2 x2 x−2 x+2 2x 2x
(a) (2logx + 1) + c (b) (2logx − 1) + c (a) (b) (c) (d)
4 4 x x 1−x 1+x
2 1 (e) 2x
(c) x (2logx − 1) + c (d) (2logx + 1) + c
4 Solution
x2
(e) (2logx) + c To find the inverse of a function three
4 steps are needed
Solution
1 (1) let f(x)=y
let u =logx ; du = dx
x (2) Make x the subject of the formula
1 2 (3) replace y with x
dv = x ; v = x
R2 x
R
udv = uv − vdu now let see f (x) = put f(x)=y
x+2
R 1 R 1 2 1 x
xlogxdx = x2 logx − x ( dx) y=
2 2 x x+2
130

make x the subject of formula f (−x) = (−x)2 = x2


y(x + 2) = x f (x) = f (−x) even function
2y f(x) = sin2 x + x = sin x × sin x + x
x=
1−y f (−x) = sin(−x) × sin(−x) + (−x)
now replace y with x i.e f (−x) = − sin x × sin x − x
2
f −1 (x) = (c) f (−x) = sin2 x − x
1−x f(−x) 6= f (x) also f (−x) 6= −f (x)
dy Hence neither even nor odd
4. If x = e2t and y = sin 2t then =? Ans E
dx
2t −2t −2t
(a) 4e cos 2t (b) e cos 2t (c) e sin 2t dy
(d) e2t sec 2t (e) None of these 7. If y = sec x + cos x then is
dx
Solution (a) sec x tan x+cosec2 x (b) sec2 x−cosec2 x
This is a parametric equation (c) sec x+cosecx (d) sec x tan x−cosec2 x
dy
y = sin 2t ; = 2 cos 2t (e) sec2 x + cosec2 x
dt Solution
dx
x = e2t ; = 2e2t y = sec x + cot x
dt dy
dy dy dt 2 cos 2t = sec x tan x − cosec2 x(D)
= × = dx
dx dt dx 2e2t
dy
= e2t cos 2t(B) 8. If f(x) = x2 − 2x + 1 then f(−3) is
dx
(a) −8 (b)10 (c) 16 (d) −16 (e) -34
5. The limit x cot x = x → 0 Solution
(a) 1 (b) −1 (c) 2 (d) −∞ (e) ∞ f (−3) = (−3)2 − 2(−3) + 1 = 16(C)
Solution
x+1
The correct ans is A because 9. Evaluate the limx→−1 2
x x − 2x − 3
limx→0 x cot x = limx→0 1 1 1 1
tan x (a) − (b) − (c) (d) ∞ (e) −
x
and limx→0 =1 2 4 4 5
tan x Solution
By factorizing the de-numerator we have
6. Which of the following function is neither x+1 1
odd or even function limx→−1 =
(x + 1)(x − 3) x−3
(a) none of the above (b) f (x) = x sin x 1 1
(c) f (x) = x2 (d) f (x) = x cos x (e) at x=−1 = = − (B)
−1 − 1 4
f (x) = sin2 x + x
Solution dy
10. x3 − sin xy = y 3 ,
let us analyze it dx
option b f(x)= x sin x 3x2 − y cos xy 3x2 − y cos xy
(a) (b)
x and sin are both odd function 3y 2 + x cos xy 3y 2 − x cos xy
2
Note : the product of two odd function 3x + y sin xy 3x2 + y cos xy
(c) 2 (d) 2
give an even function 3y + x cos xy 3y + x cos xy
2
f (x) = x sin x 3x + y cos xy
(e) 2
f (−x) = (−x) sin(−x) = x sin x 3y − x cos xy
f (x) = f (−x) ,Hence is an even function Solution
In case of option D,now x is an odd func- x3 − sin xy = y 3
dy dy
tion while cos x is an even function 3x2 − (y cos xy + x cos xy) = 3y
dx dx
Note: the product of odd and even func- dy
2 2 dy
tion gives an odd function 3x − y cos xy − x cos xy = 3y
dx dx
f (x) = x cos x 3x2 − y cos xy dy (3y 2 + x cos xy)
f (−x) = (−x) cos(−x) = −x cos x =
3y 2 + x cos xy dx (3y 3 + x cos xy)
f (−x) = −f (x) Hence is an odd function dy 3x2 − y cos xy
while f (x) = x2 = 2 (A)
dx 3y + x cos xy
0.26. MATHS 105 2011/2012 131

1 − cos2 2x
 
11. ltx→0 = find α and β
x2 (a) 20 and −23 (b) 10 and −25
(a) −2 (b) 0 (c) 1 (d) 2 (e) 4 (c) 10 and −23 (d) 20 and −25
Solution (e) 1 and −5
Using trig identity cos2 2x + sin2 2x = 1 Solution
sin2 2x= 1 − cos 2
 2x Expanding (5x+1)(2x−5)= 10x2 −23x−
2
 
sin 2x sin 2x sin 2x 5
ltx→0 = × dy
x2 x x = 20x − 23
2 sin 2x 2 sin 2x dx
ltx→0 × ltx→0 dy
2x 2x comparing with = αx + β
= 2 × 2 = 4(E) dx
sin x α = 20 and β = −23(A)
Recall that ltx→0 =1
x
Rb 16. What is the Domain √ of the function f(x)
12. If 0 (2bxx2 )dx = 18 then b= x2 − 4
(a) −9 (b) 9 (c) −3 (d) 3 (e) 18 given by f (x) = is (a) None (b)
x−3
Solution
Rb Rb Rb {x : |x| ≥ 2} (c) {x : x 6= 3}
0
(2bx − x2 )dx = 0 2bdx − 0 x2 dx = 18 (d) {x : x 6= 2 and x 6= 3}
b
2bx2 x3 (e) {x : x 6= 2 and x 6= 3}

− = 18 Solution
2 3 0 √
x2 − 4
2b(b)2 b3 f (x) = by taking what is inside
− = 18 x−3
2 3 the root we have
b3
b3 − = 18 x2 − 4 ≥ 0
3
b3 = 27 x≥0
b = 3(D) And also x 6= 3 otherwise the function
f(x) will be undefined . though negative
2+x sign has to be put into consideration but
13. If y = then y 0
3x + 1 here the square (power 2) will take care of
−4 6x − 5 −5
(a) (b) (c) the negative sign so no need of absolute
(3x + 1) 2 (3x + 1) 2 (3x + 1)2 value
5 7 − 6x
(d) 2
(e) Ans D
(3x + 1) (3x + 1)2
Solution R
17. cos(2x + 3)dx =
du dv
V −U 1
Using dx 2 dx (a)−2 sin(2x+3)+c (b) − cos(2x+3)+c
V 2
(3x + 1) × 1 − (2 + x) × 3 1
0
Y = (c) 2 sin(2x + 3) + c (d) sin(2x + 3) + c
(3x + 1)2 2
(e) sin(2x + 3) + c
(3x + 1) − (6 + 3x) −5
= = Solution
(3x + 1) 2 (3x + 1)2 du
−5 let u= 2x + 3 then dx =
y0 = (C) 2
(3x + 1)2 R du 1R
cos u. = cos u
2 2
dy 1
14. if = cosec2 x,then y= = sin u + c but u=2x+3
dx 2
(a) cot2 x+c (b) − cot x+c (c) lncosecx+ R 1
cos(2x + 3)dx = sin(2x + 3) + c(D)
c (d) cosec2 x + c (e)ln sin x + c 2
Solution
dy R 1
18. √ dx is
R
= cosec2 x then y cosec2 xdx
dx 36 − x2
y = − cot x + c(B) 1 x
(a)arcsin x + c (b) arcsin + c
6 6
dy x x
15. If y = (5x + 1)(2x − 5) and = αx + β (c) arc sin + c (d) arc cos + c
dx 6 6
132
√ R x2 + 2x
(e) x2 − 36 + c 22. dx is
Solution x3 + 3x2
1 1 x (a) ln(x3 + 6x) + c (b) x2 + 6x + c
dx = sin−1
R R
√ dx = p + 1
36 − x 2 2
(6) − (x) 2 6 (c) x3 + 6x + c (d) ln(x3 + 3x2 ) 3 + c
c (e) ln(x2 + 2x) + c
but sin−1 x is the same as arcsin x (C) Solution
du
let u= x3 + 3x2 then dx =
3(x2 + 2x)
R
19. lnxdx
(a) (lnx)2 − x + c (b) xlnx − x + c R x2 + 2x R x2 + 2x du
dx = .
1 3
x + 3x 2 u 2
3(x + 2x)
(c) +c (d) (lnx)2 +x+c (e) x2 lnx+x+c 1R 1 1
x = du = lnu + c but u = x3 + 3x2
Solution
R R 3 u 3
lnxdx = 1 × R lnxdx Using law of logarithm (power log)
1
R
Using by part udv = uv − vdu 1
ln(x3 + 3x2 ) + c = ln(x3 + 3x2 ) 3 + c(D)
1 3
let u = lnx then du = dx
R x
dv = 1.dx then v = 1.dx = x dz
23. If z = t2 + 1 and k = sin 2t then is
R R 1 dk
lnx × 1dx = xlnx − x × dx 2t t
R x (a) (b) 2t cos t (c) (d) t cos 2t
= xlnx − 1.dx = xlnx − x + c(B) cos t cos 2t
(e) ∞
Solution
dy dz
20. If 2x3 + 2xy − 3y 2 = 2 then at the z = t2 + 1 then = 2t
dx dt
point (1, 1−) is dk
1 1 1 k = sin 2t then = 2 cos 2t
(a) − (b) − (c) −1 (d) 0 (e) dt
2 4 4 dz dz dt 2t t
Solution = × = = (C)
dk dt dk 2 cos 2t cos 2t
If 2x3 + 2xy − 3y 2 = 2
dy dy 24. Suppose f (x) = sin x − x3 then which of
6x2 + (2y + 2x ) − 6y =0
dx dx the following is not true about f(x)
dy
6x2 + 2y + (2x − 6y) = 0 (a) f(x) is odd (b) f(x) + sin x is even
dx
dy (6x2 + 2y) (c) f (x) + x3 is odd (d) xf(x) is even (e)
=− at point (1, −1) − sin x + f (x) is odd
dx 2x − 6y
dy (6(1)2 + 2(−1)) 1 Solution
=− = − (A) f (x) = sin x − x3
dx 2(1) − 6(−1) 2
f(−x) = sin(−x) − (−x)3 = − sin x + x3
f (−x) = −(sin x − x3 )
3x6 + 2x5 + x
21. limx→∞ is f (−x) = −f (x) Hence Option A is true
x6 + 4x2 + 7x f (x) + sin x = sin x − x3 + sin x
(a) 7 (b) 6 (c) 5 (d) 4 (e)8
= 2 sin x − x3
Solution
f (−x) = 2 sin(−x) − (−x)3 = −2 sin x +
Divide both the numerator and de nu-
x3
merator by the highest power
3x6 5
3+ ∞2 1
+ (inf) f (−x) = −(2 sin x − x3 )
x6
+ 2x
x6
+ xx6 5
limx→∞ x6 4x2 7x = 4 7
f (−x) = −f (x) Option B is not true it
x6
+ x6 + x6 1 + (∞)4 + (∞)5 is odd not even Ans B
3+0+0
= =3 √ dy
1+0+0 25. If x = t3 − t and y= 3t + 1,then at
SHORT CUT dx
The ratio of the coefficient of the high- t=1 is
3 3 8 1
est power or degree of x is the answer (a) (b) 8 (c) (d) (e)
,here the highest power is x6 the ration 8 4 3 8
Solution
3x6 3 dx
is 6 = = 3(E) If x = t3 − t ; = 3t2 − 1
x 1 dt
0.26. MATHS 105 2011/2012 133

√ 1 dy 3 d2 y
y= 3t + 1 = (3t+1) 2 ; = 1 = 2 − cos x(C)
dt 2(3t + 1) 2 dx2
dy dy dt 3
= × = |x|2
dx dt dx 1
2
2(3t + 1) 2 (3t − 1) 29. lim x →0 is
x
dy 3 1 3 1 (a) 0 (b) 1 (c) -1 (d) None existent (e)
at t=1 = √ × = ×
dx 2( 4) 3 − 1 4 2 None of these
dy 3 Solution
= (A) |x|2 |0|2
dx 8 limx→0 = = 0 Non-existent (D)
 x 0
 x2 − 9 2 sin 3x
if x 6= −3 30. lim
26. If the function f(x) = x + 3 x→0
 k if x 6= −3 sin 5x
6 4 3 1
is continuous at x = −3,what is the value (a) (b) (c) (d) 1 (e)
5 3 4 5
of k Solution
(a) 6 (b) −8 (c) undefined (d) (e) −6 2 sin 3x
limx→0
Solution sin 5x
6 4 3 1
Since it is continuous at x = −3 it means (a) (b) (c) (d) 1 (e)
x2 − 9 5 3 4 5
and k will have the same value Solution
x+3 sin mx
now , NOTE : limx→0 =m
x2 − 9 (x + 3)(x − 3) mx
= sin mx m
x+3 (x + 3) limx→0 =
sin nx n
x − 3 at x = −3 2 sin 3x sin 3x
−3 − 3 = −6(E) like wise here lim x→0 = 2×limx→0
sin 5x sin 5x
3 6
R1 x = 2 × = (A)
27. 02 √ dx 5 5
1 + x2 √ R dx
1 3 π π 5 31. Evaluate
(a) ln (b) (c) − 1 (d) − 1 (e) 4x2 + 9
2√ 4 6 6 2 2x
2− 3 (a) 6 tan−1 ( ) + c
3
Solution 1 −1 3x
By using integration method of substitu- (b) tan ( ) + c
6 2
tion −1 2x
R1 x 2
(c) tan ( )
0
2
√ let u = 1 + x 3
1 + x2 3x
du du (d) 6 tan−1 ( ) + c
= 2x ; dx = 2
dx 2x 1 −1 2x
R1 x du 1 R 12 − 1 (e) tan ( ) + c
2
√ × = u 2 du
6 3
0
u 2x 2 0 Solution
" 1 # 12 R dx R dx
2
=
1 U 2 4x + 9 (2x) + (3)2
2
= 1 2x
2 12 = tan−1 ( )
0
1h i1 6 3
1 2
= 2(1 + x2 ) 2 Rπ
2 √0 32. 02 x sin xdx =
1 √ 5 π π π
= ( 5 − 2) = − 1(D) (a) − (b) 1 − (c) −1 (d) (e) − 1
2 2 2 2 2
Solution
d2 y Using integration by part R u =Rx du = dx
28. If y= x2 + cos x then 2 is dv = sin xdx v = dv = sinxdx =
dx
(a) 1 + sin x (b) 1 − sin x (c) 2 − cos x (d) −
R πcos x
2 − sin x (e) 2 + sin x
R
0
x sin xdx =R −x cos x− 1×− cos xdx
Solution = −x cos x + cos xdx
dy π
= 2x − sin x = [−x cos x + sin x + c]02
dx
134
 3
= (−90 cos 90+sin 90)−(−0 cos 0+sin 0) 1 R R 3 1 3
= ( 1dx+ dx) = x + ln(2x − 3)
= −90 × 0 + 1 + 0 + 0 = 1(D) 2 2x − 3  2 2 2
1 3 3
= 3 + ln3 − ln1
33. Which of the following
 is false about the 2 2 2
2x x ≥ 0 1 3 1 3
function f(x) = = (1 + (ln3 − ln1)) = (1 + ln( 31 ))
x+1 x>0 2 2 2 2
(a) limx→0 f (x) exist (b) limx→0 f (x) = 0 1 3 1 3
= (1 + ln3) = + ln3 (A)
(c) f(x) is continues at x=0 (d) f(0)=0 (e) 2 2 2 4
limx→0 f(x) does not exist
Solution
A function is said to be continuous at
 
−1 2x gy
point x → a if 36. If y = sin 2
=
1+x dx
(1) f(a) is defined 2x 2x 2
(2) ltx→a f (x) exist (a) − sin−2 ( 2
) (b) 2
(c)
1+x 1+x 1 − x2
(3) ltx→0 f (x) = f (a) 2x 2
(d) cos−1 ( 2
) (e)
This above condition does no hold for the 1+x 1 + x2
above equation,hence Ans is C Solution  
−1 2x
If y = sin
R dx 1 1 + x2
34. is (a) ln(2 − x) + lnx + c (b) dy 1 du
2x − x 2 2 =√ ×
1 x 2−x dx 1 − u2 dx
ln( ) + c (c) 2ln( )+c 2x du 2(1 − x2 )
2 2−x x put u= ; =
x 1 1 + x2 dx (1 + x2 )2
(d) ln( ) + c (e) ln(2 − x) + lnx + c
2−x 2 dy 1 2(1 − x2 )
Solution =q ×
R dx R dx dx 1 − ( 2x )2 (1 + x2 )2
= 1+x2
2x − x 2 x(2 − x) 1 2(1 − x2 )
1 A B =q ×
= + 4x2
1 − (1+x (1 + x2 )2
x(2 − x) x (2 − x) 2 )2

1 1 1 2(1 − x2 )
by partial fraction A = & B = =q ×
2 2 1+2x2 +x4 −4x2 (1 + x2 )2
R dx R 1 R 1 (1+x2 )2
= dx + dx
2x − x2 2x 2(2 − x) 1 2(1 − x2 )
1 1 1 =s ×
= lnx− ln(2−x) = (lnx−ln(2−x)) 1 − 2x2 + x4 (1 + x2 )2
2 2 2
1 x (1 + x2 )2
= ln( ) + c(B)
2 2−x The factor of 1 − 2x2 + x4 can be gotten
as put k= x2 the equation will change to
R 3 xdx 1 − 2k + k 2
35. 2
2x − 3 k 2 − 2k + 1 = 0
1 3 1 1 3
(a) + ln3 (b) + ln3 + c (c) − ln3 (k − 1)2 = 0 ; k = 1
2 4 2 2 4 but k = x2 i.e x2 = 1 ; ∴ (1 − x2 ) = 0
3 3
(d) 3 − ln3 (e) 3 + ln2 now back to our work
4 4 dy 1 2(1 − x2 )
Solution =q ×
By mathematical manipulation we can dx (1−x2 )2 (1 + x2 )2
(1+x2 )2
2 2
introduce to the integral since = 1 1 2(1 − x2 )
2 2 =q ×
and multiplying by 1 is the same thing ( 1−x
2
)2 (1 + x2 )2
1+x2
as that thing
R xdx 2 R xdx 1 R 2xdx 1 2(1 − x2 ) 1 + x2 2(1 − x2 )
= = = 2 × = ×
2x − 3 2 2x − 3 2 2x − 3
1−x
1+x2
(1 + x2 )2 1 − x2 (1 + x2 )2

1 R 2x − 3 + 3 1 R 2x − 3 3 2
= dx = + dx
= (E)
2 2x − 3 2 2x − 3 2x − 3 (1 + x2 )
0.27. MATHS 105 2013/2014 135

dy 0.27 MATHS 105 2013/2014


37. If y = x sin x then − 1 is
dx
(a) 0 (b) 1 (c) x + lnx (d) lnx (e) x 1. Solve |3 − x| ≥ 1
Solution (A)x ≥ 4 (B)−2 ≤ x ≤ 2 (C) x ≥ 2 or
dy
= lnx + 1 using product rule 1 1
dx x ≤ −1 (D) ≤ x ≤ 1 (E) x ≤ or
dy 3 3
− 1 = lnx(D) x≥1
dx Solution
If |3 − x| ≥ 1, this absolute sign has two
condition. i.e either (3 − x) ≥ 1 or −(3 −
38. If f(x) s continuous at x=a which of the
x) ≥ 1
following about the function is true
Now let see
(i) f(a) is defined (ii) ltx→0 f (x) = f (a)
3−x≥1
(iii) f 00 (a) does not exist
−1 ≥ 1 − 3
(a) ii and iii only (b) i and iii only (c )
−x ≥ −2
i and ii only (d) iii only (e) All of the
x ≤ 2 or
above
−(3 − x) ≥ 1
Solution
−3 + x ≥ 1
A function is said to be continuous at
x≥1+3
point x → a if
x≥4
(1) f(a) is defined
Answer is A
(2) ltx→a f (x) exist
(3) ltx→0 f (x) = f (a) 2. Solve x > x2
By considering the above condition i and (A)x < 0 or x > 1 (B) −1 < x < 0
ii from the question ate the required con- (C) 0 < x < 1 (D) −1 < x < 1 (E)
dition x < −1 or x > 0
Ans C Solution
x > x2

39. The fundamental period of the function x − x2 > 0


f (x) = 2 tan x is x(1 − x) > 0

(a) (b) π (c) 2π (d) 2 (e) 3 x > 0 or 1 − x > 0
3
Solution x > 0 or 1 > x
The periodic function of tan x is π
Ans B x > o or x < 1
Now we have x > 0 =⇒ 0 < x and
x < 1 which can be arranged as 0 < x <
d x 1
40. Evaluate x at x=1 Answer is C
dx
(a) 1 (b) (1+ log 2) (c) (4+log 2) (d)
1
(1+log 3) (e) 2 3. Find the domain of f (x) = √
Solution 1 − x2
(A) [−1, 1] (B) (−1, 1) (C) (−∞, −1)
let y = xx
(D) (1, +∞) (E) None of these
lny = xlnx on differentiating
1 dy 1 Solution
= x × + 1 × lnx = 1 + lnx 1
y dx x f (x) = √
1 − x2
dy
= y(1 + lny) but y= xx 1
dx The formula or expression f (x) = √
dy 1 − x2
= xx(1+lnx) at x=1 gives a real f (x) value for .....
dx
dy Beyond this domain 1 − x2 is negative
= 11 (1 + ln1) = 1(A)
dx and its square root is not a real number,
136

the value of 1 − x2 vary from 0 to 1 on By applying the rule of logarithms


the given domain and the square root do log a + log b = log(a × b)
1
the same. the range of √ is [0, 1] √ !
1 − x2 √ 2 √
and the domain is [−1, 1] now, log( 2) + log = log 2 ×
2

Alternatively,
 
2 2
1 = log = log 1
f (x) = √ 2 2
1 − x2 log 110 = x
By solving the domain 1 − x2
10x = 1
1 − x2
√ √ 10x = 100
x2 = 1
x=0 or
x = ±1
log 110 = 0
the domain is ± i.e +1 and −1.
Note: [a,b] describe the close interval of hence option A is the correct answer
all numbers between a and including a,
6. Period of tan x and cot x is (A) 2π (B)
and b. π 3π
(a,b) describe the close interval of all num- (C) (D) π (E) ∞
2 2
bers a and b excluding a and b likewise Solution
here +1 and −1 ..... included. i.e [−1/1] A periodic function is a function that re-
not (−1, 1) peat itself at regular or constant interval.
Answer is B. The periodic function of tan θ is π and
cot θ is the inverse of tan θ, hence they
4. f (x) = x2 − 1 then f (sec θ) − f (tan θ) =? have the same periodic function.
(A)0 (B) 2 (C) 1 (D) −1 (E) −2 The correct answer is option D.
Solution Note: The periodic function of both sin θ
If f (x) = x2 −1 then f (sec θ) = (sec θ)2 − and cos θ is 2π while that of tan θ is π.
1 and f (tan θ) = (tan θ)2 − 1
but 1 + tan2 θ = sec2 θ 1
7. If f (x) = x + , x 6= 0 then f ( x1 ) =?
then f (sec θ) − f (tan θ) is x
1 1
sec2 θ − 1 − (tan2 −1) then (A) x (B) (C)f (x) (D) x −
x x
(1 + tan2 θ) − 1 − 1 tan2 θ + 1 = 1 (E) None of these
Answer is C Solution
1
5. If f (x) = log x, then and f (2 sin x) + If f (x) = x + , then
π x
f (cos x) at x =
 
4 1 1
f ( x1 ) = + 1
(A) 0 (B) 2 (C) 1 (D) −1 (E) −2 x (x)
Solution 1 1 1
= +1× = +x
If f (x) = log x x x x
then f (2 sin x) + f (cos x) is log(2 sin x) + 1
π now x + = f (x)
log cos x at x = x
4 Therefore, the correct answer is C
here π = 180
π 180 8. If f is a function defined from set A =
Therefore, x = = = 450
4 4 2, 3, 4 to set B = 4, 9, 16, 25 by f (X) =
By substituting x = 450 , we have x2 then the range of function f will be
log(2 sin 450 ) + log cos 450 (A) (2,3,4) (B) (4,9,2) (C) (9,16,25)
√ ! √ ! (D) (4,9,16,25) (E) (4,9,16)
2 2 Solution
log 2. + log
2 2 f (x) = x2
0.27. MATHS 105 2013/2014 137

now, f (2) = 22 = 4 1 1 −1
f (−x) = 3
= 3
= 3
(−x) − 1 −x − 1 (x + 1)
f (3) = 32 = 9
−1 −1
f (4) = 42 = 16 3
6= 3 therefore is not an
−x + 1 x +1
hence, the range of function f is (4, 9, 16) odd function and also is not a periodic
which is option E function
 
2x + 1 x−1 1
9. If f (x) = , then (f(2)) = ? 11. lim 3 is equal to (A) 0 (B) (C)
3x − 2 x→1 x − 1 3
5 1 1
(A) 2 (B) 1 (C) 3 (D) 4 (E) (D) (E) None of these
4 2 4
Solution Solution
x−1
 
2x + 1
f (x) = , then lim can be solved using
3x − 2 x→1 x3 − 1

(f (2)), there is two (2) function here, L’ HOPITAL RULE


and you are always required to start from In a certain situation where by x is sub-
the inner function before outer function. stituted directly to a function of f (x) and
The inner function is f (2) while the outer either f (x) = ∞, 00 , ∞ , 0 , ∞ is found ,
∞ ∞ 0
function is the result of f (2) then we have to differentiate both the
 
2(2) + 1 4+1 numerator and the denominator at the
f (2) = =
3(2) − 2 6−2 same time till a stage where we do not
5 have above values as our answer.
f (2) =
4 Now, using L’ Hopital rule
now, f (f (2)) = f ( 45 ) we have to differentiate a number of times
2( 54 ) + 1 5
+1 till a stage where we dont have zero
2
= = 1−1 0
3( 54 ) − 2 15
4
−2 substituting x = 1, =
5+2 7 1−1 0
2 2 Now differentiating the numerator and
= 15−8 = 7 =2
4 4
denominator
the correct answer is A
d
(x − 1) 1−0
1 dx
=
10. The function f (x) = 3 is ..... func- d
(x3 − 1) 3x2 − 0
x −1 dx
tion (A)odd (B) even (C) periodic 1 1
substituting x = 1, 2
=
(D) symmetrical (E) None of these 3(1) 3
Solution ∴ the answer is B
1 √
f (x) = 3 1
x −1 12. lim (1 + 2x) x =? (A) 2 (B) e (C)
x→0 √
for even function f (x) = f (−x), let us e2 (D) (E) e
check Solution
1
1 1 lim (1+2x) x is an exponential limit Note:
f (−x) = 3
= 3 x→0
(−x) − 1 −x − 1 1
lim (1 + k) k = e
1 1 x→0
3
6= 3
hence, it is not even Now our target is to convert or change
x −1 −x − 1 1 1
function (1 + 2x) x to (1 + 2x) 2x .2
1 1 2
(1 + 2x) 2x ×2 = (1 + 2x) x × 2
Let us check for odd 1 1
= (1+2x) 2x .2 = e2 since lim (1+k) k = e
f (x) = −f (x) 1
x→0

1

−1 Therefore lim (1 + 2x) = e2 x
x→0
−f (x) = − = The correct answer is C
x3 − 3 x3 − 1
−1 1 −1 1 − cos 2x
= = 13. Evaluate lim
x3 − 1 −(x3 − 1) −x3 + 1 x→0 x2 (1 + cos 2x)
138

1 d
(2 sin x − sin 2x) 2 cos x − 2 cos 2x
(A)1 (B) 0 (C) (D) 2 (E) none of dx
=
2 d
(x3 ) 3x2
the above dx
Differentiating for the second time will
Solution
give
d
1 − cos 2x dx
(2 cos x − 2 cos 2x) − sin x − (−4 sin 2x)
lim , substitute x = 0 d
=
x→0 x2 (1 + cos 2x)
dx
(3x2 ) 6x
1 − cos 2(0) 0 Since the denominator will still give us
2
=
0 (1 + cos 2(0)) 0 zero, we have to differentiate for the third
Applying L Hopital rule, time
d
d
(1 − cos 2x) 0 − (−2 sin 2x) dx
(−2 sin x + 4 sin 2x) −2 cos x + 8 cos 2x
dx
= d
=
d
(x2 (1 + cos 2x)) 2x(−2 sin 2x) dx
(6x) 6
dx
−1 Now, substituting our limit (x = 0) will
=
2x give
substituting the limit i.e x = 0 will give
−1 −1 −2 cos 0 + 8 cos 2(0) −2 × 1 + 8 × 1
= =
2(0) 0 6 6
Therefore, we have to differentiate for the
−2 + 8 6
second time = = =1
6 6
The correct answer is B
d −1 −1
( x) = π
dx 2 2
 1
Hence the correct answer is E 16. lim x − arctanx (A) −1 (B)
x→−∞ 2 2
 x −1
4 (C) (D) 1 (E) 2
14. lim 1 + is equal to 2
x→∞ x Solution
1 1 4
(A) e 4 (B) 4e (C) e (D) (D) e π 
e lim x − arctanx
Solution
 x x→−∞ 2
4 By applying L Hopital rule
lim 1 + is equal to d h π i 
1

x→∞ x lim x − arctanx = 1 0 −
 1  1× x4 x→−∞ dx 2 1 + x2
4 4 4 
1
 
1

1+ x = 1+ at x = ∞, 1 0 − =1 0−
x x 1 + (∞)2 1
 x4 ×4  x× 44   x4 .4
= 1(−1) = −1

4 4 4
= 1+ = 1+ = 1+ Answer is A.
x x x
  x4
4
but 1 + =e A function is defined by
x
 x× 44
4

Therefore 1+ =e 4  x + 1 if x ≥ 2
x f (x) = 2x − 1 if 1 < x < 2
The correct answer is E 
x − 1 if x ≤ 2
2 sin x − sin 2x Use this function to answer question 17
15. lim (A) 0 (B) 1 (C) to 19
x→0 x3
2 (D) 3 (E) 4
Solution 17. Which of the following is false about the
function (A) f (x) is continuous at x = 1
2 sin x − sin 2x (B) f (x) is discontinuous at x = 1 (C)
lim , substitute x = 0
x→0 x3 f (x) is continuous at x = 2 (D) f (x) is
2 sin 0 − sin 2(0) 2 sin 0 − sin 0 0 continuous at x = ... (E) f (x) is contin-
= = uous at x < 1
03 0 0
Using L Hopital rule,
0.27. MATHS 105 2013/2014 139

Solution The correct answer is F


A function is continuous if lim f (x) = dy
x→a
f (a). 21. If y = log sin x, find
dx
lim f (x) 6= f (1), so it is discontinuous 1
x→1 (A)cot x (B) tan x (C) (D) sin x
at x = 1. Answer is A sin x
(E) −cosx
Solution
18. Find the point of discontinuity for the
If y = log sin x, put u = sin x
function (A) x < 1 (B) x = 2 (C)
then y = log u
x = 1 (D) x > 2 (E) 1 < x < 2
now, we have two function
Solution
u = sin x and y = log u
The point of discontinuity by considering
Using chain rule, which is given by
the condition in no17 is at x = 1 dy dy dv
Answer is C = ×
dx dv dx
19. Which of the following is false about the du
u = sin x then = cos x
function ? (A) lim+ f (x) = 1 (B) dx
x→1 dy 1
lim− f (x) = 1 (C) lim+ f (x) = 3 (D) lim− f (x) =Also y = log u then du = u
x→1 x→2 x→2
3 (E) lim f (x) = 7 dy dy dv
x→6 = ×
dx dv dx
Solution dy 1
The answer is B because lim− f (x) = 1 = × cos x
x→1 dx u

which is not true i.e x −→ 1 means the dy cos x
value tending x by negative side or be- = cot x
dx sin x
fore x . values like 0.99, 0.98 e.t.c Therefore, option A is the correct an-
f (x) = x − 1 f (0.99) = 0.99 − 1 6= 1 swer.
dy
22. If y 0 = 3sin x , =? (A) 3sin x (B)

 x2 − 16 dx
, x 6= 4
20. If the function f (x) = −4 3sin x
 xH, x=4 3sin x
log 3 (C) 3sin x
cos x log 3 (D)
log 3
is continuous, what is the value of H ? (E) 3cos x
Solution
Solution  y 0 = 3sin x
 x2 − 16 By taking natural log (i.e In) of both
, x 6= 4
f (x) = x − 4 is continu-
 H, side, we have
x=4
Iny = In3sin x
ous since the function is continuous, they
Iny = sin xIn3
must have the same answer both H and
x2 − 16 differentiating both side
at x = 4 1 dy
x−4 = cos xIn3
2 y dx
x 16 (x + 4)(x − 4) dy
= = y(cos xIn3) but y = 3sin x
x−4 x−4 dx
dy
x + 4 at x = 4, 4 + 4 = 8 ∴ = 3sin x cos xIn3
dx
Alternatively, using L’ Hopital rule x2
23. Derivative of with respect to x
d
1 + x2
dx
(x2 − 16) 2x − 0 will be
d
= = 2x 2x 1 x
dx
(x − 4) 0 (A) 2 2
(B) 2 2
(C)
(1 + x ) (1 + x ) (1 + x2 )2
2x at x = 4 2
x 2
2×4=8 ∴ H =8 (D) (E)
(1 + x2 )2 (1 + x2 )
140

Solution d
Since (2 sin2 x) = 4 sin x cos x and
dx
x2 d
put u = x2 and v = 1 + x2 (cos 2x) = −2 sin 2x
1 + x2 dx
Using quotient rule d
∴ (2 sin2 x + cos 2x) = 4 sin x cos x +
v du
dx
dv
− u dx dx
(−2 sin 2x)
v2
= 4 sin x cos x + 2 sin 2x
du
If u = x2 , = 2x
dx 26. Differentiate with respect to x, the func-
dv √ 1
v = 1 + x2 , = 2x tion 2arctan x (A) 2 (B)
dx x (1 + x2 )
2x(1 + x2 ) − 2x(x2 ) 2x + 2x3 − 2x3 1 2 2
Now, = (C) (D)
(1 + x2 )2 (1 + x2 )2 x2 (1 + x) (1 + x2 )2 1 + x2
2x 2
= (E) 2 √
(1 + x2 )2 x (1 + x)
Hence, option A is the correct answer Solution
√ 1 d √
2arctan x = 2 × √ 2 × ( x)
(1 + ( x) ) dx
d3 y d √ 1
24. If y = x4 − 7x3 + 3, then at x = 2 but ( x) = √
dx3 dx 2 x
is (A) 6 (B) −6 (C) 4 (D) −4 (E) 2
√ 1 1
Solution 2arctan x = 2 × √ 2 × √
4 3 d3 y (1 + ( x) ) 2 x
y = x − 7x + 3, to find , you 1 1
dx3 = √ =√
have to differentiate 3times x(1 + x) x(1 + x)
dy No correct option, but the likely answer
= 4x3 − 21x2 + 0
dx is D
d2 y
= 12x2 − 42x d
dx 2 27. Find f (x) where f (x) = cosh−1 (2x)
dx
d3 y at x = 2
√ √
= 24x − 42 1 −2
dx3 (A) √ (B) 2 15 (C) 15 (D)
15 15
d3 y 2
at x = 2 , = 24(2) − 42 = 6 (E)
dx3 15
option A is the correct answer. Solution
d 1
Note that (cosh−1 x) = √
d π dx x2 − 1
25. (2 sin2 x+cos 2x) (A) π (B) (C) 1
dx 2 Therefore , cosh−1 (2x) = p
0 (D) 1 (E) none of these (2x)2 − 1
Solution 1
d = √
(2 sin2 x + cos 2x) 4x2 − 1
dx 1
Since we are to differentiate both 2 sin2 x at x = 2 we have p
d 4(2)2 − 1
and cos 2x and we know that (cos 2x) =
dx 1 1
−2 sin 2x = √ =√
16 − 1 15
let us find that of 2 sin2 x, if y = 2 sin2 x Answer is A
put u = sin x y = 2u2
Using chain rule x2 log y 2 dy
dy dv 28. If + = 1 , then =?
= 4u , = cos x a2 b2 dx
du dx b2 xy −a2 xy −b2 xy
(A) 2 (B) (C) (D)
dy dy dv a b2 a2
2 2
= × a −a
dx du dx 2
(E) 2
dy b xy b xy
= 4u × cos x = 4 sin x cos x Solution
dx
0.27. MATHS 105 2013/2014 141

x2 log y 2 dy dy
dy dt
If 2 + =1 = dt
= ×
a b2 dx dx dt dx
By solving the fraction dt
2
dy 6t − 6
now, =
x2 b2 + a2 log y 2 dx 2t + 2
=1
a2 b 2 dy dy 6(0)2 − 6
at t = 0 is =
cross multiply dx dx 2(0) + 2
x2 b2 + a2 log y 2 = a2 b2 dy 0−6 −6
= = = −3
now, you can differentiate it dx 0+2 2
differentiating a2 log y 2 dy
let y = a2 = log y 2 , put u = y 2 Therefore, = −3
dx
2
y = a log u The correct option is A
du dy
then = 2y 30. Differentiate xsin x with respect to x
dy dx
(sin x)xsin x + x(log x) cos x
dy 1 a2 (A)
= a2 × = 2 x
dx u u
dy dy du sin x
x
= × (B)
dx dv dx sin x + x(log x) cos x
2
a dy 2a2 y dy
= × 2y = sin x + xsin x (log x) cos x
u dx u dx (C)
d 2 2a2 y dy x
2
Since (a log y ) = . 
sin x + x(log x) cos x

dx u dx (D) xsin x
d 2 2 d 2 2 x2
Now (x b + a2 log y 2 ) = (a b )
dx dx  
2
2a y dy sin x sin x + x(log x) cos x
2xb2 + 2 =0 (E) x
y dx x
2a2 dy Solution
= −2xb2 Let y = xsin x
y dx
Take log of both side
2a2 dy
y dx −2xb2 log y = log xsin x
=
2a2 2a2 log y = sin x log x
y y
dy y Now, differentiating with respect to x but
= −2xb2 × 2 to differentiating sin x log x, we have to
dx 2a
2
use product rule since the relationship
dy −b xy between sin x and log x is multiplication
=
dx a2 du
Let u = sin x, = cos x
The correct option is C dx
dv 1
Also v = log x , =
29. A particle moves in the plane according dx x
du dv
to x = t2 + 2t, y = 2t3 − 6t. Find the Using v +u
dx dx
slope of the tangent line when t = 0 sin x x log x(cos x) + sin x
(A) −3 (B)−4 (C) −5 (D)−6 (E) log x.(cos x)+ =
x x
−7 d
where 4 (sin x log x) is
Solution dx
dy x log x(cos x) + sin x
Note, slope = gradient =
dx x
Now y = 2t3 − 6t now back to our work
dy log y = sin x log x
= 6t2 − 6
dx 1 dy x log x(cos x) + sin x
=
x = t2 + 2t y dx  x 
dx dy x log x(cos x) + sin x
= 2t + 2 =y
dt dx x
142
 
dy sin x x log x(cos x) + sin x = tan x − x + c
=x .
dx x The correct option is A
The correct answer is E
R (x2 − x + 1) R 1
31. Evaluate √ dx 33. Find log xdx (A) +C (B) x log x+
x x
x+C (C) x log x−x+C (D) log x+x+C
5 5 3 3 1
(A) x 2 − x 2 + x2 + C (E) log x − x + C
2 2 2 Solution
2 5 2 3 1
R R
log xdx = 1 × log xdx
(B) x 2 − x 2 + 2x 2 + C
5 3 Using integralRby part
5 3 1
R
(C) 5x 2 − 3x 2 + 2x 2 + C udv = uv − vdu
put u = log x, and dv = 1dx
2 5 2 3 1 du 1 R R
(D) x 2 − x 2 − 2x 2 + C = , v = dv = 1dx = x
5 3 dx x
R 1
5 5 3 3 1 udv = x log x − x. dx
(E) x 2 − x 2 − x2 + C x
2 2 2 R
x log x − 1dx
Solution
R (x2 − x + 1) = x log x − x + C
By splitting √ dx, we will have
The correct option is C
x
R x2 R x R 1
√ dx − √ dx + √ dx dx 1
 
3x
x x x R
34. √ =? (A) tan−1 +C
R x2 R x R 1 4 − 9x   3
2 2
= 1 dx − 1 dx + 1 dx 1 2x
x2 x2 x2 (B) tan−1 +C
R 2 −1 R −1 R 2 3 
= x × x 2 dx − x × x 2 dx + 1 × 1

3x
−1
x R2 dx (C) cos−1 +C
2− 12
R 1− 12
R −1 3 2
= x dx − x dx + x dx 2
1

3x
(D) sin−1
R −1
+C
R 3 R 1
= x 2 dx − x 2 dx + x 2 dx 3  2
3 1 −1 1 2x
x 2 +1 x 2 +1 x 2 +1 (E) sin−1 +C
= 3 − 1 + −1 2 3
2
+1 2
+1 2
+1
5 3 1 Solution
x2 x2 x2 dx dx
= 5 − 3 + 1
R R
√ = √
4 − 9x 2 2
2 − 9x 2
2 2 2 
R dx −1 3x
= p = sin
2 5 2 3 1
2 2 − (3x)2 2
= x 2 − x 2 + 2x 2 + C
5 3 1 x
= sin−1 + C
R
recall that √
The correct answer is B
2
a −x 2 a
R
32. Evaluate (sec2 x + tan2 x)dx R dx 1 −1 x
 
35. =? (A) tan
(A) 2 tan x − x + C (B) tan x − 2x + C 4 + x2   4 2
(C) tan x − x + C (D) 2 tan x + x + C 1 2
(B) tan−1 +C
(E) tan x + x + C 4 x
Solution 1 x
(C) tan−1
From trigonometry identity , 3  3
2
sin x cos x 2
1 2
+ = (D) 2 tan−1
cos2 x cos2 x cos2 x x
Also tan2 x + 1 = sec2 x 1 x
2 2
(E) tan−1 +C
Substitute tan x = sec x − 1 2 2
R 2 2 Solution
(sec
R x2+ tan x)dx
= R (sec x + sec2 x − 1)dx R dx R dx
=
R R
= (2 sec2 x−1)dx = 2 sec2 xdx− 1dx 4+x 2 22 + x2
0.28. MATHS 105 2014/2015 143

1 −1 x
 
Solution
= tan +C R1 x
2 2 xe dx
The correct answer is C 0
du
put u = x, =1
R (log x)2 1 dxR
36. dx =? (A) (log x)2 + C dv = e v = ex = ex =⇒ v = ex
x
x 2
1 Using integralRby part
(B) (log x) + C (C) 3(log x)3 + C
3 R
3 udv = uv − vdu
1 1 1
R1 x x
R x
(D) (log x) 3 + C (E) 2(log x) 2 + C 0
xe dx = xe − e .1dx
3 x
= [xe − e ]0 x 1
Solution
substituting upper and lower limit,
R (log x)2 =(1 × e1 − e1 ) − (0 × e0 − e0 )
dx, put u = log x
x = 0+1=1
du 1 The correct answer is A
= dx = xdu
dx x
R u2 R Rπ
× xdu = u2 du 39. Find 0 | cos x|dx (A)4 (B)0 (C)1 (D)2 (E)3
x
u2+1 u3 Solution

=
2+1
+C =
3
+C
0
| cos x|dx = [sin x + C]π0
substituting upper and lower limit
1
= (log x)3 + C = [sin x + C]π − [sin x + C]π
3
The correct answer is B =sin π + C − sin 0 − C
0−0=0
R 1 (tan−1 x)2 π2 π B is the correct answer
37. 0
dx =? (A) (B)
1 + x2 64 16
π3 π2 π3 R2 32 125 64 16
(C) (D) (E) 40. −2
x4 dx =? (A) (B) (C) (D) (E)0
194 182 192 3 5 5 5
Solution Solution 
2
x4+1

R2 4
R 1 (tan−1 x)2 −2
x dx = + C
dx =? 4+1 −2
0
1 + x2 substituting upper and lower limit,
du 1  5 2  5 2
put u = tan−1 x then = x x
dx 1 + x2 = +C − +C
dx = (1 + x2 )du 5 2
25 (−2)5
R 1 (tan−1 x)2 R 1 u2 = +C − −C
5 55
0
dx = 0
.(1 + x2 )du 32

−32 32 32 64
1 + x2 1 1 + x 2
= − = + =
R1 2 u2 + 1 5 5 5 5 5
o
u du = +C The correct answer is C
2+1 0
 3 1
u
= +C
3 0

(tan2 x)3
1 0.28 MATHS 105 2014/2015
+C
3 o
4θ dy
Bysubstituting the upper and lower limit 1. If y = e ln7θ, then is
dx
(tan2 1)3 (tan2 0)3
  
1 1
= +C − +C (a) e4θ + (b) e4θ ( + 4ln7θ)
3 3 7θ θ
3 3 4θ 1
(45) (0) 91125 (c) ln7θ + e (d) 0 (e) ln7θ( )
− = − 0 = 30375 7θ + e4θ
3 3 3 Solution
The answer is option E because
y = e4θ ln7θ using Product rule
π3 (180)3 dy 7
E= = = 3037 = 4e4θ ln7θ + e4θ .
192 192 dx 7θ
R1 x 4θ 1
38. Evaluate 0 xe dx (A) 1 (B)2 (C)3 (D)4 (E)5 = e (4ln7θ + )(B)
θ
144

x 3 − a2 No correct option
2. limx→a is ?
x−a
(a) 2a3 (b) a2 (c) ∞ (d) a (e) 3a2 x5 + x3 + x1
Solution 7. limx→∞ 6 is
x − x2 + 1
Using L’Hospital Rule that is differenti- 6
(a) x (b) 1 (c) ∞ (d) 0 (e) 2
ating Solution
limx x → a3x2 = 3a2 (E) Dividing both Numerator and de-numerator
by the highest power of x
dy x5 3 1
3. If y = xx then is x6
+ xx6 + xx6 1
x
+ x13 + x15
dx limx→0 x6 x3 = lim x→∞
(a) xx (1 + lnx) (b) ln x (c) 0 x6
− x6 + x16 1 − x14 + x16
(d) ex (e) xx lnx 0+0+0
= = 0(D)
Solution 1−0−0
y = xx taking the ln of both side R
lny = lnxx = xlnx differentiating both 8. lndx is
side 1
(a) + c (b) x(lnx − 1) + c (c) xlnx + c
y0 1 x
= lnx + x × (d) xlnx − x2 + c (e) x2 + c
y x
y 0 = y(lnx + 1) = xx (lnx + 1)(A) Solution
Using by part
tan θ R R 1
4. limθ→0 is 1.lnx = xlnx − x × dx = xlnx −
θ R x
(a) ∞ (b) 2 (c) 1 (d) 0 (e) −∞ 1dx
Solution = xlnx − x + c = x(lnx − 1) + c(B)
Using L’hospital rule
1 1 9. If f (x) = − sin x − x3 ,then which of the
limθ→0 sec2 θ = limθ→0 2
= = ∞(A)
tan θ 0 following is not true about f(x)
5. f : A → B is a function if (a) f(x) is odd (b) f(x) is even
(a) it can map each element of A to more (c) f (x) − x3 is odd
element of B (d) f (x)+ sin x is even
(b) it is a relation (e) f(x) in neither even nor odd
(c) it is equal to the certesian product of Solution
A and B f (x) = − sin x − x3
(d) it maps a proper subset of A to B f (−x) = − sin(−x) − (−x)3 = sin x + x3
(e) it maps each a ∈ A to a unique b ∈B ∴ f (−x) = −f (x) This is an odd
Solution function (A)

dy
dy 10. of y= sin φ + tan 4φ, x = φ − lnφ is
−1 x
6. if y = tan e then dx
dx x sin φ φ cos φ + 4φ(sec 4φ)2
ex e (a) 0 (b) (c)
(a) 2
(b) 0 (c) (d) tan−1 x sec 4φ φ−1
1+x 1 + x4 cos φ + 4(sec 4φ)2 φ cos φ + 4(sec 4φ)2
1 (d) (e)
(e) φ−1 1−φ
1 + x2 Solution
Solution
y= tan1 ex y = sin θ + tan 4θ
dy 1 du dy
Recall = × = cos θ + 4 sec2 4θ
dx 1 + u2 dx dθ
du x = θ − lnθ
u = ex ; = ex dx 1 θ−1
dx =1− =
dy 1 dθ θ θ
= × ex dy dy dθ
dx 1 + (e ) x 2 = ×
dx dθ dx
ex dy θ
= = (cos θ + 4 sec2 4θ) ×
1 + (ex )2 dx θ−1
0.28. MATHS 105 2014/2015 145

θ cos θ + 4θ(sec2 4θ)2 Solution


= (C)
θ−1 y = xlnx
dy 1
dv = lnx + x × = lnx + 1
11. v = 5t sin 2t then , is ? dx x
dt dy
(a) 10t cos 2t+1 (b) 10(t cos 2t+sin 2t) (c) − 1 = lnx + 1 − 1 = lnx(A)
0 (d) cos 2t+sin 2t (e) 5(2t cos 2t+sin 2t) dx
Solution R x2 + 2x
v = 5t sin 2t 17. dx is
dv x3 + 3x2 1
= 5 sin 2t + 5t(2 cos 2t) (a) ln(x3 + 3x2 ) 3 +c (b) ln(x3 + 36) + c
dt
= 5(sin 2t + 2t cos 2t)(E) (c) x3 + 6x + c (d) x2 + 6x + c (e) 0
Rπ Solution
12. 02 (sin x + cos x)dx is R x2 + 2x
dx
(a) 1 (b) 2 (c) 3 (d) 4 (e) 0 x3 + 3x2
du
Solution
Rπ π let u = x3 + 3x2 ; dx = 2
3(x + 6x)
0
2
(sin x + cos x)dx = [− cos x + sin x]02
π π R x2 + 2x R x2 + 2x du
= (sin − cos ) − (sin 0 − cos 0) 3 2
dx = × 2
2 2 x + 3x u 3(x + 2x)
= (1 − 0) − (0 − 1) = 1 + 1 = 2(B) 1R 1 1
du = lnu + c but u= x3 + 3x2
x2 + 5x + 6 3 u 3
13. limx→−3 is ? 1 1

x+3 = ln(x +3x2 )+c = ln(x3 +3x2 ) 3 +c(A)


3
3
(a) −1 (b) −2 (c) 2 (d) 1 (e) 3
Solution 3x6 + 2x5 + x
By factorizing the numerator 18. limn→∞ is ?
x6 + 4x2 + 7x
(x + 3)(x + 2) (a)0 (b) 1 (c) 2 (d)3 (e) 4
limx→−3 limx→−3 (x+2) =
x+3 Solution
−3 + 2 = −1(A) Dividing both the numerator and denu-
d2 y merator by x6
14. of y = x3 lnx is ? 3 + x2 + x15 3
dx2 limn→∞ 1 7 = = 3(D)
(a) lnx + 6x (b) 0 (c) ∞ (d) 10x + 6xlnx2 + x4 + x5 1
(e) 5x + xlnx2
Solution dy
19. Let y = sin θ and x = cos θ then is ?
dy 1 dx
= 3x2 ln + x3 × = 3x2 lnx + x2 (a) cot θ (b) − cot θ (c) tan θ (d) −tanθ
dx x
d2 y 2 1 (e) sec θ
= 6xlnx + 3x × + 2x
dx 2 x Solution
= 6xlnx + 3x + 2x = 6xlnx + 5x dy
y = sin θ ; = cos θ
No correct option dθ
dx
x = cos θ ; = − sin θ
15. Which of the following is true about f (x) = dθ
sin x + cos x dy dy dθ −1
= × = cos θ× = − cot θ(B)
(a) it is odd (b) xf(x) is odd (c) it is even dx dθ dx sin θ
(d) xf(x) is even (e) it is neither even or R 2x
odd 20. dx is ?
(x − 1)(2x + 1)
Solution 2
f (x) = sin x + cos x (a) ln(x − 1)(2x + 1) (b) 0 (c) (ln(x −
3
f (−x) = sin(−x) + cos(−x) = − sin x + 2x
cos x 1)(2x+1)) (d) lnx−1 (e)
ln(x − 1)(2x + 1)
Hence it is neither even or odd (E) Solution
R 2x A B
dy dx = +
16. If y= xlnx then − 1 is (x − 1)(2x + 1) (x − 1) (2x + 1)
dx 2
(a) ln x (b) 0 (c) x + lnx (d) 1 (e) x By partial fraction A = B =
3
146

2x 2R 1 2R 1
dxSolution
R
= dx+ tan x
(x − 1)(2x + 1) 3 x−1 3 2x + 1 lim + 2x2 + 4
x→0 f (x) =
2 1 x
= (ln(x − 1) + ln(2x + 1)) + c Applying L’hospital rule
3 2
2 1 0 sec2 x
= ln(x − 1) + ln(2x + 1) + c lim x→0 f (x) = + 4x + 0
3 3 1
No correct option 1
= + 4x
2
cos2 x
x − 4x − 5 1
21. limx→−1 = + 4(0) = 1(E)
x+1 1
(a) −4 (b) −6 (c) 6 (d) 5 (e) 1
4
Solution
R
2
26. √ dx is :
x − 4x − 5 (x + 1)(x − 5) 1 − 3x2
limx→−1 = limx→−1 4 √ √
x+1 (x + 1) (a) √ sin−1 3 + c (b) sin−1 x 3 + c
= −1 − 5 = −6(B) 3
4 4
dy 2x (c) √ sin−1 x + c (d) √ sin x + c (e)
22. of 2 is ? 3 3
dx x +1 4 −1
2 2x − 1 2 sin x4 + c
(a) (b) 2 (c) 2 3
2
((x + 1) 2 (x + 1) 2 x +1 Solution
2(1 − x2 )
R 4 R 1
(d) ∞ (e) 2 √ dx = 4 q √ dx
2 1 − 3x 2
(x + 1) 2
1 − ( 3x) 3
Solution 4 √
Using Quotient rule = √ sin−1 3x + c(A)
dy (x2 + 1)(2) − (2x)(2x) 3
=
dx (x2 + 1)2 sin t
(
2 2 2 t<0
2x + 2 − 4x 2(1 − x ) 27. Suppose g(t)= t
= = 2 (E) c t≥0
(x2 + 1)2 (x + 1)2
for what value of c would the function be
dz continuous at t = 0 ?
23. Let z = t2 + 1 and k = sin 2t then is
dk (a) 1 (b) 2 (c) 3 (d) 4 (e) 0
? Solution
2t t
(a) (b) 2t cos t (c) (d) t cos 2t
cos t cos 2t
(e) ∞
Solution 28. The domain of the real valued function of
dz 2x
z = t2 + 1 ; = 2t real variables defined by k(x) = 2
dt x −1
dk is ?
k = sin 2t ; = 2 cos 2t (a) < (b)</{1, 2} (c) < {1} (d) < {−1, 1}
dt
dz dz dt 2t t (e) </{0, 1}
= × = = (C) Solution
dk dt dk 2 cos 2t cos 2x
d2 y
24. If y x + cos x then is (a) 1 + sin x (b) R
dx2 29. xex dx is ?
sin x (c) − sin x (d) 1 − sin x (e) − cos x (a) ex (1 − x) + c (b) ex (x − 1) + c (c)
Solution ex (1 − x) + x+c (d) xex (e) ex (1 + x)
Y = X + cos x Solution
dy
= 1 − sin x Using
R x by partx R x
dx
dy2 xe dx = xe − e dx = xex − ex + c
= − cos x(E) = ex (x − 1) + c(B)
dx2
tan x d
25. If f(x) = ( +2x2 +4) then limx→0 f (x) 30. (2 cos4 x) is ?
x dx
is ? (a) −10 cos4 x sin x (b) −5 cos4 x sin x
(a) 5 (b) 3 (c) 3 (d) 4 (e) 1 (c) −10cos4 x (d) −5 cos4 x (e) −cosxsinx
0.28. MATHS 105 2014/2015 147

Solution Solution
let y = 2 cos5 x also let u = cos x Ans C
du
= − sin x ; ∴ y = 2u5 2x
dx 35. The derivatives of the curve y =
dy x2 −5
= 10u4 at point (2, −4) is ?
du
dy dy du (a) 17 (b) 15 (c) −15 (d) −18 (e) 18
= × = 10u4 × − sin x
dx du dx Solution
= −10 sin x cos4 x(A) 2x
2
x −5
31. Consider the function k(t) = sin t then dy (x2 − 5)(2) − 2x(2x) −1002x2
= = at
(a) tk(t) is odd (b) t2 k(t) is even dx (x2 − 5)2 (x2 − 5)2
(c) t3 k(t) is odd (d) tk(t) is even (2,−4)
(e) k(t) is even dy −10 − 2(2)2 18
= = − = −18(E)
Solution dx ((−2)2 − 5)2 1
k(t) = sin t
k(−t) = sin(−t) = sin t R x3 + x
36. dx is
Therefore the function is odd x2
x
f(t)= t sin t (a) ln 2 + c (b) x + x2 + c (c) x + lnx + c
x
f (−t) = (−t) sin(−t) = t sin t x2 x
f (−t) = f (t) even function (D) (d) + lnx + c (e) x + ln 2 + c
2 x
Solution
R x3 + x R x3
 
R 1 x R
32. dx is ? dx = + dx = (x +
x2−1 x2 x 2 x2
1 x+1 x+1 1 x−1 1
(a) ln +c (b) ln +c (c) ln + )dx
2 x−1 x−1 2 x+1 x
x−1 x2
c (d) ln + c (e) ln(x2 − 1) + c = + lnx + c(D)
x+1 2
Solution
R 1 R 1 37. Which of the following is true about f (x) =
dx = dx
x2 − 1 (x + 1)(x − 1) x2
R A R 1 + 2x4
= dx + dx cos x
(x + 1) x−1 (a) f(x) is periodic (b) f(x) is odd (c) f(x)
1 1 is even (d) f(x) is not defined at 0 (e) f(x)
by partial fraction A = − , B =
2 2 is linear
R 1 R 1
= dx − Solution
2(x − 1) 2(x + 1) x2
1 f (x) = + 2x4
= [ln(x − 1) − ln(x + 1)] + c cos x
2 (−x)2 x2
1 x−1 f (−x) = +2(−x)4 = +2x4
= ( ) + c(C) cos(−x) cos x
2 x+1 f (−x) = f (x) even function (C)
2
33. g : < → < ,given by g(x) = √ is 38. If f(t)= 5t2 + cos t then 10−f 00 (t) is
x 4 − x2 (a) 0 (b) − cos t (c) cos t (d) − sin t (e)
defined for all x ∈ < except
(a) {0, 1} (b) {−2, 0, 2} (c) {−1, 0, 1} (d) sin t
{0, 1, 2} (e) {0, 4} Solution
Solution f (t) = 5t2 + cos t
f 0 (t) = 10t − sin t
f 00 = 10 − cos t
34. f(x) is even if it is 10 − f 00 = 10 − 10 + cos x = cos t(C)
(a) Symmetrical about x axis (b) peri-
odic (c) Symmetrical about y axis (d) dy
constant (e) Linear 39. If y= x3x+2 then is ?
dx
148

2 2 0.29 PHYS131 09/10


(a) x3x+2 (3+ +3lnx) (b) 3+ +3lnx (c)
x x
3x+2 3x+2 2
x 3(1+lnx) (d) x (e) +3lnx3x+2 1. A cubical metallic container expands isotrop-
x ically so that its length increases from
Solution
y = x3x+2 54cm to 60cm when its temperature rises
lny = 3x + 2lnx differentiating by 200 c.Calculate its lunar,superficial and
y0 3x + 2 cubic expansivity respectively.
= 3lnx + (a)5.56 × 10−30 C −1 , 1.11 × 10−20 C −1 and
y x
0 3x + 2 1.67 × 10−20 C −1
y = y(3lnx + ) = x3x+2 (3lnx +
x (b)5.56×10−30 C −1 , 1.11×10−30 C −1 and1.67×
3x + 2 10−30 C −1
)
x (c)5.56×10−20 C −1 , 1.11×10−20 C −1 and1.67×
2
= x3x+2 (3lnx + 3 + )(A) 10−30 C −1
x
(d)5.56×10−30 C −1 , 1.11×10−30 C −1 and1.67×

40. 02 3 sin 2xdx is 10−20 C −1
(a) 5 (b) 4 (c) 2 (d) 3 (e) 1 (e)5.56×10−30 C −1 , 1.11×10−20 C −1 and1.67×
Solution 10−30 C −1
π Solution
Rπ R
0
2
3 sin 2xdx = 3 0 sin 2xdx Given that;l0 = 54cm, lt = 60cm, M θ =
cos 2x π2 3 200 C, α =?, β =? and γ =?
3[− ]0 = − [−1 − 1]
x 2 ∆l 60 − 54
3 α= = = 5.56 × 10−30 C −1
= − [−2] = 3(D) lo ∆θ 54(20)
2 But β = 2α = 2 × 5.56 × 10−3 = 1.11 ×
10−20 C −1 and
γ = 3α = 3 × 5.56 × 10−3 = 1.67 ×
10−20 C −1
Ans A

2. A clock whose pendulum makes one com-


plete oscillation in 2s was correct at 100 C.The
pendulum shaft is of aluminium.What will
be its gain or loss in period when the
temperature rises to 250 C?(Given ∝ of
aluminium= 2.4 × 10−5.0 C)
(a)2.9×10−4 s(b)3.6×10−4 s(c)3.2×10−4 s
(d)2.8 × 10−4 s(e)3.0 × 10−4 s
Solution
T 2g
r
l
T = 2π ∴l=
g 4π 2
Ti2 g
f or l0 =
4π 2
Tf2 g
f or lf =
4π 2 2
Tf g T 2g
lf − lo 4π 2
− 4πi 2
α= = T2
lo ∆θ i
2 M θ

t2f − Ti2
α= 2
Ti ∆θ
Tf2 − Ti2 = αTi2 ∆θ
p
Tf = αTi2 ∆θ + Ti2
Given that Ti = 2secs, α = 2.4 × 10−5
0.29. PHYS131 09/10 149

0
C −1 − ∆d = r∆θ(do + ∆d)
∆θ =p (25 − 10) = 150 C −∆d = r∆θdo + r∆θ∆d
−5 2
Tf = (2.4 × 10 × 2 × 15) + 2 = 2.000359secs2 −∆d(1 + r∆θ) = r∆θdo
∴ change in periods,Tf −Ti = 2.000359− γ∆θdo
−4
∆d = − (C)
2 = 3.59 × 10 secs 1 + δ∆θ
= 3.6 × 10−4 secs(B) 5. A cylinder of diameter exactly 2cm at
300 C is inserted into a hole in a steel
3. What should be the length of steel and
plate.The hole has a diameter of 1.99967cm
copper rods so that the length of the steel
at 300 C.If α of steel is 1.1 × 10−50 C,what
rod is 5cm longer than that of the copper
temperature must the plate be heated?
rod at all temperature change?(Given their
1.1×10−50 C −1 ,to what temperature must
coefficient of linear expansivity to be 1.7×
the plate be heated?
10−50 C −1 and 1.1 × 10−50 C −1 ) Respec-
(a)400 C (b)600 C(c)500 C (d)700 C (e)450 C
tively
Solution
(a)12.71cm and 17.71cm ∆l
(b)10.17cm and 15.17cm α=
lo ∆θ
(c)49.17cm and 14.17cm −5 (2 − 1.99967)
(d)48.25cm and 12.52cm 1.5 × 10 =
1.99967 × ∆θ
(e)41.21cm and 13.72cm 3.3 × 10−4
∆θ = = 150 C
Solution 1.1 × 10−5 × 1.99967
Let the length of cube xcu then the length θ2 = ∆θ + θ1 = 15 + 30 = 450 C(E)
of cube steel=(x+5)cm
αcu = 1.1×10−50 C −1 , αsteel = 1.7×10−50 C −1 6. A metallic sphere of diameter D has a
∆lcu ∆lsteel cavity of diameter d at it center.If the
αcu = , αsteel = sphere is heated,the diameter of the cav-
lcu ∆θ lsteel ∆θ
∆lcu ∆lsteel ity will
∆θ = = (a)Remain unchanged(b)decreases
αcu lcu αsteel lsteel
at all temperature∆lcu = ∆lsteel D
(c)increases (d)decreases if d <
αcu lcu = αsteel lsteel 2
−5
1.1 × 10 × x = 1.7 × 10 × (x + 5) −5 D
(e)increases if d <
1.1x = 1.7x + 8.5 2
Solution
x=−14.17cm Ans C
∴ that of steel=x+5= −14.17cm + 5 =
−9.17cm 7. A metallic container is completely filled
9.17cm and 14.17cm(C) with a liquid.The co-efficient of linear ex-
pansion of the metal is 2.0×10−60 C −1 and
4. The density of a liquid of co-efficient of the co-efficient of cubical expansion of
cubical expansion γ is d0 at00 C.When the the liquid is 6.0 × 10−60 C −1 .On heating
liquid is heated to a temperature θ,the the vessel,
change in density will be (a)The liquid will overflow
(−1 + γθd0 ) −δ0 (1 + γθ) −δ0 γθ (b)The level of the liquid will remain un-
(a) (b) (c)
γθ γθ 1 + γθ changed
δ0 (1 + γθ) δ0 1 + γθ (c)The level of the liquid will fall
(d) (e)
γθ 1 − γθ (d)The level of the liquid will rise de-
Solution
do − df pending on the nature of the metal
γ= (e)None of the above
df ∆θ
−∆d Solution
γ= Since the co-efficient of cubic expansion
df ∆θ
but df = do + ∆d of both the metal and that of the liq-
−∆d uid,therefore the level of the liquid will
γ= remain unchanged (B)
∆θ(do + ∆d)
150
1
8. Two rods of different materials having P 4
∴T = (B)
co-efficient of thermal expansionα1 andα2 Aεt
and young‘s moduli Y1 andY2 are fixed
11. If the temperature of a block increases
between two rigid and massive walls.The
from 70 C, and then the rate of energy
rods are heated to the same tempera-
radiation increases by
ture.If there is no bending of rods,the 7 4 287 1
thermal stress developed in them are equal (a)4 (b)2(c)16 (d)( ) (e)( )2
287 7
provided r Solution
Y1 α1 Y1 α1 Using P = AεγT 4
(a) = (b) =
Y2 α2 Y2 r α2 p = Aεt(T + Tatm )4
Y1 α2 Y1 α2 Y1 α2 p1 = Aεt(273 + 7)4 = (280)4 Aεt
(c) = (d) = (e) =
Y2 α1 Y2 α1 Y2 α1 p2 = Aεt(273 + 287)4 = (560)4 Aεt
 4
Solution P2 560
Thermal stress=αγ∆T = = (2)4
P1 280
α1 Y1 ∆T = α2 γ2 ∆T P2 : P1 = 16 : 1(C)
Y1 α2
= (C) 12. The filament of an incandescent lamp at
Y2 α1
17270 C emits 2000J per minutes.If its
9. Two different metal rods of the same length surface area is 10−4 M 2 .Find its emissiv-
have their ends kept at the same temper- ity.Given Stefan‘s constant=5.67×10−8 W M −2
ature θ1 and θ2 with θ2 > θ1 .If A1 andA2 and the room temperature is 270 C.
are their cross sectional areas and K1 and K2 (a)0.37 (b)0.36 (c)0.35 (d)0.34 (e)0.32
their thermal conductivities,the rate of Solution
flow of heat in the two rods will be the p = Aεt(T 4 − Tatm4
)
same if 2000
p= = 33.33J/s
A1 K2 A1 K1 A1 K1 θ1 60
(a) = (b) = (c) = Tatm = 273 + 27 = 300k
A2 K1 A2 K 2 A2 θ2 K2
A1 K2 θ2 A1 K1 θ2 T = 17270 C = 2000K
(d) = (e) = 33.33 = 10−4 × ε × 5.67 × 10−8 × (20004 −
A2 K1 θ1 A2 K2 θ1
Solution 3004 )
Q 33.33
Rate of heat flow= ε= = 0.37(A)
t 90.674
Q KA(θ1 − θ2 )
we know that, = 13. The thermal conductivity of a brick is 20
t l
k1 A1 (θ1 − θ2 ) k2 A2 (θ1 − θ2 ) times that of an air,calculate the thick-
=
l l ness of brick which conducts the same
k1 A1 = K2 A2 quantity of heat per second per units as
K1 A2
= (A) 0.1m of air.
K2 A1 (a)4m(b) 1m(c)5m (d) 2m(e)6m
Solution
10. The tungsten filament of an electric lamp
has a surface area A and a power rating
P.If the emissivity of the filament is ε and 14. The thermal conductivity of ice is
σ is the Stefan‘s constant,the steady tem- 1.6Js−1 m−1 K −1 .calculate the rate of heat
perature of the filament will be loss by water in a 10m by 6m swimming
p 2 p 2
(a)T = ( ) (b)T = ( )4 pool covered by a layer of ice 3cm thick,if
Aεσ Aεσ the water is at a temperature of 00 C and
p 1 p p 1
(c)T = ( ) (d)T =
2 (e)T = ( ) 3
the surrounding air at a temperature −120 C
Aεσ Aεσ Aεσ
Solution (a)38000js−1 (b)38480js−1 (c)38500Js−1
Power Rating=P (d)38400Js−1 (e)34000Js−1
4
p = AεtT Solution
4 P Q KA(θ1 − θ2 )
T = =
Aεt t l
0.29. PHYS131 09/10 151

1.6 × 10 × 6(0 − (−12)) 1152 ratio of 1.67 and calculate the value of
−2
= =
3 × 10 3 × 10−2 Cv .If molar gas constant is 8.314×10−23 J(Kmol)−1
38400J/s(D) (a)0.08J/(KgK) (b)0.32J/(KgK)(c)3.11J/(KgK)
15. The following are temperature measuring (d)12.47J/(KgK)(e)12.4J/(KgK)
instrument which of these sets are elec- Solution
tric contact thermometer? Cp : Cr = 1.67
i.Dilatation thermometer cp = 1.67Cv using, cp − cv = R
ii.Quartz thermometer 1.67Cv − Cv = 8.314 × 103
iii.Fibre optic thermometer Cv = 12.41 × 103 (E)
iv.Disappearing filament pyrometer 19. The volume of flask X is four times that
v.Gas thermometer of a flask Y and are join together.If the
vi.Total radiation pyrometer system is filled with an ideal gas and a
(a)i,ii and vi (b)ii and iii (c)iv and vi steady state is established with the flask
(d)iii,iv and vi (e)iv,v and vi held at 250k and 500k respectively,if the
mass of the gas in X is m.What is the
Use fig below the P V -diagram of a carnot mass of the gas in Y?
cycle and answer the question below. (a)4m (b)m/4 (c)m/8 (d)8m (e)2m
Solution

20. A beaker of volume V is pushed verti-


cally downward into a pool of water with
its mouth downwards.If the atmospheric
pressure is equivalent to 10.0m of wa-
ter,how far below the water surface must
the beaker be pushed so that the volume
of air in it becomes 0.8v?The tempera-
16. The area representing the work done by ture of the pool may be considered to be
the gas is described by constant.
(a)HIJNLH(b)HIJNMKH (a)2m (b)9.92m (c)−2m(d)12.5m(e)2.5m
(c)HIJKH(d)HKJNLH(e)KJNMK Solution
Solution Given that v1 = v, V2 = 0.8v, h1 = 10m
Work done=Area enclose by p-v.graph=HIJKH(C) of water
p1 = ρgh = 100 × 10 × 10 = 100000Pa
Using Boyle‘s eqn,p1 v1 = p2 v2
17. Estimate the root mean square speed of 100000 × v = p2 × 0.8v
carbon mono-oxide gas at 273K.if avogadro‘s p2 = ρgh2
number is 6.022×1026 moleculeskilomol−1 1.25 × 105
and Boltzmann constant is 1.381×10−23 J(Kmol)h −12 =
1000 × 10
(a)2.01E −11(b)4.04E −22 (c)493.24m/s h2 = 12.5m of water(C)
(d)243.29E2 (e)1.72E − 11
Solution 21. At the triple point of water the pressure
:Molar Mass of CO = 3Og r of a fixed mass of gas is 2680pa.The tem-
3KB To perature is charged to T while the vol-
using the relation:Urms = = ume of gas is kept constant.The pressure
M
r is then 4870pa.Find the value of T
3 × 1.38 × 10−23 × 273
(a)496.83K(b)496.38K(c)150.32K
30 (d)3.57E9K(e)47.780K
Ur.m.s = 1.94 × 10−11 m/s(A)
Solution
18. Assume the equipartition energy of a gas Given that pressure at triple point pt =
having N degree of freedom has Cp to Cv 2680P a and at a particular temperature=4870P a
152

Temperature at triple point of water=273.16k process BC 200j of heat is added to the


Temperature,T at the particular pressure. system.Calculate the change in internal
Pt 2680 energy in the process ABC.
Using,T= × 273.16 = × 273.16 =
p 4870
150.32k(C)
22. A uniform capillary tube closed at one
end contained air trapped by thread mer-
cury 80mm long.When the tube was held
through an angle 80 the length of the air
column was 50mm then the tube shift to
an angle 20 with air column of 40mm.Deduce
the atmospheric pressure of the system
(a)23.23P a(b)231.232P a(c)23P a
(d)32.23P a(e)45P a
(a)600J(b)800J(c)560J(d)640J(e)300J
Solution
Solution
Process AB is isochloric ∆VAB = 0
23. Calculate the r.m.s speed of oxygen gas ∴ Work done in AB = 0.
molecule at a pressure of 100Kpa when In BC pressure=constant
the density is 1.43Kgm−1 W.D = PB × (VD − VA ) = 8 × 104 × (5 ×
(a)485m/s(b)458m/s(c)2.10E05m/s 10−3 − 2 × 10−3 ) = 240J
(d)485(m/s2 )(e)458(m/s2 ) Change in internal energy=∆u = dQ −
Solutionr r dw = 800 − 240 = 560J(C)
3p 3 × 100 × 10−3
Ur.m.s = = 26. A block of ice at −100 C is slowly heated
ρ 1.43
Ur.m.s = 458m/s(E) and converted to steam at 1000 C.Which
of the following curves represents the phe-
24. The potential energy that exists between nomenon qualitatively?
a b
two molecules is given by V = 8 − 3
r r
where a and b are constant and r is the
distance.The inter atomic spacing at equi-
librium is therefore
4.3a 1/4 a b
(a)( ) (b)1.22( )1/5 (c)0.5( )1/4
b b a
a 1/5 b 1/5
(d)(0.5 ) (e)2.2( )
b a
Solution
−du
At equilibrium, =0
dr
−du 8a 3b
= 9 − 4 =0
dr r r
8a 3b
= 4
r9 r
5 8a
r =
3b
r = 1.22(a/b)1/5 (B)
25. A thermodynamics process as shown in Solution
the p-v diagram (Figure below),the pres- Ans D
sures and volumes corresponding to some
points in the figure arePA = 3×10−3 m3 , PB =27. The equation of the F-r curve for a solid
8 × 104 pa, VD = 5 × 10−3 m3 .In the pro- 1.2x10−24 2.8x10−10
cess AB,600J of heat is added and in NaCl is given by F = − .
r5 r2
0.30. PHYSICS 131 2010/2011 153

Calculate the force constant K within the T A(L1 α1 + L2 α2 )


(c) F =
elastic limit of the molecules at r angstroms ( LY12 + LY21 )
and state the nature of the slope. T A(L1 α1 + L2 α2 )
(d) F =
(a)2.8 × 1010 N/m; −veslope ( LY21 + LY12 )
(b)1.55 × 1034 N/m; −veslope T A(L1 α1 − L2 α2 )
(c)2.8 × 1010 N/m; +veslope (e) F =
( LY11 + LY22 )
(d)3.1 × 1034 N/m; +veslope
Solution
(e)2.8 × 1034 N/m; +veslope
Solution
−dF
k=
dr
dF 5(1.2 × 10−24 ) 2(2.8 × 10−20 )
= −
dr ro6 ro3
−24
5(1.2 × 10 ) 2.8 × 10−10 2
k= −
(2.7 × 10−10 )6 (2.7 × 10−10 )3
= 1.55 × 10 − 2.845 × 1019
34

= 1.55 × 1034 N/M (B)

28. Capillary rise of liquid in tubes is due to


(a)The meniscus being convex L1 , γ1 , α1 , A, L1 , γ2 , α2 , A2 ,
(b)The angle of contact being obtuse Temperature raise = T
(c)The force of adhesion being greater Increase in length of the first metal =
than the force of cohesion ∆L1
(d)The force of cohesion being greater ∆L1 = L1 α1 T
than the force of adhesion and second metal = ∆L2
(e)None of the above ∆L2 = L2 α2 T
Solution Total increase = ∆L1 + ∆L2
Answer:C ∆L1 + ∆L2 = T (L1 α1 + T L2 α2 )....(1) be-
cause the walls are massive, the increase
in length will not occur, due to the com-
pressor force provided by the wall.
0.30 Physics 131 2010/2011 γ(A)αL FL
F= ⇒ ∆L =
L A(γ)
Two rods of different metals having the same F L1 F L2
area of cross-section A are placed between two ∆L1 = andγ(L2 ) =
Aγ1 A(γ2 )
massive walls as shown in the figure below. F L1 F L2
The first rod has length L1 coefficient of lin- 4L1 = and 4L2 =
Ay1 Ay2
ear expansion α1 and young modulus Y1 .The F L1 F L2
corresponding quantities for the second rod are M L1 + M L2 = +
Ay1 Ay2
L2 , α2 , Y2 .The temperature of both rods is now 
F L1 L2

raised by T degree .Use the information to an- = + − − − (1)
A Y1 Y2
swer question 1 and 2. Equating (1) and (2)
F L1 L2
⇒ T (L1 α1 + l2 α2 ) = ( + )
1. Find the force exerted by one rod on the A T1 T2
T A(L1 α1 + L2 α)2
other (at higher temperature) in terms of ⇒F =
L1 L2
the given quantities. +
T A(L1 α1 + L2 α2 ) T1 T2
(a) F = Ans A
( LY11 + LY22 )
T A(L1 α2 + L2 α1 ) 2. Also find the length of each rod at higher
(b)F =
( LY11 + LY22 ) temperature
154

F vMp
(a) L = L1 (1 + α1 T − ), Mv=−
Y2 A B
F −100 × 1.25 × 105
L = L2 (1 + α2 T − ) M V = = −2.0 ×
Y1 A 6 × 1011
F 10−8 cm3
(b) L = L1 (1 + α1 T − ), original volume = v1
Y1 A
F v1 = v2 − M v = 100 − (−2.08 × 10−8 )
L = L2 (1 + α2 − )
Y2 A v1 = 100.0000208cm3
F
(c) L = L1 (1 + α2 T − ),
Y2 A 4. The density of a metal at normal pressure
F is ρ;its density when it is subjected to
L = L2 (1 + α1 − )
Y2 A an excess pressure is ρ1 .if B is the bulk
(d) L = L1 (1 + α1 T ), L = L2 (1 + α2 ) modulus of the metal ,the ratio ρ/ρ
F
(e) L = L1 (α1 T − ), Solution
Y2 A vMp
F B=−
L = L2 (1 + α2 − ) Mv
Y2 A Mv p
=−
Solution m
v m B
If the massive wall where not in place the ρ1
− ρ ρ
=−
new length = lf m B
1
lf = l0 + l0 α1 T ρ1
− ρ1 p
=−
but because of the compressive force pro- ρ B
vided by the wall,there will be a decrease ρ − rho1 p
×ρ=−
in length = M l ρρ1 B
∴ lf = l0 + l0 αT − M l ρ − ρ1 p
=−
F l0 ρ B
lf = l0 + l0 αT − ρ p
AY =1−
F L1 ρ1 B
∴ for the first rod lf = l1 + l1 α1 T − ρ1 1
AY1 =
f l2 ρ (1 − Bp )
Also,the second lf = l2 + l2 α2 T − Ans C
Ay2
Ans B
5. The density of water at the surface of the
ocean is 1.029 × 103 kgm−3 .What is the
3. Artificial diamond are made by subject- density of ocean water at a depth where
ing graphite to a high pressure 1.25 × 105 the pressure is 80.45 atm.Bulk modulus
Pa at a high temperature ,Assuming that of water is 2.1844 × 109 Pa and 1 atm =
natural were found at such high temper- 1.013 × 105 P a
ature within the earth .what have be- (a) 2.033 kgm−3 (b) 1.033 kgm−3 (c) 3.033
ing the original volume of the diamond kgm−3 (d) 0.033 kgm−3 (e) 4.033 kgm−3
whose mass is 340g.The density of dia- Solution
mond = 3400kgm−3 and its bulk modu- Using the equation in question 4
lus = 6 × 1011 N m−2 ρ1 1
(a) 90.08cm−3 (b) 95.08cm−3 (c)99.08cm−3 =
ρ (1 − Bp )
(d)102.08cm−3 (e)100.08cm−3 Then substituting the values given
Solution p = 80.45 × 1.013 × 105 = 8149588P a
new volume = v2 B = 2.1844×109 P a ρ = 1.029×105 Kgm−3
mass ρ
Using density = ρ1 =
volume 1 − Bp
340 × 10−3 kg
v2 = = 1.0 × 10−4 m3 1.029 × 103 1.029 × 103
3400kgm−3 ρ= =
814985 0.996269
v2 = 100cm3 1− 9
vMp 2.1844 × 10
but B = − ρ1 = 1033Kgm−3
Mv
0.30. PHYSICS 131 2010/2011 155

1
6. A soap bubble of radius r is blown up to Work done = 4 × R2 σ(n 3 − 1)for n= γ
1
form a bubble of radius 2r under isother- Work done = 4πR2 σ(8 3 −1) = 4πR2 σ(2.1)
mal condition .If σ is the surface tension = 4πR2 σ(A)
of the soap solution,the energy spend in
doing so is 10. Eight spherical rain drops of the same
(a) 3 πσr2 (b)6πσr2 (c)12πσr2 (d)24πσr2 mass and radius are falling down with a
(e) πσr2 terminal speed of 6cms−1 .If they coa-
Solution lesce to form one big drop,what will be
Surface area = 4πra2 but for ra = 2r then its terminal speed?Neglect the buoyancy
S.A= 4π(2r)2 = 16πr2 due to air .
M S.F = 16πr2 − 4πr2 = 12πr2 (a)1.5cms−1 (b) 6cms−1 (c) 24cms−1 (d)
for soap M S A= 2(12πr2 ) = 24πr2 32cms−1 (e) 5cms−1 Given thermal ve-
∴ Energy = (24πr2 ) × γ = 24γπr2 locity of small drop (vts) = 6ms−1
Ans D 2 ρr2 g
Vts =
9 o
7. A cylindrical jar has a radius r,to what 2 ρr2 g
6=
height h should it be filled with a liquid 9 o
so that the force exerted by the liquid on 27o
r2 =
the side of the jar equals the force ex- ρg
erted on the bottom The volume of small drops = Vs
r 4
(a) h = (b) h=r (c) h=2r (d)h=4r (e) Vs = πr3
2 3
h=5r Volume of big drops= 8Vs
Solution 4
8 × πr3 let R = radius of big drop
Force at the bottom = Pressure × Area 3
4 3 4
at the bottom= p × πr2 = ρgh × π 2 = πR = 8 × πr3
ρghπr 3 3
R3 = 8r3 = (2r)3
Force at the side = Average pressure × R = 2r − − − − − −(1) then
ρgh
side of one cylinder= ×2πrh = ρgrπh2 2 ρR2 g
2 Velocity of big drop VtB =
Given that force at bottom= force at the 9 o
side 2ρ(2r)2 g 2ρg
VtB = = × 4r2
ρghπr2 = ρgrπh2 9o 9o
r = h Ans B 2ρg 27o
= ×4× = 24cms−1
9h ρg
8. If the surface tension of soap is σ,what is
11. In a test experiment on a model aero-
the work done in blowing soap bubble of
plane in a wind tunnel,the flow speeds on
radius r?
the lower √and upper surfaces of the wing
(a) πr2 σ (b) 2πr2 σ (c) 4πr2 σ (d)8πr2 σ
are v and 2v respectively .if the density
(d) πr2 σ 2 Solution
of the air is ρ and the surface area of the
Energy = σ× surface area
wing is A ,the dynamic lift of the wing is
S.A = 4πr2 for soap bubble
given by
S.A = 2(4πr2 ) = 8πr2 1 1 √
Energy = σ × 8πr2 = 8πr2 σ (D) (a) √ ρv 2 A (b) ρv 2 A (c) 2ρv 2 A (d)
2 2
1
9. The work done to breakup the drop of 2ρv 2 A (e) √ ρvA
2
liquid of radius R and surface tension σ 1
into eight drops,all of equal size Dynamic lift = ρ(v22 − v12 )A
1 1 2
1 √ 2
(a)4 πσR2 (b) 2πσR2 (C) πσR2 (d)

2 4 = ρ ( 2v) − v 2
2 2
2
πσR (e) πσR 2 1 1
3 ρ(2v 2 − v 2 )A = ρv 2 A(B)
2 2
Solution
156

12. Two capillary tubes A and B of equal Solution


radii ra = rb = r and length la = lb = 1
Flow rate (Q) = AV,but V ∝ where
l are held horizontally .When the same A
flow rate is constant at narrow part.Velocity
pressure difference P is maintain across Time Ans D
each tube ,the rate of flow of water in
each is Q.if the tubes are connected in 15. The volume of a metallic ball increases by
series and the same pressure difference P 0.18% when its temperature is increase
is maintained across the combination,the by 300 C .what is the coefficient of linear
rate of flow through the combination will expansion of the metal
be (a) 2 × 10−5 C −1 (b) 3 × 10−5 C −1 (c) 6 ×
Q
(a) (b) Q (c) 2Q (d) 3Q (e) none of 10−5 C −1 (d) 1.8×10−5 C −1 (e)5×10−5 C −1
2 Solution
the above
Solution increase in volume M v = 0.18%, v1 =
Rate of flow Q=Qa = Qb (i.e before con- 100cm3 , M T = 300 C
Mv 0.18
nection) Y = = = 6 × 10−5 c−1
πpr4 vMT 100 × 30
Q= Y 6 × 10−5
8oL α= = = 2 × 10−5 c−1 (A)
When connected in series,the resultant 3 3
length = 2L 16. On a railway track,steel rails each 50.om
πρr4 πpr4 Q long are laid at 200 C .How much gap
Q= = = (A)
8 o (2L) 8 o L(2) 2 must be left between consecutive rails if
the highest day temperature during sum-
13. In Q12 ,ra = rb = r and lb = la = l mer is 450 C
if a pressure difference P is maintained (a) 13.75mm (b) 13.57mm (c) 12.57mm
across tube A ,the rate of flow of water (d) 1475mm (e) 12.75mm
in it is Q,if the tubed are connected in Solution
series and the same pressure difference P M T = 45 − 20 = 250 C, l1 = 50m
is maintained across the combination,the Ml
α=
rate of flow through the combination will l0 M T
be incomplete data
Q
(a) (b) Q (c)2Q (d) none of these (e) 17. The wavelength of the radiation emitted
2
all of these by a body depends upon
Solution (i) The nature of its surface (ii) The area
Given Qa = Q of its surface
πpra4 πpr4 (iii) The temperature of its surface
Q= =
8 o (2l) 16 o l (a) i and ii (b) ii and iii (c) i only (d) ii
when connected in series ,resultant length only (e) iii only
La + Lb = 2l + l = 3l Solution
2 πpr4
Qr = × from wien’s law λm T = c where c is a
3 16 o l constant
2Q 1
Qr = (D) λm ∝
3 T
The wavelength depends only on the tem-
14. Which one of the following statement is perature of the body (E)
is correct ? when a fliud passes through
the narrow path of non-uniform pipe. 18. A black body is at a temperature of 2880K
(a) its velocity and pressure both increase .The energy of radiation emitted by this
(b) its velocity and pressure both decreases body between 499mm and 500mm is U1 ,between
(c) its velocity decreases but its pressure 999mm and 1000mm is U2 and between
increases (d) its velocity increases but its 1499mm and 1500mm is U3 .The Wien’s
pressure decreases (e) None of the above constant b = 288 × 106 mmK.Then
0.30. PHYSICS 131 2010/2011 157

(a)U1 = 0 (b) U3 = 0 (c) U1 > U2 (d) Mass of ice =m but mlf + mc M θ =


U2 > U1 (e) U1 = U3 Solution mw cw 4 θ
b 336 × m + m × 4.2(8 − 0) = 1000 × 4.2 ×
Maximum emitted radiation = λm =
Tm (25 − 8)
2 × 106 336m + m(4.2)(8) = 1000(4.2)(17)
= = 1000nm
25 336m + 33.6m = 71400
Max radiation is at u2 i.e u2 > u1 (D) m = 193g(D)
19. If the temperature of the sun were to in-
22. A beaker contains 500g of water at 200 C
crease from T to 2T and its radius from
and is heated steadily .If takes 480s to
R to 2R ,then the ratio of radiant energy
reach boiling of 1000 C,how long will it
received on the earth to what it eas pre-
then take for all the water to boil away
viously will be
(latent heat of vaporization LV = 2.268×
(a) 1 (b) 16 (c) 32 (d) 64 (e) 256
j/kg,specific heat capacity of water Cw =
Solution
4200j/kg.k)
Emitted radiated heat = σ4πR2 × T 4
(a)50min (b)51min (c)54min (d)52min (e)53min
R2 = 2R and T2 = 2T
Solution
Q2 = σ4π(2R)2 × (25)4 mc M θ
Q2 256σπR2 T 4 Rate of heat generated (H) = =
= = 64(D) t
Q1 4πR2 σT 4 0.5 × 4200 × (100 − 20)
= 350j/s
480
20. Two cylindrical rod of length L1 and L2 Heat required to evaporate water = Hv =
,radii r1 and r2 have thermal conductiv- 500
× 2.268 × 106
ities k1 and k2 respectively .The ends of 1000
the rods are maintained at the same tem- Hv = 1.134 × 106 J
1.134 × 106
perature difference .If L1 = 2L2 and r1 = ∴ time for H2 O to boil away = =
r2 350
,the rates of heat flow in them will be 324sec = 54min
2
k1
the same if is Water flows at the rate of 0.15kg/min
k2
(a) 1 (b)2 (c)4 (d)8 (e) 16 through a tube and heated by a heater
Solution dissipating 25.3W.The inflow and outflow
Q kπr2 M Q water temperature are 14.40 C and 16.60 C
Rate of heat flow = =
t l1 respectively .When the rate of flow is in-
Q1 k1 πr12 ⇔ Q creased to 0.2318kg/min and the rate of
for first rod = =
t L1 heating to 37.8W,the inflow and outflow
Q2 k2 πr22 ⇔ Q temperature are kept constant.Use this
for Second rod = =
t L2 information to answer question 23 and
but l2 = 2l1 and r2 = 2r1 24
Q k2 πr22 M θ
∴ = × from here
t 4 2l2 23. Calculate the specific capacity of water
Q1 Q2 k1 πr22 M θ k2 πr22 4 θ
= = = (a)42J/Kg.K (b)420J/Kg.K (c) 4200J/Kg.K
t t 8l2 l2 (d)4020J/Kg.K (e)402J/Kg.K
k1 8
= (D) Solution
k2 1 0.15KG
In flow rate Q1 = = 2.5×10−3 kgs−1
60s
21. What mass of ice at 00 C need to be added ⇒ 0.2318kg
to 1000g of water at 250 C to lower the Outflow rate Q= = 3.86 ×
60s
−3 −1
temperature to 80 C (Specific latent heat 10 kgs
of fussion of ice =336j/g and specific ca- using p2 − p1 = (Q2 − Q1 )Cw M θ
pacity of water =4.2j/gk) 37.8−25.2 = (3.86×10−3 −2.56×10−3 Cw (16.6−
(a)500g (b)200g (c)150g (d)190g (e)93 14.4))
Solution Cw = 4200.9/kgk(C)
158

24. Find the rate of loss of heat from the tube per second.It is suddenly stoped and 50%
(a) 2.1W (b) 2.0W (c) 2.2 W (d)2.3W of its energy is used in increasing its tem-
(e)2.4W perature.Then the rise in temperature of
Solution the sphere is
h= rate of heat loss 2π 2 n2 r2 π 2 r2 π 2 n2 r2
m i cw M θ (a) (b) (c) (d)
p= +h 5S 10r2 S 5S
2
t1 7 2 2 5(πrn)
0.15 × 4200 × (16.6 − 14.7) πr n S (e)
h = 25.2 − 8 14S
60 Solution
h = 2.1W (A)

25. With natural convention the rate of loss 29. The snow on the mountain does not melt
5
of heat was proportional to θ 4 where θ is at once when its is heated bu sun because
the excess temperature.Whose statement (i) it becomes very hard (ii) it has high
is this latent heat of fusion
(a) Dulong and petit (b) Newton (c) Gra- (iii) it has low specific capacity
ham (d) Dulong (e)Petit Solution (a)i only (b) ii only (c) iii only (d) i and
Ans B ii only (e) ii and iii only
Solution
26. Newton’s law of cooling appears to be Ans B
true :
(i)In still air only for a temperature ex- 30. A gas of 500 cm3 volume at a tempera-
cess of about 20K or 30K ture of 300K and a pressure and 20Pa is
(ii) For all excess temperature in condi- enclosed in a sphere A.If sphere A,connected
tion of forced convention of the air to an empty sphere B of 100cm3 by a
(iii) For all higher temperature from 50K tube and valve .If the valve is removed
to 300K and gas flow fron A to B ,the tempera-
(a) i only (b) ii only (c) iii only (d) i and ture in A remain constant and that of B
i (e) i and iii Solution change to 500K.What is the pressure of
Newton’s law of cooling holds true when the system
their is a small temperature difference (a) 50000pa (b) 2800pa (c)85.17pa (d)17.85pa
,greater than that of the surrounding (e)5000pa
Ans D
31. A polyatomic gas of γ=1.33 and R =
27. 300g of water at 250 C is added to 100g of 8.31jmol−1 k −1 ,its specific heat capacity
ice at 00 C .The final temperature of the at constant pressure is
mixture is (a)2.0775 jkg −1 k −1 (b) 3.324jkg −1 k −1 (c)
50 50
(a) − C (b) − C (c) −50 C (d) 00 C 20.775jkg −1 k −1 (d)33.24jkg −1 k −1 (e)332.4jk
3 2 Solution
(e)50 C CP
Solution Y= but CP − R = Cv
Cv
Temperature of mixture = Q CP
Mw Cw (M Q) = Mc Cw M θ + Mc L Y =
CP − R
300 × 4.2 × (25 − Q2 ) = 100 × 4.2(Q2 − CP
1.33 =
0) + 100 × 33 CP − 8.31
315 − 12.6Q2 = 4.2Q2 + 336 11 − 0.53
CP = = 33.493/kgk(D)
16.8θ2 = −21 0.33
50
θ2 = − C 32. Which of the following represent the work
4 done on a triatomic gas that is compressed
28. A metal sphere of radius r and specific adiabatically ?
heat S is rotated about an axis passing (a)nR(T1 −T2 )/0.33 (b) 2.303nRT Log(P2 /P1 )
through its center at a speed of n rotation (c) p(v2 −v1 ) (d) nR(T1 −T2 )/(1−/gamma)
0.30. PHYSICS 131 2010/2011 159

m
(e) p(v1 − v2 ) P1 V1γ = P2 V2γ but V =
Solution d
m1 γ m2 γ
Ans B P1 ( ) = P 2 ( )
d1 d2
P1 P2
33. Given that the ratio of specific heat ca- = γ
dγ1 D2
pacity at constant pressure to that at P2 7

constant volume of a diatomic gas is 1.4.Cal- = (32) 5 = 128(C)


P1
culate both the specific heat capacities if
the molar gas constant is 8.31jmol−1 k −1 37. A sample of an ideal gas volume V,pressure
−1 −1
(a) 29.085 jlg k and 21.775jlg k −1 −1 P and temperature T.The mass of each
−1 −1
(b) 20.775jlg k and 20.085jlg k −1 −1 molecule of the gas is m.The density of
(c) 29.085jlg −1 k −1 and 20.775jlg −1 k −1 the gas is (k is the Boltzmann’s constant)
−1 −1
(d) 20.775jlg k and 2.0775jlg k −1 −1 (a) mKT (b) p/Kt (c) p/kvT (d) mp/kT
−1 −1
(e)29.085 jlg k and 20.085 jlg k −1 −1 (e) mp/kvT
Solution Slution
CP Using P V = nkT
Given = 1.4 but Cv = Cp − R m
Cv pv = KT
Cp Mm
1.4 = m m
CP − 8.31 M mp = kT but d =
Cp = 29.0853j/kgk but also Cv = CP −R v v
M mp
then d= (D)
kT
Cv = 29.085 − 8.31 = 20.775j/kgk(C)
38. A gas does 4.5j of external work during
34. A monoatomic gas isothermically expands adiabatic expansion.its temperature falls
from a volume v1 to v2 at 300K .If the by 2K.its internal energy will
ratio of the volume v2 to v1 is 2.Calcu- (a) increase by 4.5j (b) decrease by 4.5j
late the work done by the gas .Assume (c) decrease by 2.25j (d) increase by 9.0j
−1 −1 (e) increase by 2.25j
R=8.31jmol k
(a)1246.5j (b)2493j (c)1728.15j (d)24930j Solution
(e)12465j In adiabatic process dw = −du
Solution ∴ du=−4.5j(B)
Work done = RT = 8.31 × 300= 2493J
(B) 39. A certain amount of heat energy is sup-
plied to a monoatomic ideal gas which
35. In the old west,a desperado fires a bul- expands at constant pressure.What frac-
let into an open water tank,creating a tion of the heat energy is converted into
hole at a distance h below the water sur- work
2 2 5
face.What is the speed v of the water (a) 1 (b) (c) (d) (e) 2
emerging from the hole 3 5 7
Solution
(a) [2g(h2 −h1 )]1/2 (b) x2 /t (c) [p+ρhg]1/2 Heat energy supplied dQ = CP dT also
(d) [2(p + ρhg)]1/2 (e) t/x2 du = Cv dt
Solution √ ∴ dw= dQ − du = CP − Cv dT
Velocity = 2gh where h= h2 − h1 dw CP − Cv CP − Cv
v = (2g(h2 − h1 )) 2 (A)
1 = =
dθ Cp dT CP
1 dw
36. The pressure and density of a diatomic 1− =
γ dθ
gas (γ = 7/5) change adiabatically from dw 3 2
d2 p2 = 1 − = (C)
(p1 , d1 ) to (p2 , d2 ) .If = 32 then dθ 5 5
d1 p1
(a) 1/128 (b) 32 (c) 128 (d) 256 (e) 23 40. One mole of an ideal gas (γ = 1.4) is adi-
Solution abatically compressed so that its temper-
γ
In adiabatic change P V = constant ature rise from 250 C to 350 C.The change
160

in the internal energy of the gas is (given 0 = u + w ⇒ 0 = u + 4.5


R= 8.3j/mol/k) ∴ u = −4.5j
(a)−166j (b) 166j (c)−168j (d) 168j (e) So -ve sign indicate decreasing (B)
165j
Solution 4. A vessel contain 1 mole of O2 gas (molar
RMT 8.31 × 8 mass 32) at a temperature T.The pres-
Mu= = = 166J(B)
(Y − 1) 1.4 − 1 sure of the gas is P.An ideal vessel con-
taining one mole He gas (molar mass 4)
at a temperature 2T has a pressure of
0.31 PHY131 2011/2012 p
(a) (b)p (c) 2p (d)8p (e)
p
8 2
1. Which of the following statement is true Solution
P1 P2
about a gas undergoing an adiabatic change Using pressure law =
? T1 T2
P1 = P, T1 = T, P2 =?, T2 = 2T
(a) The temperature of the gas remain P P2 2P T
constant = i.e p2 = = 2P (C)
T 2T T
(b) The pressure of the gas remain con-
stant 5. Which one of the following statement are
(c) The volume of the gas remain con- not true for gas undergoing an isothermal
stant change
(d) The gas is completely insulated from (a) The temperature of the gas remains
the surroundings constants
(e) All of the above (b) The pressure of the gas remains con-
Solution stants
Ans D; The gas is completely isolated (c) The volume of the gas remain con-
from the surrounding ∴ Q=0 stants
(d) The gas is completely insulated from
5 the surrounding
2. One mole of a monoatomic gas (γ = )
3 (e) b,c and d above
is mixed with one mole of a diatomic gas Solution
7
(γ = ).What will be value of γ for the If the gas is undergoing isothermal change
5
mixture the temperature remain constant (A)
(a)1.5 (b)1.54 (c)1.4 (d)1.45 (e) none of
the above 6. Which of the following statement is not
Solution true for gas undergoing an adiabatic change
5 7 (a) The temperature of the gas remains
γ= , γ=
3 5 constants
5 7 (b) The pressure of the gas remain con-
γ + γ = + = 3.067
3 5 stants
2γ = 3.067
3.067 (c) The volume of the gas remain con-
γ= = 1.54 stants
2
(d) The gas is completely insulated from
3. A gas does 4.5j of external work dur- the surroundings
ing adiabatic expansion,if its tempera- (e) a,b and c above
ture falls by 2K,its internal energy will Solution
(a) increase by 4.5j (b) Decrease by 4.5j Ans E because only D satisfy adiabatic
(c)Decrease by 2.25j (d) increase by 9.0j condition
(e) None of the above
Solution 7. A certain mass of an ideal gas at pres-
w = +4.5j, M T = 2k, u =? sure p1 ,volume v1 and absolute tempera-
Q = u + w but Q=0 (adiabatic expan- ture T1 undergoes an adiabatic change to
sion) pressure p2 ,volume v2 and absolute tem-
0.31. PHY131 2011/2012 161

perature T2 .Choose the incorrect relation Solution


from the following (γ is the ratio of the a b
V = 8 − 3 = ar−8 − br−3
two specific heats of the gas ) r r
 γ  γ−1 dV
v1 v1 on differentiating = −8ar−9 + 3br−4
(a)P2 = P1 (b) T2 = T1 dr
v2 v2 dV −8a 3b
  γ−1 =− 9 + 4
P2 x dr r r
(c) T2 = T1 (d) P1 V1 = P2 V2 dV 8a 3b
P1 at = 0; 9 = 4
(e) P1 V1γ = P2 V2γ dr r ra 
5 8a
Solution r = = 2.667
3b   1 b
So if the gas undergoes Adiabatic change a 5
∴ Q=0,it has 2 equation P V γ = k & T V γ−1 = r = 1.22 (B)
b
k
by usingthe  first equation P1 V1γ = P2 V2γ 9. Which of the following statement is (are)
γ true
V1
P2 = P1 (i) in a covalent bond the shared elec-
V2
Using eqn 2 make T2 the subject of for- trons stay midway between the two atoms.
mula   (ii) Metallic bond is stronger than vander
γ−1
V1 waal’s bond
T2 = T1 (iii) All types of bonds are as a result of
V2
so also
 γ by comparing the 2 eqn electrostatic force .
V1 P2 (a) i and iii only (b) i only (c) ii and iii
= − − − −(1)
V2 P1 only (d) i and ii only (e) iii only
 γ−1
T2 V1 Solution
also = Ans D
T1 V2
 γ   γ−1 1
P2 V1 T2 10. An experimental result for extension of
∴ = =
P1 V2 T1 a thin wire for increasing loads are pre-
  1   γ−1 1 sented in the table below.And it is used
P2 γ T2
∴ = to plot a graph of extension against load.
P1 T1 Load:(W)kg 0 2 4 6 8 10
multiply by the power by γ − 1
Extension(cm) 0 0.12 0.24 0.36 0.48 0.60
 γ − 1   DIAGRAM
P2 γ T2
= The point A on the graph is called
1 T1
  γ−1 (a) Elastic limit (b) stress limit (c) pro-
T2 P2 γ
portional limit (d) strain limit (e) Exten-
∴ =
T1 P1 sion limit
  γ−1
P2 γ Solution
∴ T2 = T1 A: i.e proportional limit also point B is
P1
So A,B,C and E obeys adiabatic Ans D elastic limit ,point C is the breaking point

11. The curve BC on the graph in Q(11) rep-


8. The potential energy that exist between resent :
a b
two molecules is given by V = 8 − 3 (a) Elastic deformation (b) lattice defor-
r r mation (c) structural deformation (d) plas-
where a and b are constants and r is
the distance,The interatomic spacing in tic deformation (e) plane deformation
therefore Solution
 1 1  a  15 D: plastic deformation while AB = elas-
4.3a 4 a 5

(a) (b) 1.22 b (c) 0.5 tic deformation
b b
 15   15
12. Suppose a mass of 2kg is attached to the

0.5a b
(d) (e) 2.2 end of a suspended wire of length 2m
b a
162

and cross-sectional are 0.625 × 10−3 m2 .if


the ectension produced is 0.6mm,what is
the energy stored in the wire (take g= 16. A rectangular tank of sides 3m by 2m is
10ms−2 ) fikked with water to its brim.If the depth
(a)4.8j (b) 2.4j (c) 3.4j (d) 4.2j (e) 4.0j of the tank is 5m and a hole at its bot-
Solution tom is unplugged,how long will it take
the tank to be empty?Take acceleration
due to gravity to be 10m/s2
13. When Hook’s law is obeyed , an increase (a) 0.5s (b)1s (c) 3s (d) 2s (e) 5s
in pressure (M P ) on an object can pro- Solution
duce a proportional fractional change in 2
l = 3m,√ B = 2m.h√ − 5m, g = 10m/s√
volume (M V /V ) Which of the follow- v = 2gh = 2 × 10 × 5 = 100 =
ing expression correctly defined the bulk 10ms−1
v
modulus of the object Q = = AV
M pV V MV MP t
(a) (b) − (c) v
Mv M v/ M V v = AT
t
Mp Mp 3×2×5
(d) − (e) − = 3 × 2 × 10
M v/v p/ M v t
Solution 30
MP V MP t= = 0.5sec
Bulk modulus = − MV = − (D) 60
V
V
17. A liquid flows through two capillary tubes
14. A hydraulic press contains a certain vol- A and B connected in series.The length
ume of oil.When it is subjected to a pres- and radius B is twice those of A.Find the
sure increase M P = 1.6 × 107 N m−2 ,the ratio of the pressure difference across A
decrease in volume is found to be (−8.0× and across B
10−4 m3 ).Determine the original volume (a) 8:1 (b) 8:2 (c) 7:2 (d) 7:1 (e) 8:3
of the oil (Bulk modulus of the oil B = Solution
5.0 × 109 N m−2 ) πρr4
Q=
(a)2.50 m3 (b) 1.50m3 (c) 2.30 m3 (d) 8oL
0.25 m3 (e) 1.50m3 the capillaries are connected in series QA =
Solution QB
4 4
πPA rA πPB rB
M p = 1.7 × 107 , M v = −8 × 10−4 m3 , v = =
?, B = 5 × 109 N m−2 8 o lA 8 o lB
Length and radius of B is twice those of
−1.6 × 107 v
 
V MP
B=− =− A
V 8 × 10−4 πPA rA 4
πPB (2rA )4
1.6 × 107 v = then cross multi-
5 × 109 = 8 o lA 16 o lA
8 × 10−4 ply
3
v = 0.25m (D) 2pa ra4 = pb .16ra4
pa 8pb 8
15. A body having a low density and rela- = =
pb pb 1
tively large surface are for its weight will; pa : pb = 8 : 1
(a)Falls through a long distance before it
acquires its terminal velocity 18. A steel ball of radius 2mm falls under
(b) Accelerates through a short distance gravity in oil of coefficient of viscosity
before it acquires its terminal velocity 2.5N sm−2 ,if the density of the oil is 890Kgm−3
(c) Acquires a very high terminal veloc- and that of the steel ball is 8000Kgm−3 ,calculate
ity the terminal velocity attained by the ball
(d) acquires terminal velocity before ac- (a)0.0025m/s (b) 0.25m/s (c) 0.025m/s
celerating downward (d) 0.055m/s (e) 0.0055m/s
(e) Can never acquire terminal velocity Solution
Solution γ = 2mm = 2 × 10−3 m, o = 2.5N sm−2
0.31. PHY131 2011/2012 163

ρl = 890kgm−3 , ρs = 8 × 103 kgm−3 , Vt = 89.98


= 4.5ms−1 (D)
?g = 9.8m/s−2 20
2gr2 (ρs − ρl ) 21. The static pressure in a horizontal pipeline
Vt =
9o is 4.3 × 104 pa ,the total pressure is 5.2 ×
2 × 9.8 × (2 × 10−3 )2 (8000 − 890) 104 pa and the cross sectional area is 20 ×
Vt = =
9 × 2.5 10−4 m2 .Find the volume flow rate in the
0.557424
pipeline if the liquid is incompressible and
22.5
Vt = 0.025ms−1 (C) non-viscous .Take density of liquid to be
1700Kgm−3
19. Air streaming past a horizontal air plane (a) 4.3 ×10−2 m3 /s (b) 6.5×10−3 m3 /s (c)
wing such that its speed is 120ms−1 over 5.5×10−3 m3 /s (d) 0.9 ×10−3 m3 /s (e)3.3
−1
the upper surface and 90ms at the lower ×10−3 m3 /s
surface.If the density of air is 1.3kgm−3 .if Solution
the wing is 10m long and 2m wide,calculate P2 = 4.3 × 10−4 P a, Pt = 5.2 × 104 pa, A =
the gross lift of the wing 20 × 10−4 m2
(a) 8.4 × 10−4 N (b) 8.2 × 104 N (c) 8.4 × Q =?, ρ = 1700kgm−3
102 N (d) 8.4 × 10−2 N (e) 8.2 × 102 N Ptotal = Pstatic + Pdynamic
Solution Pdynamic = Pt − Ps = 5.2 × 104 − 4.3 ×
V1 = 120ms−1 , V2 = 90ms−1 , ρ = 1.3kgm−3 , l = 104 = 9000
1
10m, b = 2m But Pdynamic = ρv 2
1 2
Gross lift = ρA(V12 − V22 ) 1
2 9000 = × 1700v 2
A = L × b = 2 × 10 = 20m 2 2
−1
1 1 v = 3.25ms
f = × 1.3 × 20(1202 − 902 ) = × 26 × but Q=AV= 3.25 × 20 × 10−4 = 6.5 ×
2 2
6300 10−3 m3 /s(B)
4 4
= 8.19 × 10 = 8.2 × 10 N (B)
22. If the radius of the human artery is 1.5cm
20. The cylindrical tube of a spray pump has and the average systolic blood velocity
diameter of 3cm one end of the tube 20 0.08ms−1 determine the Reynolds num-
fine holes each of diameter 0.5mm.if the ber for the flow pattern of the blood its
speed of flow of the liquid in the tube is coefficient of blood is 2.5 × 10−2 N sm−2
−1
2.5cms ,calculate the speed of ejection and density id 11000Kgm−3
of the liquid through the fine holes (a) 96 (b) 9600 (c)990 (d)9600 (e) 960
(a) 1.5m/s (b) 2.5m/s (c) 3.5m/s (d)4.5m/s Solution
(e) 5.5m/s r = 1.5 × 10−2 m, v = 0.08ms−1 NR =
Solution ?, o = 2.5 × 10−2 N sm−2 , ρ = 103 kgm−3
−2 −2
M1 = 3cm = 3 × 10 m, r1 = 1.5 × 10 m V ρD 0.8 × 103 × 2 × 1.5 × 10−2
−1
v1 = 2.5cms ≈= 2.5 × 10 ms −2 −1 N R = = =
o 2.5 × 10−2
M2 = 0.5mm = 0.5 × 10−3 m 96
M −3
∴ r2 = = 0.25 × 10 m, v2 =?
2 23. A body radiates heat energy per second
but A1 V1 = A2 V2 of 5.083js−1 at 7220 C .if its surface area
2 22 −2 −2
A1 = πr1 = (1.5 × 10 ) = 7.069 × is 4.8cm2 ,what is its emissivity ? Ste-
7
10−9 m2 fans constants = 5.67 × 10−8 W m−2 K −4
22 (a) 0.019 (b) 0.203 (c) 19.0 (d)0.19 (e)1.90
A2 = (0.25 × 10−3 )2 = 1.96 × 10−7 m2
7 Solution
A1 V1 = A2 V2
M E = 5.083Js−1 , T = 7220 C = 995K.A =
7.069×10−4 ×2.5×10−2 = 1.964×10−7 v2
7.069 × 10−4 × 2.5 × 10−2 4.8cm2 = 4.8 × 10−4 m2
−1
v2 = = 89.93ms e =?, σ = 5.67 × 10−8 wm−2 k −4
1.964 × 10−7 ME 5.083
v2 e= = =
but speed through 20 holes will be = AσT 4 4.8 × 10−4 × 5.67 × 10−8 × (995)4
20
164

5.083 pressure of 8.20 × 104 Nm−2 at the tem-


≈= 0.19(D)
26.68 perature of melting ice,1.0 × 105 N m−2
24. Which of the following statement is cor- at the temperature of steam and 8.50 ×
rect about a perfectly black body? 104 N m−2 at temperature of boiling point
(a) Reflects completely the radiations of of calcuim.Calculate the temperature of
all wavelength incident on it. calcuim .
(b) Have absorbing power greater than (a) 160 C (b) 170 C (c) 180 C (d)150 C (e)
unity 140 C
(c) it reflects and transmits radiation Solution
(d)it does have an ideal surface with which P0 = 8.2×104 N m−2 , P100 = 1×105 N m−2 , Pθ =
we can compare teal surfaces 8.5 × 104 N m−2
pθ − p 0
(e) Absorbs completely the radiation of θ= × 1000 C
p100 − p0
all wavelength incident on it 8.5 × 104 − 8.2 × 104
Solution θ= × 100
1 × 105 − 8.2 × 104
E; it has absorbing power of 1 (unity) 3000 300
θ= × 100× = ≈ 170 C(B)
18000 18
25. The density ρ of a liquid is 1930Kgm−3
at 200 C and its coefficient of linear ex- 28. How many joules are required to raise the
pansivity is 1.4 × 10−4 K −1 .Calculate the temperature of 500g of copper from 160 C
density at 800 C to 1160 C.Take the specific capacity to be
(a) 1873Kgm−3 (b) 1783Kgm−3 (c) 2113Kgm−3 400j/kg.K
(d) 1883Kgm−3 (e) 1623Kgm−3 (a) 20kj (b) 2kj (c) 200kj (d) 2000kj (e)
Solution 20000kj
ρ1 = 1930kgm−3 , θ1 = 200 C, γ = 1.4 × Solution
10−4 k −1 , ρ2 =?, θ2 = 800 C H=? M = 500g = 0.5kg, T1 = 160 C, T2 =
(γ2 − γ1 ) 1160 C, C = 400jkg −1 k −1
γ=− H = mc M θ = 0.5 × 400 × (116 − 16) =
ρ2 M θ
(ρ2 − 1930) 20kj(A)
3(1.4 × 10−4 ) = −
ρ(80 − 20)
0.0252ρ + ρ = 1930 29. In a particular gasoline stove,30% of en-
1930 ergy release in burning the fuel actually
ρ2 = = 1882.55kgm−3 goes to heating the water in the pot on
1.0252
the stove.if we heat 1L (1kg) of water
26. The absolute zero on kelvin and rankin from 200 C to 1000 C and boil 0.35Kg of
scale are ok and 00 R respectively ,what it away,how many gasoline do we burn in
will be the corresponding value on celsius the process (Given the Cw = 4190j/kg.k
and Ferhenheit scale repectively and Lv = 2.256 × 106 j/kg)
(a) −273.160 C, −459.670 F (b) −273.150 C, −491.67 0
F
(a) 62g (b)63g (c)65g (d)66g (e)64g
(c)−273.150 C, −459.670 F
(d) −273.160 C, −491.670 F (e)−459.670 C, −273.15 beaker contains 500g of water at 200 C
30. A0 F
Solution and is heated steadily.if takes 80s to reach
0
Tk = C + 273.15 boiling of 1000 C ,how long will it then
0k =0 C + 273.15 take for all the water to biol away?(Latent
0
∴ C = 0 − 273 − 15 = −273.15 C 0 heat of vaporization Lv = 2.268×106 j/kg,specific
0
F − 32 −273 heat capacity of water Cw = 4200j/kg.k)
=
9 5 (a)50min (b)51min (c)54min (d) 52min
50 F − 160 = −2458.32 (e)53min
0
F = −459.670 f Solution
0
k = −273.150 C = −459.670 f (C) mc M θ
Rate of heat generated =
N.B 273.16 IS tripple point temperature t
mw = 0.5kg, θ1 = 200 C, t1 = 480s, θ2 =
27. A constant Volume gas thermometer is at 1000 C
0.32. PHYS 2013/2014 165

Lv = 2.268 × 106 , Cw = 4200jkg −1 k −1 dv 9a b


= − 10 + 2
0.5 × 4200 × (100 − 20) dr r r
Rate = = 350j/s dv
4800 recall that F= −
Heat to boil away H = mlv dr
= 0.5 × 2.268 × 106 = 1.134 × 106 9a b
F = 10 − 2
1.134 × 106 r r
rate of heat =
t 6. At 270 C a gas occupies a certain volume
1.134] × 106
350 = say v
t
1.134 × 106 At T2 the volume will be 3V.
t= = 3240sec = 54min(C) v1
350 At constant pressure i.e charles eqn =
T1
V2
0.32 PHYS 2013/2014 T2
V 3V
=
1. Given that ; re = 2.8 × 10−15 m, ρ =? 27 + 273 T2
Assuming the shape of an electron to be T2 = 3 × 300 = 900K
0
spherical i.e its volume C = k − 273 = 900 − 273 = 6270 C
4
Ve = πre3 7. v = 2 × 10−2 m3 , T = 400K
3
me me M P = (2.5 × 105 − 1.3 × 105 )pa = (2.5 −
We know that ρ = = 4 3
ve 3
πre 1.3)105 pa
3me 3 × 9.11 × 10−31 = 1.2 × 105 pa
= = =
4πre3 4 × 3.142 × (2.8 × 10−15 )3 Mo2 = 32g = 32 × 10−3 kg using ideal gas
9.9 × 1012 kgm−3 equation
m
P V = nRT But n =
2. Given that proton number = 19 mm
Newton number =20 mRT
PV =
We know that atomic number =no of pro- M
pvM
tons m=
RT
∴ Atomic number=19 Where R is universal gas constant
mass no= atomic no + neutron number 1.2 × 105 × 2 × 10−2 × 32 × 10−3
A=Z +n Mo2 = =
8.314 × 400
A + 19 + 20 = 39 0.023kg
A 39
Z X =19
∴ the atom is potassium 39 8. p = 1.65 × 104 P a, v1 = 0.20m3 expands
19 X
to v2 = 0.40m3
3. Ans D : because they do not form ions M Q or M H = 1000J
when dissolve in solution,hence they ere M v = v2 − v1 = (0.40 − 0.20) = 0.20m3
bad conductors of heat and electricity Recall that M Q =M u + W
M u =M Q − p M v = 1000 − (1.65 × 104 ×
4. Given that r = 10−20 m, f =? 0.2)
Charge of proton = +1.6 × 10−19 C will = −2.3 × 103 j
be given.the same as charge of electron
only the sin differ Q1 = Q2 = Q 9. W= 820 j,we know that during adiabatic
kQ2 expansion M Q = 0
F = 2
r M T = 270 C, M U =?
1
where k= = 9.0 × 109 N m2 C −2 Recall M Q =M U + W
4π0 M U = −W = −820J
9.0 × 109 × (1.6 × 10−19 )2
F =
(10−20 )2
12
= 2.3 × 10 N 10. Less than proportional limit
a b (i) the bumper will have sprung back to
5. Given that V = 6
− , f =? its original shape since it did not pass its
r r
166

elastic point regardless of proportional 19. A1 = 3m2 , V1 = 2ms−1 , A2 =?, v2 = 10ms−1


limit Using the formula A1 V1 = A2 V2
(ii) sice it has touched the post therefore A1 V1 3×2
A2 = = = 0.6m2
the bumper paint may be scratched. V2 10
(iii) It will not have a permanent dent
20. r=2m ,ρs = 10.5kgm−3 , o = 9.8pa; ρl =
Ans (i)and (ii) only (B)
1.5kgm−3
2gr2 (ρs − ρl )
11. At the fracture point Using the relation VT =
9o
(i) The bunper will never come back to 2 × 9.8 × 22 (10.5 − 1.5)
its original shape = = 8ms−1
9 × 9.8
(ii) The bumper will have of course a per-
manent deformation . 21. Laminar flow of liquid is not character-
(iii) It must be turn or broken at the case ized by high velocities and high Reynold
may be number
(ii) and (iii) only (E) Ans E

Energy 1
Fe 22. V1 = 2ms−1 , p1 = 4 × 104 pa, p2 =?, v2 =
12. = 2 4ms−1
volume v
stress for horizontal pipe Bernouli’s eqn becomes
recall that Young modulus = 1 1
strain P1 + ρV12 = P2 + ρV22
F
F L 2 2
i.e E = Ae = 1 2 2 1
Ae P2 = ρ(V1 − V2 ) + P1 = × 1000 ×
l 2 2
EAe (22 − 42 ) + 4 × 104
F = = 4 × 104 − 6 × 103 = 3.4 × 104 pa
l
1 EAe
1
F e e 23. Ans E
So 2 = 2 l
v v
EAe2 1 e 1 24. Ans B
= = (E × ( )2 ) = (E × ε2 )
2l.Al 2 l 2 25. d1 = 0.2m, d2 = 0.1m, v1 = 2ms−1 , v2 =?
13. Weight = Force = 20N,extension ,e= 1mm= A1 V1 = A2 V2
1 × 10 m−3 πd21 v1 πd2 v2
= 2
1 1 4 4
E = F e = × 20 × 10−3 = 0.01j d 2
v1 = d 2
v2
2 2 1 2
 2
d1 0.2 2
F 5390 v2 = v1 = 2 × ( )
14. stress =? ,stress = = = 1.8×10 8 d 2 01
A 0.3 v2 = 8ms−1
stress 26. A constant volume gas thermometer uses
15. E =
strain the properties of pressure change eith tem-
stress 1.8 × 108 perature of a gas at constant volume
Strain = = = 9.0 ×
E 20 × 1010
10−4 Tθ − T0
27. Using θT = × 100
T100 − T0
16. The bulk modulus of a gas on - a pressure 1.64 − 1.5
θR = × 100 = 350 C
i.e initial pressure . 1.9 − 1.5
Ans A
28. Given that Eθ = Eθ (1 + aθ + 0.01aθ2 )
17. Ans E E100 = E0 (1 + 100A + 10−2 A.1002 )
E100 = E0 (1 + 200a)
18. The formula representing Stroke’s law is E0 = E0 (1 + 0.A + 10−2 A.02 ) = E0
F = 6π o rv (A) E10 = E0 (1 + 10a + 10−2 .a.102 )
0.32. PHYS 2013/2014 167

E10 = E0 (1 + 11a) 35. Given that ; e = (1 − 0.99967)cm, αs =


E10 − E0 1.1 × 10−5 0 C −1
∴ θ= × 100
E100 − E0 θ1 = 300 C, θ2 =?
E0 (1 + 11a) − E0 e
= × 100 αs =
E0 (1 + 200a) − E0 l0 M θ
e
11 Mθ=
θ= θ100 = 5.50 C lo αs
200 e
θ2 = + θ1
l0 αs
29. Given that 1 litre =2kg then 6 litres = (1 − 0.99967)
= + 30 = 600 C
2 × 6 = 12kg 0.99967 × 1.1 × 10−5
and 30 litres = 2 × 30 = 60kg
36. Given that θ1 = 00 C, ρ0 = 2.7×103 kg/m3 ,
Heat lost = heat gain
ρt = 2.695 × 103 , θ2 = 600
mh Ch M θh = mc Cc M θc ρ0 − ρt
12×Ch ×(100−θf ) = 560×Cc ×(θf −40) γ=
ρ0 M θ
100 − θf = 5θf − 200 2.7 × 103 − 2.695 × 103
300 =
θf = = 500 C 2.7 × 103 × (60 − 0)
6 103 (2.7 − 2.695)
30. Given that Cw = 10Cc , mc =?, θc = 500 C, mw = = = 3.1 × 10−5 k −1
2.7 × 103 × 60
100g = 0.1kg, θw = 100 C, θf = 120 C
Assuming no energy is lost to the sur- 37. Given that vg = 1 litres = 1000cm3
rounding αg = 9 × 10−6 0 C −1 , γm = 1.8 × 10−4 0
Heat lost by copper = heat gain by water C −1 , γg = 3 × 6 × 10−6 = 2.7 × 10−5 0
mc Cc M θc = mw Cw M θw C −1
Mv
mc Cc (50 − 12) = 0.1 × 10Cc (12 − 10) using the relation γ =
vMθ
0.1 × (12 − 10) × 10 Mv
mc = = γv
(50 − 12) Mθ
γg vg = γm vm
mc = 5.26 × 10−2 kg = 52.6g 2.7 × 10−5 × 1000 = 1.8 × 10−4 × vm
31. Given that C = 450j/kg 0 C, h = 30.6m, M 2.7 × 10−5 × 1000
vm = −4
= 150cm3
θ =? 1.8 × 10
mc M θ = mgh Q KA M T
gh 9.8 × 30.5 38. Considering the relation =
Mθ= = = 0.660 C t l
c 450 Ans E
32. The Zeroth law of thermodynamic can be 39. Given that γHg = 18 × 10−5 0 C −1
stated as Two bodies are in thermal equi- volume of bulb = 10−6 m3 ,cross sectional
librium separately with third body,then area of the steam= 0.002cm2 = 0.002 ×
the two bodies are also in thermal equi- 10−4 m2
librium with one another θ1 = 00 C, θ2 = 1000 C
Ans A M θ = 100 − 0 = 1000 C
Mv
33. Given that ; θ1 = 00 C, θ2 = 1000 C Using the relation γ =
vMθ
M θ = 100 − 0 = 100k M v = γv M θ = 18 × 10−5 × 10−6 × 100
0.05
e = 0.05% = l0 α =? = 1.8 × 10−8 m3 we know that M v = Ah
100 Mv 1.8 × 10−8
e h= = = 9cm(B)
α= A 0.002 × 10−4
l0 M θ
0.05 Q KA(θ1 − θ2 )
× l0 40. =
α = 100 t l
l0 × 100 Ql
= 5.0 × 10−6 0 C −1 k=
A(θ1 − θ2 )t
lt − l0 M L2 T −2 × L
34. α = Ans E = 2 = M LT −3 K −1
l0 M t L ×K ×T
168

0.33 CHEM 161 2010/2011 9. What is the oxidation number of chromium


in CrO42−
1. Which of the following statement is cor- (a) +5 (b) +7 (c) +6 (d) 5
rect ?
(a) The end point in titration is always 10. A solution of sodium trioxocarbonate(iv)
determined by change in indicator colour contains 10.6g in 250cm3 of solution ,cal-
(b) Experimentally determined stoichio- culate the concentration of the solution
metric point of reaction is known as equiv- (a) 0.4moldm−3 (b) 1.0moldm−3
alence point . (c) 10.6moldm−3 (d) 25.0moldm−3
(c) A standard solution contains 1moldm−3
of solute 11. Which of the following apparatus can be
(d) End point is experimentally deter- used to measure accurate a specific vol-
mined ume of a liquid
(a) Beaker (b) conical flask
2. How many moles of AgN O3 are there in (c) measuring cylinder (d) pipette
500cm3 of 0.01M AgN O3 solution .
(a) 0.005 mol (b) 0.05mol 12. If a solution has a pH of 2,it can be con-
(c) 0.50 mol (d) 1 mol cluded that it :
3. Which of the following is not a type of (a) It is a weak electrolyte
titrimetric analysis (b) it has hydrogen ion concentration of
(a) Acid-base (b) iodometric 0.2 moldm3
(c) complexometric (d)Redox (c) it is twice as acidic as a solution of
pH 1
4. Which of the following is not a type of (d) will produce effervescence with mag-
quantitative analysis nesium ribbon
(a) Replacement analysis
(b) Instrumentals 13. If 10cm3 of distilled water is added to
(c) Gravimetric analysis 10cm3 of an aqueous salt solution,the con-
(d) volumetric analysis centration of the solution
(a) increases (b) decreases
5. What volume of distilled water should be (c) remains constant (d) doubles
added to 400cm3 of 2.0moldm−3 H2 SO4
to obtain 0.20molm−3 of solution ? 14. Calculate the mass of ZnSO4 Produced
(a) 600cm3 (b) 800cm3 when excess of ZnCO3 is added to 50cm3
(c) 1000cm3 (d) 3600cm3 of 4.0moldm−3 H2 SO4 .The equation for
the reaction is :
6. Which of the following is not a way of
ZnCO3 + H2 SO4 = ZnSO4 + CO2 +
representing concentration
H2 O(ZnSO4 = 161gmol−1 )
(a) Normality (b) weight percent
(a) 0.200g (b) 1.29g (c) 32.2g (d) 39.0g
(c) mass percent (d) volume percent

7. What is the percentage by mass of sul- 15. Which of the following acid form normal
phur in Al2 SO4 with salts only
(a) 9.36 % (b) 14.71% (a) Tetraoxosulphate (vi) acid
(c) 21.33% (d) 28.07% (b) trioxosulphate (iv) acid
(c) tetraoxosulphate(v) acid
8. What volume of hydrogen is produced at (d) trioxonitrate (v) acid
S.T.P when 2.06g of zinc reacts with ex-
cess HCl according to the following reac- 16. Calculate the mass of sodium hydroxide
tion Zn + 2HCl = ZnCl2 + H2 in 5.0dm3 of a 0.125 moldm−3 solution
(a) 0.040dm3 (b) 0.896dm3 [NaOH =40gmol−1 ]
(c) 5.82dm3 (d) 8.62dm3 (a)0.015g (b) 0.625 (c) 1.00g (d) 25
0.33. CHEM 161 2010/2011 169

17. A solution of Sodium hydroxide contain- 25. The color phenophthalein indicate in di-
ing 6.0g in 250 cm3 of solution has a con- lute Hcl is
centration of (a) colorless (b) orange (c) pink (d) pur-
(a) 0.04M (b) 0.60M (c) 0.96M (d) 0.015 ple

18. What process is involved in the reaction 26. Consider the reaction represented by this
represented by the equation : equation
Alcl3 + 3H2 O = Al(OH)3 + 3Hcl −−−−→
N aHCO3 (s)HEAT N aCO3 +CO2 +H2 O
(a) Dehydration (b) hydrolysis What is the mass of soduim hydrogen tri-
(c) double decomposition oxocarbonate (iv) [N aHCO3 = 84; N aCO3 =
(d) neutralization 106]
19. If 20cm3 of distilled water is added to (A)10.6 (B) 16.8g (c) 21.2g (d) 42.4g
80cm3 of 0.50moldm−3 hydrochloric acid,the
27. What volume of 0.20 moldm−3 solution
new concentration of acid will be
would yield 5.00g of NaOH on evepora-
(a) 0.10 moldm−3 (b) 0.20 moldm−3
tion to dryness [N aOh = 40]
(c) 0.40 moldm−3 (d) 2.0moldm−3
(a) 400 cm3 (b) 625cm3
20. Consider the reaction represented by the (c) 1000cm3 (d) 1600cm3
equation
N aHCO3 (s) → N aCO3 + CO2 + H2 O 28. In the titration of KMnO4 against thio-
What volume of carbon(iv) oxide at S.T.P sulphate the indicator used is
is evolved when 0.50 moles of NaHCO3 is (a) phenolphthalien (b) methyl orang
heated [Molar volume of gas = 22.4dm3 ] (c) litmus (d) none of the above
(a) 1.12dm3 (b) 2.24 dm3 (c) 5.6 dm3 (d)
56.0 dm3 consider the reaction between KM nO4
and H2 C2 O4 (oxalic acid) and answer the
21. 14.8g of 0.80 moldm−3 hydrochloric acid question that follow (28- 32)
3
neutralized 20cm of distilled water gives
concentration 0.080molm−3 .calculate the 29. What type of reaction is this
molar mass of the salt (Z) (a) Acid-base (b) redox
(a) 13.5 gmol−1 (b) 18.5 gmol−1 (c) oxidative (d) reductive
−1 −1
(c) 47.4 gmol (d) 74.0 gmol
30. The stiochiometry of the reaction in in
22. 25 cm3 of 0.80 moldm−3 hydrochloric acid the ratio
3
neutralized 20cm of sodium hydroxide (a)2:4 (b) 3:5 (c) 2:5 (d) 4:5
solution.What is the concentration of sodium
hydroxide in moldm−3 31. What is the oxidation number of iodine
N aOH + Hcl → N acl + H2 O in IO3−
(a) 0.08 (b) 0.10 (c) 0.80 (d) 1.00 (a) +5 (b) +3 (c) +4 (d) +2

23. When concentrated H2 SO4 is added to 32. What number of moles of tetraoxosul-
N acl(s) ,the gas evolved phate(vi)acid would you require
(a) Bleaches damp blue litmus paper (a)2 (b) 3 (c) 4 (d) 5
(b)forms a white precitate with AgN O3
(c) form a white precipitate with Bacl2 33. What is the oxidation number of carbon
(d) turn moist red litmus paper blue in C2 O42−
(a) +3 (b) +4 (c) +1 (d) +5
24. what volume of 0.25moldm−3 KOH Would
be required to completely neutralize 40cm3 of 34. Which of the following is a common re-
0.10moldm−3 of Hcl ducing agents
(a) 40 cm3 (b) 32cm3 (a) HN O3 (b) H2 SO4 (c) KM nO4 (d)
(c) 24 cm3 (d) 16 cm3 F eSO4
170

35. In iodometry,the type of indicator that is 5. CA VA = CB VB


normally used is : 400cm3 × 2.0M = 0.20M × VB
(a) phenolphthalein (b) starch solution 400 × 2
VB = = 4000cm3
(c) methyl orang (d) methyl red 0.02
The volume to be added = (4000−400) =
36. In the standardization of CuSO4 .5H2 O 3600cm3 (D)
using standard thiosulphate,the CuSO4
reacts : 6. C
(a) Directly with the sulphur in thiosul-
7. Al2 SO4
phate
Total mass = (27 × 2) + 32 + (4 × 16) =
(b) Reacts first with the iodide
150g
(c) Displaces Na from solution
S= 32
(d) Reacts first with KM nO4 32
Weight percent of S= ×100% = 21.33%(C)
150
37. When Cu reacts with iodide,the equation
of the reaction can be represented as 8. Zn + 2Hcl → Zncl2 + H2 O
(a) 2Cu2+ + 4I − → Cu2 I2 + I2 Mass of Zn = 2.06g,Zn= 64g/mol
(b)Cu1− + 4I − → CuI2 + 3I − 2.06
no of moles= = 0.0322mol
(c) 2Cu2+ + 4I − → 2CuI2 64
(d) none of the above
9. CrO42−
38. In the reaction between iodine (I2 ) and
x + 4(−2) = −2
thiosulphate S2 O32− Changes from 2 to
x = +6(C)
(a) 3 (b)4 (c) 2.5 (d) 5

39. Which of the following is arrange in or- 10. Na2 CO3


der of increasing oxidation number of the Molar mass = 23 × 2 + 12 + 3 × 16 =
non-metal 106g/mol
10.6
(a) CO2 < CO32− < C2 O62− no of moles = 106g/mol = 0.1mol
(b) CO2 < C2 O42− < CO32− 0.1 moles of N a2 CO3 is present in 250
(c)S2 O32− < S4 O62− < SO32− cm3
(d) none of the above x mol of Na2 CO3 will be present in 1000cm3
x = 0.1×1000
250
= 0.4moldm−3 (A)
40. 30cm3 of distilled water was added to a
20cm3 solution containing 0.8mols Hcl,what 11. C
is the number of moles of Hcl present in
12. B
the new solution
(a) 0.5 mols (b) 0.6cm3 (d) 0.8 mol (d) 13. B
1.2
14. equation of reaction
0.33.1 ANSWER TO CHEM 161 ZnCO3 +H2 SO4 → ZnSO4 +CO2 +H2 O
4mol of H2 SO4 is present in 1000cm3
2010/2011
x mol of H2 SO4 WILL BE PRESENT
1. A IN 50cm3
x = 0.2mol
2. 0.01 mols of AgN O3 is present in 1000cm3 from equation of reaction
x mols of AgN O3 WILL BE PRESENT 1 mol of H2 SO4 Produced 1mol of ZnSO4
IN 500CM 3 0.2 mol of H2 SO4 will produce x mol
3
X = 0.01mols×500cm
1000cm3 = 0.005mols(A) x= 0.2 mol of ZnSO4
mass
3. B no of moles = molar mass
Molar mass = 161gmol−1
4. A mass= 0.2 × 161= 32.2g(C)
0.33. CHEM 161 2010/2011 171

15. A 24. Hcl + KOH → KCL + H2 O


0.25mols of KOH is present in 1000cm3
16. 0.125mol of NaOH is present in 1000cm3 0.10 mols of Hcl is present in 1000cm3
x mol of NaOH will be present in 5000cm3 x mol of Hcl will be present in 40cm3
5000 × 0.125mol
x= = 0.625mol x = 0.10×40
1000
= 0.004moles
1000
mass = no pf moles × molar mass from the equation of reaction
= 0.625 × 40= 25g(D) 1 mol of Hcl neutralizes 1 mol of KOH
0.004 moles of Hcl will neutralizes x mol
mass 6
17. no of mole = molar mass
= 40 = 0.15moles of KOH
This implies that x= 0.004 moles
0.15 moles of N aOH is present in 250cm3 Therefore
x mol of NaOH will be present in 1000cm3 0.25 moles of KOH is present in 1000cm3
x = 0.6M (B) 0.004 moles of KOH will be in x cm3
x= 16.00 cm3 (D)
18. B
25. A
19. CA VA = CB VB
80cm3 × 0.50moldm−3 = 100cm3 × CB 26. 2N aHCO3 → N a2 CO3 + CO2 + H2 O
CB = 80×0.50
100
= 0.40M (C) Mass of NaHCO3 = 33.6g
20. 2N aHCO3 → N aCO3 + CO2 + H2 O Molar mass of N aHCO3 = 84g/mol
mass
From equation of reaction no moles = molar mass
= 33.6
84
= 0.4
2 moles of N aHCO3 Produced 1 mol of from equation of reaction
CO2 2 moles of NaHCO3 produce 1 mole of
0.5Mols of N aHCO3 will produce x N aCO3
x = 0.5×1mol = 0.25mol 0.4 mol of N aCO3 will produce x
2
1 mol of any gas at STP Occupies 22.4 x= 0.2 mols of N aCO3
dm3 Molar mass of N aCO3 = 106gmol−1
This implies that mass= 0.2 ×106 = 21.2g(C)
1 mol of CO2 at STP occupies 22.4 dm3 27. Molar mass of N aOH = 40g/mol
0.25mols of CO2 will occupy x mass = 5.00g
x = 0.25×22.4
1mol
= 5.6dm3 (C) no of moles = 405
= 0.125moles
21. 0.8 mol of Z is present in 1000cm3 0.20 mol is present in 1000cm3
x mol of Z will be present in 250cm3 0.125mol will be present in x
x= 0.2mol x = 625cm3 (B)
mass
no of mole = molar mass 28. D
Molar mass= 0.2mol = 74gmol−1 (D)
14.8

29. B
22. NaOH + Hcl → N acl + H2 O
0.80mol of Hcl is present in 1000cm3 30. C
x mol of Hcl will be present in 250cm3
x = 0.02mols of Hcl 31. IO3−
From equation of reaction x + 3(−2) = −1
1 mol of Hcl reacted with 1mol of NaOH x − 6 = −1
0.02MOL of Hcl will react with mol x = +5(A)
x= 0.02 mol of NaOH 32. C
This implies that
0.02 mol of NaOH is present in 20cm3 33. C2 O42−
c mol of NaOH will be present in 1000cm3 2(x) + 4(−2) = −2
x = 1moldm−3 (D) x = +3(A)

23. D 34. B
172

35. B 4. What is the amount,in moles,of the iron(II)


that reacted?
36. B (A)4.47 × 10−3 (B)1.79 × 10−4
37. A (C)4.21 × 10−3 (D)8.43 × 10−4

38. C 5. The molar concentration of F e2+ is:


(A)0.447 moldm−3 (B)0.0179 moldm−3
39. C
(C)0.421 moldm−3 (D)0.0843 moldm−3
40. 0.8 mols of Hcl is present in 20cm3
x mols of Hcl will be present in 30cm3 6. Which of the following is NOT a unit of
x = 1.2mols(D) concentration?
(A)Mole fraction (B)Normality
(C)Molality (D)Molarity
0.34 CHEM 161 2011/2012
7. The most common indicator for the titra-
Carefully study the information below and an- tion of potassium permanganate and ox-
swer question 1-5.A 100 level student of Metal- alic acid is:
lurgical Engineering used 0.0893 moldm−3 so- (A)Starch indicator (B)Methyl orange
lution of KM nO4 to standardize a sample of (C)Phenolphthalein (D)None of the above
iron(II) as follows:10cm3 of the unknown iron(II)
solution transferred into a conical flask using 8. Why is potassium thiocyanate added to
a pipette.The solution was diluted to about the titrating solution in the determina-
80cm3 with distilled water and 20cm3 of 2M tion of the percentage purity of an im-
H2 SO4 was added.The cold solution of iron(II) pure sample of CuSO4 solution using stan-
was titrated with the standard solution until dard N aS2 O3 ?
the desirable colour change was observed.9.20cm3 (A)To enhance the reduction of Cu(II)
of the standard solution was used in the titra- (B)To enhance the liberation of iodine
tion.Following is the balanced equation of the
reaction: (C)To avoid the disturbance to the de-
M nO4− + 5F e2+ + 8H + −→ M n2+ + 5F e3+ + tection of the end point
4H2 O (D)To enhance the efficiency of the indi-
cator
1. An indicator was not used in the above
expirement because: 9. Which of the following utilizes indirect
(A)Iodine was not liberated titration in the determination of the un-
known?
(B)KM nO4 is self-indicating (A)Determination of the amount of as-
(C)KM nO4 is a strong oxidizing agent pirin in aspirin tablet using standard NaOH
(D)The reaction is slow in nature (B)Standardization of Fe(II) using stan-
dard KM nO4
2. What colour indicates the endpoint of
(C)Standardization of N a2 S2 O3 using stan-
the above reaction?
dard KM nO4
(A)Permanent yellow (B)Colourless
(C)Permanent faint pink (D)Standardization of NaOH solution us-
(D)Permanent purple ing HCl
3 −3
3. What is the amount,in moles,of the M nO4− 10. 15cm of 0.0782moldm hydrochloric acid
that reacted? neutralized 23cm3 of sodium hydroxide
solution.What is the molar concentration
(A)8.34 × 10−4 (B)4.45 × 10−3 of sodium hydroxide?
−4 −4
(C)1.69 × 10 (D)8.93 × 10 (A)0.051moles (B)0.0051moldm−3
0.34. CHEM 161 2011/2012 173

(C)0.051moldm−3 (D)0.051gmol−1 (C)0.0942moldm−3 (D)0.924moldm−3


11. Which of the following statement does
not appropriately describe oxidation-reduction Use the given information to answer ques-
reactions? tions 16 and 17.The approximate concen-
(A)They sometimes require elevated tem- tration of hydrochloric acid,HCl,in the
peratures stomach (stomach acid) is 0.17M.Calculate
the mass of the following antacid required
(B)They are very fast and rigorous
to neutralize 50cm3 of this acid:
(C)They require the addition of a cata-
lyst 16. Aluminium hydroxide,Al(OH)3
(A)0.0714g (B)0.714g
(D)They involve change in oxidation state
of the titrant (C)0.221g (D)0.0221g

12. In the standardization of CuSO4 .5H2 O 17. Bicarbonate of soda,N aHCO3


using standard thiosulphate,the CuSO4 : (A)0.0714g (B)0.714g (C)0.221g (D)0.0221g
(A)Displaces Na from the solution
18. The end point of the reaction between
(B)Reacts first with iodine
aspirin and NaOH is when a permanent
(C)Reacts directly with the thiosulphate pink colour is attained.After standing for
(D)Reacts with the iodide several minutes,the pink colour disappears.This
is due to
13. Which of the following statements about (A)Reverse neutralization reaction
phenolphthalein indicator is absolutely cor-
rect? (B)Hydrolysis of the acetyl group
(A)It is red in alkali,colourless in acid (C)Polarization of the acetyl group
and colourless in neutral solutions
(D)Esterification of the acetyl group
(B)It is pink in alkali,colourless in acid
and faint pink in neutral solutions 19. Which of the following best describes mo-
(C)It is faint pink in alkali,pink in acid lality?
and colourless in neutral solutions (A)Known amount of solute in a given
(D)It is pink in alkali,colourless in acid volume of solvent
and colourless in neutral solutions (B)Amount of solute in 1000g of solution
14. A 0.58g of potassium hydrogen phtha- (C)Amount of solute in 100g of solution
late [KH(C8 H4 O4 )] required 20.22cm3 of (D)Amount of solute in 1000g of solvent
NaOH for complete neutralisation.What
is the molarity of NaOH? 20. The REDOX reaction between KM nO4
(A)0.00284moldm−3 (B)0.141moldm−3 and N a2 C2 O4 is a very slow reaction.Therefore,all
(C)0.0871moldm−3 (D)0.00437moldm−3 but one of the following is true:
(A)The reduction process takes plays sev-
15. A 0.19g sample pure K2 Cr2 O7 was dis- eral minutes after the oxidation
solved,acidified and treated with an ex-
cess of KI.The liberated iodine required (B)Heat can be used to hasten the reac-
3
41.15cm of the thiosulphate solution.What tion
is the molarity of the thiosulphate solu- (C)The end point of the reaction is when
tion? a persistent pink colour is obtained
Hint:K2 Cr2 O7 +6KI+7H2 SO4 −→ Cr2 (SO4 )3 +
(D)No external indicator is needed
4K2 SO4 + 7H2 O + 3I2
2N a2 S2 O3 + I2 −→ 2N aI + N a2 S4 O6 21. In titrimetric analysis,the other name for
(A)0.00942moldm−3 (B)0.942moldm−3 the titrant is:
174

(A)Primary standard (B)Unknown solu- (D)Normal solution


tion
29. Calculate the amount in mole of a solute
(C)Standard solution (D)Basic solution in 250cm3 solution of 3.0moldm−3
22. A bottle of dilute HCl is 2M.What mass,in (A)0.012 (B)0.45 (C)0.75 (D)0.25
grams,of HCl would be dissolved in 100cm3 30. During a chemistry practical session,a stu-
of this acid? dent fainted suddenly.What is your ad-
(A)0.2 (B)7.3 (C)3.7 (D)6.5 vice?
23. Which of the following is not an applica- (A)Pour cold water over the student and
tion of acid-base titration? tickle the back of his throat to enhance
recovery
(A)Standardization of a solution of an
acid or base (B)Raise the patient’s head to normalize
blood circulation then shake gently
(B)Determination of the relative molec-
ular mass of an acid or base (C)Administer excess amount of sodium
bicarbonate till the student recovers
(C)Determination of the degree of tem-
porary hardness of a water sample (D)Lower the student’s head,raise the feet
and pass ammonia vapour beneath the
(D)Determination of the enthalpy of neu-
nostrils
tralization
31. During titration,the function of an indi-
24. If 19.33cm3 of 0.0892M of NaOH neutral-
cator is to detect the
ized a solution of one aspirin tablet,what
(A)Colour of the solution being titrated
is the amount of acetyl salicylic acid(C9 H8 O4 )
in the tablet? (B)End point
(A)317mg (B)300mg (C)312mg (D)310mg (C)Equivalence point
25. The experimentally determined position (D)pH of the solution
of the stoichiometric point of a titration
32. The oxidation number of Cr in K2 Cr2 O7
is called:
is:
(A)Equivalence point
(A)4 (B)5 (C)6 (D)7
(B)Stoichiometric point
33. Which of the following is NOT a type of
(C)End point (D)Flash point
quantitative analysis?
26. Which of the following is not a descrip-
tion of a primary standard? (A)Instrumental analysis
(A)Anhydrous (B)Volumetric analysis
(B)Non-deliquescent (C)Conductimetric analysis
(C)High purity (D)Gravimetric analysis
(D)Insoluble in water
34. The oxidation number of sulphur in S2 O32−
27. In iodometry,which of the following indi- and S4 O62− are respectively:
cator is used? (A)2 and 3 (B)2 and 2.5
(A)KM nO4 (B)Starch solution (C)Potassium (C)1 and 2 (D)3 and 6
thiocyanate (D)Methyl blue
35. When HCl solution in a burette is added
28. Dissolving two moles(80g) of NaOH in drop-wise to NaOH solution in a solution
1000g of water gives: in a conical flask,which of the following
(A)Molal solution (B)Molar solution best describes what happens in the con-
(C)Concentration solution ical flask?
0.34. CHEM 161 2011/2012 175

(A)The pH of the conical flask content 2. C


increases
3. 0.0893mol of KM nO4 is present in 1000cm3
(B)NaCl crystals begin to appear
x mol of KM nO4 will be present in 9.20cm3

(C)Concentration of the OH in the con-
0.0893mol × 9.20cm3
ical flask decreases x=
1000cm3
(D)Concentration of H + in the conical x =8.2156 × 10−4 mols of KM nO4
flask is neutralized
4. From equation of reaction,
36. What will be the mass of 0.0125 moles of
1mol of KM nO4 reacted with 5mols of
CuSO4 .5H2 O?
F e2+
(A)2.0grams (B)0.3125grams
8.2156 × 10−4 mols of KM nO4 will react
(C)0.20grams (D)3.125grams
with x
37. If it takes 10.30cm3 of 3.57M H2 SO4 in a 8.2156 × 10−4 mols × 5mols
x=
battery to neutralize 38.70cm3 NaOH,what 1mol
−3
is the molarity of NaOH? x = 4.1078 × 10 mols × 1000cm3
(A)1.90M (B)0.901M (C)1.09M (D)0.19M
5. 4.1078 × 10−3 molsof F e2+ is present in
38. Indicators are: 10cm3
(A)Secondary standards x mols of F e2+ will be present in 1000cm3
(B)Buffer solutions 4.1078 × 10−3 mols × 1000cm3
x=
(C)Weak acids or bases 10cm3
(D)Colourful x = 0.41078moldm−3

6. A
39. Why is the conical flask swirled properly
during titration? 7. D
(A)To detect the correct end point
8. C
(B)To prevent secondary reactions
(C)To obtain a homogeneous mixture 9. C
(D)To increase the rate of the reaction 10. Equation of reaction
40. When copper (II) reacts with an iodide,the HClaq + N aOHaq −→ N aCls + H2 Ol
ionic equation of the reaction can be rep- 0.0782mol is present in 1000cm3
resented as:
x mols of HCl will be present in 15cm3
(A)Cu2+ + 4I − −→ CuI2 + I2
0.0782mol × 15cm3
(B)2Cu2+ + 4I − −→ Cu2 I2 + I2 x=
1000cm3
(C)2Cu2+ + 6I − −→ 2CuI2 + I2 x = 0.001173mols of HCl
(D)2Cu2+ + 4I − −→ 2CuI2 + 3e− 1mol of HCl reacted with 1mol of Na
0.001173mols of HCl will react with x mols
0.34.1 ANSWER TO CHEM 161 Na
0.001173mols×1mol
2011/2012 x= 1mol

Equation of reaction x = 0.001173mols of NaOH


M nO4− + 5F e2+ + 8H + −→ M n2+ + 5F e3+ + This implies that,
4H2 O 0.001173mols of NaOH is present in 23cm3
1. x mols of NaOH will be present in 1000cm3
176

0.001173mol × 1000cm3 0.000646mols of K2 Cr2 O7 will react with


x=
23cm3 x
x = 0.051moldm−3 3mols × 0.000646mols
x=
The answer is A 1mol
x = 0.001938mols of I2
11. B
1mol of I2 reacted with 2mols of N a2 S2 O3
12. 0.001938mols will react with x
13. D x = 2 × 0.01938mols
x = 0.003876mols of N a2 S2 O3
14. Equation of reaction KH(C8 H4 O4 )aq +
N aOHaq −→ N aK(C8 H4 O4 ) + H2 O This implies that
No of mole = mass 0.003876mols is present in 41.15cm3
molarmass
molar mass of [KH(C8 H4 O4 )] x mols of N a2 S2 O3 will react with 1000cm3
=39+1+8(12)+4(1)+4(16) 1000cm3 × 0.003876mols
x=
41.15cm3
=39+1+96+4+64
x = 0.0942M The answer is C
=204g/mol
0.58g 16. Equation of reactions
No of mole= 204g/mol
=0.00284mols
3HClaq + Al(OH)3 −→ AlCl3 + 3H2 Ol
From equation of reaction,
0.17mols of HCl is present in 1000cm3
1mol of KHP required 1mol of NaOH
x mols of HCl will be present in 50cm3
0.00284mols of KHP will require x mol
0.00284mols × 1mol 0.17mols × 50cm3
x= x=
1mol 1000cm3
x = 0.00284mols of NaOH x = 0.0085mols of HCl

This implies that, From equation of reaction


0.00284mols of NaOH is present in 20.22cm3 3mols of HCl reacted with 1mol of Al(OH)3
x mol of NaOH will be present in 1000cm3 0.0085mols will react with x mols
0.0085mols × 1mol
0.00284mols × 1000cm3 x=
x= 3mols
20.22cm3
x = 0.00283mols of Al(OH)3
x = 0.141moldm−3 The answer is B
mass
No of moles= molarmass
15. Equation of reactions
mass= no of moles× Molar mass
K2 Cr2 O7 +6KI+7H2 SO4 −→ Cr2 (SO4 )3 +
Molar mass of Al(OH)3
4K2 SO4 + 7H2 O + 3I2
=27+(16+1)3 =27+(17)3 =27+51=78g/mol
2N a2 S2 O3 + I2 −→ 2N aI + N a2 S4 O6
mass Mass= 0.00283 × 78 Mass= 0.22074g
No of moles= molarmass
Mass= 0.221g The answer is C
Molar mass= 2(39)+2(52)+7(16)
= 78+104+112 17. Equation of reaction
= 294g/mol HClaq + N aHCO3 −→ N aCls + H2 O +
No of moles= 0.19g
=0.000646mol CO2
294g/mol
From equation of reaction No of moles of HCl= 0.0085mols

1mol of K2 Cr2 O7 reacted with 3mols of From equation of reaction


I2 1mol of HCl reacted with 1mol of N aHCO3
0.34. CHEM 161 2011/2012 177

0.0085mols of HCl will react with x 25. C


0.0085mols × 1mol
x= 26. D
1mol
x = 0.0085mols of N aHCO3
mass 27. B
No of moles= molarmass
Molar mass of N aHCO3 28. A
=23+1+12+3(16)=84g/mol
29. 3.0mols of solute is present in 1000cm3
Mass= 0.0085mols × 84g/mol= 0.714g
x mols of solute will be present in 250cm3
The answer is B
3 × 250cm3
18. x=
1000cm3
19. D x = 0.75mols The answer is C

20. B 30. A

21. C 31. B
22. 2mols of HCl is present in 1000cm3 32. K2 Cr2 O7
x mols of HCl will be present in 100cm3
2(1)+2(x)+7(-2)=0
2mols × 100cm3
x= 2 + 2x − 14 = 0
1000cm3
mass
No of moles= molarmass 2x = 12 =⇒ x = 6 The answer is C
Molar mass of HCl= 36.5g/mol
33. C
Mass= 0.2mols × 36.5g/mol= 7.39
The answer is B 34. S2 O32−

23. 2(x) + 3(−2) = −2


2x = 4 =⇒ x = 2 and
24. 0.0892mols of NaOH is present in 1000cm3
x mols of NaOH will be present in 19.33cm3 S4 O62−
0.0892mols × 19.33cm3 4x + 6(−2) = −2
x=
1000cm3 4x=10
x = 0.00172mols of NaOH
x= 10
4
=2.5
1mol of NaOH required 1mol of aspirin
2 and 2.5 respectively ,The answer is B
0.00172mols of NaOH will require x mols
x = 0.00172mols of C9 H8 O4 35. C
Molar mass of C9 H8 O4
36. CuSO4 .5H2 O
Molar mass= 9(12)+8(1)+4(1)
Molar mass= 64+32+4(16)+5(2×1+16)
= 108+8+64
=64+32+64+5(2+16) =250g/mol
= 180g/mol
mass= noof moles × molarmass No of mole= 0.0125mols

mass= 0.00172 × 180= 0.3096g mass= noof moles × molarmass


mass= 0.3096g × 1000m = 309.6mg mass= 0.0125mol × 250g/mol
mass= 310mg The answer is D mass= 3.125grams The answer is D
178

37. 2N aOH + H2 SO4 −→ N a2 SO4 + 2H2 O (A)0.265g (B)1.06g (C)0.53g (D)2.12g


3
3.57mols of H2 SO4 is present in 1000cm
x mols of H2 SO4 will be present in 10.30cm3 5. Phenolphtalien indicator in an acidic medium
3.57mols × 10.30cm3 is
x= (A)purple (B)colorless (C)Pink (D)Orange
1000cm3
x = 0.036771mols of H2 SO4 6. In the titration of KMnO against , which
From the equation of reaction is the reductant?
(A)KM N4 (B)N a2 C2 O4 (C)H2 SO4 (D)M nSO4
1mol of H2 SO4 reacted with 2mols of
NaOH 7. The blue black coloration in iodometry
0.036771mols of H2 SO4 will react with x is as a result of a reaction between
2mols × 0.036771mols (A)I2 /KM nO4 (B)N a2 S2 O4
x= (C)I2 /starch (D)starch/KM N O4
1mol
x =0.07354mols 8. 25cm3 of a 6M solution of was diluted to
0.07354mols of NaOH is present in 38.70cm3 with distilled water, what is the concen-
tration of the solution in glitre−1
x mols of NaOH will be present in 1000cm3
(A)135 (B)90 (C)1.5 (D)67.5
0.07354mols × 1000cm3
x= 9. The most frequently use indicator for the
38.70cm3
x = 1.90moldm−3 of NaOH titration of against fe( is
(A)phenopthalein (B)Such indicator (C)methyl
x = 1.90M The answer is A orange (D)none of the above
38. 10. The theoretically determined position of
the stoichiometric point of a titration is
39. C
called
40. B (A) Equivalence point (B) Stoichiometric
point (C) End point (D) Flash point

0.35 CHEM 161 2012/2013 11. Which of these properties is undesirable


of a primary standard?
1. The oxidation numbers of N a2 S2 O3 and (A) High purity (B) High solubility in
N a2 S4 O6 respectively are water (C) Anhydrous (D) Deliquescence
(A)2 and 2.5 (B)4 and 2 (C)3 and 1.5
12. A drug company claims to have 350mg
2. Which of the following statements is true of acetylsalicylic acid per 500mg tablet
3
about a molar solution of KHP [KH(C8 H4 O6 ] of aspirin. Upon titration, 20.00cm of
(A)12.50g of 10003 of solution 0.09M solution of NaOH was needed to
(B)10 /0 of solution neutralize the acid. Is the claim true?
3
(C)1g of KHP in 1000cm of solution (A) Yes (B) No (C) Approximately (D)None
(B)1 of KHP of solution of the above
(D)204g of kHP in of solution 13. Two student A and B performed the same
3. Dissolving 106g of in 1000g 0f water gives titration, with A swirling his conical flask
a and B leaving his undisturbed. Which
(A)Formal solution (B)Molar solution will be more accurate and why?
(C)Molal solution (D)Normal solution (A) A, homogeneity of the mixture is at-
tained
4. How many grams of a student would a (B) B, the reaction proceeds undisturbed
student require to prepare of a 0.01M so- (C) A and B, reaction is unaffected
lution of the salt (D) A, prevents secondary reaction
0.35. CHEM 161 2012/2013 179

14. A 100L chemistry student present the ta- A 0.01M solution of standard HCl was
ble below in an experiment to standard- titrated against a 10CM3 mixture ofNaOH
ize KM nO4 using HC2 O4 solution. In and Na2 CO3 . The amount of acid needed
an earlier titration the student found out to reach the end point as indicated by
that 10.00cm3 Of the H2 C2 O4 required phenolphthalein was 13.50cm3 and the
10.50cm3 of 0.02M NaOH for complete amount of HCI required to reach the end
neutralization. The student dilute the point as indicated by methyl orange was
10.00cm3 of the H2 C2 O4 solution to a 3.70cm3 .
85.00cm3 with distil water before adding
15.00cm3 of 2M H 2 SO4 and the mixture 21. The volumes of the HCI that reacted with
heated to about 800c, the mixture was NaOH and Na2 CO3 respectively are.
titrated against kmno4 solution to a per- a. 9.80cm3 and 7.40cm3
manent pink coloration. b. 3.70cm3 and 9.80cm3
c. 7.40cm3 and 9.80cm3
1st 2nd d.3rd7.40cm3 and 15.50cm3
3
Final Burette Reading (cm ) 12.20 12.90 25.00
22. Concentration of Na2CO3 in Mol/dm3
Initial Burette Reading (cm3 ) 0.00 0.00 12.90
a. 0.0013M b. 0.0098M
Use the table above to answer question
c. 0.0074M d. 0.0037M
15-20
23. Concentration of NaOH in mol/dm3
15. What is the average volume of KMnO4
a. 0.0098M b. 0.000098M
Used?
c. 0.001M d. 0.0015M
(A) 12.40cm3 (B) 12.50cm3
(C) 12.15cm3 (D)12.35cm3 24. Concentration of NaOH in g/dm3
16. What is the molarity of the H2 C2 O4 so- a. 0.3920 b. 0.00392
lution? c. 0.0600 d. 0.0400
(A) 0.0105M (B) 0.105M
25. Concentration of N a2 CO3 in g/dm3
(C) 0.075M (D) 0.20M
a. 0.1378 b. 0.3922 c. 0.7844 d. 0.6996

26. If 21.30cm3 of 0.0785M NaOH Neutral-


17. What is the stoichiometry in the redox
ized a solution containing one dissolved
reaction?
aspirin tablet, what is the amount of acetyl-
a. 2:1 b. 2:5 c. 1:5 d. 5:10
salicylic acid (molar mass of aspirin is
18. What is the molarity of KMnO4? 180g) per tablet to the nearest mg?
a. 0.00015M b. 0.0015M a. 301 b. 300 c. 300.9 d. 300.97
c. 0.0030M d. 0.0346M
27. A solution with a relatively small amount
19. What function does the 2M H2 SO4 added of solute is said to be
serves? a. Concentrated b. Standard
a. Indicator b. catalyst c. Dilute d. Minute
c. buffer d. oxidizing agent
28. The end point of the titration between
20. Should the student accidentally spill some NaOH and aspirin is when a permanent
acid into his eyes, after flushing with wa- pink colour is observed . after standing
ter, what would you advise him to add for several minutes, the pink colour dis-
as a first aid? appears. This is due to
a. 5% Boric acid b. 5% NaHCO3 a. Esterification of the acetyl group
c. 5% potassium thiocyanate d. 5% NaOH b. Reverse neutralization reaction
Use the information below to answer the c. hydrolysis of the acetyl group
questions 21-25. d. polarization of the acetyl group
180

29. All except one are types of titrimetric A. presence of certain ions in solution
analysis b.amounts of elements or compounds present
a. Acid Base titration in a sample
b. Precipitation titration c. nature of constituent in a sample
c. complexometric titration D.what the constituents in a sample are
d. lionization titration

30. Normality is defined as 36. what is the morality of a solution con-


a. Number of moles of solute in 1000g of taining 12.3g of NaOH dissolved in 250cm3 ?
solvent A. 1.23m b. 12.3M c. 14.6M d. 1.46M
b. Number of moles of solute in 1dm3 of 37. Vinegar contains acetic acid, CH3COOH.
solute Titration of 5.00g of vinegar with 0.15M
c. number of gram equivalent weights of NaOH requires 28.00cm3 to reach the equiv-
solute per dm3 of solution alence point. Use this information to an-
d. Mass of solute per dm3 of solution swer question 38 and 39.
use the information below to answer ques-
tions 31 and 32
38. what is the mass of CH3 COOH in Vine-
3 gar?
To 10.00cm of a 0.005M KMnO4 an ex-
cess of KI and acid was added. The lib- A. 0.252g b. 1.252g c.0.334g d.3.34g
erated iodine was titrated with Na2 S2 O3
39. what is the weight percentage of CH3 COOH
of the latter 12.00cm3 was required.
in vinegar?
A. 5.04% b. 25.04% c.6.68% d. 66.8%
31. What is the molar concentration of the 40. which of the following is correct about
N a2 S2 O3 back titration?
a. 0.03M b. 0.0025M c. 0.005M d. 0.0208M A. two standard solutions are required
32. what is the mass concentration of the b. the titrant an dtirand must be equal
Na2S2O3 c. a standard an unknown solution must
a. 4.7401 gdm−3 b. 3.2684 gdm−3 react stoichiometrically
c. 0.7910 gdm−3 d. 5.3200gdm−3 d. the acid is back titrated and the base
frontally titrated.

33. If 10.00cm3 of a solution of H2 S2 O4 is 0.35.1 Answers to 2012/2013 CHEM


titrated with 9.50cm3 of 0.4m NaOH and
161
if 10.00cm3 of the same H2 S2 O4 solution
required 16.50cm3 of a KmnO4 solution. 1. N a2 S2 O3 = 2(1) + 2(x) + 3(−2) = 0
What is the molarity of the KMnO4 so- 2x = 4 ∴ x = 2
lution? N a2 S4 O6 = 2(1) + 4(x) + 6(−2) = 0
a. 0.76M b. 0.19M c. 0.046M d. 0.45M 4x = 10 ∴ x = 2.5(A)

34. A bottle of HCI is labeled 2M. what mass 2. KHP= [KH(C8 H4 O4 )]


in grams of HCI would be found dissolved Molar mass= 39 + 1 + (8 × 12) + (4 × 1) +
in 500cm3 of this solution? (4 × 16) = 204g/mol
a. 36.5 b. 73 c. 7.3 d. 3.68 204 g of KHP in 1000cm3
mass 204g
no of moles = molar mass
= 204g/mol =1
35. If 1.84g of a salt (X) is dissolved in 100cm3 This implies that
of solution. What is the mass concentra- 1mol of KHP is present in 1000cm3
tion of the solution. x mol of KHP will be present in 1000cm3
2 ×1
A. 18.4 b. 184 c. 10.84 d. 3.68 x = 1000cm
1000cm3
36. Quantitative analysis determines? = 1M (D)
0.35. CHEM 161 2012/2013 181

3. C 14.

4. Molarity of N a2 CO3 = 0.01M 15. C


3
volume of Na2 CO3 = 250cm
16. volume of NaoH = 10.50cm3
0.01 moles of Na2 CO3 is present in 1000cm3
Molarity of NaOH = 0.02M
x mols of Na2 CO3 will be in 250cm3
0.01mols × 250cm3 volume of H2 C2 O4 = 10cm3
x= = 0.0025mols 2N aOH + H2 C2 O4 → N a2 C2 o4 + H2 O
1000cm3
Molar mass of Na2 CO3 = (23 × 2) + 12 + 0.02 mols of NaOH is present in 1000cm3
(16 × 3) = 106g/mol x mols of NaOH will be present in 10.50
mass
no of moles = molar mass cm3
3 ×0.02mols
mass required = no of moles × molar x = 10.50cm
1000cm3
= 0.00021mols
mass From equation of reaction
=0.0025mols × 106g/mol = 0.265g(A) 2mols of NaOH react with 1 mol of H2 C2 O4
0,0021mols of NaOH will react x mols
5. B 0.00021mols × 1mol
x= = 0.000105mols
2mol
6. B This implies that
0.000105 mols of H2 C2 O4 is in 10cm3
7. C x mols of H2 C2 O4 will be in 1000cm3
3 3
X = 1000cm ×0.0000105cm
10cm3
= 0.0105moldm−3 (A)
8. Molar concentration of H2 C2 O4 = 6M
Volume of H2 C2 O4 = 25CM 3
6 mols of H2 C2 O4 is present in 1000cm3 5H2 C2 O4 +2KM nO4 +3H2 SO4 → K2 SO4 +
x mols of H2 C2 O4 will be present in 25 2M nSO4 + 10CO2 + 8H2 O
cm 3 2N aOH + H2 C2 O4 → N a2 C2 O4 + 2H2 O
25cm3 × 6mols 0.02 moks of NaOH is present in 1000cm3
x= = 0.015mols
1000cm3 mass x mols of NaOH will be present in 10.50cm3
no of moles = molar mass 0.02mols × 10.50cm3
molar mass of H2 C2 O4 = (2×1)+2(12)+ x= = 0.00021mols
1000cm3
4(16) = 90g/mol 2 mols of NaOH reacted with 1 mol of
−1
mass= 90gmol × 0.15mols = 13.5g H2 C2 O4
Concentration in g litre−1 = volumn(lt)
mass
0.00021 mol of NaOH will react with x
1 litre is equivalent to 1000cm3 mols
−1 13.5g 0.00021mol × 1mol
concentration in glitre = 1lt x= = 0.000105mols
concentration in glitre−1 = 13.5glitre−1 2mols
This implies that
9. D,because no indicator is required in re- 0.000105mols of H2 C2 O4 is present in 10cm3
dox tireation x mols of H2 C2 O4 will be present in 1000cm3
3
x = 0.000105mols×1000cm
10cm3
= 0, 0105moldm−3 (A)
10. A
17. B from the equation of reaction
11. D
18. 0.0105 mol of H2 C2 O4 is in 1000cm3
12. 0.09mol of NaoH is present in 1000 cm3 x mol of H2 C2 O4 will be present in 10cm3
3 ×0.0105mol
x mols of NaoH will be present in 20cm3 x = 10cm1000cm 3 = 0.0000105mols
x= 20cm3 ×0.09mols
= 0.0018mols from equation of reaction
1000cm3
mass
no of moles = molar mass 5 mols of H2 C2 O4 react with 2 mols of
mass= 180gmol−1 ×0.0018mols = 0.324g KM nO4
But 1000mg is equivalent to 1g 0.000105 mols of H2 C2 O4 will react with
x mg wil be equivalent to 0.324g x
x= 0.324g×1000mg
= 324g(B) x = 2×0.000105mol
5mols
= 0.000042mols
1g
This implies that
13. A 0.000042 mol of KM nO4 is present in
182

3
12.15 x = 0.0785mols×21.30cm
1000cm3
= 0.001672mols
x mols of KM nO4 will be in 1000cm3 1mol of NaOH reacted with 1mol of as-
3
x = 0.000042mols×1000cm
12.15cm3
pirin
x = 0.00346moldm−3 of KM nO4 0.001672mol of NaOH will react with x
x = 0.00167mols of aspirin
19. mass = 0.001672mol×180gmol−1 = 0.30097g
20. D mass ≈ 301mg(A)

27. C
21. A
28.
22. 2HCL+N a2 CO3 → 2N acl+CO2 +H2 O
0.01 mols of Hcl is present in 1000 cm3 29. D
x mols of Hcl will be present in 7.4 cm3
3
x = 0.01m×7.40cm = 0.000074mols 30. C
1000cm3
from the equation of reaction
31. Equation of reaction
2 mol of Hcl reacted with 1mol of N a2 CO3
2KM nO4 + 10KI + 8H2 SO4 → 5I2 +
0.000074mols will react with x mol
6K2 SO4 + 2M nSO4 + 8H2 O
x = 1mol×0.000074mol
2mols
= 0.000037mols
And
This implies that
2N a2 S2 O3 + I2 → 2N aI + N a2 S4 O6
0.000037 mols is present in 10cm3
Molarity of KM nO4 = 0.005M
x mol of N a2 CO3 will be present in 1000cm3
3 volume of KM nO4 = 10.00CM 3
x = 0.000037mols×1000cm = 0.0037(D)
10cm 3
0.005MOLS of KMnO4 is present in 1000cm3
23. Concentration of NaOH in moldm−3 x mols of KM nO4 will be present in 10.00cm3
equation of reaction x= 0.0005 mols of KM nO4
N aOH + HCL → N acL + H2 O From equation of reaction
1 : 1 2mols of KM nO4 required 5mols of I2
0.01mols of Hcl is present in 1000cm 3 0.00005mols of KM nO4 will require xmols
x mols of Hcl will be present in 9.80 cm 3 x= 0.000125 mols of I2
0.01mol×9.80cm3 From the second equation
x 1000cm3
= 0.000098mols
1mol of Hcl reacted with 1 mol of NaOH 1 mols of I2 required 2 mols of N a2 S2 O3
0.000098MOL of Hcl will react with x 0.00125MOLS OF I2 will require x mol
x= 0.000098mol×1mol
= 0.000098molof N aOH x = 0.00025mols of N a2 S2 O3
1mol
0.000098MOL OF NaOH is in 10cm 3 VOLUME OF N a2 S2 O3 =12.00cm3
x mol of NaOH will be in 1000cm3 This implies that
x= 0.000098mol×1000cm3
= 0.0098moldm (A) −3 0.00025mols of N a2 S2 O3 is in 12.00 cm3
10cm3
x mols of N a2 S2 O3 will be in 1000 cm3
−3
24. Concentration of NaOH in g/dm x= 0.0208 moldm−3 (D)
Mass concentration = Molarity × Molar
mass 32. Mass Concentration of N a2 S2 O3
Molar mass of NaOH = 40g/mol Molarity = 0.0208M
Molarity of NaOH = 0.0098moldm−3 Molar mass of N a2 S2 O3 = (23×2)+(2×
Mass concentration = 0.0098M OL 40g
× mol = 32) + (3 × 16) = 158gmol−1
dm3
0.092gdm−3 (A) Mass Concentration = Molarity × Molar
mass
25. Concentration of N a2 CO3 in g/dm−3 = 0.0208moldm−3 × 158gmol−1 (B)
molarity of N a2 CO3 = 0.0037M OLDM −3
Mass concentration = 0.003moldm−3 × 33. Equation of reaction
106gmol−1 = 0.3922gdm−3 (B) H2 C2 O4 + 2N aOH → N a2 C2 O4 + 2H2 O
2KM nO4 +5H2 C2 O4 +3H2 O → K2 SO4 +
26. 0.0785 of NaOH is present in 1000cm3 2M nSO4 + 10CO2 + 8H2 O
3
x mol of NaOH will be present in 21.30cm volume of NaOH = 9.50 cm3
0.36. CHEM 161 2013/2014 183

Molarity of NaOH = 0.4 M 0.36 CHEM 161 2013/2014


volume of H2 C2 O4 = 10.00cm3
0.4 mols of NaOH is present in 1000cm3 1. Qualitative analysis is different from quan-
x mols of NaOH will be present in 9.50cm 3 titative analysis in that
x= 0.0038mols of NaOH A.the former requires heat
from equation of reaction
B.the latter is restricted to organic chem-
2mol of NaOH required 1mol of H2 C2 O4
istry only
0.0038mol of NaOH will required x mol
x= 0.0019mols of H2 C2 O4 C.the former involves only instrumental
From second equation of reaction analysis
2mols of KM nO4 required 5 mols of H2 C2 O4 D.the latter involves determination of the
X mols of KM nO4 will require 0.0019 amount of element or compounds present
mols in a sample
x= 0.00076mol
This implies that 2. The point at which stoichiometrically equiv-
3 alent quantities of acid and base have re-
0.00076 mol of KMnO4 is present in 16.50cm
x mol of KM nO4 will be present in 1000cm3 acted in a titration is called
x= 0.046M (C) A.end point B.equivalence point
34. 2 mol of Hcl is present in 1000cm3 C.reaction point D.standard point
x mols of Hcl will be present in 500cm3
m= 1 mol 3. In the determination of the amount of
mass
no of mole = molar acetylsalicylic acid in aspirin tablet,the
mass
Molar mass of Hcl = 1+ 35.5=36.5g/mol tablet is dissolved in water and heated
mass= 1mol ×36.5g/mol = 36.5g(A) to about 90 C
mass A.break the bonds in the compound
35. Mass concentration = volum(dm 3 ) Mass con-
1.84g
centration = 100/1000dm 1.84
3 = 0.1 = 18.4gdm
−3
(A) B.help the aspirin tablet dissolve in water
C.make the solution colourless
36. B
D.catalyze the reaction
37. Molar mass of NaOH = 23 + 16 + 1 =
40g/mol 4. Which of the following indicators is a suit-
mass 12.3 able choice for a weak acid and strong
no of moles = =
molar mass 40g/mol base titration?
= 0.3075mols
no of moles 0.3075moles A.Phenolphthalein B.Methyl red
Molarity = =
volume in dm3 250/1000 C.Methyl orange D.Starch
0.3075moles
= = 1.23(A) 5. In the iodometric titration of CuSO4 .5H2 O
0.25
against N a2 S2 O3 the indicator used is
38. 0.15 mols of NaOH is present in 1000cm3
x mols of NaOH will be present in 28cm3 A.Phenolphthalein B.Methyl red
x = 0.0042mols of NaOH C.Methyl orange D.Starch
This implies that CH3 COOH has 0.0042
mols 6. In the redox titration of KM nO4 with
mass KI,the oxidant and reductant species are,
no of moles =
molar mass respectively
mass = 0.0042 moles × 60 = 0.252g(A)
mass
A.KM nO4 and KI
39. Weight percentage = molar mass
× %
0.252 B.KM nO4 andKM nO4
= × 100% = 5.04%(A)
5g C.KI and KM nO4
40. A D.KI and KI
184

7. Which of these solutions is used as a first 13. If the approximate concentration of the
aid measure to neutralize acid on skin? hydrochloric acid in the stomach (stom-
A.5 percent sodium bicarbonate ach acid) is 0.17 M,calculate the mass of
the antacid N aHCO3 required to neu-
B.5 percent glycerol tralized 50cm3 of this acid.
C.5 percent boric acid A.0.714 g B.0.221 g C.0.515 g D.0.005 g
D.5 percent sodium hydroxide
14. The indicator used in the redox titration
8. One of the following is not a type of titri- of KM nO4 and N a2 C2 O4 is
metric analysis. A.Methyl orange
A.Complexometric titration B.Phenolphthalein
B.Precipitation titration C.Starch
C.Oxidation-Reduction titration D.None of the above
D.Solubility titration
15. The stoichiometry of the reaction of F e2+
9. The amount of a substance expressed in and M nO4− (in that order) is
number of moles of solute per 1000g of A.1:5 B.2:5 C.5:1 D.5:2
solvent is known as
A.Molarity B.Molality 16. An aspirin tablet is suspected to have a
higher amount of acetylsalicylic acid per
C.Normality D.Concentrated solution mg of tablet than indicated by the drug
company(190 mg per 250 mg tablet).12.50cm3
10. The colour change of phenolphthalein in-
of a 0.1M NaOH solution was required
dicator from acid to base is
to completely neutralize the acid in one
A.colourless to pink tablet.What amount of acetyl salicylic acid
B.yellow to orange is present per 250 mg tablet?(Molar mass
of aspirin is 180g)
C.blue-black to colourless
A.190 mg B.225 mg
D.purple to colourless C.180 mg D.203.5 mg
11. Which of the following statements is true Use the information below to an-
about a 0.5 molar solution of KHP [KH (C8 H4 O4swer
)]? questions 17-21
A. 12.5g of KHP in 1000cm3 of solution A 0.5 M solution of standard HCl was
titrated against a 10cm3 mixture of NaOH
B.1 percent of KHP in 1000cm3 of solu-
and N a2 CO3 . The amount of acid needed
tion
to reach the end-point as indicated by
C.1g of KHP in 1000cm3 of solution phenolphthalein was 21.00cm3 and the
D.204g of KHP in 2000cm3 of solution amount of HCl required to reach the end-
point as indicated by methyl orange was
12. Which of the statements following is cor- 7.50cm3 .
rect about back titration?
17. The volumes of the HCl that reacted with
A.Two standard solutions are required
NaOH and N a2 CO3 respectively are
B.The titrant and titrand must be equal
A. 28.50cm3 and 11.00cm3
C.A standard and unknown solution must
B. 10.50cm3 and 9.80cm3
react stoichiometrically
C. 13.50cm3 and 15.00cm3
D.The acid is back titrated and the base
frontally titrated D. 3.25cm3 and 15.50cm3
0.36. CHEM 161 2013/2014 185

18. The concentration of N a2 CO3 in mol/dm3 24. What is the molarity of the oxalic acid
is solution?
A. 0.130 B. 0.098 C. 0.089 D. 0.375 A.0.0515M B.0.098M
C.0.001M D.0.103M
19. The concentration of NaOH in mol/dm3
is 25. The stoichiometry of the oxidant to re-
A. 0.675 B. 0.98 C. 0.15 D.0.015 ductant is:
A. 1:5 B. 1:2 C. 5:1 D. 2:5
20. The concentration of NaOH in g/dm3
26. The concentration of the KM nO4 is
A. 39.20 B. 3.92 C. 27.00 D. 2.70
A. 0.000206M B. 0.0196M
3
21. The concentration of N a2 CO3 in g/dm C. 0.0015M D. 0.0206M
is
27. What gives oxalic acid its special feature
A.39.75 B.3.97 C.15.66 D.1.56 in this experiment?
22. To prepare a 0.8M CaCO3 solution,what A. Its dual function as an acid and a re-
amount,in grams of CaCO3 ,would be dis- ducing agent
solved in 250cm3 ? B. its low concentration
A.40 B.125 C.20 D.80 C. its being an organic acid
3
Titrate 10cm of oxalic acid which had D.its dibasic nature
previously been diluted with distilled wa-
ter to about 50cm3 and 3 drops of phe- 28. The change in oxidation state of Mn in
nolphthalein indicator is added,with a stan- KM nO4 is from
dard NaOH solution (0.098M) to the first A. +2 to +7 B. +7 to +4
3
faint pink appearance.To a fresh 10 cm C. +4 to +1 D. +7 to +2
of oxalic acid solution,add 85cm3 distilled
water and add 15cm3 of 2 M H2 SO4 and 29. The oxidation number of S in N a2 S4 O6
◦ ◦
heat to about 60 C-80 C.Titrate the mix- and N a2 S2 O3 are respectively
ture with KM nO4 solution to be stan- (a) 2.5 and 2 (b) 4 and 2
dardized until it retains a permanent faint (c) 6 and 4 (d) 3 and 1.5
pink colour. 30. Two students(A and B) performed the
A 100L Chemistry student followed the same titration,with A swirling his conical
above procedure and generated the data flask and B leaving his undisturbed.Which
in the table below will be more accurate and why?
Burette Reading NaOH A. A,prevents secondary
KM nO4 reaction
st st st st
1 2 B. 3B,the reaction
1 2st
proceeds 3st
undisturbed
Final Burette Reading (cm3 ) 10.50 10.50 12.00 11.20 11.20 21.70
Initial Burette Reading (cm3 ) 0.00 21.00 C.0.00 A and 0.00
B,reaction is unaffected
21.70 32.20
D. A,homogeneity of the mixture is at-
Use the information in the table above
tained
to answer questions 23-28
31. What amount,in grams,of Al(OH)3 will
23. What is the average volumes of NaOH be equivalent 40g NaOH?
and KM nO4 used?
A. 78 B. 40 C. 80 D. 100
A.11.00cm3 and 10.73cm3
Use the options below to answer ques-
B.10.50cm3 and 10.50cm3 tions 32 and 33
C.12.00cm3 and 11.20cm3 A. Always true B. Never true
D.10.50cm3 and 11.20cm3 C. Sometimes true D. None of the above
186

32. A desirable property of a primary stan- 40. One of the following is true of redox titra-
dard is hygroscopy tions
A. They are self-indicating
33. Potassium hydrogen phthalate is a strong
acid B. They can only be carried out via iodom-
etry
34. What determines the choice of indicators C. The number of electrons gained by re-
in titration? duced species is same as that (those) lost
A. The nature of acid/base pair by oxidized species
B. The concentration of the acid and the D. Can only take place in an acidic medium
base
Answers to 2013/2014 CHEM 161
C. The nature of the expirement
D. The temperature of the reaction 1. D

2. B
35. Which of the following is not a common
way of expressing concentration in ana- 3. B
lytical chemistry?
4. A
A. ppm B. mole fraction
C. specific gravity D. percent purity 5. D

Use the information given below to 6. A


answer questions 36-38
7. D
Vinegar contains acetic acid (CH3 COOH).Titration
of 5.00g of vinegar with 0.50M NaOH re- 8. D
3
quires 20.00cm to reach the equivalence
point 9. B (Molality is also defined as the amount
of a substance expressed in number of
36. What is the mass of CH3 COOH in the moles of solute per 1kg of solvent)
vinegar?
10. D
A. 0.60g B. 0.198g C. 4.20g D. 0.10g
11. D
37. What is the weight percent of CH3 COOH KHP=KH(C8 H4 O4 )
in the vinegar?
K=39;H=1;C=12 and O=16
A. 3.96 percent B. 20.0 percent
molar mass of KH(C8 H4 O4 ) = 39 + 1 +
C. 0.15 percent D. 12.0 percent [(8 × 12) + (4 × 1) + (4 × 16)]
=40+[96+4+64]=40+164=204g/mol
38. If the concentration of the NaOH solu-
tion used is increased to 1.0M,what will mass given=204g
be the volume of the NaOH solution used? volume given=2000cm3
A. 20.10cm3 B. 10.00cm3 and molar mass = 204 g/mol
3 3 mass given
C. 15.00cm D. 12.50cm
no of mole =
molar mass
39. The oxidant in a titration is ........ at the 204g
end of the titration no of mole =
204g/mol
A. balanced B. reduced no of mole of KH(C8 H4 O4 ) = 1 mol
C. oxidized D. precipitated This implies that,
0.36. CHEM 161 2013/2014 187

1 mol of KHP is present in 2000cm3 Note: 1mol of NaOH will require x mol of
x mol of KHP will be present in 1000cm 3 aspirin
1mol × 1000cm3 0.00125mol of NaOH will require x mol of
x= aspirin
2000cm3
x = 0.5 moldm−3 0.00125mol × 1mol
x=
1mol
x = 0.5 Molar (D)
x = 0.00125mol of aspirin
12. mass
No of mole =
molar mass
13. Molality of HCl= 0.17M mass = noof moles × molar mass
Volume of HCl= 50cm3 g
mass = 0.00125mol × 180 = 0.225g
0.17mol of HCl is present in 1000cm3 mol
To convert to milligram (mg) you multi-
x mol of HCl will be present in 50cm3 ply by 1000
0.17mol × 50cm3
x= Therefore, mass = 0.225 × 1000mg
1000cm3
x= 0.0085 mol of HCl mass = 225mg The answer is B
HCl+N aHCO3 −→ N aCl+H2 O+CO2 16. Molarity of HCl = 0.5M
From equation of reaction Amount of HCl that reacted with NaOH
1mol of HCl neutralize 1 mol of N aHCO3 = 21.00cm3 − 7.50cm3
0.0085mol of HCl will neutralize x mol =13.5cm3
1mol × 0.0085mol Volume of HCl that reacted with N a2 CO3 =
x mol of N aHCO3 =
1mol 2 × 7.50cm3
x mol of N aHCO3 = 0.0085mol =15.00cm3 The answer is C
Molar mass of N aHCO3 = ?
17. C
Na=23,H=1,C=12, and O=16
N aHCO3 = 23+1+12+3(16) 18. Equation of reaction
=23+1+12+48 =84 g/mol 2HCl + N a2 CO3 −→ 2N aCl + CO2 +
H2 O
massrequired
no of mole = This implies that 2 moles of HCl reacted
molarmass
mass required = noof mole×molarmass with 1mol of N a2 CO3
mass required = 0.00085mol × 84g/mol For HCl
mass required = 0.714g Answer is A 0.5mols of HCl is present in 1000cm3
x mols of HCl will be present in 15.00cm3
14. D (No indicator is required in redox titra-
tion) 0.5mols × 15.00cm3
x =
2mols
15. Molar mass of aspirin= 180g x = 0.0075mols
molarity of NaOH= 0.1M For N a2 CO3
3
volume of NaOH= 12.50cm From equation of reaction
3
0.1M of NaOH is present in 1000cm 2mols of HCl reacted with 1 mol of N a2 CO3
3
x M of NaOH will be present in 12.50cm 0.0075mols of HCL will react with x
12.5cm3 × 0.1M 0.0075mols × 1mol
x = x =
1000cm3 2mols
x = 0.00125mols x = 0.00375mols of N a2 CO3
188

This implies that 21. Concentration of N a2 CO3 in g/dm3


0.00375mols of N a2 CO3 is in 10cm3 Mass conc. = M olarity × M olarmass
x mols of N a2 CO3 will be in 1000cm3 Molarity of N a2 CO3 = 0.375mol/dm3
0.00375 × 1000 Molar mass = 106g/mol
x =
10 Mass conc. = 0.375mol/dm3 × 106g/mol
x = 0.375mol/dm3 answer is D Mass conc. = 39.75g/dm3 Answer is A

19. Equation of reaction 22. 0.8mols of CaCO3 is present in 1000cm3


HCl + N aOH −→ N aCl + H2 O x mols of CaCO3 will be present in 250cm3
1: 1 250cm3 × 0.8mols
x =
1000cm3
For HCl x = 0.2mols
0.5mols of HCl is in 1000cm3 mass
no of moles =
molarmass
x mols of HCl is in 13.50cm3 Molar mass of CaCO3 = 40+12+(3 ×
0.5 × 13.50cm3 16)=40+12+48
x =
1000cm3 =100g/mol
x = 0.00675mols
Mass = noof moles × molarmass
For NaOH
mass = 0.2mols × 100g/mol
From equation of reaction mass = 20g answer is C
1mol of HCl reacts with 1mol of NaOH
23. B
0.00675mols of HCl will react with xmol
of NaOH 24. Molarity of oxalic acid solution
0.00675mols × 1mol Equation of reaction
x=
1mol 2NaOH +H2 C2 O2 −→ N aC2 O4 + 2H2 O
x = 0.00675mols of NaOH 2 : 1
This implies that, Molarity of NaOH = 0.098M
0.00675mols of NaOH is present in 10cm3 Volume of NaOH = 10.50cm3
x mols of NaOH will be present in 1000cm3 This means,
0.00675mols × 1000cm3 0.098mols of NaOH is present in 1000cm3
x =
10cm3 x mols of NaOH will be present in 10.50cm3
x = 0.675mol/dm3 0.098mols × 10.50cm3
x =
The answer is A 1000cm3
x = 0.001029mols of NaOH
20. The concentration of NaOH in g/dm3 For H2 C2 O4 (oxalic acid)
Mass concentration = M olarity×M olarmass From equation of reaction
Molarity of NaOH = 0.675mol/dm3 2mols of NaOH reacted with 1mol of H2 C2 O4
Molar mass of NaOH = 23+16+1=40g/mol 0.001029mols of NaOH will react with
x mole
Mass concentration = 0.675mol/dm3 × 0.001029mols × 1mol
40g/mol x =
2mols
Mass concentration = 27g/dm3 (C) x = 0.0005145mols of H2 C2 O4
0.36. CHEM 161 2013/2014 189

This implies that, 2(1)+2x+3(-2)=0


3
Volume of H2 C2 O4 (oxalic acid) = 10cm 2+2x-6=0
3
0.0005145mols is present in 10cm 2x = 4 ⇒ x = +2
3
x mols of H2 C2 O4 will be present in 1000cm Oxidation state of S is +2
0.0005145 × 1000cm3 The oxidation state respectively are +2.5
x =
10cm3 and +2
x = 0.0515mol/dm3 answer is A
The answer is A
25. 2KM nO4 +5H2 C2 O4 +3H2 SO4 −→ K2 SO4 +
2M nSO4 + 10CO2 + 8H2 O 30. D
Oxidant is the oxidizing agent and is KM nO431. Al(OH)3 =27 + (16 + 1)3
while the reductant is the reducing agent
which is H2 C2 O4 .Therefore the stoichiom- = 27+3(17)= 27+51=78 (A)
etry of the oxidant to reductant is 2:5
32. B
The answer is D
33. C
26. Concentration of the KM nO4
For H2 C2 O4 34. C
0.0515mols of H2 C2 O4 is present in 1000cm3 35. C
x mols of H2 C2 O4 will be present in 10cm3
0.0515 × 10 36. CH3 COOH+N aOH −→ CH3 COON a+
x= =0.000515mol H2 O
1000
From equation of the reaction, For NaOH
2mols of KM nO4 react with 5mols of 0.5mols of NaOH is present in 1000cm3
H2 C2 O4
x mols of NaOH will be present in 20.00cm3
x mols of KM nO4 will react with 0.000515mol
0.5mols × 20.00cm3
2mol × 0.000515mols x=
x= 1000cm3
5mols
x = 0.01mols
x = 0.000206mols
From equation of reaction
0.000206mols is present in 10.5cm3
x mols will be present in 1000cm3 1mol of CH3 COOH reacted with 1mol
of NaOH
0.000206 × 1000
x= x mol of CH3 COOH will react with 0.01mol
10.5
x = 0.0196moldm−3 = 0.0196M (B) of NaOH
1mol × 0.01mol
27. A x=
1mol
x = 0.01mols of CH3 COOH
28. D
mass
No of mole =
29. N a2 S4 O6 molarmass
2(1)+4x+6(-2)=0 Molar mass of CH3 COOH
2+4x-12=0 = 12+3(1)+12+16+16+1
4x = 10 ⇒ x = 10/4 = 2.5 = 12+3+12+33=60g/mol
0xidation of S is +2.5 mass = noof mole × molarmass
N a2 S2 O3 = 0.01mol × 60g/mol=0.6g(A)
190

weight
37. Weight percent = × 100%
T otalweight
0.6
Weight percent of CH3 COOH = ×
5
100%
Weight percent of CH3 COOH = 12 %
The answer is D

38. CA VA = CB VB
0.5M × 20cm3 = x × 1M
x = 10cm3
The answer is B

39. B

40. C

You might also like